Sie sind auf Seite 1von 94

TOPNOTCH MEDICAL BOARD PREP SURGERY SUPEREXAM

For inquiries visit www.topnotchboardprep.com.ph or email us at topnotchmedicalboardprep@gmail.com


DEAR TOPNOTCH FRIENDS:

PLEASE FOLLOW THESE INSTRUCTIONS:

1. These questions are previous diagnostic, midterm, and finals exams of Topnotch, almost all of them made by Topnotch Board Exam Topnotchers.
2. Answer this Topnotch Superexam seriously 100-items at a time. Cover the “Explanations” Column. Do not immediately look at the answers from the
answer key. That’s not the correct way of answering sample exams. You need to treat these MCQs as exercises and not as handouts.
3. Time yourself. 1.5 hours per 100-item block.
4. After answering each 100-item block, refer to the Topnotch Answer Key for the correct answers. Please be careful of “frameshift mutations” when
checking your answers – check every 10 items. (the format of the answer key was designed for you to practice against “frameshift mutations”)
5. The Topnotch Superexams are EXERCISES for the actual med boards. They will not appear verbatim in your future exams. More than knowing what’s
the correct answer, it’s more important for you to:
a. Know why the other choices are wrong
b. Know why the other choices were included in the first place
c. Know the explanation to the correct answer
6. Sharpen your mind by answering the Topnotch Superexams. Most of these questions based on past feedback are more difficult than the actual questions
in the med boards. In these exams made by Board Exam Topnotchers, if you’re getting a score of 60/100 , that’s already a good score. More than 80/100
is outstanding.

Item QUESTION EXPLANATION AUTHOR TOPNOTCH
# EXAM
1 The amount of time kidneys for transplantation 6-8hrs – heart and lungs KRISTEL TANHUI (TOP DIAGNOSTIC
can be preserved is: 16hrs - liver 3 - AUG 2015 MED EXAM -
A. 6-8hrs 24hrs - pancreas BOARDS; TOPNOTCH MARCH 2016
B. 16hrs 36-40hrs – kidneys MD FROM LA SALLE)
C. 24hrs
D. 36-40hrs
E. 48-72hrs Source: Schwartz’s Principles of Surgery 8th ed
p 282

2 Which of the following treatment used for the Silver sulfadiazine – antimicrobial activity, KRISTEL TANHUI (TOP DIAGNOSTIC
management of burn wounds can be absorbed soothing quality, inexpensive and easily applied, 3 - AUG 2015 MED EXAM -
systemically and cause metabolic acidosis? neutropenia BOARDS; TOPNOTCH MARCH 2016
A. Silver nitrate Mafenide acetate – antimicrobial activity, MD FROM LA SALLE)
B. Silver sulfadiazine painful, metabolic acidosis
C. Mafenide acetate Silver nitrate – antimicrobial activity,
D. A and B hyponatremia, methemoglobinemia, black
E. All of the above staining

SIMILAR TO PREVIOUS BOARD EXAM
CONCEPT/PRINCIPLE
Source: Schwartz’s Principles of Surgery 8th ed
p 202
3 A 70 year old male incurred 20% TBSA burns. If Parkland formula = 4ml/kg/%burned KRISTEL TANHUI (TOP DIAGNOSTIC
PE revealed normal and stable vital signs, how ½ given in the first 8hrs then the next half given 3 - AUG 2015 MED EXAM -
much fluids should be given to him in the first over the next 16hrs. BOARDS; TOPNOTCH MARCH 2016
hour? MD FROM LA SALLE)
A. 250cc 70x20x4/2/8 = 350cc
B. 350cc
C. 450cc Source: Schwartz’s Principles of Surgery 8th ed
D. 550cc p 200
E. 1 L fast drip

4 In which type of hiatal hernia does the fundus of Type I (sliding) – cardia KRISTEL TANHUI (TOP DIAGNOSTIC
the stomach herniate? Type II (rolling) – fundus 3 - AUG 2015 MED EXAM -
A. Type I Type III – cardia and fundus BOARDS; TOPNOTCH MARCH 2016
B. Type II Type IV – intestines MD FROM LA SALLE)
C. Type III
D. Type IV SIMILAR TO PREVIOUS BOARD EXAM
E. Type V CONCEPT/PRINCIPLE
Source: Schwartz’s Principles of Surgery 8th ed
p 842

5 Which of the following maneuvers involve SIMILAR TO PREVIOUS BOARD EXAM KRISTEL TANHUI (TOP DIAGNOSTIC
clamping of the portal triad and is used for CONCEPT/PRINCIPLE 3 - AUG 2015 MED EXAM -
hemostasis and control of bleeding during hepatic Source: Schwartz’s Principles of Surgery 8th ed BOARDS; TOPNOTCH MARCH 2016
surgery? p 1128 MD FROM LA SALLE)
A. Madison meneuver
B. Pringle maneuver
C. Henderson’s maneuver
D. Koch maneuver
E. Rovsing maneuver

6 Patient is diagnosed with a superior sulcus tumor This is a case of pancoast syndrome. Invasion of KRISTEL TANHUI (TOP DIAGNOSTIC
(a form of lung adenocarcinoma). Invasion of the stellate sympathetic ganglion results to the 3 - AUG 2015 MED EXAM -
nearby structures results to which clinical classic Horner’s triad – ptosis, miosis, BOARDS; TOPNOTCH MARCH 2016
findings? anhidrosis. MD FROM LA SALLE)
A. Hoarseness
B. Back pain SIMILAR TO PREVIOUS BOARD EXAM
C. Ptosis, miosis, anhidrosis CONCEPT/PRINCIPLE
D. Shoulder pain, hiccups and dyspnea Source: Schwartz’s Principles of Surgery 8th ed
E. Conjunctival edema, swelling of the head and p 534
neck, headache

TOPNOTCH MEDICAL BOARD PREP SURGERY SUPEREXAM Page 1 of 94


For inquiries visit www.topnotchboardprep.com.ph or email us at topnotchmedicalboardprep@gmail.com
TOPNOTCH MEDICAL BOARD PREP SURGERY SUPEREXAM
For inquiries visit www.topnotchboardprep.com.ph or email us at topnotchmedicalboardprep@gmail.com
Item QUESTION EXPLANATION AUTHOR TOPNOTCH
# EXAM
7 A fistula with an external opening located Goodsall’s rule dictates that fistulas with an KRISTEL TANHUI (TOP DIAGNOSTIC
anteriorly 2.75cm from the anal margin will have: external opening anteriorly connect to the 3 - AUG 2015 MED EXAM -
A. Short, radial tract to the anterior midline internal opening by a short, radial tract. Fistulas BOARDS; TOPNOTCH MARCH 2016
B. Curvilinear tract to the posterior midline with an external opening posteriorly track in a MD FROM LA SALLE)
C. Curvilenear tract to the anterior midline curvilinear fashion to the posterior midline.
D. A and C However, if the anterior external opening is
E. All of the above greater than 3cm from the anal margin, such
fistulas tract to the posterior midline.

SIMILAR TO PREVIOUS BOARD EXAM
CONCEPT/PRINCIPLE
Source: Schwartz’s Principles of Surgery 8th ed
p 1064
8 The Poupart’s ligament is derived from which of The poupart’s ligament/inguinal ligament forms KRISTEL TANHUI (TOP DIAGNOSTIC
the following structures? the inferior margin of the inguinal canal. Inside 3 - AUG 2015 MED EXAM -
A. External oblique courses the spermatic cord. BOARDS; TOPNOTCH MARCH 2016
B. Internal oblique MD FROM LA SALLE)
C. Trasversus abdominis Source: Schwartz’s Principles of Surgery 8th ed
D. Transversalis fascia p 1308
E. Peritoneum

9 A 26 year victim of a vehicular accident is brought This is a case of neurogenic shock. The classic KRISTEL TANHUI (TOP DIAGNOSTIC
on a stretcher in the ER. On physical examination description of neurogenic shock consists of 3 - AUG 2015 MED EXAM -
his blood pressure is 70/40 and heart rate is 65. decreased BP associated with bradycardia BOARDS; TOPNOTCH MARCH 2016
The extremities are warm and he is noted to have (absence of reflexive tachycardia due to MD FROM LA SALLE)
decreased sensorium. Which of the following is the disrupted sympathetic discharege), warm
appropriate initial management? extremities (loss of peripheral vasoconstriction),
A. Fluid resuscitation motor and sensory deficits indicative of a spinal
B. Blood transfusion cord injury, and radiographic evidence of a
C. Dopamine/Norepinephrine vertebral column fracture.
D. Dobutamine/Digoxin
E. Epinephrine/Antihistamine/ Corticosteroids After the airway is secured and ventilation is
adequate, fluid resuscitation and restoration of
intravascular volume often will improve
perfusion in neurogenic shock. Most patients
with neurogenic shock will respond to
restoration of intravascular volume alone, with
satisfactory improvement in perfusion and
resolution of hypotension. Administration of
vasoconstrictors will improve peripheral
vascular tone, decrease vascular capacitance,
and increase venous return, but should only be
considered once hypovolemia is excluded as the
cause of the hypotension, and the diagnosis of
neurogenic shock established. If the patient's
blood pressure has not responded to what is felt
to be adequate volume resuscitation, dopamine
may be used first.

**Note: Be careful of doing fluid resuscitation in
patients with decreased cardiac function.
Always ask for history of MI and auscultate for
crackles.

Source: Schwartz’s Principles of Surgery 8th ed
p 108
10 A 43 year old male incurred corneal scarring from Penetrating keratoplasty – full thickness KRISTEL TANHUI (TOP DIAGNOSTIC
eye herpes and lost vision of his left eye. A full Partial thickness 3 - AUG 2015 MED EXAM -
thickness corneal transplant was done. Examples - Deep anterior lamellar keratoplasty BOARDS; TOPNOTCH MARCH 2016
of full thickness keratoplasty include: - Descemet stripping with endothelial MD FROM LA SALLE)
A. Penetrating keratoplasty keratoplasty
B. Descemet stripping with endothelial - Descemet membrane keratoplasty
keratoplasty
C. Descemet membrane keratoplasty SIMILAR TO PREVIOUS BOARD EXAM
D. A and B CONCEPT/PRINCIPLE
E. All of the above Source: Corneal Endothelial Transplant DSEAK,
DMEK and DLEK By John Thomas (MD.), Thomas
John
11 Which of the following structures are removed in a Breast conservation – resection of primary KRISTEL TANHUI (TOP DIAGNOSTIC
skin sparing mastectomy? breast cancer + margin of normal appearing 3 - AUG 2015 MED EXAM -
A. All breast tissue breast tissue + assessment of regional lymph BOARDS; TOPNOTCH MARCH 2016
B. Nipple areola complex node status MD FROM LA SALLE)
C. Scars from previous biopsy procedures
D. A and B Radical mastectomy – Medial/Lateral pectoral
E. All of the above nerves, Long thoracic nerve, Thoracodorsal
nerve + Pectoralis Major and minor + All LN +
All breast

Modified radical mastectomy – Same as Radical
mastectomy except all nerves are spared, level
III LN spared, Pectoralis major spared

Source: Schwartz’s Principles of Surgery 8th ed
p 460-461 + Topnotch Lecture on Surgery

TOPNOTCH MEDICAL BOARD PREP SURGERY SUPEREXAM Page 2 of 94


For inquiries visit www.topnotchboardprep.com.ph or email us at topnotchmedicalboardprep@gmail.com
TOPNOTCH MEDICAL BOARD PREP SURGERY SUPEREXAM
For inquiries visit www.topnotchboardprep.com.ph or email us at topnotchmedicalboardprep@gmail.com
Item QUESTION EXPLANATION AUTHOR TOPNOTCH
# EXAM
12 Which of the following pertains to Osteonecrosis Legg-Calve-Perthes disease also known as cox KRISTEL TANHUI (TOP DIAGNOSTIC
of the proximal femoral epiphysis of the pediatric plana is a condition fo the pediatric hip 3 - AUG 2015 MED EXAM -
hip and is thought to be due to vascular characterized by a flattened misshapen femoral BOARDS; TOPNOTCH MARCH 2016
compromise? head. The etiology is related to osteonecrosis of MD FROM LA SALLE)
A. Slipped capital femoral epiphysis the proximal femoral epiphysis and is thought to
B. Developmental dysplasia of the hip result from vascular compromise.
C. Talipes equinovarus
D. Legg-Calve-Perthes disease SIMILAR TO PREVIOUS BOARD EXAM
E. Osgood-Schlatter disease CONCEPT/PRINCIPLE
Source: Schwartz’s Principles of Surgery 8th ed
p 1605
13 Which of the following orthopedic conditions It may also occur with several other diseases KRISTEL TANHUI (TOP DIAGNOSTIC
describe a form of neuropathic arthropathy that that affect the sensory nervous system 3 - AUG 2015 MED EXAM -
affects some diabetics with peripheral (alcoholism, leprosy, syphilis, Charcot-Marie- BOARDS; TOPNOTCH MARCH 2016
neuropathy? Tooth Disease to name a few). In the United MD FROM LA SALLE)
A. Charcot joints States, diabetes is the number-one cause.
B. Osgood Schlatter Disease
C. Paget Schroetter Disease SIMILAR TO PREVIOUS BOARD EXAM
D. Kimmel-Steil Wilson Lesions CONCEPT/PRINCIPLE
E. None of the above Source:
https://www.aofas.org/footcaremd/conditions/
diabetic-foot/Pages/Charcot-Joints-or-
Neuropathic-Arthropathy.aspx
14 A 57 year old male with poorly controlled Also called amaurosis fugax. Management KRISTEL TANHUI (TOP DIAGNOSTIC
hypertension presents with acut onset painless options include direct infusion of thrombolytic 3 - AUG 2015 MED EXAM -
loss of vision in the left eye. PE reveals a relative to the ophthalmic artery or by decreasing IOP BOARDS; TOPNOTCH MARCH 2016
afferent pupillary defect and fundoscopy of the (IV acetazolamide or vitreous paracentesis) MD FROM LA SALLE)
affected eye showed “cherry red spot” over a pale
background. What is the diagnosis? SIMILAR TO PREVIOUS BOARD EXAM
A. Rhegmatogenous retinal detachment CONCEPT/PRINCIPLE
B. Age related macular degeneration Source:
C. Central serous retinopathy http://www.williamsoneyeinstitute.com/retina-
D. Central retinal artery occlusion center/retinal-artery-occlusions
E. Central retinal vein occlusion

15 Which of the following aortic aneurysms Indications for repair of aortic aneurysms in KRISTEL TANHUI (TOP DIAGNOSTIC
merit/merits operative management? asymptomatic patients without connective 3 - AUG 2015 MED EXAM -
A. Sudden onset lower back pain in a man tissue disorders BOARDS; TOPNOTCH MARCH 2016
diagnosed with aortic aneurym 6 months ago. Last - Ascending aortic aneurysm >5.5cm diameter MD FROM LA SALLE)
CT showed diameter of 4.5cm. - Descending aortic aneurysm >6.5cm diameter
B. Asymptomatic ascending thoracic aortic - Abdominal aorta >5.5cm
aneurysm of 5.75cm - Rate of dilatation >1cm/yr for thoracic aorta/
C. Asymptomatic descending thoracic aortic >0.5cm for abdominal aorta
aneurysm of 5.75cm
D. A and B only Normal aortic diameter: 2-3cm
E. All of the above
Source: Schwartz’s Principles of Surgery 8th ed
p 671, 723

16 Dr. Juan dela Cruz is assisting in a bowel surgery Tensile strength KRISTEL TANHUI (TOP DIAGNOSTIC
and is about to do an anastomosis. Which of the Aorta: Tunica media 3 - AUG 2015 MED EXAM -
following is the strongest layer of the of the bowel GIT: submucosa BOARDS; TOPNOTCH MARCH 2016
wall that he has to pay special attention to? MD FROM LA SALLE)
A. Mucosa Source: Schwartz’s Principles of Surgery 8th ed
B. Submucosa p 667 + Topnotch handout in surgery
C. Muscularis Propria
D. Serosa
E. Adventitia

17 A 42-year-old jaundiced female with pancreatic This is a case of ascending cholangitis secondary KRISTEL TANHUI (TOP DIAGNOSTIC
cancer is admitted at the female ward of the to biliary tract obstruction secondary to a 3 - AUG 2015 MED EXAM -
hospital you are on duty in. She has been febrile pancreatic mass. The patient presented with BOARDS; TOPNOTCH MARCH 2016
for 24 hours now and the nurse refers her to you Raynaud’s pentad. The initial management in MD FROM LA SALLE)
for changes in sensorium. You evaluate her and on such patients is IV antibiotics and fluid
PE, her vitals were HR 98 RR 20 BP 80/50 T38.9. resuscitation. About 15% of patients will not
What is the next step in management? respond to antibiotics and fluid resuscitation
A. IV antibiotics and fluids and an emergency biliary decompression may
B. Laparoscopic cholecystectomy with be required.
sphincterotomy
C. Emergency exploratory laparotomy
D. Whipple’s procedure Source: Schwartz’s Principles of Surgery 8th ed
E. Bowel resection with end to end anastomosis p 1150
18 A 15 year old female has a history of intermittent Choledochal cyst type I – saccular/fusiform KRISTEL TANHUI (TOP DIAGNOSTIC
abdominal pain and jaundice. On PE, dilatation 3 - AUG 2015 MED EXAM -
hepatosplenomegaly was noted. A ultrasound Choledochal cyst type II – isolated diverticulum BOARDS; TOPNOTCH MARCH 2016
revealed intrahepatic biliary cysts. What is the Choledochal cyst type III - choledochocele MD FROM LA SALLE)
diagnosis? Choledochal cyst type IV – dilatations of the
A. Choledochal cyst type I intra and extrahepatic/ extrahepatic biliary tree
B. Choledochal cyst type II only
C. Choledochal cyst type III Choledochal cyst type V – dilatation of the
D. Choledochal cyst type IV intrahepatic biliary tree
E. Choledochal cyst type V
Source: Schwartz’s Principles of Surgery 8th ed
p 1158
TOPNOTCH MEDICAL BOARD PREP SURGERY SUPEREXAM Page 3 of 94
For inquiries visit www.topnotchboardprep.com.ph or email us at topnotchmedicalboardprep@gmail.com
TOPNOTCH MEDICAL BOARD PREP SURGERY SUPEREXAM
For inquiries visit www.topnotchboardprep.com.ph or email us at topnotchmedicalboardprep@gmail.com
Item QUESTION EXPLANATION AUTHOR TOPNOTCH
# EXAM
19 The law of Laplace states that as the diameter of Normal cecum has a luminal diameter of 7.5- KRISTEL TANHUI (TOP DIAGNOSTIC
the lumen of cavity increases, the wall tension also 8.5cm and becomes at risk at 10cm. Sigmoid 3 - AUG 2015 MED EXAM -
increases. Which segment of the GIT has the colon is the narrowest part and most vulnerable BOARDS; TOPNOTCH MARCH 2016
widest luminal diameter and is thus most prone to to obstruction and also most mobile and MD FROM LA SALLE)
rupture and least prone to obstruction? vulnerable to volvulus.
A. Cecum
B. Appendix Source: Schwartz’s Principles of Surgery 8th ed
C. Sigmoid colon p 1015
D. Transverse colon
E. Duodenum

20 A 42-year old man with gunshot wound to the Patient is already unstable hence should be KRISTEL TANHUI (TOP DIAGNOSTIC
RUQ and epigastric area arrives in the ER drowsy, opened for exploratory laparotomy. 3 - AUG 2015 MED EXAM -
pale with a BP = 80/60, HR = 120, RR = 35. BOARDS; TOPNOTCH MARCH 2016
Abdomen is distended, tense and with diffuse Source: Topnotch handout on Surgery MD FROM LA SALLE)
rebound tenderness. What is the most appropriate
procedure?
A. Laparotomy
B. Stat CT scan of the abdomen
C. Stat Diagnostic peritoneal lavage
D. Stat FAST
E. Observation
21 Thyroglossal duct cysts are most commonly Thyroglossal duct cysts are the most commonly LESTER BRYAN CO MIDTERM 1
located encountered congenital cervical anomalies. (TOP 10 - AUG 2015 EXAM -
A. On the anterior border of the During the fifth week of gestation, the MED BOARDS; MARCH 2016
sternocleidomastoid muscle thyroglossal duct lumen starts to obliterate, and TOPNOTCH MD FROM
B. In the midline at the level of the hyoid the duct disappears by the eighth week of UST)
C. Over the medial clavicular head gestation. Rarely, the thyroglossal duct may
D. In the midline just superior to the thyroid gland
persist in whole, or in part.
E. Behind the base of the tongue near the
foramen cecum
22 A patient with hypertension is diagnosed with If an adrenal tumor is not seen or if both LESTER BRYAN CO MIDTERM 1
hyperaldosteronism. A CT scan shows bilaterally adrenals are abnormal on CT scan, selective (TOP 10 - AUG 2015 EXAM -
enlarged adrenals without a mass. The most venous catheterization and adrenal vein MED BOARDS; MARCH 2016
appropriate next intervention is sampling for aldosterone can help determine if TOPNOTCH MD FROM
A. Unilateral adrenalectomy the aldosteronoma is unilateral, because this UST)
B. Bilateral adrenalectomy can benefit from surgery. Bilateral adrenal
C. Selective venous catheterization tumors or adrenal hyperplasia are best managed
D. Medical management medically.
E. Observation

23 A 60-year-old woman develops a right-sided Sessile serrated adenomas can be very subtle LESTER BRYAN CO MIDTERM 1
colonic adenocarcinoma. She has a history of and are easily missed on endoscopy. Under the (TOP 10 - AUG 2015 EXAM -
alleged colonoscopies; however, review of images microscope, they closely resemble hyperplastic MED BOARDS; MARCH 2016
from her most recent colonoscopy reveals a polyps but show more complicated glands, often TOPNOTCH MD FROM
prominent fold in the region that subsequently with “boot-shaped” configurations at the gland UST)
developed cancer. Biopsy from this area would bases. Unlike tubular adenomas, they lack the
have most likely revealed which of the following? overtly dysplastic epithelium and often do not
A. Tubular adenoma form polypoid lesions. Hyperplastic polyps,
B. Hyperplastic polyp Peutz-Jeghers polyps, and inflammatory polyps
C. Peutz-Jeghers polyp are not malignant precursors.
D. Sessile serrated adenoma
E. Inflammatory polyp

24 The most common type of hiatal hernia is SIMILAR TO PREVIOUS BOARD EXAM LESTER BRYAN CO MIDTERM 1
A. Type I CONCEPT/PRINCIPLE. There were 3 questions (TOP 10 - AUG 2015 EXAM -
B. Type II about esophageaWith the advent of clinical MED BOARDS; MARCH 2016
C. Type III radiology, it became evident that a TOPNOTCH MD FROM
D. Type IV
diaphragmatic hernia was a relatively common UST)
E. Type V abnormality and was not always accompanied
by symptoms. Three types of esophageal hiatal
hernia were identified: (a) the sliding hernia,
type I, characterized by an upward dislocation of
the cardia in the posterior mediastinum; (b) the
rolling or paraesophageal hernia (PEH), type II,
characterized by an upward dislocation of the
gastric fundus alongside a normally positioned
cardia; and (c) the combined sliding-rolling or
mixed hernia, type III, characterized by an
upward dislocation of both the cardia and the
gastric fundus. The end stage of type I and type
II hernias occurs when the whole stomach
migrates up into the chest by rotating 180°
around its longitudinal axis, with the cardia and
pylorus as fixed points. In this situation the
abnormality is usually referred to as an
intrathoracic stomach. In some taxonomies, a
type IV hiatal hernia is declared when an
additional organ, usually the colon, herniates as
well.
When radiographic examinations are done in
response to GI symptoms, the incidence of a
sliding hiatal hernia is seven times higher than
that of a PEH. The PEH is also known as the giant
hiatal hernia. Over time the pressure gradient
between the abdomen and chest enlarges the
TOPNOTCH MEDICAL BOARD PREP SURGERY SUPEREXAM Page 4 of 94
For inquiries visit www.topnotchboardprep.com.ph or email us at topnotchmedicalboardprep@gmail.com
TOPNOTCH MEDICAL BOARD PREP SURGERY SUPEREXAM
For inquiries visit www.topnotchboardprep.com.ph or email us at topnotchmedicalboardprep@gmail.com
Item QUESTION EXPLANATION AUTHOR TOPNOTCH
# EXAM
hiatal hernia. In many cases the type 1 sliding
hernia will evolve into a type III mixed hernia.
Type II hernias are quite rare.

25 Which of the following infections have been There are plenty of ENT questions in our LESTER BRYAN CO MIDTERM 1
correlated with nasopharyngeal carcinoma? Surgery Exam. Risk factors for nasopharyngeal (TOP 10 - AUG 2015 EXAM -
A. Herpes simplex virus carcinoma include area of habitation, ethnicity, MED BOARDS; MARCH 2016
B. Epstein Barr virus and tobacco use. There is an increased incidence TOPNOTCH MD FROM
C. Cytomegalovirus of nasopharyngeal cancer in southern China, UST)
D. Human immunodeficiency virus Africa, Alaska, and in Greenland Eskimos. A
E. Human Papilloma Virus strong correlation exists between
nasopharyngeal cancer and the presence of EBV
infection, such that EBV titers may be used as a
means to follow a patient’s response to
treatment.
26 A classic Le Fort Type I fracture involves the: Le Fort I fractures occur transversely across the LESTER BRYAN CO MIDTERM 1
A. Forehead alveolus, above the level of the teeth apices. In a (TOP 10 - AUG 2015 EXAM -
B. Nose pure Le Fort I fracture, the palatal vault is MED BOARDS; MARCH 2016
C. Maxilla mobile while the nasal pyramid and orbital rims TOPNOTCH MD FROM
D. Mandible are stable. The Le Fort II fracture extends UST)
E. Zygomatic through the nasofrontal buttress, medial wall of
the orbit, across the infraorbital rim, and
through the gomaticomaxillary articulation. The
nasal dorsum, palate, and medial part of the
infraorbital rim are mobile. The Le Fort III
fracture is also known as craniofacial
disjunction. The frontozygomaticomaxillary,
frontomaxillary, and frontonasal suture lines are
disrupted. The entire face is mobile from the
cranium.
27 The yearly mammogram of a 65-year-old This patient is at high risk of recurrence by LESTER BRYAN CO MIDTERM 1
postmenopausal woman shows an irregular area virtue of having disease in her lymph nodes. (TOP 10 - AUG 2015 EXAM -
of microcalcification that has grown in size Radiation therapy or surgical excision of the MED BOARDS; MARCH 2016
compared with her mammogram from 2 years ago. breast cancer usually provides local control and TOPNOTCH MD FROM
She missed her mammogram last year. Physical adequate protection against recurrence of UST)
examination is unrevealing without disease in the tumor bed. However,
lymphadenopathy or nodularity in the breasts. micrometastatic disease can only be addressed
You refer her to a surgeon, and a 2-cm invasive through the administration of systemic therapy
ductal carcinoma is removed from her left breast. such as cytotoxic chemotherapy. In patients with
Sentinel node biopsy shows two positive lymph hormonally sensitive tumors (e.g., estrogen-
nodes, and axillary dissection reveals five receptor positive), hormonal therapy has been
additional positive nodes. The tumor expresses shown to be very effective in the prevention of
the estrogen receptor (ER ). Which of the following disease recurrence after chemotherapy.
interventions would increase her chance of cure? Randomized clinical trials have shown that
A. Chemotherapy followed by hormonal therapy lumpectomy and radiation therapy is equal to a
B. Radiation therapy total mastectomy in terms of overall survival.
C. Total mastectomy For tumors that can be completely removed
D. Hormonal therapy alone with adequate margins (10 mm), lumpectomy
E. High-dose chemotherapy with stem cell support offers better cosmetic and psychological results,
is less invasive, and allows for faster recovery
time.
28 The Norwood procedure is used in the treatment Hypoplastic left heart syndrome - Norwood + LESTER BRYAN CO MIDTERM 1
of Glenn + modified Fontan; TGA - palliative: (TOP 10 - AUG 2015 EXAM -
A. Coarctation of the aorta Rastelli, arterial switch: Jenning, atrial MED BOARDS; MARCH 2016
B. TGA switch:Senning/Mustard; TOF - palliative: TOPNOTCH MD FROM
C. Hypoplastic left heart syndrome Blalock-Taussig shunt UST)
D. Truncus Arterious
E. TAPVC

TOPNOTCH MEDICAL BOARD PREP SURGERY SUPEREXAM Page 5 of 94


For inquiries visit www.topnotchboardprep.com.ph or email us at topnotchmedicalboardprep@gmail.com
TOPNOTCH MEDICAL BOARD PREP SURGERY SUPEREXAM
For inquiries visit www.topnotchboardprep.com.ph or email us at topnotchmedicalboardprep@gmail.com
Item QUESTION EXPLANATION AUTHOR TOPNOTCH
# EXAM
29 An aortic dissection that extends from the left Dissections are categorized according to their LESTER BRYAN CO MIDTERM 1
subclavian artery to the aortic bifurcation is a anatomic location and extent to guide treatment. (TOP 10 - AUG 2015 EXAM -
A. DeBakey Type I dissection The two traditional classification schemes that MED BOARDS; MARCH 2016
B. DeBakey Type II dissection remain in common use are the DeBakey and the TOPNOTCH MD FROM
C. DeBakey Type IIIa dissection Stanford classification systems. In their current UST)
D. DeBakey Type IIIb dissection forms, both of these schemes describe the
E. DeBakey Type IV dissection segments of aorta that are involved in the
dissection, rather than the site of the initial
intimal tear. The main drawback of the Stanford
classification system is that it does not
distinguish between patients with isolated
ascending aortic dissection and patients with
dissection involving the entire aorta. DeBakey
type I: involves ascending and descending aorta
(= Stanford A)
DeBakey type II: involves ascending aorta only
(= Stanford A)
DeBakey type III: involves descending aorta
only, commencing after the origin of the left
subclavian artery (= Stanford B). Type III
dissections are further divided into IIIa and IIIb.
DeBakey Type IIIa refers to dissections that
originate distal to the left subclavian artery but
extend proximally and distally, mostly above the
diaphragm.
DeBakey Type IIIb refers to dissections that
originate distal to the left subclavian artery,
extend only distally, and may extend below the
diaphragm
30 The treatment of stage II pressure sores is Stage I and II ulcers are treated conservatively LESTER BRYAN CO MIDTERM 1
A. Local wound care only with dressing changes and basic pressure ulcer (TOP 10 - AUG 2015 EXAM -
B. Extensive débridement and local wound care prevention strategies as already discussed. MED BOARDS; MARCH 2016
C. Direct closure Patients with stage III or IV ulcers should be TOPNOTCH MD FROM
D. Skin grafting evaluated for surgery. UST)
E. Observation

31 The most common location for a congenital SIMILAR TO PREVIOUS BOARD EXAM LESTER BRYAN CO MIDTERM 1
diaphragmatic hernia is CONCEPT/PRINCIPLE. The most common (TOP 10 - AUG 2015 EXAM -
A. Left posterolateral variant of a congenital diaphragmatic hernia MED BOARDS; MARCH 2016
B. Right posterolateral (CDH) is a posterolateral defect, also known as a TOPNOTCH MD FROM
C. Left anteromedial Bochdalek’s hernia. This anomaly is UST)
D. Right anteromedial encountered more commonly on the left (80 to
E. Right anterolateral 90% of cases).

32 A 60-year-old man with newly diagnosed prostate The correct answer is to do a bone scan. This is LESTER BRYAN CO MIDTERM 1
cancer comes to see you. He underwent the first staging test done in the workup of (TOP 10 - AUG 2015 EXAM -
transrectal ultrasonography with a needle biopsy prostate cancer. If abnormalities are found on MED BOARDS; MARCH 2016
showing adenocarcinoma. Except for elevated PSA, the bone scan, you then proceed to do plain film TOPNOTCH MD FROM
he is currently asymptomatic. The fi rst test to x-rays of the areas to exclude other possible UST)
order in staging for his prostate cancer is: causes. Then you would perform surgical
A. Full-body plain film x-rays staging with removal and examination of the
B. Bone scan surrounding nodes (often done with
C. CT of head prostatectomy).
D. CEA level
E. Repeat PSA now
33 Patients with symptoms from a Chiari I SIMILAR TO PREVIOUS BOARD EXAM LESTER BRYAN CO MIDTERM 1
malformation may complain of: CONCEPT/PRINCIPLE. Chiari I malformation is (TOP 10 - AUG 2015 EXAM -
A. Seizures the caudal displacement of the cerebellar tonsils MED BOARDS; MARCH 2016
B. Extremity weakness below the foramen magnum. It may be seen as TOPNOTCH MD FROM
C. Eye pain an incidental finding on MRI scans in UST)
D. Ataxia asymptomatic patients. Symptomatic patients
E. Dysautonomia usually present with headache, neck pain, or
symptoms of myelopathy, including numbness
or weakness in the extremities. A syrinx may be
associated, but the brain stem and lower cranial
nerves are normal in Chiari I malformations.
Chiari II malformations are more severe and
involve caudal displacement of the lower brain
stem and stretching of the lower cranial nerves.
Symptomatic patients may be treated with
suboccipital craniectomy to remove the
posterior arch of the foramen magnum, along
with removal of the posterior ring of C1.
Removal of these bony structures relieves the
compression of the cerebellar tonsils and
cervicomedullary junction, and may allow
reestablishment of normal CSF flow patterns.

TOPNOTCH MEDICAL BOARD PREP SURGERY SUPEREXAM Page 6 of 94


For inquiries visit www.topnotchboardprep.com.ph or email us at topnotchmedicalboardprep@gmail.com
TOPNOTCH MEDICAL BOARD PREP SURGERY SUPEREXAM
For inquiries visit www.topnotchboardprep.com.ph or email us at topnotchmedicalboardprep@gmail.com
Item QUESTION EXPLANATION AUTHOR TOPNOTCH
# EXAM
34 A clerk rotating on a radiology elective was The patient has a mechanical intestinal LESTER BRYAN CO MIDTERM 1
presented an x-ray of the abdomen, which showed obstruction, as the description of the air–fluid (TOP 10 - AUG 2015 EXAM -
multiple air–fluid levels with dilated loops of small levels indicates a mechanical intestinal MED BOARDS; MARCH 2016
bowel, paucity of air in the colon, and no air in the obstruction. Mechanical intestinal obstruction TOPNOTCH MD FROM
rectum. The radiology consultant asked the clerk may be the result of extrinsic, intramural, or UST)
which of the following clinical features would intraluminal causes. Symptoms include crampy
most likely be found in this patient: abdominal pain that waxes and wanes,
A. Hypoactive bowel sounds obstipation or constipation, nausea and
B. Pain out of proportion to physical examination vomiting, and abdominal distention. Physical
C. Crampy abdominal pain that waxes and wanes examination of the abdomen reveals high-
D. Diarrhea pitched bowel sounds and rushes and tinkles, as
E. A flat, rigid abdomen well as marked abdominal distention and
tympany on percussion. Pain out of proportion
to the physical examination is most suggestive of
acute mesenteric ischemia.
35 A 51-year-old man with recurrent peptic ulcer Zollinger–Ellison syndrome is caused by a non– LESTER BRYAN CO MIDTERM 1
disease had a fasting gastrin level of 1000. Which islet cell tumor that produces gastrin and is (TOP 10 - AUG 2015 EXAM -
of the following organs is the most common site of associated with gastric acid hypersecretion and MED BOARDS; MARCH 2016
origin of the tumor associated with this peptic ulcer disease. Tumors are biologically TOPNOTCH MD FROM
syndrome? malignant in 60% of cases, and the most UST)
A. Stomach common site involved is the pancreas. Most
B. Duodenum gastrinomas are found in the gastrinoma
C. Lymph nodes triangle (Passaro's triangle): this is bound by the
D. Spleen junction of cystic and common bile ducts,
E. Pancreas junction of the second and third parts of the
duodenum, and the junction of the neck and
body of the pancreas.
36 Paraphimosis refers to: Paraphimosis is a common problem that LESTER BRYAN CO MIDTERM 1
A. Inability to retract the foreskin represents a true medical emergency. When (TOP 10 - AUG 2015 EXAM -
B. Inability to reduce the foreskin after it has been foreskin is retracted for prolonged periods, MED BOARDS; MARCH 2016
retracted constriction of the glans penis may ensue. This is TOPNOTCH MD FROM
C. Infection of the foreskin near the urethral particularly likely in hospitalized patients who UST)
meatus are confined to bed or who have altered
D. Excessive length of foreskin
mentation. Edema often forms in the genitals of
E. Incomplete embryologic formation of the supine patients due to the dependent position of
foreskin that area. Patients with diminished
consciousness will not be aware of the penile
pain from paraphimosis, which may delay
recognition of the problem until too late. Delay
can be catastrophic as penile necrosis may occur
due to ischemia.
37 Which of the following is NOT part of Cantrell’s Omphalocele has an incidence of approximately LESTER BRYAN CO MIDTERM 1
pentalogy? 1 in 5000 live births and occurs in association (TOP 10 - AUG 2015 EXAM -
A. Omphalocele with special syndromes such as exstrophy of the MED BOARDS; MARCH 2016
B. Ectopia cordis cloaca (vesicointestinal fissure), the Beckwith- TOPNOTCH MD FROM
C. Posterolateral diaphragmatic hernia Wiedemann constellation of anomalies UST)
D. Cardiac anomalies
(macroglossia, macrosomia, hypoglycemia,
E. cleft sternum visceromegaly, and omphalocele) and the
Cantrell pentalogy (lower thoracic wall
malformations such as cleft sternum, ectopia
cordis, epigastric omphalocele, anterior midline
diaphragmatic hernia, and cardiac anomalies)
38 A 30-year-old man is evaluated for a thyroid For the patient described, the markedly LESTER BRYAN CO MIDTERM 1
nodule. T he patient reports that his father died increased calcitonin level indicates the diagnosis (TOP 10 - AUG 2015 EXAM -
from thyroid cancer and that a brother had a of medullary carcinoma of the thyroid. In view of MED BOARDS; MARCH 2016
history of recurrent renal stones. Blood calcitonin the family history, the patient most likely has TOPNOTCH MD FROM
concentration is 2000 pg/mL (normal is < 100); multiple endocrine neoplasia (MEN) type 2A, UST)
serum calcium and phosphate levels are normal. T which includes medullary carcinoma of the
he patient is referred to a thyroid surgeon. Which thyroid gland, pheochromocytoma, and
of the following studies should also be obtained? parathyroid hyperplasia. Pheochromocytoma
A. Obtain a liver scan. may exist without sustained hypertension, as
B. Measure parathormone level. indicated by excessive urinary catecholamines.
C. Measure urinary catecholamines. Before thyroid surgery is performed on this
D. Administer suppressive doses of thyroxine and patient, a pheochromocytoma must be ruled out
measure levels of thyroid-stimulating hormone. through urinary catecholamine determinations;
E. Treat the patient with radioactive iodine. the presence of such a tumor might expose him
to a hypertensive crisis during surgery. T he
serum calcium serves as a screening test for
hyperparathyroidism. At surgery, the entire
thyroid gland must be removed because foci of
parafollicular cell hyperplasia, a premalignant
lesion, may be scattered throughout the gland.
Successful removal of the medullary carcinoma
can be monitored with serum calcitonin levels.
Medullary carcinoma of the thyroid rarely
metastases to the liver, so a liver scan would be
unnecessary if liver enzymes are normal.
Thyroxine will be needed after surgery, but MEN
type 2 is not associated with hypothyroidism.
Radioactive iodine can be used to treat
malignancies that arise from the follicular cells
of the thyroid; parafollicular cells, however, do
not take up iodine and do not respond to
radioactive iodine. Hyperparathyroidism, while
unlikely in this eucalcemic patient, is probably
present in his brother.

TOPNOTCH MEDICAL BOARD PREP SURGERY SUPEREXAM Page 7 of 94


For inquiries visit www.topnotchboardprep.com.ph or email us at topnotchmedicalboardprep@gmail.com
TOPNOTCH MEDICAL BOARD PREP SURGERY SUPEREXAM
For inquiries visit www.topnotchboardprep.com.ph or email us at topnotchmedicalboardprep@gmail.com
Item QUESTION EXPLANATION AUTHOR TOPNOTCH
# EXAM
39 A 61-year-old man with a history of Barrett’s esophagus with high-grade dysplasia LESTER BRYAN CO MIDTERM 1
gastroesophageal reflux disease and Barrett’s requires defi nitive treatment, and referral for (TOP 10 - AUG 2015 EXAM -
esophagus is undergoing a screening endoscopy. esophagectomy is recommended. For patients MED BOARDS; MARCH 2016
The biopsy results show high-grade dysplasia. who are medically unstable or for those who TOPNOTCH MD FROM
What is the recommended treatment approach? refuse surgery, endoscopic resection or UST)
A Refer the patient to a surgeon for photodynamic therapy are viable alternatives.
esophagectomy. Surveillance alone is inappropriate, making B
B Repeat surveillance endoscopy in 6 months. and C incorrect. (Surveillance endoscopy at 6
C Repeat surveillance endoscopy in 1 year. months and 1 year are recommended for
D No further endoscopy is indicated unless the patients with Barrett’s esophagus with low-
patient becomes symptomatic. grade dysplasia and no dysplasia, respectively.)
E Initiate chemotherapy. Chemotherapy is not indicated unless carcinoma
in found.
40 A 6-year-old presents with a tibial fracture of the Classification of growth plate injuries has LESTER BRYAN CO MIDTERM 1
metaphysis extending across the growth plate. important implications as doctors communicate (TOP 10 - AUG 2015 EXAM -
This would be a about the treatment of a patient. The exact type MED BOARDS; MARCH 2016
A. Salter-Harris type 1 fracture of physeal injury is important for the prognosis TOPNOTCH MD FROM
B. Salter-Harris type 2 fracture and treatment of the fracture. Salter and Harris UST)
C. Salter-Harris type 3 fracture described a very useful classification of growth
D. Salter-Harris type 4 fracture plate injuries. A type I injury is a simple
E. Salter-Harris type 5 fracture transverse failure of the physis without
involvement of the ossified epiphysis or
metaphysis. A Salter-Harris type II fracture
contains a component of fracture through the
growth plate in continuity with a fracture of the
metaphysis. Salter-Harris type III fracture
occurs partially through the epiphysis and
partially through the growth plate. These
fractures are essentially always intra-articular. A
Salter-Harris type IV injury is one which has a
fracture line extending through the physis
extending from the metaphysis through into the
epiphysis. Finally, a Salter-Harris type V injury is
a subtle injury where the physis itself is injured
but not displaced.
41 Absolute contraindications to Breast Conservative Absolute contraindications to BCS include A. GEORGE MICHAEL MIDTERM 2
Surgery Prior RT SOSUAN (TOP 5 - AUG EXAM -
A. Prior RT B. Preganancy 2015 MED BOARDS; MARCH 2016
B. Preganancy C. Positive margins TOPNOTCH MD FROM
C. Positive margins D. Multicentric UST)
D. Multicentric E. Diffuse microcalcifications
E. AOTA

42 To confirm a diagnosis of achalasia, the following To confirm a diagnosis of achalasia, manometry GEORGE MICHAEL MIDTERM 2
should be requested should be requested. SOSUAN (TOP 5 - AUG EXAM -
A. EGD 2015 MED BOARDS; MARCH 2016
B. Manometry TOPNOTCH MD FROM
C. CT scan UST)
D. Barium swallow
E. 24 hours pH monitoring

43 What is the basal caloric requirement basal requirement: 25; mild stress: 25-30; GEORGE MICHAEL MIDTERM 2
(kcal/Kg/day) of a normal healthy individual? moderate stress: 30; severe stress 35-40 SOSUAN (TOP 5 - AUG EXAM -
A. 20 2015 MED BOARDS; MARCH 2016
B. 25 TOPNOTCH MD FROM
C. 30 UST)
D. 35
E. 40

44 Single most important test in the evaluation of FNAB - single most important test in evaluation GEORGE MICHAEL MIDTERM 2
thyroid nodules: of thyroid nodules; core needle biopsy - for SOSUAN (TOP 5 - AUG EXAM -
A. Core needle biopsy breast nodules 2015 MED BOARDS; MARCH 2016
B. Fine needle aspiration biopsy TOPNOTCH MD FROM
C. Thyroid ultrasound UST)
D. Excision biopsy
E. Incision biopsy

45 A 65 y/o M, smoker, obese presented at the ER Abdominal pain with severity out of proportion GEORGE MICHAEL MIDTERM 2
with sudden, severe abdominal pain. The PE of the to PE findings is the hallmark of acute SOSUAN (TOP 5 - AUG EXAM -
abdomen was normal. What is the most probable mesenteric ischemia. 2015 MED BOARDS; MARCH 2016
diagnosis? TOPNOTCH MD FROM
A. Acute mesenteric ischemia UST)
B. Acute perforated PUD
C. Acute appendicitis
D. Acute diverticulitis
E. NOTA
46 Most common presentation of Meckel's Intestinal obstruction is the most common GEORGE MICHAEL MIDTERM 2
Diverticulum in adults presentation of Meckel's diverticulum in adults. SOSUAN (TOP 5 - AUG EXAM -
A. Bleeding 2015 MED BOARDS; MARCH 2016
B. Perforation TOPNOTCH MD FROM
C. Intractability UST)
D. Obstruction
E. NOTA

TOPNOTCH MEDICAL BOARD PREP SURGERY SUPEREXAM Page 8 of 94


For inquiries visit www.topnotchboardprep.com.ph or email us at topnotchmedicalboardprep@gmail.com
TOPNOTCH MEDICAL BOARD PREP SURGERY SUPEREXAM
For inquiries visit www.topnotchboardprep.com.ph or email us at topnotchmedicalboardprep@gmail.com
Item QUESTION EXPLANATION AUTHOR TOPNOTCH
# EXAM
47 The most common abnormality of hemostatis in GEORGE MICHAEL MIDTERM 2
the surgical patients SOSUAN (TOP 5 - AUG EXAM -
A. Anemia 2015 MED BOARDS; MARCH 2016
B. Hypovolemia TOPNOTCH MD FROM
C. Thrombocytopenia UST)
D. Leukopenia
E. DIC

48 What is the risk of perforation of acute The risk of perforation of acute appendicits is as GEORGE MICHAEL MIDTERM 2
appendicitis? follows: 25% by 24 hours rom onset of SOSUAN (TOP 5 - AUG EXAM -
A. 25% by 24 hours from onset of symptoms symptoms, 50% by 36 hours, and 75% 2015 MED BOARDS; MARCH 2016
B. 50% by 24 hours from onset of symptoms by 48 hours TOPNOTCH MD FROM
C. 75% by 24 hours from onset of symptoms UST)
D. 100% by 24 hours from onset of symptoms
E. NOTA

49 What is the most common appendicial tumor? Carcinoid tumors are the most common GEORGE MICHAEL MIDTERM 2
A. Carcinoid tumor appendicial tumor. <5% of which are malignant. SOSUAN (TOP 5 - AUG EXAM -
B. Mucinous tumor 2015 MED BOARDS; MARCH 2016
C. Serous tumor TOPNOTCH MD FROM
D. Brenner tumor UST)
E. Dysplastic tumor

50 A 67-year-old otherwise healthy male presents A. TAA > 6 cm GEORGE MICHAEL MIDTERM 2
with a TAA. What are the indications for repair? B. TAA in a symptomatic patient (e.g., aortic SOSUAN (TOP 5 - AUG EXAM -
A. TAA > 8 cm insufficiency) 2015 MED BOARDS; MARCH 2016
B. TAA in asymptomatic patient C. TAA growth rate > 1 cm/year TOPNOTCH MD FROM
C. TAA growth rate > 0.5 cm/year D. TAA > 5 cm in the setting of a connective UST)
D. TAA > 5 cm in the setting of a connective tissue disorder
tissue disorder
E. AOTA

51 What are the six P’s of acute limb ischemia? Pain, pallor, paralysis, paresthesias, GEORGE MICHAEL MIDTERM 2
A. Pain, purple, paralysis, paresthesias, pulselessness, poikilothermia SOSUAN (TOP 5 - AUG EXAM -
pulselessness, poikilothermia 2015 MED BOARDS; MARCH 2016
B. Pain, purple, pathologic, paresthesias, TOPNOTCH MD FROM
pulselessness, poikilothermia UST)
C. Pain, pallor, paralysis, paresthesias,
pulselessness, poikilothermia
D. Pain, pallor, pathologic, paresthesias,
pulselessness, poikilothermia
E. Pain, pallor, paralysis, paretic, pulselessness,
poikilothermia
52 Charaterized by chronic eczematoid eruption of Paget's disease is a chronic, eczematous GEORGE MICHAEL MIDTERM 2
the nippe eruption of the nipple. Large pale vacuolated SOSUAN (TOP 5 - AUG EXAM -
A. Tubular carcinoma cells in the rete pegs of epithelium (Paget cells) 2015 MED BOARDS; MARCH 2016
B. Mucinous carcinoma are pathognomonic. TOPNOTCH MD FROM
C. Paget's disease UST)
D. Inflammatory carcinoma
E. Medullary carcinoma

53 A 10-year-old presents with a painless swelling The second branchial cleft anomaly is the most GEORGE MICHAEL MIDTERM 2
along her lateral neck. Ultrasound reveals a fluid- common and usually presents on the first SOSUAN (TOP 5 - AUG EXAM -
filled cyst. What is the most common origin of this decade of life. Treatment is by complete excision 2015 MED BOARDS; MARCH 2016
anomaly? TOPNOTCH MD FROM
A. Thyroglossal duct cyst UST)
B. First branchial cleft cyst
C. Second branchial cleft cyst
D. Third branchial cleft cyst
E. Fourth branchial cleft cyst
54 A 25-year-old female with protein C deficiency The syndrome described is Budd-Chiari GEORGE MICHAEL MIDTERM 2
presents with the acute onset RUQ pain and Syndrome. First line therapy is systemic SOSUAN (TOP 5 - AUG EXAM -
jaundice. Her evaluation reveals occlusion of her anticoagulation 2015 MED BOARDS; MARCH 2016
hepatic veins at the level of her IVC. What is the TOPNOTCH MD FROM
immediate first treatment? UST)
A. IVC filter
B. Systemic anticoagulation
C. Systemic fibrinolysis
D. Thrombolectomy
E. AOTA
55 What is the most common location of an Insulinoma: Evenly distributed throughout the GEORGE MICHAEL MIDTERM 2
insulinoma? pancreas SOSUAN (TOP 5 - AUG EXAM -
A. Pancreatic head Gastrinoma: Gastrinoma triangle 2015 MED BOARDS; MARCH 2016
B. Pancreatic tail Glucagonoma: Pancreatic tail TOPNOTCH MD FROM
C. Evenly distributed throughout the pancreas UST)
D. Gastrinoma triangle
E. Pancreatic body

56 What is the most common cause of fever in the Atelectasis is the most common cause of in the GEORGE MICHAEL MIDTERM 2
first 48 hours post-operatively? first 48 hours post-operatively. SOSUAN (TOP 5 - AUG EXAM -
A. Atelectasis 2015 MED BOARDS; MARCH 2016
B. Pneumonia TOPNOTCH MD FROM
C. UTI UST)
D. Wound infection
E. Seroma

TOPNOTCH MEDICAL BOARD PREP SURGERY SUPEREXAM Page 9 of 94


For inquiries visit www.topnotchboardprep.com.ph or email us at topnotchmedicalboardprep@gmail.com
TOPNOTCH MEDICAL BOARD PREP SURGERY SUPEREXAM
For inquiries visit www.topnotchboardprep.com.ph or email us at topnotchmedicalboardprep@gmail.com
Item QUESTION EXPLANATION AUTHOR TOPNOTCH
# EXAM
57 What sedative medication has caloric value? Propofol delivers 1 kcal/cc in the form of lipid. GEORGE MICHAEL MIDTERM 2
A. propofol SOSUAN (TOP 5 - AUG EXAM -
B. midazolam 2015 MED BOARDS; MARCH 2016
C. desflurane TOPNOTCH MD FROM
D. fentanyl UST)
E. NOTA

58 Most common origin of masses in the pre-styloid 45% of the pre-styloid tumors/masses are pre- GEORGE MICHAEL MIDTERM 2
compartment of the parapharyngeal space styloid in origin. SOSUAN (TOP 5 - AUG EXAM -
A. Salivary 2015 MED BOARDS; MARCH 2016
B. Paraganglioma TOPNOTCH MD FROM
C. Neurogenic UST)
D. Lymph nodes
E. Vascular

59 Most common site of perforation of the colon The cecum is the widest portion but has the GEORGE MICHAEL MIDTERM 2
A. Cecum thinnest wall; thus, it is the most common site of SOSUAN (TOP 5 - AUG EXAM -
B. Ascending colon perforation. The sigmoid is the narrowest 2015 MED BOARDS; MARCH 2016
C. Transverse colon portion; thus, it is the most common site of TOPNOTCH MD FROM
D. Descending colon obstruction. UST)
E. Sigmoid colon

60 What is the most common indication for GEORGE MICHAEL MIDTERM 2


intubation? SOSUAN (TOP 5 - AUG EXAM -
A. Facial trauma 2015 MED BOARDS; MARCH 2016
B. Hypotension TOPNOTCH MD FROM
C. Flail chest UST)
D. Pulmonary contusion
E. Altered mental status

61 Which of the ff is not a component of the SIRS Source of the SIRS criteria wherein RR>20 is in JAN CHRISTIAN MIDTERM 3
criteria? Schwartz. If the question was asked in the IM FELICIANO (TOP 2 - EXAM -
A. Temp of =/<36 C boards, then the SIRS criteria is RR>24 accdg to AUG 2015 MED MARCH 2016
B. HR of =/>90 bpm Harrisons. I think it depends on the subject and BOARDS; TOPNOTCH
C. RR =/> 24 the source book to be used . RR should be equal MD FROM UST)
D.WBC =/< 4,000/ul or more than 20 not 24. All the other parameters
E. Bandemia are correct.

62 This interluekin promotes eosinophil proliferation IL-5 is secreted by mast cells and basophils in JAN CHRISTIAN MIDTERM 3
and airway inflammation? order to promote eosinophil proliferation and FELICIANO (TOP 2 - EXAM -
A. IL-3 airway infllammation AUG 2015 MED MARCH 2016
B. IL-5 BOARDS; TOPNOTCH
C. IL-7 MD FROM UST)
D. IL-9
E. IL-11

63 This statement is true of Von Willberand's it is both a qualitative and quantitiave defect JAN CHRISTIAN MIDTERM 3
disease? char by mucocutaneous bleeding. Menorrhagia FELICIANO (TOP 2 - EXAM -
A. It is the most common congenital bleeding is common in women. It is assoc with F8 AUG 2015 MED MARCH 2016
disorder deficinecy not F9. BOARDS; TOPNOTCH
B. It is a pure quantitiative defect of the vWF MD FROM UST)
C. Patients with this disorder have characteristic
visceral and joint hemorrhages
D. Menorrhagia is also common in men
E. Patient also have prolonged PTT due to
deficiency of F9
64 The ff statement regarding transfusion therapy is FFP is the usual source of the vitamin K JAN CHRISTIAN MIDTERM 3
correct except? dependent factors and is the only source of F5 FELICIANO (TOP 2 - EXAM -
A. Rh negative blood should ideally be transfused not F8. All the other statements are correct. AUG 2015 MED MARCH 2016
with Rh negstive blood only. BOARDS; TOPNOTCH
B. In emergency situations, type O negative blood MD FROM UST)
may be transfused to all recipients.
C. Cross matching shoild always be performed
before the administration of dextran because it
interferes with the typiong procedure
D. Platelet preparations are capable of
transmitting infectious infectious diseases and
accounts for allergic disease as well.
E. FFP is the usual source of the vitamin K
dependent factors and is the only source of F8
65 What electrolyte abnormality occurs after a In refeeding syndrome, you expect decreased JAN CHRISTIAN MIDTERM 3
refeeding syndrome? serum potassium, magnesium and phosphate FELICIANO (TOP 2 - EXAM -
A. Decreased serum sodium especially when refeeding via TPN or NGT. AUG 2015 MED MARCH 2016
B. Increased serum potassium BOARDS; TOPNOTCH
C. Decreased serum magnesium MD FROM UST)
D. Increased serum phosphate
E. Increased serum calcium

TOPNOTCH MEDICAL BOARD PREP SURGERY SUPEREXAM Page 10 of 94


For inquiries visit www.topnotchboardprep.com.ph or email us at topnotchmedicalboardprep@gmail.com
TOPNOTCH MEDICAL BOARD PREP SURGERY SUPEREXAM
For inquiries visit www.topnotchboardprep.com.ph or email us at topnotchmedicalboardprep@gmail.com
Item QUESTION EXPLANATION AUTHOR TOPNOTCH
# EXAM
66 How much percentage of blood loss is expected in Pateint has Class II hemorrhage with JAN CHRISTIAN MIDTERM 3
a patient with normal blood pressure but with approximately 15-30% blood loss or 750-1500 FELICIANO (TOP 2 - EXAM -
orthostatic hypotension? cc. Although cghoice C could be correct it is AUG 2015 MED MARCH 2016
A. 10% borderline Class II and III therefore Choice B is BOARDS; TOPNOTCH
B. 20% the best answer. MD FROM UST)
C. 30%
D. 40%
E. 50%

67 All of the following rpactice guidelines are part of The goal MAP in the Surviving Spesis guidelines JAN CHRISTIAN MIDTERM 3
the Surviving Sepsis Campaign Bundle EXCEPT ? is >65 mmHg not 60mmHg FELICIANO (TOP 2 - EXAM -
A. Measure lactate levels AUG 2015 MED MARCH 2016
B. Obtain blood cultures prior to the admistration BOARDS; TOPNOTCH
of antibiotics MD FROM UST)
C. Administer 30cc/kg crystalloid for hypotension
D. Maintain a mean arterial pressure of >60
mmHg
E. Remeasure lactate levels if initial lactate was
elevated
68 What antiobitic is recommended as prophylactic Accdg to Schwartz's 10 ed pg 146 Table 6-5: No JAN CHRISTIAN MIDTERM 3
use for a laparoscopic low risk biliary tract antibiotic is needed for a low risk laparasocopic FELICIANO (TOP 2 - EXAM -
surgery biliary tract surgery. AUG 2015 MED MARCH 2016
A. Cefazolin BOARDS; TOPNOTCH
B. Cefazolin + Metronidazole MD FROM UST)
C. Cefoxitin
D. Ampicilin-sulbactam
E. None of the above

69 An extensive introduction of bacteria into a Contaminated or Class 3 wounds includes open JAN CHRISTIAN MIDTERM 3
normally sterile are of the body due to major accidental wound encountered early after FELICIANO (TOP 2 - EXAM -
breaks in the sterile technique is identified as injury, those with extensive extensive AUG 2015 MED MARCH 2016
what wound classification? introduction of bacteria into a normally sterile BOARDS; TOPNOTCH
A. Clean are of the body due to major breaks in the sterile MD FROM UST)
B. Clean/Contaminated technique penetrating abdominal trauma, large
C. Contaminated tissue injury and enterotomy during bowel
D. Dirty obstruction
E. Somewhat dirty
70 In relation to the previous question, what is the Accdg to Schwartz's 10 ed pg 148 Table 6-7; JAN CHRISTIAN MIDTERM 3
expected infection rate? Choice A- Class 1, Choice B- Class 2 and so forth. FELICIANO (TOP 2 - EXAM -
A. 1.0-2.0% AUG 2015 MED MARCH 2016
B. 2.1-9.5% BOARDS; TOPNOTCH
C. 3.4-13.2% MD FROM UST)
D. 3.1-12.8%
E. None of the above

71 All of the ff are criteria for referral to a burn center Accdg to Schwartz's 10 ed pg 228 Table 8-1; A JAN CHRISTIAN MIDTERM 3
EXCEPT? partial thickness burns of greater than 10% FELICIANO (TOP 2 - EXAM -
A. Partial thickness burns greater than 20% TBSA TBSa should already be referred to a burn AUG 2015 MED MARCH 2016
B. Burns involving the genitalia center. Burns involving the face, hands, feet, BOARDS; TOPNOTCH
C. Electrical burns genitalia, perineum or major joints are also MD FROM UST)
D. Chemical burns referred to a burn center.
E. Burns involving the hands

72 This is true of Basal Cell Carcinoma? BCC arises from the basal layer of non JAN CHRISTIAN MIDTERM 3
A. Arises from the basal layer of keratinocytes keratinocytes. IT accounts for 75% of all skin FELICIANO (TOP 2 - EXAM -
B. It accounts for half of all diagnosed skin cancers cancers. The natural history is local invasion not AUG 2015 MED MARCH 2016
C. The natural history is BCC is distant metastasis distant mets. The msot common form is the BOARDS; TOPNOTCH
D. The most common form of BCC is the nodular form. MD FROM UST)
superficial form accounting for 60% of all forms
E. Bleeding, ulceration and itching are often part of
the clinical presentation.

73 It is the treatment of choice for morpheaform, Moh's microsurgery provides histologic JAN CHRISTIAN MIDTERM 3
poorly delinated, recurrent and infiltrative BCC, confirmation of excision and maximal FELICIANO (TOP 2 - EXAM -
particularly facial lesions. conservation of tissue, which is important to AUG 2015 MED MARCH 2016
A. Excisional surgery keep in mind in cosmetically senstivie areas BOARDS; TOPNOTCH
B. Incisional surgery such as the face. MD FROM UST)
C. Wide local excision
D. Cautererization
E. Moh's microsurgery

74 Among the ff breast conditions, which has the Florid hyperplasia- 1.5-2 fold JAN CHRISTIAN MIDTERM 3
highest relative risk in terms of breast CA? Sclerosing adenosis- No risk
FELICIANO (TOP 2 - EXAM -
A. Florid hyperplasia Intraductal papilloma- No risk AUG 2015 MED MARCH 2016
B. Sclerosing adenosis Atypical ductal hyperplasia- 4-fold BOARDS; TOPNOTCH
C. Intraductal papilloma Lobular carcinoma in situ- 10-fold MD FROM UST)
D. Atypical ductal hyperplasia
E. Lobular carcinoma in situ

TOPNOTCH MEDICAL BOARD PREP SURGERY SUPEREXAM Page 11 of 94


For inquiries visit www.topnotchboardprep.com.ph or email us at topnotchmedicalboardprep@gmail.com
TOPNOTCH MEDICAL BOARD PREP SURGERY SUPEREXAM
For inquiries visit www.topnotchboardprep.com.ph or email us at topnotchmedicalboardprep@gmail.com
Item QUESTION EXPLANATION AUTHOR TOPNOTCH
# EXAM
75 Nasopharngeal carcinoma will likely spread to NPCA will classically spread to the Level 5 or JAN CHRISTIAN MIDTERM 3
what level of neck lymph nodes? posterior triangle lymph nodes. FELICIANO (TOP 2 - EXAM -
A. Level 2 AUG 2015 MED MARCH 2016
B. Level 3 BOARDS; TOPNOTCH
C. Level 4 MD FROM UST)
D. Level 5
E. Level 6

76 All of the ff are true regardign the principles of The operation should NOT increase the JAN CHRISTIAN MIDTERM 3
anti-reflux surgery EXCEPT? resistance of the relaxed sphincter to a level that FELICIANO (TOP 2 - EXAM -
A. Following a fundoplication, the expected exceeds the peristaltic power of the body of the AUG 2015 MED MARCH 2016
increase is to a level twice the resting pressure esophagus. All other statements are true. BOARDS; TOPNOTCH
B. The operation should place an adequate length MD FROM UST)
of the distal esophageal sphincter of about 2 or
more cm in the stomach.
C. The operation should allow the recontructed
cardia to relax on deglutition for about 10 secs
D. The operation should increase the resistance of
the relaxed sphincter to a level that exceeds the
peristaltic power of the body of the esophagus.
E. The operation should ensure that the
fundoplication can be placed in the abdomen
without undue tnesion and maintained there by
approximation the crura of the diaphragm above
the repair.
77 It is known as the upward sliding of the gastric Type I/Sliding hernia- upward dislocation of the JAN CHRISTIAN MIDTERM 3
fundus alongside a normally positioned cardia? cardia; Type 2/Rolling hernia- upward FELICIANO (TOP 2 - EXAM -
A. Hiatal hernia type I dislcoation of the fundus; Type 3- upward AUG 2015 MED MARCH 2016
B. Hiatal hernia type II dislocation of both cardia and fundus; Type 4- in BOARDS; TOPNOTCH
C. Hiatal hernia type III addition to above, an addtl organ such as the MD FROM UST)
D. Hiatal hernia type IV colon is dislocated.
E. Sliding hernia

78 Herniation involving a Meckel's diverticulum is Spigelian- hernia through the semilunaris; JAN CHRISTIAN MIDTERM 3
also known as? Petersen's- internal henriation of the small bwel FELICIANO (TOP 2 - EXAM -
A. Littre's hernia through the mesenteric defect from the Roux AUG 2015 MED MARCH 2016
B. Spigelian hernia limb; Pantaloon's- hernia sac exists as both BOARDS; TOPNOTCH
C. Petersen's hernia direct and indirect hernia straddling the inferior MD FROM UST)
D. Pantaloon's hernia epigastric vessels; Richter's- incarcerated or
E. Richter's hernia strnagulated hernia involving only one sidewall
of the bowel
79 It is the single most important test in the Please refer to the figure in Schwartz's 10 ed pg JAN CHRISTIAN MIDTERM 3
evaluation of thyroid masses and is the first 1539 Figure 38-14. FELICIANO (TOP 2 - EXAM -
diagnostic step of a solitary thyroid nodule AUG 2015 MED MARCH 2016
according to the Bethesda criteria? BOARDS; TOPNOTCH
A. TSH MD FROM UST)
B. Thyroid UTZ
C. FNAB
D. Thyroid scan
E. fT4 and fT3
80 All fo the ff statements are true regarding epidural Choice D refers to subdural hematoma. In JAN CHRISTIAN MIDTERM 3
hematoma EXCEPT? epidural hematoma there is tearign of the FELICIANO (TOP 2 - EXAM -
A. Epidural hematomas usually appear convex in middle mengineal artery instead AUG 2015 MED MARCH 2016
shape because their expansion stops at the skull's BOARDS; TOPNOTCH
sutures, MD FROM UST)
B. May present with a lucid period immediately
following the trauma and a delay before symptoms
become evident
C. It is an extra-axial bleed
D. Associated with tearing of the bridging veins in
the dural venous sinuses
E. The most common cause of intracranial epidural
hematoma is traumatic
81 Which of the following giant diaphragmatic Schwartz Principles of SUrgery 9th edition p. ANDREW TIU (TOP 1 - FINAL EXAM -
hernias refers to an upward dislocation of the 842 AUG 2015 MED MARCH 2016
gastric fundus alongside a normally positioned BOARDS; TOPNOTCH
cardia? MD FROM CIM)
a. type II rolling
b. type I sliding
c. type III sliding-rolling
d. type IV
e. NOTA
82 2. A 16 year old male stowaway came in for fistulas with an external opening anteriorly ANDREW TIU (TOP 1 - FINAL EXAM -
persistent feculent discharge over his underwear. connect to the internal opening by a short radial AUG 2015 MED MARCH 2016
Upon rectal examination, an indurated tract is tract. Fistulas with an external opening BOARDS; TOPNOTCH
palpable. Which of the following is TRUE? posteriorly tract in a curvilinear fashion to the MD FROM CIM)
a. an anterior opening 4cm from the anal margin posterior midline. D - transphinteric fistula; E -
will have an internal opening through a short extrasphincteric fistula are rare and treatment
radial tract depends upon both anatomy of fistula and its
b. goal of treatment is eradication of sepsis etiology
c. a posterior opening will track in a long radial Schwartz Principles of SUrgery 9th edition p.
route in posterior midline 1064
d. intersphincteric fistulas usually result from an
ischiorectal abscess and extends to both internal
and external sphincters

TOPNOTCH MEDICAL BOARD PREP SURGERY SUPEREXAM Page 12 of 94


For inquiries visit www.topnotchboardprep.com.ph or email us at topnotchmedicalboardprep@gmail.com
TOPNOTCH MEDICAL BOARD PREP SURGERY SUPEREXAM
For inquiries visit www.topnotchboardprep.com.ph or email us at topnotchmedicalboardprep@gmail.com
Item QUESTION EXPLANATION AUTHOR TOPNOTCH
# EXAM
e. extrasphincteric fistulas are the most common

83 3. Which of the following techniques for breast http://www.breastpreservationfoundation.org/ ANDREW TIU (TOP 1 - FINAL EXAM -
surgery will leave most of the breast skin that will nipple-sparing-mastectomy AUG 2015 MED MARCH 2016
create a pocket to be filled with implants or with BOARDS; TOPNOTCH
patient’s own tissue from another part of her MD FROM CIM)
body?
a. skin sparing mastectomy
b. simple mastectomy
c. modified radical mastectomy
d. total mastectomy
e. radical mastectomy
84 4. AJ Perez, a young actor, died in a MVA in When the entire cornea is replaced it is known ANDREW TIU (TOP 1 - FINAL EXAM -
Moncada, Tarlac. He donated his cornea to a 28 as penetrating keratoplasty and when only part AUG 2015 MED MARCH 2016
year old and 8 year old. He is the famous face on of the cornea is replaced it is known as lamellar BOARDS; TOPNOTCH
the Eye Bank of the Philippines posters. Which of keratoplasty. Keratoplasty simply means MD FROM CIM)
the following refers to corneal transplantation surgery to the cornea.Endothelial keratoplasty
wherein the endothelial cells and descemet’s replaces the patient's endothelium with a
membrane are left in place and the anterior layers transplanted disc of posterior
of the cornea are removed and replaced with stroma/Descemets/endothelium (DSEK) or
donor tissue? Descemets/endothelium (DMEK).
a. penetrating keratoplasty https://en.wikipedia.org/wiki/Corneal_transpla
b. superficial anterior lamellar keratoplasty ntation
c. deep anterior lamellar keratoplasty
d. endothelial keratoplasty
e. NOTA
85 5. which of the following refers to an esophageal Schwartz Principles of Surgery 9th edition p. ANDREW TIU (TOP 1 - FINAL EXAM -
hiatal hernia characterized by an upward 842 AUG 2015 MED MARCH 2016
dislocation of the cardia in the posterior BOARDS; TOPNOTCH
mediastinum? MD FROM CIM)
a. type II rolling
b. type I sliding
c. type III sliding-rolling
d. type IV
e. NOTA
86 6. Which of the following bones of the orbital wall Vaughan and Asbury’s General Ophthalmology ANDREW TIU (TOP 1 - FINAL EXAM -
will be involved in a blowout fracture? 17th edition p.1 AUG 2015 MED MARCH 2016
a. frontal process of the maxilla BOARDS; TOPNOTCH
b. zygomatic bone MD FROM CIM)
c. orbital plate of the maxilla
d. orbital process of palatine bone
e. ethmoid bone

87 7. Which of the following maxillary fractures Type I - below the orbital floor ANDREW TIU (TOP 1 - FINAL EXAM -
refers to a fracture line which passes through the type III - involves the medial and lateral walls AUG 2015 MED MARCH 2016
nasal and lacrimal bones in addition to the maxilla and the orbital floor in the presence of BOARDS; TOPNOTCH
forming the medial orbital floor? separation of the facial skeleton MD FROM CIM)
a. type I Le Fort Vaughan and Asbury’s General Ophthalmology
b. type II Le Fort 17th edition p.375
c. type III Le Fort
d. type IV Le Fort
e. type V Le Fort
88 8. Which of the following refers to a delayed A - recurring membranous conjunctivitis ANDREW TIU (TOP 1 - FINAL EXAM -
hypersensitivity reaction to microbial antigens B - most commonly are chalazia AUG 2015 MED MARCH 2016
and consists of a perivasculitis with lymphocytic D - results from exudative process (coagulum on BOARDS; TOPNOTCH
cuffing of a vessel? the surface of the epithelium and when MD FROM CIM)
a. ligneous conjunctivitis removed, epithelium remains intact)
b. granulomas Vaughan and Asbury’s General Ophthalmology
c. phlyctenules 17th edition p.101
d. pseudomembrane
e. chalazion
89 9. A 4 year old male came in for proptosis, Vaughan and Asbury’s General Ophthalmology ANDREW TIU (TOP 1 - FINAL EXAM -
hyperemia, edema, chemosis, limitation of eye 17th edition p.254 AUG 2015 MED MARCH 2016
movement and blurring of vision. Patient had a BOARDS; TOPNOTCH
history of recurrent watery nasal discharges and MD FROM CIM)
nasal congestion for the past 3 months. Which of
the following is the most common cause of
proptosis in children?
a. preseptal cellulitis
b. orbital cellulitis
c. rhabdomyosarcoma
d. capillary hemangioma
e. meningocele
90 10. Which of the following diaphragmatic hernias Schwartz Principles of Surgery 9th edition p. ANDREW TIU (TOP 1 - FINAL EXAM -
refers to the herniation of an additional organ 842 AUG 2015 MED MARCH 2016
such as the colon? BOARDS; TOPNOTCH
a. type II rolling MD FROM CIM)
b. type I sliding
c. type III sliding-rolling
d. type IV
e. NOTA
TOPNOTCH MEDICAL BOARD PREP SURGERY SUPEREXAM Page 13 of 94
For inquiries visit www.topnotchboardprep.com.ph or email us at topnotchmedicalboardprep@gmail.com
TOPNOTCH MEDICAL BOARD PREP SURGERY SUPEREXAM
For inquiries visit www.topnotchboardprep.com.ph or email us at topnotchmedicalboardprep@gmail.com
Item QUESTION EXPLANATION AUTHOR TOPNOTCH
# EXAM
91 11. JCR 24 years old medical student frequently it is grade 2. grade 3 would be lethargic with ANDREW TIU (TOP 1 - FINAL EXAM -
complaints of throbbing headache which is mild focal neurologic deficit (pronator drift) AUG 2015 MED MARCH 2016
immediately relieved with paracetamol. Initially Schwartz Principles of Surgery 9th edition BOARDS; TOPNOTCH
he thought it was just tension headache. After MD FROM CIM)
taking the bimonthly examination, he went to the
ER now with double vision and stiff neck which he
attributed to lack of sleep due to excessive
studying. On physical examination, JCR was wide
awake and nuchal rigidity was noted. Which of the
following is NOT true of this case?
a. JCR should have a CT scan of the head
immediately
b. Hunt -Hess clinical grading would be grade 3
c. negative CT and LP rules out SAH
d. 4 vessel angiography is the gold standard for
diagnosis
e. if LP is performed, xanthochromia with high
RBC count which do not decrease in the tubes
would be present
92 12. RP a movie actor was involved in a motor B - temporary failure of nerve function without ANDREW TIU (TOP 1 - FINAL EXAM -
vehicular accident in Mandaue City which caused a physical axonal disruption. axonal degeneration AUG 2015 MED MARCH 2016
traffic standstill. RP sustained crushing injuries does not occur. BOARDS; TOPNOTCH
over his tibia and fibula and upon examination, he C - disruption of axons and myelin. endoneurium MD FROM CIM)
is unable to evert his foot. Based on the Seddon are intact. axons degenerate proximally and
classification, which of the following peripheral distally from area of injury.
nerve injury patterns is present when there is A - distal degeneration
disruption of axons and endoneurial tubes? D - peripheral collagenous components such as
a. wallerian degeneration perineurium may or may not be intact.
b. neurapraxia Schwartz Principles of Surgery 9th edition
c. axonotmesis
d. neurotmesis
e. none of the above
93 13. In the above scenario, RP’s right foot was due to presence of hard signs (absent pulses), ANDREW TIU (TOP 1 - FINAL EXAM -
noted to be cool, pale, with absent pulses, and a operative exploration is warranted. Other soft AUG 2015 MED MARCH 2016
hematoma over the fracture site. Which of the signs include proximity to vasculature, A-A BOARDS; TOPNOTCH
following is NOT true in the management? index of <0.9, thrill or bruit MD FROM CIM)
a. this would require operative exploration as Schwartz Principles of Surgery 9th edition
absent pulses is a hard sign
b. due to the presence of nerve injury, observation
is only warranted
c. the fractures should be realigned before
definitive vascular examination
d. significant hematoma and peroneal nerve injury
are soft signs
e. acute ischemia and pulsatile hemorrhage are
also hard signs
94 14. Due to RP’s blood loss through his leg, RP adult: 0.5ml/kg/hr; child: 1ml/kg/hr; infant: ANDREW TIU (TOP 1 - FINAL EXAM -
started to feel dizzy and disoriented time and 2ml/kg/hr AUG 2015 MED MARCH 2016
place. Upon physical examination, HR = 128, RR = Schwartz Principles of Surgery 9th edition BOARDS; TOPNOTCH
38, BP = 80/60. Which of the following is NOT true MD FROM CIM)
regarding shock classification and management?
a. patient is in class III shock
b. fluid resuscitation begins with 2L IV bolus of
isotonic crystalloid
c. PRBC is given after fluid resuscitation is
repeated
d. adequate urine output monitoring for RP would
be 1ml/kg/hour
e. none of the above
95 15. A 16 year old male came in for sudden onset of more than 80% of testes can be salvaged if ANDREW TIU (TOP 1 - FINAL EXAM -
right scrotal pain and swelling. On PE, the right surgery is performed within 6 hours, which AUG 2015 MED MARCH 2016
scrotum is swollen with a tender high riding decreases to 20% or less as time progresses BOARDS; TOPNOTCH
testicle. Which of the following is not appropriate beyond 12 hours MD FROM CIM)
in management? Schwartz Principles of Surgery 9th edition
a. golden period for immediate exploration is 8
hours
b. contralateral testes must also be explored and
fixed to dartos fascia
c. if found necrotic, orchiectomy must be
performed
d. testes are fixed with a small nonabsorbable
suture on medial and lateral aspects of dartos
fascia
e. none of the above
96 16. A 34 year old drug lord was stabbed in the left -other relative indications: large laceration of ANDREW TIU (TOP 1 - FINAL EXAM -
flank area by an unknown assailant. Which of the renal pelvis or avulsion of UPJ, persistent AUG 2015 MED MARCH 2016
following are ABSOLUTE indications for surgical urinary leakage, abnormal intraoperative one BOARDS; TOPNOTCH
intervention for renal trauma except? shot IV urogram, devitalized parenchymal MD FROM CIM)
a. renal pedicle avulsion segment with associated urine leak, complete
b. expanding pulsatile or uncontained renal artery thrombosis of both kidneys, renal
retroperitoneal hematoma vascular injuries after failed angiographic
c. coexisting bowel injuries management, renovascular hypertension
d. persistent life threatening hemorrhage Schwartz Principles of Surgery 9th edition
e. none of the above

TOPNOTCH MEDICAL BOARD PREP SURGERY SUPEREXAM Page 14 of 94


For inquiries visit www.topnotchboardprep.com.ph or email us at topnotchmedicalboardprep@gmail.com
TOPNOTCH MEDICAL BOARD PREP SURGERY SUPEREXAM
For inquiries visit www.topnotchboardprep.com.ph or email us at topnotchmedicalboardprep@gmail.com
Item QUESTION EXPLANATION AUTHOR TOPNOTCH
# EXAM
97 17. Which of the following is a feature of primary alkaline phosphatase should be normal or ANDREW TIU (TOP 1 - FINAL EXAM -
hyperparathyroidism except? increased (in the presence of high turnover bone AUG 2015 MED MARCH 2016
a. increased 1,25 dihydroxy vitamin D disease) BOARDS; TOPNOTCH
b. normal 24 hour urinary calcium Schwartz Principles of Surgery 9th edition MD FROM CIM)
c. mild hyperchloremic metabolic acidosis
d. decreased alkaline phosphatase
e. increased chloride

98 18. A 62 year old male IBV, was admitted at the percutaneous UTZ or CT guided ANDREW TIU (TOP 1 - FINAL EXAM -
SICU for flame burns 68% of his body. On the third cholecystostomy is the treatment of choice. AUG 2015 MED MARCH 2016
hospital day, patient extubated himself and started Schwartz Principles of Surgery 9th edition BOARDS; TOPNOTCH
to complain of right upper quadrant pain and MD FROM CIM)
recurrent high grade fever. Bedside HBT
ultrasound revealed distended gallbladder with
thickened wall. Which of the following is not
appropriate in the management?
a. HIDA scan is less sensitive with high false
positive rates
b. cholecystectomy is the treatment of choice
c. this requires urgent intervention
d. patient is usually unfit for surgery
e. cholecystostomy can be diagnostic and
therapeutic
99 19. IBV burn wounds were dressed appropriately mafenide acetate is effective and will penetrate ANDREW TIU (TOP 1 - FINAL EXAM -
by the interns rotating in the burn unit. Which of eschar AUG 2015 MED MARCH 2016
the following is NOT TRUE of the management? Schwartz Principles of Surgery 9th edition BOARDS; TOPNOTCH
a. mafenide acetate is not effective in eschar and it MD FROM CIM)
can cause metabolic acidosis
b. topical application of silver nitrate may cause
electrolyte extravasation
c. silver sulfadiazine is contraindicated on burns in
proximity to newly grafted areas
d. bacitracin, neomycin, and polymyxin B are
useful for superficial partial thickness facial burns
e. neutropenia from silver sulfadiazine may be due
to neutrophil margination from inflammatory
response
100 20. Which of the following types of esophageal Schwartz Principles of SUrgery 9th edition p. ANDREW TIU (TOP 1 - FINAL EXAM -
hiatal hernia is characterized by upward 842 AUG 2015 MED MARCH 2016
dislocation of both the cardia and the gastric BOARDS; TOPNOTCH
fundus? MD FROM CIM)
a. type II rolling
b. type I sliding
c. type III sliding-rolling
d. type IV
e. NOTA
101 A 22 y/o male was brought to ER due to epistaxis. Kiesselbach’s area (a richly vascularized area of ANGELA PAULINE P. DIAGNOSTIC
What is the most common site of anterior septal mucosa at the junction of the nasal cavity CALIMAG-LOYOLA EXAM - AUG
epistaxis: and vestibule.) Woodruff’s plexus – posterior (TOP 8 - FEB 2015 2015
A. Woodruff’s plexus bleeds; convergence of sphenopalatine, anterior MED BOARDS;
B. Kiesselbach’s plexus ethmoid and labial arteries TOPNOTCH MD FROM
C. Nasal valve UST)
D. Nasal sill
E. Both A and B

102 This fracture runs along the maxillary sinus, Le Fort Type I-(Transverse Maxillary Fracture)- ANGELA PAULINE P. DIAGNOSTIC
inferior orbital rim, orbital floor, medial orbital Broken pterygoid plates + fracture that runs CALIMAG-LOYOLA EXAM - AUG
wall and nasofrontal suture and includes the horizontally above the anterior maxillary (TOP 8 - FEB 2015 2015
pterygoid plate:.
alveolar process
MED BOARDS;
A. Le fort type 1 Le Fort Type II-(Pyramidal Fracture)-Broken TOPNOTCH MD FROM
B. Le fort type 2 pterygoid plates + fracture that runs along UST)
C. Le fort type 3 maxillary sinus, inferior orbital rim, orbital floor,
D. Nasoorbital ethmoidal fracture medial orbital wall, & nasofrontal suture
E. Transverse maxillary fracture Le Fort Type III-(Craniofacial Dysjunction)-
Broken pterygoid plates + zygomatic arch
fracture + craniofacial separation
103 The fossa of Rosenmuller is located ______________ SIMILAR TO PREVIOUS BOARD EXAM ANGELA PAULINE P. DIAGNOSTIC
to the pharyngeal ostia of the eustachian tube:
CONCEPT/PRINCIPLE. Behind the ostium of the CALIMAG-LOYOLA EXAM - AUG
A. Posterior eustacian tube (ostium pharyngeum tuba (TOP 8 - FEB 2015 2015
B. Superior auditiva) is a deep recess, the pharyngeal recess MED BOARDS;
C. Inferior (fossa of Rosenmüller). TOPNOTCH MD FROM
D. Lateral UST)
E. Medial

104 Modified radical neck dissection type 1 preserves :


MRND I- Excise LN I-V, SCM, IJV, Submandibular ANGELA PAULINE P. DIAGNOSTIC
A. Sternocleidomastoid muscle (SCM) salivary gland, preserves SAN. MRND II-Excise CALIMAG-LOYOLA EXAM - AUG
B. Internal jugular vein (IJV) LN I-V, IJV, preserves SAN, SCM. MRND III- (TOP 8 - FEB 2015 2015
C. Spinal accessory nerve (SAN) Excise LN I-V, Submandibular Salivary Gland, MED BOARDS;
D. Both A and C presrves SAN, SCM, IJV TOPNOTCH MD FROM
E. All of the above UST)

TOPNOTCH MEDICAL BOARD PREP SURGERY SUPEREXAM Page 15 of 94


For inquiries visit www.topnotchboardprep.com.ph or email us at topnotchmedicalboardprep@gmail.com
TOPNOTCH MEDICAL BOARD PREP SURGERY SUPEREXAM
For inquiries visit www.topnotchboardprep.com.ph or email us at topnotchmedicalboardprep@gmail.com
Item QUESTION EXPLANATION AUTHOR TOPNOTCH
# EXAM
105 Which of the following statements is true Page 19 of Topnotch Handout. All statements ANGELA PAULINE P. DIAGNOSTIC
regarding Nissen fundoplication:
are false. Principles of anti reflux surgery are: 1) CALIMAG-LOYOLA EXAM - AUG
A. The reconstructed cardia should not relax on restore LES P to a level 2x the resting gastric P (TOP 8 - FEB 2015 2015
deglutition and its length to at least 3 cms. 2) place an MED BOARDS;
B. The adequate length of the LES in the positive adequate length of the LES in the (+) P TOPNOTCH MD FROM
pressure environment of the abdomen should be environment of the abdomen (not <1 cm). 3) UST)
at least 1cm allow the reconstructed cardia to relax on
C. The LES pressure should be restored to a level deglutition. 4) the fundoplication should not
3x the resting gastric pressure increase the resistance of the LES to a level
D. The fundoplication should increase the greater than the peristaltic power of the
resistance of the LES to a level greater than the esophageal body. 5) the repair can be placed in
peristaltic power of the esophageal body the abdomen without undue tension.
E. None of the above
106 A 25 y/o male weighing 90 kgs was admitted A person weighing 90 kg with burns to 30% of ANGELA PAULINE P. DIAGNOSTIC
because of flame burns that occurred 2 hours his or her BSA would require 4 x 90 x CALIMAG-LOYOLA EXAM - AUG
earlier. Vital signs were BP = 130/70 PR -– 32.5=11,700 mL of fluid replacement within 24 (TOP 8 - FEB 2015 2015
105/min. RR- 24/min T – 37.6’C. The total body hours. The first half of this amount is delivered MED BOARDS;
surface area burned is approximately 32.5%. The within 8 hours from the burn incident, and the TOPNOTCH MD FROM
estimated fluid required for the first 24 hours is:
remaining fluid is delivered in the next 16 hours. UST)
A. 10800 ml LRS
B. 9100 ml LRS
C. 8500 ml LRS
D. 11200 ml LRS
E. 7000 ml LRS
107 If the anterior external opening of an anal fistula Goodsall's rule relates the external opening of an ANGELA PAULINE P. DIAGNOSTIC
is located 5 cm from the anal margin the fistula, anal fistula to its internal opening. It states that CALIMAG-LOYOLA EXAM - AUG
where is the track of the fistula found?
the external opening situated below the (TOP 8 - FEB 2015 2015
A. Direct track to the Anterior midline transverse anal line will open into the anal canal MED BOARDS;
B. Direct into the anal canal in the midline posteriorly. An anterior opening TOPNOTCH MD FROM
C. Direct track to the Posterior midline is usually associated with a radial tract. Anterior UST)
D. Curved track to the Anterior midline fistulas will have a direct track into the anal
E. Curved track to the Posterior midline canal. Posterior fistulas will have a curved track
with their internal opening lying in the posterior
midline of the anal canal. An exception to the
rule are anterior fistulas lying more than 3 cm.
from the anus, which may have a curved track
(similar to posterior fistulas) that opens into the
posterior midline of the anal canal.
108 A 55 y/o male was brought to the ER due to an Page 73 of surgery supplement Topnotch ANGELA PAULINE P. DIAGNOSTIC
inguinal mass of 14 hours duration, appear after Handout. CALIMAG-LOYOLA EXAM - AUG
lifting heavy objects. Before this event mass (TOP 8 - FEB 2015 2015
appears on standing and disappears when lying MED BOARDS;
down. The mass extends to scrotum. Eventually TOPNOTCH MD FROM
the patient was operated and at surgery,the UST)
transverse aponeurosis arch was sutured to
shelving edge of inguinal ligament, this method of
operation is
A. Lichenstein repair
B. Bassini repair
C. McVay repair
D. Shouldice repair
E. Pott's repair
109 A 39 y/o female notice a 2cm hard on Left breast Page 10 of Topnotch Handout. This is a ANGELA PAULINE P. DIAGNOSTIC
2 yrs ago progressively enlarging to its present phyllodes tumor. Treatment is excision with a CALIMAG-LOYOLA EXAM - AUG
size of 12x10cm ulcerating weeping lesion. The margin of normal tissue (1cm) or simple (TOP 8 - FEB 2015 2015
Left axilla appear to be free of clinically palpable mastectomy. There is no need for axillary MED BOARDS;
nodes. The microscopic picture showed leaf-like dissection (no lymph node involvement) TOPNOTCH MD FROM
pattern of complexity branching spaces UST)
invaginated by fibrous stroma. What is the
recommended treatment?
A. Radical mastectomy
B. Extended Radical mastectomy
C. Simple mastectomy
D. Lumpectomy + sentinel lymph node biopsy
E. Excision with narrow margins
110 A 55 y/o female present with dysphagia of 5 Page 21 of Topnotch Handout. Achalasia ANGELA PAULINE P. DIAGNOSTIC
months duration. Esophagram revealed a dilated esophagogram shows bird's beak esophagus, air CALIMAG-LOYOLA EXAM - AUG
esophagus with narrowing at the distal end (Bird’s fluid level, sigmoid esophagus, esophageal (TOP 8 - FEB 2015 2015
beak appearance). The most probable diagnosis is:
dilation. MED BOARDS;
A. Hypertensive LES TOPNOTCH MD FROM
B. Diffuse esophageal spasm UST)
C. Nutcracker esophagus
D. Achalasia
E. Hiatal hernia

111 The triangle of calot is composed of the ff except::


The common bile duct is not part of the triangle ANGELA PAULINE P. DIAGNOSTIC
A. Cystic duct of Calot. CALIMAG-LOYOLA EXAM - AUG
B. Common hepatic duct (TOP 8 - FEB 2015 2015
C. Inferior border of the liver MED BOARDS;
D. Common bile duct TOPNOTCH MD FROM
E. None of the above UST)

TOPNOTCH MEDICAL BOARD PREP SURGERY SUPEREXAM Page 16 of 94


For inquiries visit www.topnotchboardprep.com.ph or email us at topnotchmedicalboardprep@gmail.com
TOPNOTCH MEDICAL BOARD PREP SURGERY SUPEREXAM
For inquiries visit www.topnotchboardprep.com.ph or email us at topnotchmedicalboardprep@gmail.com
Item QUESTION EXPLANATION AUTHOR TOPNOTCH
# EXAM
112 Stones in Common duct discovered months or Stones deliberately left in place or discovered ANGELA PAULINE P. DIAGNOSTIC
years later after surgery are termed: shortly after surgery are retained stones. Stones CALIMAG-LOYOLA EXAM - AUG
A. Retained stones associated with bile stasis and infection are (TOP 8 - FEB 2015 2015
B. Recurrent stones primary stones. Stones that are formed in the GB MED BOARDS;
C. Primary stones and migrate down the cystic duct to the CBD are TOPNOTCH MD FROM
D. Secondary stones secondary stones. UST)
E. Tertiary stones

113 When progressive enlargement of a multinodular Symptomatic tracheal compression is a surgical ANGELA PAULINE P. DIAGNOSTIC
goiter causes symptomatic tracheal compression, indication. CALIMAG-LOYOLA EXAM - AUG
the preferred management in an otherwise good (TOP 8 - FEB 2015 2015
risk patient is: MED BOARDS;
A. iodine treatment TOPNOTCH MD FROM
B. thyroid hormone treatment UST)
C. surgical resection of abnormal thyroid
D. RAI
E. None of the above

114 19. A patient who has undergone parotid surgery This is a case of Frey's syndrome. ANGELA PAULINE P. DIAGNOSTIC
noticed that whenever he eats, a portion of his CALIMAG-LOYOLA EXAM - AUG
skin in the cheek area sweats. He has: (TOP 8 - FEB 2015 2015
A. Facial nerve injury MED BOARDS;
B. Injury to cervical sympathetic nerves TOPNOTCH MD FROM
C. Cervical parasympathetic nerve injury UST)
D. Great auricular nerve injury
E. Auriculo-temporal nerve injury

115 In esophageal carcinoma treatment, the best Page 22 of Topnotch Handout. Exclusion criteria ANGELA PAULINE P. DIAGNOSTIC
chance for cure is through curative resection. The for curative resection are the following: 1) Age CALIMAG-LOYOLA EXAM - AUG
following are the criteria for exclusion for curative >75, 2) >20% weight loss, 3) FEV1 < 1.25, 4) EF (TOP 8 - FEB 2015 2015
resection, except:
<40%, 5) locally advance tumor MED BOARDS;
A. Age >75 TOPNOTCH MD FROM
B. Weight loss <20% UST)
C. FEV1< 1.25
D. EF < 40%
E. Locally advance tumor
116 A 21 y/o male presents to the ER with complaints Page 23 of Topnotch Handout. This is a case of a ANGELA PAULINE P. DIAGNOSTIC
of left upper quadrant abdominal pain and Mallory-weiss tear. Majority resolves CALIMAG-LOYOLA EXAM - AUG
excreting dark stool for a week. He had mitral spontaneously. However for this patient the best (TOP 8 - FEB 2015 2015
valve replacement 4 years ago and he had been on course of action would be endoscopic MED BOARDS;
oral warfarin treatment. On physical examination, coagulation of bleeders. If endoscopic tx fails, TOPNOTCH MD FROM
he was anemic, had a blood pressure of 140/80 laparotomy+ high gastrotomy + oversewing of UST)
mmHg, a heart rate of 114/min and a hematocrit the tear is the next course of action.
level of 17.6 % and the left upper quadrant was
tender on palpation. After volume replacement
and blood transfusion, the patient underwent
upper GI endoscopy which revealed minimal
bleeding in a mucosal tear in the distal esophagus.
What is the next course of action?
A. Explore laparotomy
B. Ex-lap with high gastrotomy
C. Ex-lap with oversewing of the tear
D. Endoscopic coagulation of bleeders
E. None of the above
117 Afferent loop syndrome may be a complication Page 26 of Topnotch Handout. Afferent loop ANGELA PAULINE P. DIAGNOSTIC
after which surgical procedure?
syndrome is a complication usually following a CALIMAG-LOYOLA EXAM - AUG
A. Graham's patching Bilroth II (Gastrojejunostomy). This is (TOP 8 - FEB 2015 2015
B. Bilroth I characterized by acute or chronic intermittent MED BOARDS;
C. Bilroth II obstruction of the afferent jejunal limb. TOPNOTCH MD FROM
D. Truncal vagotomy UST)
E. Jaboulay pyloroplasty

118 In diagnosing acute appendicitis, there are Page 29. Obturator sign tells you that the tip of ANGELA PAULINE P. DIAGNOSTIC
different signs which can be elicited. Which sign the appendix is directed into the pelvis. Psoas CALIMAG-LOYOLA EXAM - AUG
tells you that the tip of the appendix is directed sign tells you that the tip is retroperitoneally (TOP 8 - FEB 2015 2015
into the pelvis?
directed. Markle sign or jar tenderness is a MED BOARDS;
A. Obturator sign clinical sign in which pain in the right lower TOPNOTCH MD FROM
B. Dunphy's sign quadrant of the abdomen is elicited by dropping UST)
C. Psoas sign from standing on the toes to the heels with a
D. Markle sign jarring landing. It is found in patients with
E. Rovsing's sign localised peritonitis due to acute appendicitis.
Dunphy's sign is a medical sign characterized by
increased abdominal pain with coughing.
Rovsing's sign, palpation of the left lower
quadrant of a person's abdomen increases the
pain felt in the right lower quadrant, the patient
is said to have a positive Rovsing's sign and may
have appendicitis.

TOPNOTCH MEDICAL BOARD PREP SURGERY SUPEREXAM Page 17 of 94


For inquiries visit www.topnotchboardprep.com.ph or email us at topnotchmedicalboardprep@gmail.com
TOPNOTCH MEDICAL BOARD PREP SURGERY SUPEREXAM
For inquiries visit www.topnotchboardprep.com.ph or email us at topnotchmedicalboardprep@gmail.com
Item QUESTION EXPLANATION AUTHOR TOPNOTCH
# EXAM
119 The ideal sites for colostomy are the following, Page 31 of Topnotch Handout. This was ANGELA PAULINE P. DIAGNOSTIC
except:
discussed by Dr. Antonio during the lecture. All CALIMAG-LOYOLA EXAM - AUG
A. Away from bony prominences of the choices given are conditions for ideal site (TOP 8 - FEB 2015 2015
B. Away from natural skin creases of colostomy placement. MED BOARDS;
C. Should be in the rectus abdominis TOPNOTCH MD FROM
D. In a place that is accessible and can be seen by UST)
the patient
E. None of the above
120 Due to the liver's anatomy, the hardest segment to Page 36 of Topnotch Handout. The hardest ANGELA PAULINE P. DIAGNOSTIC
resect is the:
lobe/segment to resect is segment 1/caudate CALIMAG-LOYOLA EXAM - AUG
A. Quadrate lobe lobe. The caudate lobe (posterior hepatic (TOP 8 - FEB 2015 2015
B. Caudate Lobe segment I, Spigelian lobe) is situated upon the MED BOARDS;
C. Left lateral segment postero-superior surface of the liver on the right TOPNOTCH MD FROM
D. Right anterior lobe lobe of the liver, opposite the tenth and eleventh UST)
E. Left medial segment thoracic vertebrae. Caudate lobe resection is one
of the most demanding procedures among
hepatic resection, owing to its deep and complex
location and its proximity to major vessels. The
caudate lobe is located beneath the major
hepatic veins, between the portal vein and
ligamentum venosum, and anterior and partially
circumferential to the inferior vena cava (IVC).
121 How many kilocalories does a 4.5 L of D5LR D5LR has 50 g of Dextrose. 1 g of glucose = 4 LYNN DARYL MIDTERM 1
provide? kcal (According to Dr. Antonio Dextrose contains FELICIANO EXAM - AUG
A. 600 kcal 3.4 kcal/g, however there is no 765 kcal/g in the VILLAMATER, MD 2015
B. 700 kcal option); 4.5L x 50 g x 4 kcal/g = 900 kcal. (TOP 5 - FEB 2015
C. 850 kcal SIMILAR TO PREVIOUS BOARD EXAM CONCEPT. MED BOARDS;
D. 900 kcal TOPNOTCH MD FROM
E. 1050 kcal EAC)

122 Patient arrives at the Emergency Department with This is a case of tension pneumothorax which is LYNN DARYL MIDTERM 1
multiple ice pick stabs onn the chest. He suddenly an emergency condition. Immediate FELICIANO EXAM - AUG
develops difficulty of breathing and hypotension. management of patients would be needle VILLAMATER, MD 2015
On physical examination, there was absent breath thoracostomy at the 2nd ICS MCL.The question (TOP 5 - FEB 2015
sounds on the right chest wall. What will you do is not definitive treatment. The answer will be MED BOARDS;
next? the next step or the immediate/emergency TOPNOTCH MD FROM
A. Do an emergency chest xray management SIMILAR TO PREVIOUS BOARD EAC)
B. Do immediate chest tube thoracostomy EXAM CONCEPT.
C. Do needle thoracostomy
D. Do endotracheal intubation
E. Intercostal nerve block
123 Which of the following finding would warrant Absent breath sounds is the most important LYNN DARYL MIDTERM 1
immediate referral to a surgeon for tube indication for chest tube thoracostomy. FELICIANO EXAM - AUG
thoracostomy? SIMILAR TO PREVIOUS BOARD EXAM CONCEPT VILLAMATER, MD 2015
A. Hypotension (TOP 5 - FEB 2015
B. Tachypnea MED BOARDS;
C. Bleeding TOPNOTCH MD FROM
D. Absent breath sounds EAC)
E. Apnea

124 The mimimum urine output for adult is? The most reliable indicator of organ perfusion LYNN DARYL MIDTERM 1
A. 15 cc/hr during rescucitation is the urine output. It FELICIANO EXAM - AUG
B. 20 cc/hr should be at least 0.5 ml/kg/hr or 30-60 ml/hr VILLAMATER, MD 2015
C. 25 cc/hr for adults; 1 ml/kg/hr in children and 2 (TOP 5 - FEB 2015
D. 30 cc/hr ml/kg/hr for infants. SIMILAR TO PREVIOUS MED BOARDS;
E. 40 cc/hr BOARD EXAM CONCEPT. TOPNOTCH MD FROM
EAC)

125 A 35-year-old male patient presented with The ileocecal and jejunoileum are the most LYNN DARYL MIDTERM 1
abdominal pain, fever, diarrhea and weight loss. common sites of involvement of GI Tuberculosis. FELICIANO EXAM - AUG
Patient had a history of Pulmonary tuberculosis 5 Definitive diagnosis is based on histopathology VILLAMATER, MD 2015
years ago. Which of the following diagnostic test and culture of biopsy specimens obtained by (TOP 5 - FEB 2015
will you request to confirm the diagnosis of colonoscopy or laparotomy. Barcium contrast MED BOARDS;
ileocecal tuberculosis? studies and colonoscopy may show ulcers, TOPNOTCH MD FROM
A. Whole abdominal ultrasound strictures, a deformed cecum, incompetent EAC)
B. Abdominal CT scan ileocecal valve or fistulas. An abdominal CT scan
C. Colonoscopy can define extraluminal pathology, especially
D. Abdominal xray with barium studies lymphadenopathy. SIMILAR TO PREVIOUS
E. MRI BOARD EXAM CONCEPT.
126 The management for carbuncle: More involved, deep-seated infections that LYNN DARYL MIDTERM 1
A. Incision and drainage result in multiple draining cutaneous sinuses are FELICIANO EXAM - AUG
B. Antibiotic called carbuncles. Along with furuncles, these VILLAMATER, MD 2015
C. Wide excision lesions often require incision and drainage (TOP 5 - FEB 2015
D. Elevation and splinting before healing can be initiiated. (Schwartz). MED BOARDS;
E. Serial debridement SIMILAR TO PREVIOUS BOARD EXAM CONCEPT. TOPNOTCH MD FROM
EAC)

TOPNOTCH MEDICAL BOARD PREP SURGERY SUPEREXAM Page 18 of 94


For inquiries visit www.topnotchboardprep.com.ph or email us at topnotchmedicalboardprep@gmail.com
TOPNOTCH MEDICAL BOARD PREP SURGERY SUPEREXAM
For inquiries visit www.topnotchboardprep.com.ph or email us at topnotchmedicalboardprep@gmail.com
Item QUESTION EXPLANATION AUTHOR TOPNOTCH
# EXAM
127 An 18 year-old male presented with right lower Stages of Appendicitis: EARLY STAGE - LYNN DARYL MIDTERM 1
quadrant pain and mild fever. Your primary obstruction of appendiceal lumen leading to FELICIANO EXAM - AUG
diagnosis is acute appendicitis. He is most likely at mucosal edema, ulceration, bacterial diapedesis VILLAMATER, MD 2015
what stage of appendicitis? and appendiceal distenstion due to increasing (TOP 5 - FEB 2015
A. Necrotizing intraluminal pressure and accumulated fluid. MED BOARDS;
B. Suppurative The visceral afferent nerve fibers are stimulated, TOPNOTCH MD FROM
C. Perforated and the patient perceives mild visceral EAC)
D. Gangrenous periumbilical or epigastric pain. SUPPURATIVE
E. Resolving - increasing intraluminal pressure exceed
capillary perfusion pressure associated with
obstructed lymphatic and venous drainage.
Transmural spread of bacteria causes acute
suppurative appendicitis. When the inflammed
seros comes in contact with parietal
peritoneum, patient experience a shift of pain
from periumbilicus to RLQ. GANGRENOUS
APPENDICITIS - intramural venous and arterial
thromboses ensue. PERFORATED - persisting
tissue ischemia results in appendiceal infarction
and perforation. SIMILAR TO PREVIOUS BOARD
EXAM CONCEPT.
128 Which of the following is true regarding acute Rupture incidene is higher in the pediatric and LYNN DARYL MIDTERM 1
appendicitis in the young and in the elderly? geriatric age group. Progression to perforation FELICIANO EXAM - AUG
A. The incidence of rupture is common in both is rapid in the elderly owing to decreased blood VILLAMATER, MD 2015
age groups supply or decreased lymphoid tissue. In (TOP 5 - FEB 2015
B. Both may present atypical clinical features. pediatric population, the more rapid MED BOARDS;
C. Progression to perforation is more rapid both progression to rupture and inability of the TOPNOTCH MD FROM
age groups owing to physiologic and anatomic underdeveloped greater omentum to contain a EAC)
factors. rupture lead to increased morbidity in children.
D. All of the above. Both age groups had atypical presentation
E. Only A and B. making diagnosis difficult. (Source: Schwartz,
Sabiston, Medscape)
129 Patient is about to undergo a small mass excision Bupivacaine is long-acting local anesthetic with LYNN DARYL MIDTERM 1
on her back under local anesthesia. She requested a duration of action of 2-9 hours. Short acting: FELICIANO EXAM - AUG
for a prolonged anesthesia since she opted to go to Procaine; Medium: Lidocaine, Mepivacaine, VILLAMATER, MD 2015
work right after the procedure. What anesthetic Prilocaine; Long-acting: Tetracaine, Bupivacaine, (TOP 5 - FEB 2015
agent will you use? Ropivacaine. SIMILAR TO PREVIOUS BOARD MED BOARDS;
A. Lidocaine EXAM CONCEPT. TOPNOTCH MD FROM
B. Mepivacaine EAC)
C. Procaine
D. Bupivacaine
E. Prilocaine
130 A 62-year-old female presented with 6 month- The most frequent tumor related to lips is LYNN DARYL MIDTERM 1
history of ulcerating lesion in the middle third of squamous cell carcinoma, with the lower lip FELICIANO EXAM - AUG
her lower lip. This is most likely a: more commonly involved than the upper lip. An VILLAMATER, MD 2015
A. Basal cell carcinoma ulcerating lesion is a common presentation in (TOP 5 - FEB 2015
B. Squamous cell carcinoma SCC. MED BOARDS;
C. Melanoma TOPNOTCH MD FROM
D. Adenocarcinoma EAC)
E. Undifferentiated carcinoma

131 A 65-year-old patient presented with difficulty in National Comprehensive Cancer Network LYNN DARYL MIDTERM 1
urination. Findings showed an enlarged prostate Guidelines (NCCN) use 3 ng/ml as PSA threshold FELICIANO EXAM - AUG
and a PSA level of 9 ng/ml. What is the most that would prompt prostate biopsy. A PSA level VILLAMATER, MD 2015
appropriate diagnostic test that you should of 4-10 ng/ml, the likelihood of finding prostate (TOP 5 - FEB 2015
request? cancer is about 25%. Needle biopsy of the MED BOARDS;
A. IVP prostate is indicated for tissue diagnosis in TOPNOTCH MD FROM
B. Prostate biopsy patients who present with elevated PSA levels or EAC)
C. Renal ultrasound abnormal DRE findings. SIMILAR TO PREVIOUS
D. CT scan BOARD EXAM CONCEPT.
E. MRI
132 A 45-year-old male presented with dysphagia and The gross appearance of adenocarcinoma LYNN DARYL MIDTERM 1
weight loss. Endoscopic finding showed an resembles that of squamous cell carcinoma. The FELICIANO EXAM - AUG
irregularly lined mass at the gastroesophageal most important etiologic factor in the VILLAMATER, MD 2015
junction. Biopsy was done which will most likely development of primary adenocarcinoma of the (TOP 5 - FEB 2015
reveal: esophagus is a metaplastic columnar-lined or MED BOARDS;
A. Adenocarcinoma Barrett's esophagus, which occurs as a TOPNOTCH MD FROM
B. Squamous cell carcinoma complication in approximately 10-15% of EAC)
C. Carcinoid tumor patients with GERD. Most adenocarcinomas of
D. Lymphoma the esophagus develop within the lower thoracic
E. GIST esophagus or gastroesophageal junction.
133 Patient presented with fever, RUQ pain and Indications for aspiration: Large abscess >10 LYNN DARYL MIDTERM 1
anorexia. Laboratory findings revealed elevated cm, failure of medical management, FELICIANO EXAM - AUG
WBC and alkaline phosohatase. Ultrasound superinfection, abscess of left lobe (may cause VILLAMATER, MD 2015
showed a 5 cm hypoechoic, unilocular lesion with rupture; affect the pericardium --> tamponade). (TOP 5 - FEB 2015
well defined border at the left lobe of the liver. * In February 2015 boards, what is asked is the MED BOARDS;
Which of the following characteristic of the most likely affectation if liver abscess is at the TOPNOTCH MD FROM
abscess warrants aspiration? left lobe. (pericardium) EAC)
A. the size of the abscess
B. presence of fever
C. presence of elevated alkaline phosphatase
D. the location of the abscess
E. all of the above

TOPNOTCH MEDICAL BOARD PREP SURGERY SUPEREXAM Page 19 of 94


For inquiries visit www.topnotchboardprep.com.ph or email us at topnotchmedicalboardprep@gmail.com
TOPNOTCH MEDICAL BOARD PREP SURGERY SUPEREXAM
For inquiries visit www.topnotchboardprep.com.ph or email us at topnotchmedicalboardprep@gmail.com
Item QUESTION EXPLANATION AUTHOR TOPNOTCH
# EXAM
134 Patient presented with a mass on the anterior Open biopsy is a reliable diagnostic method that LYNN DARYL MIDTERM 1
thigh which measures 6 cm in diameter. Which of allows adequate tissue to be sampled for FELICIANO EXAM - AUG
the following should be performed to determine definitive and specific histologic identification of VILLAMATER, MD 2015
his diagnosis? bone or soft tissue sarcoma. When adequate (TOP 5 - FEB 2015
A. Incision biopsy tissue for diagnosis cannot be obtaine by FNAB MED BOARDS;
B. Core-needle biopsy or core biopsy, an incisional biopsy is indicated TOPNOTCH MD FROM
C. Excision biopsy for deep tumors or for superficial soft tissue EAC)
D. Fine-need aspiration biopsy tumors larger than 3 cm. Excisional biopsy can
E. Stereotactic biopsy be performed for easily accessible or truncal
lesions smaller than 3 cm. (Schwartz) SIMILAR
TO PREVIOUS BOARD EXAM CONCEPT.
135 The most common site of bronchogenic Lung carcinoma arise most often in and about LYNN DARYL MIDTERM 1
carcinoma: the hilus of the lung. (Robbin's). SIMILAR TO FELICIANO EXAM - AUG
A. Trachea PREVIOUS BOARD EXAM CONCEPT VILLAMATER, MD 2015
B. Hilum (TOP 5 - FEB 2015
C. Alveolar septum MED BOARDS;
D. Alveolar sac TOPNOTCH MD FROM
E. Terminal bronchioles EAC)

136 Patient presented with progressive dysphagia to Dysphagia to both solids and liquids points to a LYNN DARYL MIDTERM 1
both solid and liquid for a year now. What is your motility probllem. . Progressive dysphagia for FELICIANO EXAM - AUG
primary impression? both solids and liquids is the hallmark of VILLAMATER, MD 2015
A. Achalasia achalasia. Progressive dysphagia (initially solid (TOP 5 - FEB 2015
B. Esophageal cancer then liquid) is most likely caused by a MED BOARDS;
C. Cancer of the stomach mechanical obstruction (peptic stricture, TOPNOTCH MD FROM
D. Epiphrenic diverticula esophageal cancer) EAC)
E. Diaphragmatic hernia

137 A 33-year old female presented with a solitary FNAB has become the single most important test LYNN DARYL MIDTERM 1
nodule on the right anterior neck and significant in the evaluation of patients with thyroid masses FELICIANO EXAM - AUG
weight loss. What is the most important test to and can be performed with or without VILLAMATER, MD 2015
determine her diagnosis? ultrasound guidance. (Schwartz) * SIMILAR TO (TOP 5 - FEB 2015
A. Free T3 PREVIOUS BOARD EXAM CONCEPT. If the MED BOARDS;
B. Neck ultrasound question is what is the next step or diagnostic TOPNOTCH MD FROM
C. CT scan test to request, the answer is TSH. EAC)
D. Fine Needle aspiration biopsy
E. Neck MRI
138 The most important diagnostic test in caustic SIMILAR TO PREVIOUS BOARD EXAM CONCEPT. LYNN DARYL MIDTERM 1
injury is: The gold standard of safely assessing depth, FELICIANO EXAM - AUG
A. Ultrasound extent of injury, and appropriate therapeutic VILLAMATER, MD 2015
B. Endoscopy regimen is esophagogastroduodenoscopy. (TOP 5 - FEB 2015
C. pH testing MED BOARDS;
D. Chest Xray TOPNOTCH MD FROM
E. Barium swallow EAC)

139 A 1-year-old male presented with chronic Full-thickness recal biopsy is the gold standard LYNN DARYL MIDTERM 1
constipation since birth, poor weight gain and for establishing the diagnosis of Hirchsprung FELICIANO EXAM - AUG
abdominal distention. He had a history passage of diseas. It will reveal absence of ganglion cells in VILLAMATER, MD 2015
meconium at48 hrs after birth. Which of the the myenteric plexus, increased AchE nerve (TOP 5 - FEB 2015
following will be most helpful in confirming his fibers and hypertrophied nerve fibers. MED BOARDS;
diagnosis? TOPNOTCH MD FROM
A. Plain abdominal radiograph EAC)
B. Barium enema
C. Anorectal manometry
D. Rectal biopsy
E. CT scan
140 The most common type of elbow dislocation Posterior elbow dislocation comprise over 90% LYNN DARYL MIDTERM 1
following an injury is: of elbow injuries. Typically, this injury is caused FELICIANO EXAM - AUG
A. Anterior by a traumatic fall onto an outstretched arm VILLAMATER, MD 2015
B. Posterior resulting in an hyper-extension injury. (TOP 5 - FEB 2015
C. Lateral MED BOARDS;
D. Medial TOPNOTCH MD FROM
E. Oblique EAC)

141 What is the most common cause of hepatic abscess Cholangitis is the most common cause of hepatic EDWARD HARRY MIDTERM 2
A. Ruptured appendicitis abscess as any obstruction in bile flow is a VALLAJERA, MD (TOP 8 EXAM - AUG
B. Acute pancreatitis medium for bacterial proliferation. - FEB 2015 MED 2015
C. Diverticulitis BOARDS; TOPNOTCH
D. Cholangitis MD FROM PERPETUAL
E. None of the above BINAN)

142 ABE, a 35 year old female was diagnosed with Metronidazole covers for the most common EDWARD HARRY MIDTERM 2
suppurative cholangitis, what is the drug of choice pathogens associated with suppurative VALLAJERA, MD (TOP 8 EXAM - AUG
A. Cloxacillin cholangitis - FEB 2015 MED 2015
B. Penicillin G BOARDS; TOPNOTCH
C. Metronidazole MD FROM PERPETUAL
D. Erythromycin BINAN)
E. None of the above

TOPNOTCH MEDICAL BOARD PREP SURGERY SUPEREXAM Page 20 of 94


For inquiries visit www.topnotchboardprep.com.ph or email us at topnotchmedicalboardprep@gmail.com
TOPNOTCH MEDICAL BOARD PREP SURGERY SUPEREXAM
For inquiries visit www.topnotchboardprep.com.ph or email us at topnotchmedicalboardprep@gmail.com
Item QUESTION EXPLANATION AUTHOR TOPNOTCH
# EXAM
143 Which of the following statements is true Lymphatic capillaries are involved in the EDWARD HARRY MIDTERM 2
A. Lymphatic capillaries are involved in the absorption of digested fats, lymphatic vessels VALLAJERA, MD (TOP 8 EXAM - AUG
absorption of digested sugars have many valves preventing backflow of lymph, - FEB 2015 MED 2015
B. Lymphatic vessels typically have very few macrophages are found in lymphoid tissue BOARDS; TOPNOTCH
valves though they are derived from monocytes of MD FROM PERPETUAL
C. Macrophages are NOT an example of lymphoid myeloid origin and the pulsation of the arteries BINAN)
cells as well as the muscle contraction and relaxation
D. The expansion and recoil of arteries assists in aid in lymphatic flow.
causing lymph flow
E. None of the above
144 RJC, a 30 year old male was diagnosed with Debridement is recommended with antibiotics EDWARD HARRY MIDTERM 2
chronic necrotizing anal fistula, what should the to promote healing by secondary intention. VALLAJERA, MD (TOP 8 EXAM - AUG
management be? - FEB 2015 MED 2015
A. Hot Sitz bath BOARDS; TOPNOTCH
B. High dose antibiotics MD FROM PERPETUAL
C. Debridement BINAN)
D. Observe
E. None of the above

145 PM, a 31 year old female presents at the clinic with Determine first whether the thyroid function is EDWARD HARRY MIDTERM 2
an anterior neck mass, what should be the initial normal or not. VALLAJERA, MD (TOP 8 EXAM - AUG
diagnostic test? - FEB 2015 MED 2015
A. TSH BOARDS; TOPNOTCH
B. Thyroid scan MD FROM PERPETUAL
C. FNAB BINAN)
D. Incision biopsy
E. None of the above

146 AJS, a 60 year old male came in to the clinic with The symptoms are suggestive that the EDWARD HARRY MIDTERM 2
painless, palpable lymph nodes and hoarseness, malignancy may be at the larynx so laryngeal CA VALLAJERA, MD (TOP 8 EXAM - AUG
you suspect that this is a malignancy but there is should be ruled out first - FEB 2015 MED 2015
no visible malignancy and the positive nodes are BOARDS; TOPNOTCH
at level II and III, what should you do? MD FROM PERPETUAL
A. Chest x-ray BINAN)
B. Laryngoscopy
C. CT scan
D. Neck dissection
E. None of the above
147 KD, a 31 year old male who came in due to right The patient is still at the suppurative stage, EDWARD HARRY MIDTERM 2
lower quadrant pain was seen at the ER, there's no there is pus in the appendix but not yet VALLAJERA, MD (TOP 8 EXAM - AUG
fever and the WBC count was normal. At what distending the organ to cause ischemia or any - FEB 2015 MED 2015
phase of appendicitis is the patient in? systemic manifestations BOARDS; TOPNOTCH
A. Suppurative MD FROM PERPETUAL
B. Gangrenous BINAN)
C. Perforated
D. Chronic
E. None of the above

148 RY, a 35 year old male was brought to the ER due The most likely cause is acute hemorrhagic EDWARD HARRY MIDTERM 2
to signs of peritonitis, history revealed that the pancreatitis, on examination, the patient may VALLAJERA, MD (TOP 8 EXAM - AUG
patient had a drinking spree 1 week prior to show tachycardia, tachypnea, hypotension, and - FEB 2015 MED 2015
consult, PE showed decreased BP with slight hyperthermia. The temperature is usually only BOARDS; TOPNOTCH
tachycardia and tachypnea and a temperature of mildly elevated in uncomplicated pancreatitis. MD FROM PERPETUAL
38.8C, what is the most likely diagnosis? Hypotension is a sign of sequestration of BINAN)
A. Acute hemorrhagic pancreatitis edematous fluid into the retroperitoneum.
B. Necrotizing pancreatitis (Schwartz, 9th ed)
C. Acute edematous pancreatitis
D. Gallstone pancreatitis
E. None of the above
149 A man was diagnosed with amebic liver abscess Because of the proximity of the left lobe to the EDWARD HARRY MIDTERM 2
localized at the left hepatic lobe, why should it be pericardial space just above the diaphragm, left VALLAJERA, MD (TOP 8 EXAM - AUG
aspirated ASAP lobe liver abscesses should be treated promptly. - FEB 2015 MED 2015
A. Can go back into the GIT through the stomach BOARDS; TOPNOTCH
B. Can extend or rupture into the pericardium MD FROM PERPETUAL
C. Can extend outward into the skin BINAN)
D. Both A and C
E. None of the above

150 SNJ, a 39 year old male working as a horse jockey Foley catheterization is contraindicated as it EDWARD HARRY MIDTERM 2
fell off from his horse straddling a fence, he was may cause further damage to the urethra, VALLAJERA, MD (TOP 8 EXAM - AUG
brought to the ER, you should perform the evaluate first the pelvis and rule out any - FEB 2015 MED 2015
following except? urethral injury before inserting a foley catheter BOARDS; TOPNOTCH
A. Foley catheterization MD FROM PERPETUAL
B. Perform cystourethrogram BINAN)
C. Pelvic x-ray
D. Both B and C
E. None of the above

TOPNOTCH MEDICAL BOARD PREP SURGERY SUPEREXAM Page 21 of 94


For inquiries visit www.topnotchboardprep.com.ph or email us at topnotchmedicalboardprep@gmail.com
TOPNOTCH MEDICAL BOARD PREP SURGERY SUPEREXAM
For inquiries visit www.topnotchboardprep.com.ph or email us at topnotchmedicalboardprep@gmail.com
Item QUESTION EXPLANATION AUTHOR TOPNOTCH
# EXAM
151 PNJ, a 29 year old male came in to the ER due to Perform a baseline esophagoscopy to determine EDWARD HARRY MIDTERM 2
ingestion of muriatic acid, 1hour after the the damage to the esophagus VALLAJERA, MD (TOP 8 EXAM - AUG
ingestion, what should you do - FEB 2015 MED 2015
A. Perform barium enema BOARDS; TOPNOTCH
B. Perform esophagoscopy MD FROM PERPETUAL
C. CT scan BINAN)
D. Insert a NGT
E. None of the above

152 After 48 hours, PNJ should undergo next? To rule out any esophageal perforation as air EDWARD HARRY MIDTERM 2
A. Chest x-ray upright from the perforated esophagus will collect in the VALLAJERA, MD (TOP 8 EXAM - AUG
B. Esophagoscopy mediastinum - FEB 2015 MED 2015
C. CT scan BOARDS; TOPNOTCH
D. NGT insertion MD FROM PERPETUAL
E. None of the above BINAN)

153 The pain from the inflammation of the cystic duct The visceral afferent fibers share the same as EDWARD HARRY MIDTERM 2
is commonly referred where? those of the cutaneous fibers of the right VALLAJERA, MD (TOP 8 EXAM - AUG
A. Epigastric shoulder. - FEB 2015 MED 2015
B. Umbilical BOARDS; TOPNOTCH
C. Hypogastric MD FROM PERPETUAL
D. Right shoulder BINAN)
E. None of the above

154 The lienorenal ligament contains all of the Both structures, Splenic artery and vein as well EDWARD HARRY MIDTERM 2
following except: as the tail of the pancreas are contained in the VALLAJERA, MD (TOP 8 EXAM - AUG
A. Splenic artery and vein lienorenal or splenorenal ligament - FEB 2015 MED 2015
B. Tail of the pancreas BOARDS; TOPNOTCH
C. Renal artery and vein MD FROM PERPETUAL
D. Both A and B BINAN)
E. All of the above

155 The majority of the lymph returns to the venous The thoracic duct drains the left side of the head EDWARD HARRY MIDTERM 2
circulation by way of up to the thorax and both abdomen and lower VALLAJERA, MD (TOP 8 EXAM - AUG
A. Right lymphatic duct extremities. - FEB 2015 MED 2015
B. Thoracic duct BOARDS; TOPNOTCH
C. Cisterna chyli MD FROM PERPETUAL
D. Azygos vein BINAN)
E. None of the above

156 Which of the following statement/s is/are true? The peyer's patches are found in the distal small EDWARD HARRY MIDTERM 2
A. The thymus and the thyroid are both lymphoid intestine specifically the ileum. VALLAJERA, MD (TOP 8 EXAM - AUG
organs - FEB 2015 MED 2015
B. Lymph nodes are the only lymphoid organs that BOARDS; TOPNOTCH
contain lymphocytes MD FROM PERPETUAL
C. Peyer’s patches are found within the distal small BINAN)
intestine
D. All lymphoid organs contain both efferent and
afferent lymphatic vessels
E. All of the above
157 Which of the following tonsils is most superior? The pharyngeal tonsil is the most superior EDWARD HARRY MIDTERM 2
A. Palatine VALLAJERA, MD (TOP 8 EXAM - AUG
B. Lingual - FEB 2015 MED 2015
C. Pharyngeal BOARDS; TOPNOTCH
D. Choroidal MD FROM PERPETUAL
E. None of the above BINAN)

158 You are about to perform a chest tube The best site to insert the chest tube would be at EDWARD HARRY MIDTERM 2
thoracostomy for a suspected malignant pleural the 4th-5th ICS MAL. VALLAJERA, MD (TOP 8 EXAM - AUG
effusion, right, you know that the best site to incise - FEB 2015 MED 2015
is at the: BOARDS; TOPNOTCH
A. 4th ICS mid axillary line MD FROM PERPETUAL
B. 5th ICS anterior axillary line BINAN)
C. 6th ICS mid axillary line
D. 8th ICS posterior axillary line
E. None of the above
159 The following are absorbable sutures except: Nylon is non-absorbable EDWARD HARRY MIDTERM 2
A. Vicryl VALLAJERA, MD (TOP 8 EXAM - AUG
B. Nylon - FEB 2015 MED 2015
C. Monocryl BOARDS; TOPNOTCH
D. Chromic MD FROM PERPETUAL
E. Cat gut BINAN)

160 Which of the following is a function of the spleen? EDWARD HARRY MIDTERM 2
A. The spleen is located in the lower right hand VALLAJERA, MD (TOP 8 EXAM - AUG
quadrant of the abdominal cavity - FEB 2015 MED 2015
B. The spleen consists primarily of white pulp, BOARDS; TOPNOTCH
which functions in RBC recycling MD FROM PERPETUAL
C. The spleen is the only lymphoid organ that BINAN)
entirely lacks white blood cells
D. If the spleen is surgically removed, many of its
blood cleansing functions can be taken over by the
liver
E. The spleen is the only lymphoid organ that
contains both afferent and efferent lymphatic
TOPNOTCH MEDICAL BOARD PREP SURGERY SUPEREXAM Page 22 of 94
For inquiries visit www.topnotchboardprep.com.ph or email us at topnotchmedicalboardprep@gmail.com
TOPNOTCH MEDICAL BOARD PREP SURGERY SUPEREXAM
For inquiries visit www.topnotchboardprep.com.ph or email us at topnotchmedicalboardprep@gmail.com
Item QUESTION EXPLANATION AUTHOR TOPNOTCH
# EXAM
vessels

161 A patient complains of severe flank pain and this stones are also called struvite stones and HAROLD JAY S. MIDTERM 3
hematuria.Upon work-up, CT stonogram showed usually associated with urease producing BAYTEC, MD (TOP 10 - EXAM - AUG
staghorn calculi. Which of the following is the microoraganism FEB 2015 MED 2015
most common component of the stone? BOARDS; TOPNOTCH
A. Cystine MD FROM FEU)
B. Calcium oxalate
C. Uric acid
D. phosphate
E. Magnesium ammonium phosphate

162 A patient came in at the ER due to head trauma. other choices are characteristic of subdural HAROLD JAY S. MIDTERM 3
Upon assesment with your senior resident, he said hematoma BAYTEC, MD (TOP 10 - EXAM - AUG
that his findings is very much consistent of an FEB 2015 MED 2015
epidural hematoma. Which of the following is true BOARDS; TOPNOTCH
regarding this case? MD FROM FEU)
A. venous blood seeping out from the vessel
B. CT scan revealed lenticular shaped lesion
C. bridging veins are involved
D. delayed presentation with fluctuating levels of
consciousness
E. none of the above
163 A patient came in at your out patient clinic for Among the choices, ropivacaine and bupivacaine HAROLD JAY S. MIDTERM 3
excision of a nodule. He said that he wants the offers the longest duration of action but BAYTEC, MD (TOP 10 - EXAM - AUG
anesthesia to have a longer duration of action ropivacaine has a lesser cardiotoxicity property FEB 2015 MED 2015
because he needs to be as painfree as possible for SIMILAR TO PREVIOUS BOARD EXAM BOARDS; TOPNOTCH
a meeting after the procedure. He also added that CONCEPT/PRINCIPLE MD FROM FEU)
he has history of arrythmia. Which of the following
local anesthetics is the most appropriate for the
patient?
A. Lidocaine
B. prilocaine
C. mepivacaine
D. bupivacaine
E. ropivacaine
164 Among the following local anesthetics, which of Schwartz 9th ed. Page 1736. SIMILAR TO HAROLD JAY S. MIDTERM 3
the following has the longest half-life? PREVIOUS BOARD EXAM CONCEPT/PRINCIPLE BAYTEC, MD (TOP 10 - EXAM - AUG
A. procaine FEB 2015 MED 2015
B. lidocaine BOARDS; TOPNOTCH
C. prilocaine MD FROM FEU)
D. Bupivacaine
E. Mepivacaine

165 In severe burns, the goal for urine output in adults Schwartz 9th ed. Page 200 HAROLD JAY S. MIDTERM 3
should be 30ml/h and to ensure optimal end- BAYTEC, MD (TOP 10 - EXAM - AUG
organ perfusion, the target MAP is _____ mmHg. FEB 2015 MED 2015
A. 50 BOARDS; TOPNOTCH
B. 60 MD FROM FEU)
C. 70
D. 80
E. 90

166 Mafenide acetate is one of the alternatives for Schwartz 9th ed. Page 202 HAROLD JAY S. MIDTERM 3
topical antibioitic treatment in burns. Which of the BAYTEC, MD (TOP 10 - EXAM - AUG
following is a major side effect of the drug that FEB 2015 MED 2015
should be observed? BOARDS; TOPNOTCH
A. Metabolic acidosis MD FROM FEU)
B. Respiratory acidosis
C. neutropenia
D. anemia
E. ototoxicity
167 A medium-built 35 year old man came in at the ER patient is at class II in which the approximated HAROLD JAY S. MIDTERM 3
due to bleeding secondary to gunshot wound. His blood loss is 750-1500 or 15-30% of blood, BAYTEC, MD (TOP 10 - EXAM - AUG
heart rate is 115, anxious and he is already having >100 HR, orthostatic hypotension, and patient FEB 2015 MED 2015
orthostatic hypotension. Based on the tends to be anxious. Class I <750ml. Class II is BOARDS; TOPNOTCH
classification of hemorrhage, approximately how 1500-2000. class IV is >2000. Patient is only on MD FROM FEU)
many mililiters is the blood loss? ORTHOSTATIC hypotension. See picture from
A. <500 schwartz 9th ed page 99
B. 500-750
C. 750-1500
D. 1500-2000
E. >2000

TOPNOTCH MEDICAL BOARD PREP SURGERY SUPEREXAM Page 23 of 94


For inquiries visit www.topnotchboardprep.com.ph or email us at topnotchmedicalboardprep@gmail.com
TOPNOTCH MEDICAL BOARD PREP SURGERY SUPEREXAM
For inquiries visit www.topnotchboardprep.com.ph or email us at topnotchmedicalboardprep@gmail.com
Item QUESTION EXPLANATION AUTHOR TOPNOTCH
# EXAM
168 Biliary colic generally includes 1-5 hours of HAROLD JAY S. MIDTERM 3
constant pain on the epigastrium or right upper BAYTEC, MD (TOP 10 - EXAM - AUG
quadrant which usually radiates to what part of FEB 2015 MED 2015
the body? BOARDS; TOPNOTCH
A. left flank area MD FROM FEU)
B. Right flack area
C. Right axillary region
D. Left scapular region
E. Right scapular region
169 In tension pneumothorax, the contralateral lung is schwartz 9th ed page 138 HAROLD JAY S. MIDTERM 3
compressed and the heart rotates about this/these BAYTEC, MD (TOP 10 - EXAM - AUG
vessel which will eventually cause circulatory FEB 2015 MED 2015
collapse. BOARDS; TOPNOTCH
A. Inferior vena cava MD FROM FEU)
B. Superior vena cava
C. aorta
D. Pulmonary vessels
E. A and B
170 A 4 year old male was rushed to the ER because of schwartz 9th ed page 141 HAROLD JAY S. MIDTERM 3
vehicular accident. The patient is already BAYTEC, MD (TOP 10 - EXAM - AUG
tachycardic, hypotensive, tachypneic, confused FEB 2015 MED 2015
and pale. Fluid resuscitation of lactated ringersn of BOARDS; TOPNOTCH
IV bolus should be immediately started at MD FROM FEU)
A. 10 ml/kg
B. 20 ml/kg
C. 30 ml/kg
D. 40 ml/kg
E. 50 ml/kg
171 A head trauma patient came in at the ER. Upon E2V3M4 HAROLD JAY S. MIDTERM 3
assessment, you elicited eye opening through pain, BAYTEC, MD (TOP 10 - EXAM - AUG
responds with inapropriate words, and withdraws FEB 2015 MED 2015
to pain. What is the GCS score of the patient? BOARDS; TOPNOTCH
A. 6 MD FROM FEU)
B. 7
C. 8
D. 9
E. 10

172 In cases of trauma, once immediate threats to life Last meal… P is previous illness and pregnancy HAROLD JAY S. MIDTERM 3
have been addressed, secondary survey is BAYTEC, MD (TOP 10 - EXAM - AUG
initiated. The patient and surrogates should be FEB 2015 MED 2015
querried to obtain an AMPLE history. All of the BOARDS; TOPNOTCH
following are component of AMPLE history MD FROM FEU)
EXCEPT:
A. Allergies
B. Medications
C. Pregnancy
D. Last voiding episode or urine output
E. Events related to injury
173 This cytokine induces fever through prostaglandin HAROLD JAY S. MIDTERM 3
activity in anterior hypothalamus and it also BAYTEC, MD (TOP 10 - EXAM - AUG
promotes beta endorphin release form pituitary FEB 2015 MED 2015
A. Interleukin 1 BOARDS; TOPNOTCH
B. Interleukin 2 MD FROM FEU)
C. Interleukin 6
D. Interleukin 8
E. Interferon gamma

174 In response to injury, this cytokine is among the HAROLD JAY S. MIDTERM 3
earliest responders and it induces muscle BAYTEC, MD (TOP 10 - EXAM - AUG
breakdown and cachexia through increased FEB 2015 MED 2015
metabolism. BOARDS; TOPNOTCH
A. Interleukin 1 MD FROM FEU)
B. Interleukin 2
C. Interleukin 8
D. Interferon gamma
E. TNF alpha
175 In most series reports, which of the following is schwartz 9th ed page 1088 HAROLD JAY S. MIDTERM 3
the most common type of primary appendiceal BAYTEC, MD (TOP 10 - EXAM - AUG
tumors which has also the best 5-year survival? FEB 2015 MED 2015
A. Goblet cell carcinoma BOARDS; TOPNOTCH
B. Mucinous carcinoma MD FROM FEU)
C. carcinoid
D. Signet-ring ccarcinoma
E. Adenocarcinoma

TOPNOTCH MEDICAL BOARD PREP SURGERY SUPEREXAM Page 24 of 94


For inquiries visit www.topnotchboardprep.com.ph or email us at topnotchmedicalboardprep@gmail.com
TOPNOTCH MEDICAL BOARD PREP SURGERY SUPEREXAM
For inquiries visit www.topnotchboardprep.com.ph or email us at topnotchmedicalboardprep@gmail.com
Item QUESTION EXPLANATION AUTHOR TOPNOTCH
# EXAM
176 During appendectomy, a firm yellow bulbar mass the mass is most likely carcinoid. If the mass in HAROLD JAY S. MIDTERM 3
approximately 1.8cm located at the tip of the <1cm or if it is >1-2cm but located at the tip or BAYTEC, MD (TOP 10 - EXAM - AUG
appendix was seen. Quick histologic study was mid appendix, appendectomy alone should be FEB 2015 MED 2015
done and shows malignancy. Which of the done. Appendectomy lang. See Picturefrom BOARDS; TOPNOTCH
following is the procedure of choice to be done? schartz 9th ed page 1088. MD FROM FEU)
A. appendectomy alone
B. right hemicolectomy
C. extended hemicolectomy
D. total colectomy
E. exploratory laparotomy
177 A patient with fistula in ano came in to your clinic aplication of the Goodsall's rule. HAROLD JAY S. MIDTERM 3
for consult. On assessment, you saw that the BAYTEC, MD (TOP 10 - EXAM - AUG
anterior external opening is approximately 4 cm FEB 2015 MED 2015
fromm the anal margin. With this, you expect that BOARDS; TOPNOTCH
the interior opening is: MD FROM FEU)
A. connected with a short radial tract to the
anterior midline
B. connected with a short radial tract to the
posterior midline
C. connected in a curvilinear fashion to the
anterior midline
D. connected in a curvilinear fashion to the
posterior midline
E. none of the above
178 A 60 year old 20 pack year smoker came in at the schwartz 9th ed page 490.. Lymphadenopathy of HAROLD JAY S. MIDTERM 3
OPD for check up. Your senior resident suspects the posterior triangle of the neck should BAYTEC, MD (TOP 10 - EXAM - AUG
that the patient has nasopharyngeal carcinoma provoke consideration for a nasopharyngeal FEB 2015 MED 2015
because of the location of the lymphadenopathy. primary BOARDS; TOPNOTCH
The lymph node is most likely located at what MD FROM FEU)
level?
A. I
B. II
C. III
D. IV
E. V
179 A 50 year old woman came in to your clinic HAROLD JAY S. MIDTERM 3
because of painless anterior neck mass which was BAYTEC, MD (TOP 10 - EXAM - AUG
associated with dysphagia, dyspnea, choking and FEB 2015 MED 2015
hoarseness and signs and symptoms of BOARDS; TOPNOTCH
hypothyroidism. Upon palpation, the mass in hard, MD FROM FEU)
woody thyroid gland with fixation ro the
sorrounding tissue.
A. Reidel's thyroiditis
B. Thyroid nodule
C. de quervain's thyroiditis
D. acute suppurative thyroiditis
E. Papillary carcinoma
180 The following are true with Thyroid Papillary usually spread via lymphatic route HAROLD JAY S. MIDTERM 3
carcinoma EXCEPT: BAYTEC, MD (TOP 10 - EXAM - AUG
A. It is the most common thyroid malignancy in FEB 2015 MED 2015
iodine sufficient areas BOARDS; TOPNOTCH
B. It has excellent prognosis MD FROM FEU)
C. Usual metastatic sites are lungs, bones, liver
and brain
D. Age is the most important prognostic factor in
determining long term survival
E. spreads via hematogenous route
181 What is the most appropriate definitive Dalrymple sign is upper lid retraction, von JEAN PAOLO M. FINAL EXAM -
management for patients with Grave's disease Graefe sign is lid lag on downgaze, both seen in DELFINO, MD (TOP 10 AUG 2015
who is positive for Dalrymple and von Graefe sign? Grave's patient with ophthalmopathy. Since the - FEB 2015 MED
A. Subtotal thyroidectomy patient already have ophthalmopathy, BOARDS; TOPNOTCH
B. Near Total thyroidectomy radioactive ablation is no longer considered MD FROM FATIMA)
C. Isthmusectomy because it worsens the ophthalmopathy. Near
D. Radioactive Ablation therapy total thyroidectomy is done in patients with
E. Medical treatment coexistent thyroid Ca, refuse RAI,
ophthalmopathy, compressive symptoms and
severe reactions to thyroid drugs.
182 What is the most common location of lung cancer? SIMILAR TO PREVIOUS BOARD EXAM JEAN PAOLO M. FINAL EXAM -
A. 1st generation bronchi CONCEPT/PRINCIPLE.. Bronchogenic DELFINO, MD (TOP 10 AUG 2015
B. 2nd generation bronchi carcinoma accounts for about one third of all - FEB 2015 MED
C. alveolus cancer deaths in men and is becoming BOARDS; TOPNOTCH
D. Terminal bronchiole increasingly common in women. It commences MD FROM FATIMA)
E. All of the above in most patients in the mucous membrane lining
the larger bronchi and is therefore situated close
to the hilum of the lung.
183 4 weeks old infant presented with projectile non- The diagnosis is hypertrophic pyloric stenosis. JEAN PAOLO M. FINAL EXAM -
bilous vomiting. A firm, movable olive-shaped The confirmatory test is ultrasound DELFINO, MD (TOP 10 AUG 2015
mass can be palpated in the abdomen. What - FEB 2015 MED
diagnostic test will confirm the diagnosis? BOARDS; TOPNOTCH
A. MRI MD FROM FATIMA)
B. CT scan
C. Abdominal x-ray
D. Ultrasound
E. PET scan

TOPNOTCH MEDICAL BOARD PREP SURGERY SUPEREXAM Page 25 of 94


For inquiries visit www.topnotchboardprep.com.ph or email us at topnotchmedicalboardprep@gmail.com
TOPNOTCH MEDICAL BOARD PREP SURGERY SUPEREXAM
For inquiries visit www.topnotchboardprep.com.ph or email us at topnotchmedicalboardprep@gmail.com
Item QUESTION EXPLANATION AUTHOR TOPNOTCH
# EXAM
184 A patient has fistula-in-ano. The external opening In general, fistulas with an external opening JEAN PAOLO M. FINAL EXAM -
was noted at 2 o'clock position (right anteriorly connect to the internal opening by a DELFINO, MD (TOP 10 AUG 2015
anterolateral), 2.5cm from the the anal margin. short, radial tract. Fistulas with an external - FEB 2015 MED
The internal opening of the fistulous tract is opening posteriorly track in a curvilinear BOARDS; TOPNOTCH
expected to be in what position? fashion to the posterior midline. However, MD FROM FATIMA)
A. posterolateral exceptions to this rule often occur if an anterior
B. posterior midline external opening is greater than 3 cm from the
C. right anterolateral anal margin. Such fistulas usually track to the
D. left posterolateral posterior midline. This is known as the
E. left anterolateral Goodsall's rule.
185 In doing pericardiocentesis, what technique JEAN PAOLO M. FINAL EXAM -
utilizes the shortest distance? DELFINO, MD (TOP 10 AUG 2015
A. Subxiphoid approach - FEB 2015 MED
B. Transdiaphragmatic BOARDS; TOPNOTCH
C. Left 4th ICS MD FROM FATIMA)
D. Left 5th ICS
E. None of the above

186 15 year old patient was bitten by a stray dog and You do not suture the wound from dog bites, JEAN PAOLO M. FINAL EXAM -
sustained a 4cm lacerated wound in the middle healing by secondary intention is preferred. DELFINO, MD (TOP 10 AUG 2015
1/3 of the leg, lateral side. The following are true - FEB 2015 MED
of the management for this patient except? BOARDS; TOPNOTCH
A. Thoroughly clean the wound with water and MD FROM FATIMA)
antiseptic solution
B. Suture the wound
C. Give Anti-rabies vaccine and Ig
D. Give tetanus prophylaxis
E. None of the above
187 After an appendectomy, patient was given 3L of SIMILAR TO PREVIOUS BOARD EXAM JEAN PAOLO M. FINAL EXAM -
D5LR as fluid maintenance for 24 hours. How CONCEPT/PRINCIPLE.. 1 L of D5LR contains 50g DELFINO, MD (TOP 10 AUG 2015
many kilocalories does it provide to the patient? glucose. 1 g glucose contain 4 kcal. 4x50= - FEB 2015 MED
A. 600kcal 200kcal per liter. Patient consumes 3 L so 200 x BOARDS; TOPNOTCH
B. 700kcal 3= 600 kcal MD FROM FATIMA)
C. 800kcal
D. 300kcal
E. 900kcal

188 What is the most commonly used suture in SIMILAR TO PREVIOUS BOARD EXAM JEAN PAOLO M. FINAL EXAM -
circumcision? CONCEPT/PRINCIPLE. DELFINO, MD (TOP 10 AUG 2015
A. Silk - FEB 2015 MED
B. Nylon BOARDS; TOPNOTCH
C. Chromic MD FROM FATIMA)
D. Vicryl
E. Prolene

189 8 year old female suffered a lacerated wound, 3cm SIMILAR TO PREVIOUS BOARD EXAM JEAN PAOLO M. FINAL EXAM -
in the left temporal area due to self-accident. CONCEPT/PRINCIPLE.. toxic dose of lidocaine DELFINO, MD (TOP 10 AUG 2015
Wound suturing was done. 2% lidocaine was used is 5mg/kg. 5mg/kg x 25kg= 125mg. 2% - FEB 2015 MED
as local anesthetic. What is the toxic dose of lidocaine is 20mg/ml so 125mg/20mg/ml= BOARDS; TOPNOTCH
lidocaine if the patient's weight is 25kg? 6.25ml MD FROM FATIMA)
A. 6.50 ml
B. 6.35 ml
C. 6.45 ml
D. 6.25 ml
E. 6.55 ml
190 A patient was brought to the emergency room 12 SIMILAR TO PREVIOUS BOARD EXAM JEAN PAOLO M. FINAL EXAM -
hours after ingestion of "liquid sosa". After history CONCEPT/PRINCIPLE.. If a patient presents DELFINO, MD (TOP 10 AUG 2015
and PE, what should be done initially in the within the first hour of ingestion, neutralization - FEB 2015 MED
management? is attempted. Early endoscopy is recommended BOARDS; TOPNOTCH
A. Induce emesis 12 to 24 hours after ingestion to identify the MD FROM FATIMA)
B. Neutralize with half-strength vinegar or citrus grade of the burn. The treatment of caustic
juice lesions of the esophagus is determined by the
C. Do immediate esophagectomy extent of the injury and addresses the injuries
D. Observe that occur both in the acute and chronic phase.
E. Do early endoscopy
191 30 minutes prior, an 18 year old female attempted SIMILAR TO PREVIOUS BOARD EXAM JEAN PAOLO M. FINAL EXAM -
suicide by ingesting hydrochloric acid. What CONCEPT/PRINCIPLE.. If a patient presents DELFINO, MD (TOP 10 AUG 2015
should be done initially in your management? within the first hour of ingestion, neutralization - FEB 2015 MED
A. Induce emesis is attempted. BOARDS; TOPNOTCH
B. Neutralize with milk, egg whites, or antacids MD FROM FATIMA)
C. Do immediate esophagectomy
D. Observe
E. Do early endoscopy

192 58 year old patient underwent cholecystectomy. 2 SIMILAR TO PREVIOUS BOARD EXAM JEAN PAOLO M. FINAL EXAM -
days post-operation, patient developed fever. CONCEPT/PRINCIPLE.. Fever in the first 3 days DELFINO, MD (TOP 10 AUG 2015
There were no other associated symptoms. What after surgery most likely has a noninfectious - FEB 2015 MED
is the most likely cause of the fever? cause. Atelectasis is considered if fever BOARDS; TOPNOTCH
A. Atelectasis developed within 24 hours. Fever developed 2 MD FROM FATIMA)
B. UTI days post op int he case given so phlebitis is the
C. Surgical wound infection most likely cause.
D. Nosocomial pneumonia
E. Phlebitis

TOPNOTCH MEDICAL BOARD PREP SURGERY SUPEREXAM Page 26 of 94


For inquiries visit www.topnotchboardprep.com.ph or email us at topnotchmedicalboardprep@gmail.com
TOPNOTCH MEDICAL BOARD PREP SURGERY SUPEREXAM
For inquiries visit www.topnotchboardprep.com.ph or email us at topnotchmedicalboardprep@gmail.com
Item QUESTION EXPLANATION AUTHOR TOPNOTCH
# EXAM
193 What is the most important component of healing SIMILAR TO PREVIOUS BOARD EXAM JEAN PAOLO M. FINAL EXAM -
in a large surface superficial wound? CONCEPT/PRINCIPLE.. If only the epithelium DELFINO, MD (TOP 10 AUG 2015
A. Collagen deposition and superficial dermis are damaged, such as - FEB 2015 MED
B. Wound contraction occurs in split-thickness skin graft donor sites or BOARDS; TOPNOTCH
C. Epithelialization in superficial second-degree burns, then repair MD FROM FATIMA)
D. Maturation consists primarily of re-epithelialization with
E. Inflammation minimal or no fibroplasia and granulation tissue
formation.

194 43 year old male is diagnosed with Papillary Patients with well differentiated thyroid cancer JEAN PAOLO M. FINAL EXAM -
carcinoma of the thyroid gland. Lymph nodes in < 45 years old is staged as follows: Stage I: Any DELFINO, MD (TOP 10 AUG 2015
the central compartment and the jugular chains T, Any N, M0; Stage II: Any T, Any N, Mi - FEB 2015 MED
are positive and there is documented metastasis in BOARDS; TOPNOTCH
the lungs. What is the cancer stage? MD FROM FATIMA)
A. I
B. II
C. III
D. IV
E. 0
195 22 years old female patient complained of pituitary tumors <1cm are considered as JEAN PAOLO M. FINAL EXAM -
amenorrhea and galactorrhea. Diagnostic work- microadenoma while those that are > 1 cm are DELFINO, MD (TOP 10 AUG 2015
ups revealed a pituitary tumor 0.7cm in size. How macroadenomas. Microadenomas are managed - FEB 2015 MED
will you manage the patient? medically with bromocriptine. BOARDS; TOPNOTCH
A. Observe MD FROM FATIMA)
B. Open craniotomy
C. Transphenoidal surgery
D. Bromocriptine therapy
E. Craniectomy
196 There are how many parathyroid glands? giveaway question last boards JEAN PAOLO M. FINAL EXAM -
A. 1 DELFINO, MD (TOP 10 AUG 2015
B. 2 - FEB 2015 MED
C. 3 BOARDS; TOPNOTCH
D. 4 MD FROM FATIMA)
E. 5

197 What is the most important risk factor in the Gender is the most important risk factor in JEAN PAOLO M. FINAL EXAM -
development of breast cancer? Breast cancer DELFINO, MD (TOP 10 AUG 2015
A. age - FEB 2015 MED
B. sex BOARDS; TOPNOTCH
C. Family history MD FROM FATIMA)
D. smoking
E. Early menarche, late menopause

198 The following are the nerves that are spared in All the given choices are spared in MRM JEAN PAOLO M. FINAL EXAM -
doing Modified Radical Mastectomy except? DELFINO, MD (TOP 10 AUG 2015
A. Long thoracic nerve - FEB 2015 MED
B. Thoracodorsal nerve BOARDS; TOPNOTCH
C. Medial pectoral nerve MD FROM FATIMA)
D. Lateral pectoral nerve
E. None of the above

199 Widened mediastinum is a finding most commonly Widened mediastinum is the most sensitive CXR JEAN PAOLO M. FINAL EXAM -
associated with what condition? finding suggesting tear of the aorta DELFINO, MD (TOP 10 AUG 2015
A. Aortic dissection - FEB 2015 MED
B. Pericardial effusion BOARDS; TOPNOTCH
C. Pleural effusion MD FROM FATIMA)
D. Esophageal rupture
E. Peptic ulcer perforation

200 What is the drug of choice for primary peritonitis? Third-generation cephalosporins such as JEAN PAOLO M. FINAL EXAM -
A. Penicillin cefotaxime (2 g q8h, administered IV) provide DELFINO, MD (TOP 10 AUG 2015
B. Cefazolin reasonable initial coverage in peritonitis - FEB 2015 MED
C. Cefotaxime BOARDS; TOPNOTCH
D. Metronidazole MD FROM FATIMA)
E. Clindamycin

201 A post-operative patient is to be given a penicillin- Among all antibiotics, carbapenems possess the GRACE ARVIOLA, MD DIAGNOSTIC
type of antibiotic for an abdominal infection. highest cross-reactivity with penicillins. (TOP 3 - AUG 2014 EXAM - FEB
However, history revealed a strong allergy to this MED BOARDS; 2015
class of medications. Which among the following TOPNOTCH MD)
drugs should be MOST avoided as it possesses the
highest cross-reactivity with penicillin?
A. Cefazolin
B. Clindamycin
C. Metronidazole
D. Imipenem + Cilastatin
E. Sulbactam + Ampicillin

TOPNOTCH MEDICAL BOARD PREP SURGERY SUPEREXAM Page 27 of 94


For inquiries visit www.topnotchboardprep.com.ph or email us at topnotchmedicalboardprep@gmail.com
TOPNOTCH MEDICAL BOARD PREP SURGERY SUPEREXAM
For inquiries visit www.topnotchboardprep.com.ph or email us at topnotchmedicalboardprep@gmail.com
Item QUESTION EXPLANATION AUTHOR TOPNOTCH
# EXAM
202 A 4 kg-infant with 20% TBSA partial thickness In burn patients, adequate fluid resuscitation of GRACE ARVIOLA, MD DIAGNOSTIC
burns on the trunk was admitted at the Burn Unit an infant achieves at least 2 mL/kg/hr of urine (TOP 3 - AUG 2014 EXAM - FEB
of your hospital. To accurately assess whether output. MED BOARDS; 2015
your fluid resuscitation is adequate, this infant's TOPNOTCH MD)
urine output should measure at least:
A. 2 mL/hr
B. 4 mL/hr
C. 6 mL/hr
D. 8 mL/hr
E. 10 mL/hr
203 A patient with a gunshot wound at the torso is Because the descending thoracic aorta is located GRACE ARVIOLA, MD DIAGNOSTIC
suspected to have a tear of the descending posteriorly, this structure is best approached (TOP 3 - AUG 2014 EXAM - FEB
thoracic aorta. For optimal exposure, what using a left posterolateral thoracotomy. MED BOARDS; 2015
surgical approach is appropriate? TOPNOTCH MD)
A. Left anterolateral thoracotomy
B. Left posterolateral thoracotomy
C. Left anterolateral thoracotomy and median
sternotomy with supraclavicular extension
D. Bilateral anterolateral thoracotomy
E. Median sternotomy

204 A 43 year old female patient with a 1 cm palpable Adjuvant chemotherapy for early invasive GRACE ARVIOLA, MD DIAGNOSTIC
mass of the right breast for 5 years underwent breast cancer is considered for all node-positive (TOP 3 - AUG 2014 EXAM - FEB
core-needle biopsy which revealed invasive ductal cancers, all cancers that are larger than 1 cm in MED BOARDS; 2015
carcinoma. Breast conservation surgery was done. size, and node-negative cancers larger than TOPNOTCH MD)
Lymph nodes were negative for cancer cells. ER 0.5 cm in size when adverse prognostic
and PR tests were also negative. What is the features are present.
appropriate next step?
A. Hormone therapy
B. Adjuvant chemotherapy
C. Combined hormone therapy and
chemotherapy
D. Prophylactic BCS of the left breast.
E. No more intervention is warranted. Careful
follow-up is advised.
205 The Sistrunk procedure is performed for: The Sistrunk procedure consist of en bloc GRACE ARVIOLA, MD DIAGNOSTIC
A. Mandibular masses cystectomy and excision of the central hyoid (TOP 3 - AUG 2014 EXAM - FEB
B. Laryngeal cancer bone to minimize recurrence. MED BOARDS; 2015
C. Thyroglossal duct cysts TOPNOTCH MD)
D. Cholesteatoma
E. Nasal polyps

206 In a patient with severe aortic stenosis, the The Ross procedure may be indicated for GRACE ARVIOLA, MD DIAGNOSTIC
patient's native pulmonary valve is used to replace younger patients who require aortic valve (TOP 3 - AUG 2014 EXAM - FEB
the stenotic aorta and a homograft is used in place replacemnt and want to avoid the need for MED BOARDS; 2015
of the pulmonary valve. The procedure described anticoagulation. TOPNOTCH MD)
is:
A. Norwood procedure
B. Ross procedure
C. Fontan procedure
D. Blalock-Taussig procedure
E. Brock's procedure
207 The triple layer descibed by Bassini in his method Bassini repair involves reconstruction of the GRACE ARVIOLA, MD DIAGNOSTIC
of inguinal hernia repair consist of: posterior wall by suturing this triple layer (TOP 3 - AUG 2014 EXAM - FEB
A. Skin, external oblique, internal oblique, medially to the inguinal ligament laterally and MED BOARDS; 2015
B. External oblique, internal oblique, rectus possibly the iliopubic tract. TOPNOTCH MD)
abdominis
C. Skin, rectus abdominis, external oblique
D. Rectus abdominis, internal oblique,
transversus abdominis
E. Internal oblique, transversus abdominis,
transversalis fascia
208 The colon of a 55 year-old male was inadvertently The sigmoid colon is least to perforate but most GRACE ARVIOLA, MD DIAGNOSTIC
perforated during a colonoscopy procedure. What likely to undergo volvulus. The cecum is the part (TOP 3 - AUG 2014 EXAM - FEB
segment is LEAST likely to be involved? of the colon most likely to perforate due to its MED BOARDS; 2015
A. Cecum thinner wall. TOPNOTCH MD)
B. Ascending colon
C. Transverse colon
D. Descending colon
E. Sigmoid colon

209 During work-up of a 40 year-old female patient The size of the adrenal mass on imaging studies GRACE ARVIOLA, MD DIAGNOSTIC
presenting with virilizing features, an adrenal is the sinbgle most important criterion to help (TOP 3 - AUG 2014 EXAM - FEB
mass was identified by imaging studies. What is diagnose malignancy. MED BOARDS; 2015
the single most important criterion to diagnose TOPNOTCH MD)
carcinoma in this patient?
A. Irregular margins
B. Size of 8 cm
C. Heteregoneity
D. Adjacent lympadenopathy
E. Presence of hemorrhage within the tumor

TOPNOTCH MEDICAL BOARD PREP SURGERY SUPEREXAM Page 28 of 94


For inquiries visit www.topnotchboardprep.com.ph or email us at topnotchmedicalboardprep@gmail.com
TOPNOTCH MEDICAL BOARD PREP SURGERY SUPEREXAM
For inquiries visit www.topnotchboardprep.com.ph or email us at topnotchmedicalboardprep@gmail.com
Item QUESTION EXPLANATION AUTHOR TOPNOTCH
# EXAM
210 Which statement is NOT correct regarding HPS classically present between 3 and 6 weeks GRACE ARVIOLA, MD DIAGNOSTIC
hypertrophic pyloric stenosis? of age. (TOP 3 - AUG 2014 EXAM - FEB
A. HPS is never a surgical emergency. MED BOARDS; 2015
B. It can be accurately diagnosed by ultrasound. TOPNOTCH MD)
C. Affected patients present with non-bilious
vomiting.
D. It presents early in the neonatal period; 95%
present within one week of life.
E. It commonly affects first-born males.
211 A suicidal patient drank a bottle of Liquid Sosa In caustic injury, the midportion of the GRACE ARVIOLA, MD DIAGNOSTIC
after having failed the physician licensure exam. esophagus is most likely affected. This segment (TOP 3 - AUG 2014 EXAM - FEB
You suspect esophageal perforation. As the is best approached via a right thoracotomy. MED BOARDS; 2015
surgeon on duty, you would approach this patient TOPNOTCH MD)
by a:
A. Right thoracotomy
B. Left thoracotomy
C. Median sternotomy
D. Median sternotomy and right thoracotomy
E. Median sternotomy and left thoracotomy
212 A 65 year-old hypertensive male has developed Abdominal pain for which the severity is out of GRACE ARVIOLA, MD DIAGNOSTIC
“food fear” over the last three months. According proportion to the degree of tenderness on (TOP 3 - AUG 2014 EXAM - FEB
to his daughter, he has also lost considerable examination is the hallmark of acute mesenteric MED BOARDS; 2015
amount of weight. No changes in bowel movement ischemia. TOPNOTCH MD)
were noted. The patient is hysterically screaming
in the emergency room because of his diffuse
abdominal pain. However, on physical
examination, you only noted minimal tenderness
in the epigastric area. At this point, you strongly
suspect:
A. Perforated peptic ulcer
B. Acute cholecystitis
C. Acute pancreatitis
D. Diverticulitis
E. Acute mesenteric ischemia
213 Surgical wound infections usually become GRACE ARVIOLA, MD DIAGNOSTIC
manifest ________ after surgery. (TOP 3 - AUG 2014 EXAM - FEB
A. 1-3 days MED BOARDS; 2015
B. 3-5 days TOPNOTCH MD)
C. 5-7 days
D. 7-10 days
E. 10-14 days

214 Lobular carcinoma in situ of the breast is best Because LCIS is considered a marker for GRACE ARVIOLA, MD DIAGNOSTIC
managed by: increased risk rather than an inevitable (TOP 3 - AUG 2014 EXAM - FEB
A. Breast conservation surgery precursor of invasive disease, the current MED BOARDS; 2015
B. MRM treatment of LCIS is observation with or without TOPNOTCH MD)
C. Radiotherapy tamoxifen.
D. Chemotherapy
E. Observation with or without tamoxifen

215 During thyroidectomy, a nerve was accidentally GRACE ARVIOLA, MD DIAGNOSTIC


severed that lies in close proximity to the inferior (TOP 3 - AUG 2014 EXAM - FEB
thyroid artery. This nerve is: MED BOARDS; 2015
A. Superior laryngeal nerve TOPNOTCH MD)
B. Internal laryngeal nerve
C. Recurrent laryngeal nerve
D. External laryngeal nerve
E. Galla Curci nerve

216 In adults, Meckel's diverticulum usually presents In children, bleeding is the most common GRACE ARVIOLA, MD DIAGNOSTIC
as: symptom. (TOP 3 - AUG 2014 EXAM - FEB
A. Bleeding MED BOARDS; 2015
B. Obstruction TOPNOTCH MD)
C. Abdominal mass
D. Post-prandial vomiting
E. Jaundice

217 What is the oncogenic risk of a villous adenoma of Among all colonic adenomas, the villous type GRACE ARVIOLA, MD DIAGNOSTIC
the colon? carries the highest risk of malignancy, at 40%. In (TOP 3 - AUG 2014 EXAM - FEB
A. 10% contrast, tubular adenomas carry only a 5% risk. MED BOARDS; 2015
B. 20% TOPNOTCH MD)
C. 30%
D. 40%
E. 50%

218 A 58 year-old male patient has a 5 cm carcinoma Miles procedure or abdominoperineal resection GRACE ARVIOLA, MD DIAGNOSTIC
of the anus with extension into the distal rectum. (APR) involves removal of the entire rectum, (TOP 3 - AUG 2014 EXAM - FEB
What procedure is most appropriate? anal canal, and anus with construction of a MED BOARDS; 2015
A. Miles procedure permanent colostomy from the descending or TOPNOTCH MD)
B. Low anterior resection sigmoid colon.
C. Extended low anterior resection
D. High anterior resection
E. Hartmann's procedure

TOPNOTCH MEDICAL BOARD PREP SURGERY SUPEREXAM Page 29 of 94


For inquiries visit www.topnotchboardprep.com.ph or email us at topnotchmedicalboardprep@gmail.com
TOPNOTCH MEDICAL BOARD PREP SURGERY SUPEREXAM
For inquiries visit www.topnotchboardprep.com.ph or email us at topnotchmedicalboardprep@gmail.com
Item QUESTION EXPLANATION AUTHOR TOPNOTCH
# EXAM
219 Retraction is less problematic in a colostomy than Parastomal hernia is the most common late GRACE ARVIOLA, MD DIAGNOSTIC
an ileostomy because: complication of a colostomy. (TOP 3 - AUG 2014 EXAM - FEB
A. A colostomy has a larger lumen and thicker MED BOARDS; 2015
wall. TOPNOTCH MD)
B. A colostomy is relatively stronger than an
ileostomy.
C. Stool is less irritating to skin than small
intestinal fluids.
D. A colostomy is easier to clean than an
ileostomy.
E. Movement of abdominal wall muscles does not
affect the colostomy.
220 A 45 year-old patient presents with a 2 cm ill- Basal cell carcinoma is the most common type of GRACE ARVIOLA, MD DIAGNOSTIC
defined mass on his right ear that has been skin cancer. It is usually slow growing. (TOP 3 - AUG 2014 EXAM - FEB
present for two years. The mass is cream-colored Metastasis and death are extremely rare. Choice MED BOARDS; 2015
with pearly borders and visible small blood D is melanoma; choice E is squamous cell TOPNOTCH MD)
vessels. Which statement is true regarding this carcinoma.
mass?
A. Lymph node metastasis is usually present at
the time of diagnosis.
B. It has an excellent prognosis.
C. This is best treated by wide radical resection.
D. This type of skin cancer has both a radial and a
vertical growth phase.
E. Actinic keratosis is a precursor of this
condition.
221 A 34 y/o male was brought to ER d/t MVA. BP immediate surgical intervention is necessary in LEAN ANGELO MIDTERM
130/90 PR 75 RR 10 , GCS 10 ( E2V4M4), there are acute subdural hematoma when the ff CT SILVERIO, MD (TOP 4 - EXAM 1 - FEB
multiple fractures on bilateral extremities. Cranial findings are noted: thickness >1cm, midline shift AUG 2014 MED 2015
CT revealed 1.4 cm crescent shaped mixed >5mm, or GCS drop by 2 or more points during BOARDS; TOPNOTCH
densities noted on bilateral cerebral convexity. the hospital stay. Burrholing or EVD is not ideal MD), MD
which of the following is the procedure of choice since in acute hematoma, the blood is thick and
to be done at this point ? congealed already unlike in chronic ones. the
A. perform ER external ventricular drain question is what is the best intervention.
insertion however, it should be noted that medical
B. open craniotomy decompression should be done at all points of
C. medical decompression with hypertonic saline therapeutic planning.
D. emergency third ventriculostomy
222 A 56 y/o male was brought to ER d/t sudden onset class I - asymptomatic, or mild H/A and slight LEAN ANGELO MIDTERM
of severe headache, during the ER stay, he become nuchal rigidity; II- Cr N palsy, moderate to SILVERIO, MD (TOP 4 - EXAM 1 - FEB
lethargic. PE findings revealed (+) kernig sign., severe headachr, nuchal rigidity; III- mild focal AUG 2014 MED 2015
there is also bilateral CN VI palsy. MMT (L) 3/5; ® deficit, lethargy or confusion; IV- stuporous, BOARDS; TOPNOTCH
5/5. what is the hunt and hess classification of this moderate to severe hemiparesis, early MD), MD
patient ? decerebrate rigidity, 5- deep coma, decerebrate
A. Class 1 rigidity, moribound appearance.
B. Class 2
C. Class 3
D. Class 4
E. Class 5
223 Which of the following is the most effective based on statistics, intravesical BCG is the most LEAN ANGELO MIDTERM
adjuvant therapy in preventing recurrence of effective agent. It is usually given in six weeks SILVERIO, MD (TOP 4 - EXAM 1 - FEB
bladder cancer in situ after transurethral resection instillation, with a repeat course if a complete AUG 2014 MED 2015
? response is not attained. BOARDS; TOPNOTCH
A. Alpha -interferon MD), MD
B. Bacille calmette Guerin
C. Doxorubicin
D. Mitomycin C instillation
E. any of the above
224 what is the standard treatment for a patient with the standard treatment for distant metastatic LEAN ANGELO MIDTERM
prostatic carcinoma with multiple retroperitoneal disease is androgen ablation therapy to lower SILVERIO, MD (TOP 4 - EXAM 1 - FEB
lymph node and documented vertebral serum testosterone. This is achieved by AUG 2014 MED 2015
metastasis? bilateral scrotal orchiectomy and/or BOARDS; TOPNOTCH
A. Radical prostatectomy with adjuvant administration of LHRH agonist. MD), MD
chemotherapy
B. Bilateral scrotal orchiectomy with Leuprolide
C. neoadjuvant chemotherapy with radical
prostatectomy and radiotherapy
D. palliative therapy
E. none of the above
225 which of the following is not part of cardinal signs kanavel signs indicte suppurative tenosynovitis LEAN ANGELO MIDTERM
of Kanavel tenosynovitis ? of the flexor tendon sheath. Clinical presentation SILVERIO, MD (TOP 4 - EXAM 1 - FEB
A. Pain on passive flexion include fingers held in flexion, fusiform digital AUG 2014 MED 2015
B. Fusiform swelling of the fingers swelling, tenderness of tendon sheath, pain on BOARDS; TOPNOTCH
C. Tenderness along the tendon sheath passive extension MD), MD
D. None of the above

226 what is the most common soft tissue sarcoma in LEAN ANGELO MIDTERM
the adults? SILVERIO, MD (TOP 4 - EXAM 1 - FEB
A. Fibrosarcoma AUG 2014 MED 2015
B. Rhabdomyosarcoma BOARDS; TOPNOTCH
C. Liposarcoma MD), MD
D. lymphangiosarcoma
E. Desmoid tumors

TOPNOTCH MEDICAL BOARD PREP SURGERY SUPEREXAM Page 30 of 94


For inquiries visit www.topnotchboardprep.com.ph or email us at topnotchmedicalboardprep@gmail.com
TOPNOTCH MEDICAL BOARD PREP SURGERY SUPEREXAM
For inquiries visit www.topnotchboardprep.com.ph or email us at topnotchmedicalboardprep@gmail.com
Item QUESTION EXPLANATION AUTHOR TOPNOTCH
# EXAM
227 Which of the following is true regarding Hurthle Hurthle cell cancer is a type of follicular cancer, LEAN ANGELO MIDTERM
cell carcinoma of the thyroid?? but it tends to recur more often than the other SILVERIO, MD (TOP 4 - EXAM 1 - FEB
A. It is a form of anaplastic thyroid cancer types. It spreads hematogenously to distant AUG 2014 MED 2015
B. It metastasizes via the lymphatics to regional sites. Amyloid deposits in the stroma of thyroid BOARDS; TOPNOTCH
lymph node tumor are diagnostic of medullary carcinoma. MD), MD
C. Treatment consists of near-total or total The treatment of choice is near total
thyroidectomy thyroidectomy to facilitate later body scanning
D. Microscopically, it consists of clusters of cells for metastasis and treatement with RAI.
separated by areas of collagen and amyloid
E. all of the above
228 What is the most serious complication of an end The most frequent serious complication of end LEAN ANGELO MIDTERM
colostomy?? colostomies is parastomal herniation, which SILVERIO, MD (TOP 4 - EXAM 1 - FEB
A. Bleeding commonly occurs when the stoma is placed AUG 2014 MED 2015
B. Skin breakdown lateral to, rather than through, the rectus muscle BOARDS; TOPNOTCH
C. Parastomal hernia MD), MD
D. Colonic perforation during irrigation
E. None of the above

229 A 30 y/o male with a gunshot wound came to the The patient presents with class III hemorrhage LEAN ANGELO MIDTERM
emergency room. Patient was confused with BP of since the patient is confused, hypotensive and SILVERIO, MD (TOP 4 - EXAM 1 - FEB
80/60, HR 140. Based on the physical finding, how tachycardic. Class III hemorrhage results from AUG 2014 MED 2015
much is the estimated blood loss of the patient? blood loss of 1500-2000 ml or 30-40% blood BOARDS; TOPNOTCH
A. <750 ml loss. MD), MD
B. 750-1500ml
C. 1500-2000 ml
D. >2000ml
E. none of the above

230 All of the following are true about branchial cleft The 1st branchial cleft cyst and sinuses are LEAN ANGELO MIDTERM
cyst, except? associated intimately with the external acoustic SILVERIO, MD (TOP 4 - EXAM 1 - FEB
A. It is derived from the branchial cleft apparatus canal and parotid gland. AUG 2014 MED 2015
that persists after fetal development BOARDS; TOPNOTCH
B. 1st branchial cleft cyst and sinuses are MD), MD
associated intimately with the internal acoustic
meatus and the parotid gland
C. The 2nd branchial cleft cyst is found along the
anterior border of SCM muscle
D. The 3rd branchial cleft remnant courses
posterior to the common carotid artery, ending in
the pyriform sinus region
E. none of the above
231 9. A 50 y/o patient came to your clinic presenting This result describes hypertensive lower LEAN ANGELO MIDTERM
with dysphagia. Manometric study was done, esophageal sphincter. Achalasia would result to SILVERIO, MD (TOP 4 - EXAM 1 - FEB
which revealed an elevated LES pressure, normal incomplete lower esophageal sphincter AUG 2014 MED 2015
LES relaxation, and normal peristalsis in the relaxation, aperistasis in the esophageal body, BOARDS; TOPNOTCH
esophageal body. What is your initial diagnosis? elevated LES pressure, and increased MD), MD
A. Hypertensive lower esophageal sphincter intraesophageal baseline pressures relative to
B. Achalasia gastric baselin. Diffuse esophageal spasm would
C. Diffuse esophageal spasm result to simultaneous nonperistaltic
D. Nutcracker esophagus contraction, repetitive and multipeaked
E. none of the above contractions, spontaneous contractions,
intermittent normal peristalsis. Nutcracker
esophagus has mean peristaltic amplitude in the
distal esophagus greater than or equal to 180,
increased mean duration of contraction, and
normal peristaltic sequence.
232 True about carcinoid tumor except? Carcinoid tumors are usually located at the tip of LEAN ANGELO MIDTERM
A. It is a firm, yellow, bulbar mass, most the appendix. For tumors larger than 1-2 cm SILVERIO, MD (TOP 4 - EXAM 1 - FEB
commonly seen in the appendix located at the base or with lymph node AUG 2014 MED 2015
B. Most are located at the base of the appendix metastases, right hemicolectomy is indicated. BOARDS; TOPNOTCH
C. Tumor size of <1cm rarely result in extension MD), MD
outside the appendix or adjacent to the mass,
therefore treatment is only appendectomy
D. A and B
E. B and C
233 A 69 y/o male presents with a history of suddent preoperative CT findings that determine LEAN ANGELO MIDTERM
onset of obstructive jaundice accompanied by unresectability of a pancreatic head mass SILVERIO, MD (TOP 4 - EXAM 1 - FEB
recurrent abdominal pain and involuntary weight include encasement of the SMA, celiac axis and AUG 2014 MED 2015
loss of 10%. CT scan was performed which reveals occlusion of the superior mesenteric vein or BOARDS; TOPNOTCH
3.9 cm mixed hypodensity lesion on the head of portal vein. Loss of plane b/w tumor and the MD), MD
the pancreas. percutaneous biopsy was done vessels is not a contraindication for curative
revealing irregular, anaplastic, highly mitotic cells resection.
with glandular features. which of the following CT
findings will preclude curative resection via
whipples procedure?
A. encasement of gastroduodenal artery
B. lack of pulsation of the superior mesenteric
artery
C. encroachment of the plane between the tumor
and the portal vein
D. multiple matted lymphadenopathies in the
celiac axis
E. malignant refractory ascites.

TOPNOTCH MEDICAL BOARD PREP SURGERY SUPEREXAM Page 31 of 94


For inquiries visit www.topnotchboardprep.com.ph or email us at topnotchmedicalboardprep@gmail.com
TOPNOTCH MEDICAL BOARD PREP SURGERY SUPEREXAM
For inquiries visit www.topnotchboardprep.com.ph or email us at topnotchmedicalboardprep@gmail.com
Item QUESTION EXPLANATION AUTHOR TOPNOTCH
# EXAM
234 89 year old went for consult secondary to this is a classic case of Upper esophageal LEAN ANGELO MIDTERM
dysphagia. He already had significant weight loss diverticulum aka zenkers diverticulum. The best SILVERIO, MD (TOP 4 - EXAM 1 - FEB
because of food aversion secondary persistent approach is to perform barium swallow for AUG 2014 MED 2015
reflux of swallowed food. PE of the neck shows preop work up before going to surgical BOARDS; TOPNOTCH
bulging, soft, nontender mass on the right anterior correction. Endoscopy is a relative MD), MD
cervical triangle. what is the best approach for this contraindication because of the risk for rupture.
case ? PPI will give only slight therapeutic benefit. CT
A. endoscopy scan is not necessary since barium swallow will
B. barium swallow suffice.
C. CT scan of the head and neck
D. therapeutic trial of PPI
E. perform cricopharyngeal myotomy
235 A 48 y/o male presented to your clinic with Type I - fusiform dilatation of the CBD - LEAN ANGELO MIDTERM
obstructive form of jaundice. You opted to request cholecystectomy, cyst excison and a roux en Y SILVERIO, MD (TOP 4 - EXAM 1 - FEB
for MRCP which reveal a case of type III choledochojejunostomy. Type II -diverticulum of AUG 2014 MED 2015
choledochal cyst. What is the standard treatment CBD - excision. Type III- choledochocele of the BOARDS; TOPNOTCH
for this case? intraduodenal CBD - sphincteroplasty or MD), MD
A. Cholecystectomy with cyst excision choledochoduodenostomy. Type 4- caroli's
B. transduodenal sphincteroplasty disease, cystic dilatations of intrahepatic duct, -
C. liver transplantation liver transplantation.
D. roux en y choledochojejunostomy
E. all of the above
236 A 78 y/o male was rushed to ER d/t severe back the most common surgical complication for LEAN ANGELO MIDTERM
pain associated pulsatile tender abdominal mass emergency repair of AAA is renal failure SILVERIO, MD (TOP 4 - EXAM 1 - FEB
at the periumbilical area.he was accounting about 20-25% of all cases. Ischemic AUG 2014 MED 2015
hypotensive,tachycardic and had gradual colitis usually involves the sigmoid colon. This is BOARDS; TOPNOTCH
deterioration of consciousness. he was secondary to the ligation of IMA during surgery. MD), MD
immediately brought to OR for exploratory Anterior spinal artery syndrome is secondary to
laparotomy. Which of the following is the most occlusion of the artery of adamkiewicz. acute leg
common surgical complication for this case ? ischemia is d/t clamp injury of the iliac arteries
A. renal failure
B. ischemic colitis of the ascending colon
C. anterior spinal artery syndrome
D. acute leg ischemia
E. infection
237 Which of the following is true about Paget’s Paget’s disease of the nipple is usually LEAN ANGELO MIDTERM
disease of the breast? associated with extensive DCIS and is associated SILVERIO, MD (TOP 4 - EXAM 1 - FEB
A. It presents as chronic, eczematous eruption of with invasive cancer. Pathognomonic of this AUG 2014 MED 2015
the nipple, but may progress to an ulcerated, condition is the presence of large, pale, BOARDS; TOPNOTCH
weeping lesion vacuolated cells (Paget cells) in the rete pegs of MD), MD
B. It is usually associated with extensive LCIS and the epithelium
is not associated with invasive cancer
C. Pathognomonic of this condition is the
presence of small, dark vacuolated cells in the rete
pegs of epithelium
D. all of the above
E. none of the above
238 A 26 y/o male complains of cough, upperback pain the most common posterior mediastinum mass LEAN ANGELO MIDTERM
accompanied by dysphagia. Chest xray revealed 5 is a neurogenic tumor. SILVERIO, MD (TOP 4 - EXAM 1 - FEB
cm mass located at the retrocardiac area. what is AUG 2014 MED 2015
the most likely diagnosis for this mass ? BOARDS; TOPNOTCH
A. Ectopic thymoma MD), MD
B. Neurogenic tumor
C. myxoma
D. bronchogenic carcinoma
E. none of the above

239 Which of the following is not part of the pentalogy omphalocoele and not gastroschisis is a LEAN ANGELO MIDTERM
of Cantrell? component of cantrell pentalogy. Associated SILVERIO, MD (TOP 4 - EXAM 1 - FEB
A. Gastroschisis anomalies are rare in gastroschisis. AUG 2014 MED 2015
B. Diaphragmatic hernia BOARDS; TOPNOTCH
C. Bifid sternum MD), MD
D. Absent pericardium
E. VSD

240 A 43 y/o male was rushed to the ER secondary to type 1 -m/c type near angularis incisura, acid LEAN ANGELO MIDTERM
severe epigastric pain accompanied by black tarry hyposecretion. Type 2- same as type 1 but with SILVERIO, MD (TOP 4 - EXAM 1 - FEB
stools. After stabilization with PPI. He underwent associated active or quiescent duodenal ulcer, AUG 2014 MED 2015
upper GI endoscopy and eventually classified as acid hypersecretion. Type 4- near GE junction, BOARDS; TOPNOTCH
type 3 ulcer under modified Johnson classification. acid hyposecretion. Type 5 -NSAID induced, MD), MD
where is the most probable ulcer site and what is nonspecific site.
its correlated acid secretion status ?
A. near GE junction, acid hypersecretion
B. near GE junction, acid hyposecretion
C. prepyloric area, acid hyposecretion
D. prepyloric area, acid hypersecretion
E. none of the above

TOPNOTCH MEDICAL BOARD PREP SURGERY SUPEREXAM Page 32 of 94


For inquiries visit www.topnotchboardprep.com.ph or email us at topnotchmedicalboardprep@gmail.com
TOPNOTCH MEDICAL BOARD PREP SURGERY SUPEREXAM
For inquiries visit www.topnotchboardprep.com.ph or email us at topnotchmedicalboardprep@gmail.com
Item QUESTION EXPLANATION AUTHOR TOPNOTCH
# EXAM
241 2 day old infant was brought to the emergency SIMILAR TO PREVIOUS BOARD EXAM KEVIN BRYAN LO, MD MIDTERM 2
room, infant has been noted to be feeding well but CONCEPT/PRINCIPLE, initial non invasive test (TOP 7 - AUG 2014 EXAM - FEB
would always vomit a few minutes after for hypertrophic pyloric stenosis, abdominal MED BOARDS; 2015
breastfeeding. Patient noted to be slightly ultrasound can be used TOPNOTCH MD)
dehydrated, irritable and crying. Olive shaped
mass was palpated by the physician at the ER,
which of the following diagnostic tests would be a
good initial test to request?
A. barium swallow
B. barium enema
C. abdominal ultrasound
D. scout film of the abdomen
E. upper GI endoscopy under sedation
242 50 year old patient chronically constipated SIMILAR TO PREVIOUS BOARD EXAM KEVIN BRYAN LO, MD MIDTERM 2
presents with 2 episodes of bright red bleeding CONCEPT/PRINCIPLE, hemorrhoidal cushions (TOP 7 - AUG 2014 EXAM - FEB
amount to 1 cup, this was present during episodes left lateral and right anterior and posterior MED BOARDS; 2015
of bowel movements, the doctor performs digital lateral TOPNOTCH MD)
rectal examination, all of the following are the
locations of the hemorrhoidal cushions EXCEPT
A. right posterolateral
B. left lateral
C. left posterolateral
D. right anterolateral
E. none (all are hemorrhoidal cushions)
243 A 50 year old patient complaints of anal pain, SIMILAR TO PREVIOUS BOARD EXAM KEVIN BRYAN LO, MD MIDTERM 2
occasional intermittent fever, purulent discharge CONCEPT/PRINCIPLE, goodsall's rule >3cm (TOP 7 - AUG 2014 EXAM - FEB
per anus and draining purulent sinus that was anterior, and posterior openings track MED BOARDS; 2015
found to be 5cm from the anal margin to the right curvilinear and eventually end up in the TOPNOTCH MD)
anteriorly, the doctor knows that the draining posterior midline
sinus track will eventually end at which position?
A. posterior right lateral
B. posterior left lateral
C. anterior midline
D. posterior midline
E. anterior right lateral
244 A 35 year old male comes in the out patient SIMILAR TO PREVIOUS BOARD EXAM KEVIN BRYAN LO, MD MIDTERM 2
department with complains of a 3 year history of a CONCEPT/PRINCIPLE, hallmark given at that (TOP 7 - AUG 2014 EXAM - FEB
back mass on the upper back region, he noted time was central punctum sebaceous cyst MED BOARDS; 2015
gradual changes in its size sometimes increasing TOPNOTCH MD)
sometimes shrinking in size with occasional pain
and drainage of foul smelling small amount of
yellowish discharge, the physician examines it
closely and notes a central punctum, which of the
following is the most likely diagnosis?
A. sebaceous cyst
B. epidermal inclusion cyst
C. ganglion cyst
D. trichilemmal cyst
E. dermoid cyst
245 50 year old female accidentally slipped but as she SIMILAR TO PREVIOUS BOARD EXAM KEVIN BRYAN LO, MD MIDTERM 2
fell she used her right outstretched hand to break CONCEPT/PRINCIPLE, almost the exact case (TOP 7 - AUG 2014 EXAM - FEB
her fall as she fell in the forward position, soon outstretched hand, I was looking for colles MED BOARDS; 2015
after she developed pain near the anterior fracture but there was none, clavicular was the TOPNOTCH MD)
shoulder area, which of the following injuries was next best answer
most likely sustained?
A. rotator cuff tear
B. clavicular fracture
C. fracture of the midshaft of humerus
D. fracture dislocation of the elbow
E. supraspinatus tear
246 In relation to the above question, initial SIMILAR TO PREVIOUS BOARD EXAM KEVIN BRYAN LO, MD MIDTERM 2
diagnostics revealed a linear nondisplaced CONCEPT/PRINCIPLE, nondisplaced linear (TOP 7 - AUG 2014 EXAM - FEB
fracture of the site of pathology, what is the most fracture of clavicle armsling only MED BOARDS; 2015
appropriate management of this case? TOPNOTCH MD)
A. open reduction external fixation
B. closed reduction
C. open reduction with intramedullary nailing
D. arm sling application
E. long arm cast

TOPNOTCH MEDICAL BOARD PREP SURGERY SUPEREXAM Page 33 of 94


For inquiries visit www.topnotchboardprep.com.ph or email us at topnotchmedicalboardprep@gmail.com
TOPNOTCH MEDICAL BOARD PREP SURGERY SUPEREXAM
For inquiries visit www.topnotchboardprep.com.ph or email us at topnotchmedicalboardprep@gmail.com
Item QUESTION EXPLANATION AUTHOR TOPNOTCH
# EXAM
247 A 55 year old male patient with 5 year history of SIMILAR TO PREVIOUS BOARD EXAM KEVIN BRYAN LO, MD MIDTERM 2
fairly controlled hypertension and diabetes and CONCEPT/PRINCIPLE, in SBO, scout film shows (TOP 7 - AUG 2014 EXAM - FEB
appendectomy 20 years ago came in with a chief multiple air fluid levels MED BOARDS; 2015
complaint of abdominal pain characterized as TOPNOTCH MD)
generalized, intermittent accompanied by
episodes of vomiting. There was also anorexia,
passage of flatus, last bowel movement was noted
to be 3 days ago, focused PE revealed (+) mild
abdominal tenderness near the umbilical
epigastric areas, (-) rebound tenderness, (-) pain
blood on DRE, a simple abdominal xray supine and
upright was seen, which of the following are the
most likely findings?
A. kidney shaped mass over the right upper
quadrant
B. bean shaped mass over the left upper quadrant
C. multiple air fluid levels in stepladder
appearance
D. sentinel loop sign
E. hyperlucency under the right hemidiaphragm
248 A 30 year old male was involved in a gun shot SIMILAR TO PREVIOUS BOARD EXAM KEVIN BRYAN LO, MD MIDTERM 2
incident, patient was rushed to the ER, BP was CONCEPT/PRINCIPLE, tension pneumothorax (TOP 7 - AUG 2014 EXAM - FEB
80/50, pulse was 135, RR was 33, patient noted to warrants immediate needle decompression MED BOARDS; 2015
be agitated, noted sucking chest wound on the TOPNOTCH MD)
right, decreased breath sounds on the right,
tracheal deviation to the left, hyperresonant over
the right hemithorax, which of the following is the
best course of action?
A. insert a chest tube over the right hemithorax
B. do needle decompression over the right
hemithorax
C. start fluid resuscitation 2 large bore IV cannulas
D. intubate and hook to mechanical ventilation
E. refer for emergency pericardiostomy
249 A 40 year old male smoker comes in with a 3 year SIMILAR TO PREVIOUS BOARD EXAM KEVIN BRYAN LO, MD MIDTERM 2
history of a gradually enlarging right mass located CONCEPT/PRINCIPLE, parotid mass - exision (TOP 7 - AUG 2014 EXAM - FEB
near at the anterior inferior auricular area, patient biopsy, parotid pancreatic and hepatic masses MED BOARDS; 2015
comes in for consult due to persistence of mass, no usually does not warrant needle biopsy TOPNOTCH MD)
other symptoms was noted which of the following
is the best approach for diagnosis?
A. core needle aspiration biopsy
B. fine needle aspiration biopsy
C. incisional biopsy
D. excision biopsy
E. Ultrasound
250 In relation to the above question, histopath SIMILAR TO PREVIOUS BOARD EXAM KEVIN BRYAN LO, MD MIDTERM 2
examination of the mass would most likely reveal CONCEPT/PRINCIPLE, benign features, slow (TOP 7 - AUG 2014 EXAM - FEB
which of the following pathologies? growing mass, no other symptoms parotid mass MED BOARDS; 2015
A. mucoepidermoid carcinoma - pleomorphic adenoma TOPNOTCH MD)
B. plemorphic adenoma
C. adenoid cystic carcinoma
D. warthin's tumor
E. malignant lymphoma

251 7 year old boy came to the ER with abdominal SIMILAR TO PREVIOUS BOARD EXAM KEVIN BRYAN LO, MD MIDTERM 2
pain, abdominal pain has been on for a day CONCEPT/PRINCIPLE, very difficult because it (TOP 7 - AUG 2014 EXAM - FEB
characterized as generalized. Patient had history could go anywhere as the case evolves, but best MED BOARDS; 2015
also of an upper respiratory tract infection and answer as of now is acute mesenteric TOPNOTCH MD)
vomiting diarrhea 3 days prior to the onset of lymphadenitis by history and PE
abdominal pain. On physical exam, mild cervical
lympadenoapthy, there was direct tenderness on
the umbilical and right lower quadrant areas with
minimal rebound tenderness, there was
abdominal guarding, DRE minimal pain or
discomfort no blood, CBC revealed WBCs elevated
at 14000, patient was slightly febrile at 37.8
degrees celsius, urinalysis RBC 0-1 WBC 0-1
bacterioa +2 Epithelial cells +3, no casts or
crystals, what is the most likely initial
impression?
A. acute appendicitis
B. acute mesenteric ischemia
C. acute mesenteric lymphadenitis
D. acute pancreatitis
E. acute gastroenteritis
252 Compute for the caloric intake if a patient is placed SIMILAR TO PREVIOUS BOARD EXAM KEVIN BRYAN LO, MD MIDTERM 2
on NPO and a total of 3L of D5W has been infused? CONCEPT/PRINCIPLE., D5W contains 50g x3L = (TOP 7 - AUG 2014 EXAM - FEB
A. 600 150g glucose x 4 kcal/g = 600 MED BOARDS; 2015
B. 800 TOPNOTCH MD)
C. 1000
D. 1200
E. 1500

TOPNOTCH MEDICAL BOARD PREP SURGERY SUPEREXAM Page 34 of 94


For inquiries visit www.topnotchboardprep.com.ph or email us at topnotchmedicalboardprep@gmail.com
TOPNOTCH MEDICAL BOARD PREP SURGERY SUPEREXAM
For inquiries visit www.topnotchboardprep.com.ph or email us at topnotchmedicalboardprep@gmail.com
Item QUESTION EXPLANATION AUTHOR TOPNOTCH
# EXAM
253 A 70kg 50 year old male patient sustained the SIMILAR TO PREVIOUS BOARD EXAM KEVIN BRYAN LO, MD MIDTERM 2
following burns as he was escaping from a burning CONCEPT/PRINCIPLE. asked us to compute, (TOP 7 - AUG 2014 EXAM - FEB
building, partial thickness burns over the lower whole left leg = 18, lower leg right anterior 4.5, MED BOARDS; 2015
anterior right leg, full circumferential burns of the anterior half trunk 9, posterior half trunk 9 = TOPNOTCH MD)
left leg, partial thickness burns over the anterior 40.5 x 70kg x4mL = 11340/2 =5670
half of the trunk and posterior lower half of the
back, how many mls of lactated ringers should be
infused in the first 8 hours following parkland
formula?
A. 4220mL
B. 5670mL
C. 6000mL
D. 7440mL
E. 11340mL
254 A 23 year old male sustained a facial laceration SIMILAR TO PREVIOUS BOARD EXAM KEVIN BRYAN LO, MD MIDTERM 2
over the forehead after a fist fight, which of the CONCEPT/PRINCIPLE., 5-0 or 6-0 smaller suture (TOP 7 - AUG 2014 EXAM - FEB
following will be used for closure of the wound? sizes for the skin, generally should be MED BOARDS; 2015
A. Nylon 4-0 nonabsorbable for trauma wounds TOPNOTCH MD)
B. chromic 2-0
C. Nylon 5-0
D. Silk 2-0
E. Silk-0

255 Based on the question above, the sutures on the SIMILAR TO PREVIOUS BOARD EXAM KEVIN BRYAN LO, MD MIDTERM 2
face are expected to be removed within? CONCEPT/PRINCIPLE, there are increasing (TOP 7 - AUG 2014 EXAM - FEB
A. 2 days number of stem questions, face 3-5 days, scalp MED BOARDS; 2015
B. 5 days 7-10 days, trunk 7-10 days, arms and legs 10-14 TOPNOTCH MD)
C. 7 days days
D. 10 days
E. 14 days

256 Which of the following sutures are considered SIMILAR TO PREVIOUS BOARD EXAM KEVIN BRYAN LO, MD MIDTERM 2
nonabsorbable? CONCEPT/PRINCIPLE a lot of suture questions (TOP 7 - AUG 2014 EXAM - FEB
A. polyglactin please review them, polyglactin or vicryl, MED BOARDS; 2015
B. polydioxone polydioxone or PDS chromic and catgut are TOPNOTCH MD)
C. polypropylene absorbable, polypropylene or prolene,
D. chromic polyamide nylon and silk are non absorbable
E. catgut

257 Compute for the maintenance fluid requirements first 10kg = 100ml/kg = 1000, next 10-20kg = KEVIN BRYAN LO, MD MIDTERM 2
to be given to a 30kg child surgical patient placed 50ml/kg = 500, each kg> 20 20ml/kg = 200 = (TOP 7 - AUG 2014 EXAM - FEB
on NPO 1700 in total MED BOARDS; 2015
A. 1000ml TOPNOTCH MD)
B. 1500ml
C. 1700ml
D. 1800ml
E. 2000ml

258 40 year old female patient underwent review examples of surgery of different classes, KEVIN BRYAN LO, MD MIDTERM 2
cholecystectomy for cholelithiasis, what is the 1D if with prosthesis, breast and hernia 1, (TOP 7 - AUG 2014 EXAM - FEB
class of surgical wound at which this is classified? hollow viscus not colon = 2, contaminated MED BOARDS; 2015
A. clean (class 1) penetrating trauma, enterotomy 3, diverticulitis TOPNOTCH MD)
B. clean/contaminated (class 2) perforated, traumatic, necrotizing wounds = 4
C. contaminated (class 3)
D. dirty (class 4)
E. class 1D

259 A 20 year old male patient was involved in a for blunt trauma, important xrays are the KEVIN BRYAN LO, MD MIDTERM 2
motorcycle accident and has lost consciousness, all followig except scout film of the abdomen, FAST (TOP 7 - AUG 2014 EXAM - FEB
of the following diagnostic procedures are UTZ or CT scan is usually used for abdomen, MED BOARDS; 2015
considered vital EXCEPT? cranial CT scan is important to rule out cranial TOPNOTCH MD)
A. plain cranial CT scan injuires or bleed
B. lateral cervical xray
C. chest xray
D. pelvis A-P xray
E. scout film of the abdomen

260 A 20 year old male patient sustained a lower RBC>100,000, WBC >500, amylase>19, bilirubin KEVIN BRYAN LO, MD MIDTERM 2
abdomina stab wound, a diagnostic peritoneal >0.01 (TOP 7 - AUG 2014 EXAM - FEB
lavage was done, which of the following findings MED BOARDS; 2015
are considered positive which warrants TOPNOTCH MD)
laparoscopy or exploration?
A. RBC count >50,000/ml
B. WBC count >5000/ml
C. amylase >15IU/I
D. alkaline phospatase >2IU/l
E. bilirubin >1mg/dl

TOPNOTCH MEDICAL BOARD PREP SURGERY SUPEREXAM Page 35 of 94


For inquiries visit www.topnotchboardprep.com.ph or email us at topnotchmedicalboardprep@gmail.com
TOPNOTCH MEDICAL BOARD PREP SURGERY SUPEREXAM
For inquiries visit www.topnotchboardprep.com.ph or email us at topnotchmedicalboardprep@gmail.com
Item QUESTION EXPLANATION AUTHOR TOPNOTCH
# EXAM
261 An alcoholic patient presents at the ER due to Gastrografin(water-soluble) swallow is RAYMUND MARTIN LI, MIDTERM 3
severe chest pain, back pain, and dyspnea. His preferred in demonstrating leaks/extravasation MD (TOP 1 - AUG 2014 EXAM - FEB
relatives tell you that he has had several bouts of MED BOARDS; 2015
retching prior to the onset of symptoms. If you TOPNOTCH MD)
suspect this to be a case of esophageal perforation,
what will be your diagnostic method of choice to
demonstrate leak or extravasation?
A. Chest X-ray
B. Gastrografin swallow
C. Endoscopy
D. Barium swallow
E. None of the above
262 A modified radical mastectomy will include: A refers to radical mastectomy. E is simple RAYMUND MARTIN LI, MIDTERM 3
A. Removal of all breast tissue, pectoralis major mastectomy MD (TOP 1 - AUG 2014 EXAM - FEB
and minor, and axillary lymph node dissection MED BOARDS; 2015
B. Removal of all breast tissue, pectoralis fascia, TOPNOTCH MD)
and axillary lymph node dissection of level I and II
C. Removal of all breast tissue, pectoralis minor
muscle, and axillary lymph node dissection of level
I and II
D. Removal of all breast tissue, pectoralis fascia,
and axillary lymph node dissection of level I, II and
III
E. Removal of breast tissue only

263 A 56 year old male patient presents to your clinic Oral cavity - I,II,III; Oropharynx, RAYMUND MARTIN LI, MIDTERM 3
due to weight loss and presence a palpable, Laryngopharynx, Larynx - II, III, IV MD (TOP 1 - AUG 2014 EXAM - FEB
nontender, fixed lymph node located at the MED BOARDS; 2015
posterior triangle of the neck (level V). He has an TOPNOTCH MD)
unremarkable history other than smoking for
approximately 30 years. Biopsy reveals squamous
cell carcinoma. You suspect that this malignancy
most likely originated from what region of the
head and neck?
A. Nasopharynx
B. Oropharynx
C. Laryngopharynx
D. Oral Cavity
E. Larynx
264 Surgical treatment of gastric adenocarcinoma A is D2, B and C is D1, D is D3, E is D4 RAYMUND MARTIN LI, MIDTERM 3
involves gastrectomy and lymphadenectomy. MD (TOP 1 - AUG 2014 EXAM - FEB
Some studies state that adequate MED BOARDS; 2015
lymphadenectomy requires a D2 dissection during TOPNOTCH MD)
gastric resection. Which of the following refers to
the D2 group of lymph nodes?
A. Nodes around the celiac axis, splenic hilum, and
splenic artery
B. Greater curvature perigastric nodes
C. Lesser curvature perigastric nodes
D. Hepatoduodenal and mesenteric root nodes
E. Para-aortic and paracolic nodes
265 A 34 year old patient presents to your clinic due to Completion surgery to remove as much thyroid RAYMUND MARTIN LI, MIDTERM 3
an anterior neck mass. FNAB indicates follicular tissue as possible before radioactive iodine MD (TOP 1 - AUG 2014 EXAM - FEB
type and you suspect follicular adenoma so you MED BOARDS; 2015
perform thyroid lobectomy with isthmusectomy. TOPNOTCH MD)
Post-operative biopsy reveal capsular invasion.
What will be your next step?
A. Do nothing. It is most probably benign.
B. Perform radioactive iodine ablation to destroy
the remaining thyroid tissue
C. Perform completion thyroidectomy
D. Do conservative management and monitor Tg
levels
E. None of the above
266 Which of the following statements is true Asymptomatic hemangioma requires no RAYMUND MARTIN LI, MIDTERM 3
regarding benign liver lesions? treatment; Focal nodular hyperplasia has a MD (TOP 1 - AUG 2014 EXAM - FEB
A. Hemangioma requires excision even when stellate scar and appears hot on scan with no MED BOARDS; 2015
asymptomatic due to high risk of rupture and risk of maliganant transformation. Hepatic TOPNOTCH MD)
bleeding adenoma is associated with OCP use and risk of
B. Focal nodular hyperplasia lacks a central maligant transformation
stellate scar on CT and appears cold on Tc scan
C. Hepatic adenoma is associated with OCP use
and has no risk of malignant degeneration
D. A and B
E. None of the above
267 An anal fissure that is NOT associated with other Any other position is suspicious of an underlying RAYMUND MARTIN LI, MIDTERM 3
conditions such as Crohn's disease or TB is most etiology such as TB or Crohn's MD (TOP 1 - AUG 2014 EXAM - FEB
commonly located in what position? MED BOARDS; 2015
A. anterior midline TOPNOTCH MD)
B. posterior midline
C. lateral position
D. A and C
E. A and B

TOPNOTCH MEDICAL BOARD PREP SURGERY SUPEREXAM Page 36 of 94


For inquiries visit www.topnotchboardprep.com.ph or email us at topnotchmedicalboardprep@gmail.com
TOPNOTCH MEDICAL BOARD PREP SURGERY SUPEREXAM
For inquiries visit www.topnotchboardprep.com.ph or email us at topnotchmedicalboardprep@gmail.com
Item QUESTION EXPLANATION AUTHOR TOPNOTCH
# EXAM
268 You perform suturing on a traumatic laceration 3-5 for face; 5-7 for scalp; 10-14 for trunk and RAYMUND MARTIN LI, MIDTERM 3
located on the patient's right lower leg. You will extremities MD (TOP 1 - AUG 2014 EXAM - FEB
inform the patient to come back to the OPD for MED BOARDS; 2015
suture removal after: TOPNOTCH MD)
A. 3-5 days
B. 5-7 days
C. 7-10 days
D. 10-14 days
E. 1-2 days
269 Liver abscesses may either be bacterial or amebic B and C are more consistent with bacterial RAYMUND MARTIN LI, MIDTERM 3
in etiology. Which of the following supports the abscess MD (TOP 1 - AUG 2014 EXAM - FEB
diagnosis of the latter? MED BOARDS; 2015
A. Single abscess in the right lobe TOPNOTCH MD)
B. Characterized by fever and jaundice
C. Usually caused by an ascending biliary infection
D. B and C only
E. All of the above

270 A 65 kg 32 year old male patient suffers from %TBSA x 4ml x kg; 1/2 in first 8 hours and 1/2 RAYMUND MARTIN LI, MIDTERM 3
extensive flame deep partial thickness burns next 16 hours MD (TOP 1 - AUG 2014 EXAM - FEB
involving the anterior trunk, anterior right and MED BOARDS; 2015
anterior left thigh, perineum, and right forearm. TOPNOTCH MD)
How will you give your fluid resuscitation?
A. 4225 during the first 24 hours
B. 4550 mL during the first 12 hours and 4550 mL
in the next 12 hours
C. 4550 mL during the first 8 hours and 4550 mL
in the next 16 hours
D. 4225 mL during the first 12 hours and 4225 mL
in the next 12 hours
E. 4225 mL during the first 8 hours and 4225 mL
in the next 16 hours
271 You are about to assist on a surgical repair of a McVey repair is usually done. Although femoral RAYMUND MARTIN LI, MIDTERM 3
femoral hernia. In order to impress your hernia is most commonly found in women, the MD (TOP 1 - AUG 2014 EXAM - FEB
consultant, you read up on the topic. Which of the most common type of hernia in women is still MED BOARDS; 2015
following is true of femoral hernia? indirect hernia. Femoral hernia is medial to the TOPNOTCH MD)
A. A Bassini repair is most often used and it femoral vein
involves reapproximation with the Cooper's
ligament
B. The most common type of hernia in women is
femoral hernia
C. Femoral hernias are located lateral to the
femoral vein
D. Management of femoral hernia usually involves
surgery due to high risk of strangulation
E. All are correct
272 An 11 month old infant is brought to the ER by her Primary consideration in bilious vomiting is RAYMUND MARTIN LI, MIDTERM 3
mother due to complaints of bilious vomiting and midgut volvulus. The abdominal radiograph MD (TOP 1 - AUG 2014 EXAM - FEB
irritability. Further history reveals that the infant finding described is characteristic but not MED BOARDS; 2015
has poor appetite and assessment of growth specific for the condition. TOPNOTCH MD)
reveals failure to thrive. Abdominal radiography
reveals dilated proximal bowel and paucity of air
in the distal bowel. Which is the most likely
diagnosis?
A. Intussusception
B. Malrotation of midgut volvulus
C. Duodenal atresia
D. Hirschprung disease
E. Meconium ileus
273 A 27 year old male patient in a vehicular crash is Supraumbilical approach for pregnant and RAYMUND MARTIN LI, MIDTERM 3
brought to the ER, You assess that the patient is patients with unstable pelvis/suspected pelvic MD (TOP 1 - AUG 2014 EXAM - FEB
hypotensive and you suspect blunt abdominal hematoma. Infraumbilical for all others MED BOARDS; 2015
trauma. You decide to perform a diagnostic TOPNOTCH MD)
peritoneal lavage. Where you you place the
catheter?
A. 2 cm lateral and left to the umbilicus
B. midline infraumbilical
C. midline supraumbilical
D. 2cm lateral and right to the umbilicus
E. None of the above
274 A male patient undergoes surgery for acute Right hemicolectomy for >2 cm, or if located at RAYMUND MARTIN LI, MIDTERM 3
appendicitis. Intraoperatively, an incidental the appendicial base or affecting ileum. MD (TOP 1 - AUG 2014 EXAM - FEB
finding of a yellowish mass is found located on the MED BOARDS; 2015
appendiceal base measuring 1.8 cm. What is the TOPNOTCH MD)
next step in the management?
A. Perform an appendectomy
B. Perform an appendectomy plus resection of
terminal ileum
C. Perform a right hemicolectomy
D. Do not proceed with the surgery and have the
patient undergo chemotherapy post-op.
E. None of the above

TOPNOTCH MEDICAL BOARD PREP SURGERY SUPEREXAM Page 37 of 94


For inquiries visit www.topnotchboardprep.com.ph or email us at topnotchmedicalboardprep@gmail.com
TOPNOTCH MEDICAL BOARD PREP SURGERY SUPEREXAM
For inquiries visit www.topnotchboardprep.com.ph or email us at topnotchmedicalboardprep@gmail.com
Item QUESTION EXPLANATION AUTHOR TOPNOTCH
# EXAM
275 What is the preferred site for kidney transplant? RAYMUND MARTIN LI, MIDTERM 3
A. Where the diseased kidney is originally located MD (TOP 1 - AUG 2014 EXAM - FEB
B. Intraperitoneal at the level of L1 MED BOARDS; 2015
C. Retroperitneal at the level of L1 TOPNOTCH MD)
D. Extraperitoneal iliac fossa
E. None of the above

276 A 72 year old male patient presents at the ER with A case of sigmoid volvulus. Treatment for RAYMUND MARTIN LI, MIDTERM 3
a sudden onset of abdominal distention, vague sigmoid volvulus is endoscopic decompression. MD (TOP 1 - AUG 2014 EXAM - FEB
abdominal pain, and failure to pass stool or flatus. Treatment for cecal volvulus is surgery MED BOARDS; 2015
There is a history of constipation but the rest is TOPNOTCH MD)
unremarkable. Abdominal radiograph reveals a
bent inner tube narrowing into a bird's beak. How
will this patient be managed?
A. Perform an endoscopic decompression
B. Perform emergent surgery
C. Do watchful waiting
D. Administer neostigmine to increase intestinal
motility
E. None of the above
277 Extensive introduction of bacteria into a normal Review types of surgical wound RAYMUND MARTIN LI, MIDTERM 3
sterile area or gross spillage of viscus contents MD (TOP 1 - AUG 2014 EXAM - FEB
belong to what classification of surgical wound? MED BOARDS; 2015
A. Clean TOPNOTCH MD)
B. Clean/Contaminated
C. Contaminated
D. Dirty

278 Which of the following hormones becomes Catecholamines - elevated for 24-48 hours; RAYMUND MARTIN LI, MIDTERM 3
elevated 3 to 4-fold as a response to injury and Cortisol may remain elevated in up to 4 weeks in MD (TOP 1 - AUG 2014 EXAM - FEB
returns to baseline after 24-48 hours? burn patients MED BOARDS; 2015
A. Cortisol TOPNOTCH MD)
B. Growth Hormone
C. Epinephrine
D. Glucagon
E. Insulin

279 You are managing a patient at the ER who Even when suspecting lower GI bleed, the first RAYMUND MARTIN LI, MIDTERM 3
presented with hematochezia and hemodynamic step is to rule out upper GI bleed because it's the MD (TOP 1 - AUG 2014 EXAM - FEB
instability and a rapid work-up is imperative. You most common cause MED BOARDS; 2015
suspect a lower GI bleed. After initial resuscitation TOPNOTCH MD)
and stabilization, what will be the first thing you
do in working up the patient?
A. Do an urgent colonoscopy
B. Perform an angiography with tagged RBC scan
C. Do small bowel studies
D. Do a nasogastric tube aspiration and urgent
upper endoscopy if positive
E. None of the above
280 In the management of small bowel obstruction, All cases are initially treated conservatively. RAYMUND MARTIN LI, MIDTERM 3
conservative management is the initial MD (TOP 1 - AUG 2014 EXAM - FEB
recommendation for which of the following MED BOARDS; 2015
EXCEPT? TOPNOTCH MD)
A. Carcinomatosis
B. Partial Bowel Obstruction
C. Obstruction in the early post-operative period
D. Intestinal obstruction due to Crohn's disease
E. None of the above
281 A patient presented to the ER with a temperature Sepsis = SIRS + identifiable source of infection. ERIC ROYD TALAVERA, FINAL EXAM -
of 39 C ,BP of 110/70, a heart rate of 115 bpm and SIRS: 1) temp of >38 or <36 2) HR >90, 3) RR MD (TOP 1 - AUG 2014 FEB 2015
a respiratory rate of 23. There was note of a CVA >20 4) WBC >12,000 or <4000 or>10% band MED BOARDS;
tenderness on the right. CBC showed leukocytosis TOPNOTCH MD)
at 17,000/uL and urinalysis showed pyruria of 30-
35 pus cells/hpf. Which of the following best
describes the patient's condition?
A. SIRS
B. Sepsis
C. Septic shock
D. Refractory septic shock
E. None of the above
282 A 24 year old male was involved in a motor For each rib fracture there is approximately ERIC ROYD TALAVERA, FINAL EXAM -
vehicular accident (MVA). On evaluation, he was 100-200 mL of blood loss. For tibial fracture MD (TOP 1 - AUG 2014 FEB 2015
noted to have 6 rib fractures. How much is the 300-500 mL, for femur fractures 800-1000 mL, MED BOARDS;
expected blood loss from this kind of injury? and for pelvic fractures >1000 mL TOPNOTCH MD)
A. 300-500 mL
B. 1500-1800 mL
C. 2000-2200 mL
D. 600-1200 mL
E. 100-200 mL

TOPNOTCH MEDICAL BOARD PREP SURGERY SUPEREXAM Page 38 of 94


For inquiries visit www.topnotchboardprep.com.ph or email us at topnotchmedicalboardprep@gmail.com
TOPNOTCH MEDICAL BOARD PREP SURGERY SUPEREXAM
For inquiries visit www.topnotchboardprep.com.ph or email us at topnotchmedicalboardprep@gmail.com
Item QUESTION EXPLANATION AUTHOR TOPNOTCH
# EXAM
283 A 60 year old male, asymptomatic and a known An elective lap chole is indicated for individuals ERIC ROYD TALAVERA, FINAL EXAM -
diabetic, underwent an ultrasound of the whole who are diabetic even if they are asymptomatic MD (TOP 1 - AUG 2014 FEB 2015
abdomen as part of his executive checkup. There MED BOARDS;
were noted findings of cholelithiasis in the TOPNOTCH MD)
ultrasound. What will be the best treatment option
for this patient?
A. Dietary modification
B. Ursodeoxycholic acid
C. Elective laparoscopic cholecystectomy
D. Both A and B
E. None of the above
284 A 35 year old construction worker sustained a for initial screening, a halo test can be ERIC ROYD TALAVERA, FINAL EXAM -
closed head injury. Moments later, a clear liquid is performed. A drop of fluid is allowed to fall on MD (TOP 1 - AUG 2014 FEB 2015
seen draining from his nose. Which of the an adosrbent surface such as a facial tissue. If MED BOARDS;
following tests is most sensitive in determining if blood is mixed with CSF, the drop will form a TOPNOTCH MD)
the liquid is a cerebrospinal fluid (CSF)? double ring with a darker center spot
A. Glucose level surrounded by a light halo of CSF. If it is
B. Protein electrophoresis indeterminate, the fluid can be sent for beta
C. Beta transferrin transferrin setting which will be only positive if
D. Halo test CSF is present
E. Fluid:serum LDH level
285 Panendoscopy with blind biopsies are done for 70% of unknown primaries with cervical node ERIC ROYD TALAVERA, FINAL EXAM -
patients with evidence of cervical lymph node metastasis is located above the clavicle hence a MD (TOP 1 - AUG 2014 FEB 2015
metastasis without clinical or radiologic evidence focused search on the said locations is MED BOARDS;
of a primary tumor. Which of the following sites is warranted TOPNOTCH MD)
sampled in order to search for the primary source?
A. Base of the tongue
B. Nasopharynx
C. Tonsillar fossa
D. Pyriform sinus
E. All of the above
286 A 45 year old female experieced a sudden onset of Systemic anticoagulations are generally not ERIC ROYD TALAVERA, FINAL EXAM -
painless loss of vision over her left eye. A central employed in the treatmet of CRAO. Sudden MD (TOP 1 - AUG 2014 FEB 2015
retinal artery occlusion (CRAO) is the primary decrease in intraocular pressure resulting in MED BOARDS;
impression of the attending Ophthalmologist. increased retinal perfusion can be achieved with TOPNOTCH MD)
Which of the following statement does not anterior chamber paracentesis and intravenous
describe the said condition? acetazolamide. This is particularly indicated in
A. A foveal cherry red spot is characteristically embolic central retinal artery occlusion. Inhaled
seen on fundoscopic examination oxygen–carbon dioxide mixture induces retinal
B. A relative afferent pupillary defect can vasodilation and increases the PO2 at the retinal
precede the fundal abnormalities by an hour surface. Thrombolytic therapy, infused directly
C. Irreversible damage can occur after 90 into the ophthalmic artery or administered
minutes of complete CRAO systemically, continues to be evaluated.
D. Systemic anticoagulation is the cornerstone of
treatment
E. IV acetazolamide can be given to reduce IOP
thereby increasing retinal perfusion
287 Amebic liver abscess of the left lobe should be Aspiration of the abscess is rarely needed and ERIC ROYD TALAVERA, FINAL EXAM -
primarily aspirated because of risk of rupture with should be reserved for patients with large MD (TOP 1 - AUG 2014 FEB 2015
concomittant involvement of what structure? abscesses, abscesses that do not respond to MED BOARDS;
A. Gallbladder medical therapy, abscesses that appear to be TOPNOTCH MD)
B. Duodenun superinfected, and abscesses of the left lobe of
C. Abdominal aorta the liver that may rupture into the pericardium.
D. Diaphragm
E. Pericardium

288 A 76 year old male, presented to the ER due to Simple patch closure alone should be done in ERIC ROYD TALAVERA, FINAL EXAM -
sudden onset of severe abdominal pain. He has patients with hemodynamic instability and/or MD (TOP 1 - AUG 2014 FEB 2015
been having recurrent burning epigastric pain for exudative peritonitis signifying a perforation MED BOARDS;
the past 2 weeks for which he self medicated with >24 hours. In a relatively low risk patient TOPNOTCH MD)
Ibuprofen as a pain reliever. He is a known addition of HSV may be considered due to
hypertensive and diabetic and claims to be negligible mortality with this approacj
compliant with medications. VS are as follows: BP
80/50, PR 133 RR 23 T 37.3. Abdominal
examination revealed a rigid abdomen, with
diffuse tenderness. A scout film of the abdomen
was done which showed evidence of
pneumoperitoneum. The primary impression of
the attending physician is a perforated duodenal
ulcer. What is the procedure of choice for this
patient?
A. Vagotomy + Drainage
B. Vagotomy + Antrectomy
C. Graham Patch closure only
D. Graham patch with highly selective vagotomy
E. Oversew with Vagotomy + Drainage
289 Which of the following drugs has the strongest The strongest case for a causal relationship ERIC ROYD TALAVERA, FINAL EXAM -
causal relationship with retroperitoneal fibrosis? between medication and retroperitoneal fibrosis MD (TOP 1 - AUG 2014 FEB 2015
A. Esomeprazole is made for methysergide. Other medications MED BOARDS;
B. Paroxetine that have been linked to retroperitoneal fibrosis TOPNOTCH MD)
C. Methysergide include beta blockers, hydralazine, -methyldopa,
D. Clofazimine and entacapone. The retroperitoneal fibrosis
E. Valsartan regresses on discontinuation of these
medications.

TOPNOTCH MEDICAL BOARD PREP SURGERY SUPEREXAM Page 39 of 94


For inquiries visit www.topnotchboardprep.com.ph or email us at topnotchmedicalboardprep@gmail.com
TOPNOTCH MEDICAL BOARD PREP SURGERY SUPEREXAM
For inquiries visit www.topnotchboardprep.com.ph or email us at topnotchmedicalboardprep@gmail.com
Item QUESTION EXPLANATION AUTHOR TOPNOTCH
# EXAM
290 A 24 year old baseball player was accidentally The nose is the most common facial fracture. It ERIC ROYD TALAVERA, FINAL EXAM -
struck by a bat in the face during practice. Upon is important to perform an intransal MD (TOP 1 - AUG 2014 FEB 2015
examination, the ER officer assessed the presence examination to determine whether a septal MED BOARDS;
of a septal hematoma. Which of the following is the hematoma is present. If present a septal TOPNOTCH MD)
most appropriate treatment for this case? hematoma must be incised, drained and packed
A. Incision and Drainage to prevent pressure necrosis if the nasal septum
B. Observation and long term midvault collapse
C. Aspiration
D. Operative repair of the fracture
E. None of the above
291 Which of the following statements does not HNPCC is more common than FAP, but is still ERIC ROYD TALAVERA, FINAL EXAM -
describe the Hereditary Nonpolyposis Colon extremely rare (1 to 3%). The genetic defects MD (TOP 1 - AUG 2014 FEB 2015
Cancer (HNPCC) or Lynch Syndrome? associated with HNPCC arise from errors in MED BOARDS;
A. It is more common than Familial mismatch repair. HNPCC is inherited in an TOPNOTCH MD)
Adenomatous Polyposis (FAP) autosomal dominant pattern and is
B. Genetic defects arise from errors in mismatch characterized by the development of colorectal
repair carcinoma at an early age (average age: 40 to 45
C. Cancer appears in the proximal colon more years) Cancers appear in the proximal colon
often than in sporadic colorectal cancer more often than in sporadic colorectal cancer
D. It is inherited in an autosomal dominant and have a better prognosis regardless of stage.
pattern HNPCC also may be associated with extracolonic
E. It is also asscoiated with extracolonic malignancies, including endometrial, which is
malignancies, with ovarian cancer being the most most common
common
292 A 55 year old woman, menopause for 2 years, was For postmenopausal BRCA1 and BRCA2 ERIC ROYD TALAVERA, FINAL EXAM -
tested positive for BRCA1 gene mutation. She mutation carriers who have not had a MD (TOP 1 - AUG 2014 FEB 2015
sought medical advice regarding her risk of mastectomy, it may be advisable to avoid MED BOARDS;
acquiring breast cancer. Which of the following is hormone replacement therapy, because no data TOPNOTCH MD)
the best recommendation? exist regarding the effect of the therapy on the
A. Mammogram every 6 months alone penetrance of breast cancer susceptibility genes.
B. Mammogram every 6 months with Tamoxifen Prophylactic oophorectomy is a reasonable
C. Prophylactic bilateral mastectomy alone prevention option in mutation carriers. The
D. Prophylactic bilateral mastectomy, followed American College of Obstetrics and Gynecology
by prophylactic BSO recommends that women with a documented
E. None of the above BRCA1 or BRCA2 mutation consider
prophylactic oophorectomy at the completion of
childbearing or at the time of menopause
293 What is the primary treatment for oat cell Oat cell or small cell carcinoma accounts for ERIC ROYD TALAVERA, FINAL EXAM -
carcinoma of the lung? 20% of lung CA and generally is not treated MD (TOP 1 - AUG 2014 FEB 2015
A. Surgery alone surgically due to its aggressive nature and MED BOARDS;
B. Surgery followed by chemotherapy widespread metastasis. Regardless of stage and TOPNOTCH MD)
C. Immunotherapy presentation, treatment is primarily
D. Chemotherapy and radiotherapy chemotherapy and radiation.
E. Radiotherapy alone

294 A 56 year old male presented with a gradually Dx: Medullary Thyroid Carcinoma. Total ERIC ROYD TALAVERA, FINAL EXAM -
enlarging anterior neck mass accompanied by thyroidectomy is the treatment of choice for MD (TOP 1 - AUG 2014 FEB 2015
hoarseness and dysphagia. FNAB was done which patients with MTC because of the high incidence MED BOARDS;
revealed sheets of infiltrating neoplastic cells of multicentricity, the more aggressive course, TOPNOTCH MD)
separated by collagen and amyloid. What is the and the fact that 131I therapy usually is not
best treatment option for this patient ? effective. Central compartment nodes frequently
A. Subtotal thyroidectomy are involved early in the disease process, so that
B. RAI a bilateral central neck node dissection should
C. External beam radiotherapy be routinely performed.
D. Total thyroidectomy alone
E. Total thyroidectomy + routine bilateral central
neck node dissection
295 What is the most common complication seen in Intestinal obstruction is the most common ERIC ROYD TALAVERA, FINAL EXAM -
adults with Meckel's diverticulum? presentation in adults with Meckel's divertiula. MD (TOP 1 - AUG 2014 FEB 2015
A. Diverticulitis Bleeding is the most common presentation in MED BOARDS;
B. Obstruction children with Meckel's diverticula TOPNOTCH MD)
C. Bleeding
D. Perforation
E. None of the above

296 A 40 year old female, asymptomatic with no All of the aforementioned are indications for ERIC ROYD TALAVERA, FINAL EXAM -
known co-morbids, was found to have elevated parathyroidectomy in asymptomatic individuals. MD (TOP 1 - AUG 2014 FEB 2015
serum calcium on her routine pre-employment Other indications would include: life threatening MED BOARDS;
check up. A serum PTH was done which showed hypercalcemic crisis, (+) kidney stones, TOPNOTCH MD)
elevated levels. The impression of the physician is decreased BMD (>2.5 SD)
a primary hyperparathyroidism. Which of the
following is an indication for parathyroidectomy
in an asymptomatic individual ?
A. Serum calcium >1 mg/dL above the upper
limits of normal
B. Creatinine clearance reduced by 30%
C. Age less than 50
D. Elevated 24 hour urinary calcium (>400
mg/day)
E. All of the above

TOPNOTCH MEDICAL BOARD PREP SURGERY SUPEREXAM Page 40 of 94


For inquiries visit www.topnotchboardprep.com.ph or email us at topnotchmedicalboardprep@gmail.com
TOPNOTCH MEDICAL BOARD PREP SURGERY SUPEREXAM
For inquiries visit www.topnotchboardprep.com.ph or email us at topnotchmedicalboardprep@gmail.com
Item QUESTION EXPLANATION AUTHOR TOPNOTCH
# EXAM
297 Which of the following types of renal calculi will Calcium and struvite-containing stones often are ERIC ROYD TALAVERA, FINAL EXAM -
not be visualized on CT stonogram? visible on plain radiographs, but CT scans will MD (TOP 1 - AUG 2014 FEB 2015
A. Indinavir demonstrate all calculi except those composed MED BOARDS;
B. Uric acid of crystalline-excreted indinavir, an TOPNOTCH MD)
C. Calcium oxalate antiretroviral medication.
D. Magnesium ammonium phosphate
E. Cysteine

298 A 1 year old boy presented with a foul smelling, the most common intracranial complication of ERIC ROYD TALAVERA, FINAL EXAM -
grayish yellow aural discharge over the left ear suppurative OM is meningitis MD (TOP 1 - AUG 2014 FEB 2015
accompanied by hearing loss. The attending ENT MED BOARDS;
diagnosed the patient with suppurative otitis TOPNOTCH MD)
media. If left untreated which of the following is
the most common intracranial complication of this
disease?
A. Brain abscess
B. Otitic hydrocephalus
C. Meningitis
D. Lateral sinus thrombophlebitis
E. Petrositis
299 A skin biopsy obtained from 41 year old woman Clark's level I: superficial to the basement ERIC ROYD TALAVERA, FINAL EXAM -
showed findings consistent with a malignant membrane, II: up to papillary dermis, III: up to MD (TOP 1 - AUG 2014 FEB 2015
melanoma. The malignant cells were papillary/reticular dermis junction, IV: reticular MED BOARDS;
demonstrated to have invaded up to the layer of dermis, V: up to subcutaneous fat TOPNOTCH MD)
the papillary dermis. What is the Clark's level of
staging for this case?
A. I
B. IV
C. III
D. V
E. II
300 Which valve is primarily affected in Ebstein's The predominant maldevelopment in this lesion ERIC ROYD TALAVERA, FINAL EXAM -
anomaly? is the inferior displacement of the tricuspid MD (TOP 1 - AUG 2014 FEB 2015
A. Aortic valve into the right ventricle MED BOARDS;
B. Mitral TOPNOTCH MD)
C. Pulmonary
D. Tricuspid
E. Both A and C

301 A 45 year old known duodenal ulcer patient Simple patch closure alone should be done in LEAN ANGELO BACK-UP
undergoes laparotomy for signs of peritonitis patients with hemodynamic instability and or SILVERIO, MD (TOP 4 - MIDTERM
more than 24 hrs after onset of acute symptoms. exudative peritonitis. Schwartz 9th ed p 921. AUG 2014 MED EXAM - FEB
At surgery there is massive peritoneal BOARDS; TOPNOTCH 2015
contamination and the patient is hypotensive.what MD), MD
is the most appropriate treatment for this patient ?
A. hemigastrectomy with truncal vagotomy
B. highly selective vagotomy
C. gastrojejunostomy
D. Graham’s patching with peritoneal toilette
E. Bilroth II gastrectomy
302 15. salivary gland tumor which has significant adenoid cystic carcinoma, which has a LEAN ANGELO BACK-UP
propensity to invade adjacent nerves and progress propensity for neural invasion is the second SILVERIO, MD (TOP 4 - MIDTERM
in a retrograde fashion to the central nervous most common salivary gland carcinoma in AUG 2014 MED EXAM - FEB
system adults. Scwartz 9th ed page 507 BOARDS; TOPNOTCH 2015
A. mucoepidermoid CA MD), MD
B. neurilemmoma
C. squamous cell CA
D. adenoid cystic CA
E. Warthins tumor
303 A 50 y/o female whoe practices reverse smoking B,C,E are Selective node dissection done only in LEAN ANGELO BACK-UP
presents with a 2x2cm ulcer with heaped-up patients who are clinically negative neck nodes. SILVERIO, MD (TOP 4 - MIDTERM
border at the R floor of the mouth of 10 months This is contrary to our patient. MRND has a AUG 2014 MED EXAM - FEB
duration. Pre showed 1x2xm submandibular mass better functional outcome compared to radical BOARDS; TOPNOTCH 2015
on the right together with several enlarged neck dissection. MD), MD
discrete masses on the right upper jugular area
altogether measuring 3x4cm.what is the most
appropriate neck dissection should be done in this
patient?
A. Radical neck dissection
B. Lateral neck dissection
C. Supraomohyoid neck dissection
D. Modified neck dissection
E. posterolateral neck dissection
304 . The most important criteria to distinguish a Determination of the malignant potential of an LEAN ANGELO BACK-UP
malignant adrenal incidentaloma from a benign incidentaloma is related to size. Lesions >6cm SILVERIO, MD (TOP 4 - MIDTERM
one on imaging studies is: have an appropriate risk of malignancy of about AUG 2014 MED EXAM - FEB
A. tumor size 35%. Scwartz 9th ed p1401 BOARDS; TOPNOTCH 2015
B. irregularity of margins MD), MD
C. presence of adjacent lymphadenopathy
D. tumor heterogeneity
E. vascular invasion

TOPNOTCH MEDICAL BOARD PREP SURGERY SUPEREXAM Page 41 of 94


For inquiries visit www.topnotchboardprep.com.ph or email us at topnotchmedicalboardprep@gmail.com
TOPNOTCH MEDICAL BOARD PREP SURGERY SUPEREXAM
For inquiries visit www.topnotchboardprep.com.ph or email us at topnotchmedicalboardprep@gmail.com
Item QUESTION EXPLANATION AUTHOR TOPNOTCH
# EXAM
305 A 48 y/o male presented to your clinic with Type I - fusiform dilatation of the CBD - LEAN ANGELO BACK-UP
obstructive form of jaundice. You opted to request cholecystectomy, cyst excison and a roux en Y SILVERIO, MD (TOP 4 - MIDTERM
for MRCP which reveal a case of type III choledochojejunostomy. Type II -diverticulum of AUG 2014 MED EXAM - FEB
choledochal cyst. What is the standard treatment CBD - excision. Type III- choledochocele of the BOARDS; TOPNOTCH 2015
for this case? intraduodenal CBD - sphincteroplasty or MD), MD
A. Cholecystectomy with cyst excision choledochoduodenostomy. Type 4- caroli's
B. transduodenal sphincteroplasty disease, cystic dilatations of intrahepatic duct, -
C. liver transplantation liver transplantation.
D. roux en y choledochojejunostomy
E. all of the above
306 A 35 y/o male was brought to ER secondary to treatment of appendiceal carcinoids depends on LEAN ANGELO BACK-UP
Right lower quadrant pain and fever. PE reveals the size of the mass and its location. Tumors SILVERIO, MD (TOP 4 - MIDTERM
direct and rebound tenderness on RLQ. During the <1cm, appendectomy is recommended. For AUG 2014 MED EXAM - FEB
operation, a firm , yellow bulbar mass measuring those >2.0 cm, the treatment is right BOARDS; TOPNOTCH 2015
1.3 cm located at the mid appendiceal area was hemicolectomy. For tumors 1-2cm in size, MD), MD
noted along with congested appendix. what is the resection is dependent on the location. mid and
next step in the management? distal appendiceal area tumors- appendectomy.
A. appendectomy while those located at the base or those with
B. right hemicolectomy lymph node metastasis, right hemicolectomy is
C. appendectomy with lymph node dissection warranted. Scwartz 9th ed p 1088.
D. tumor resection with exploration of small
bowels
E. none of the above
307 A 65 y/o male was referred secondary to malignancy risk is higher because of his smoking LEAN ANGELO BACK-UP
incidental finding of a 3cm mass located at the left history, therefore, observation is not indicated. SILVERIO, MD (TOP 4 - MIDTERM
upper lobe. Follow up CT scan revealed Currently the patient is on stage 1 of the disease. AUG 2014 MED EXAM - FEB
microcalcification but with an absence of lymph biopsy has clinically defined benefit over direct BOARDS; TOPNOTCH 2015
node metastasis. Patient is a smoker with a 35 surgical intervention. MD), MD
pack years history. what is the next best step in
the managment ?
A. CT guided biopsy
B. observation with repeat CXR after 3-6 months
C. excision of the mass
D. right upper lobe lobectomy
E. radiation therapy
308 which of the following is true about the Zone I LEAN ANGELO BACK-UP
neck injury: SILVERIO, MD (TOP 4 - MIDTERM
A. Refers to the area between the angle of AUG 2014 MED EXAM - FEB
mandible and cricoids BOARDS; TOPNOTCH 2015
B. Notorious for occult hemothorax MD), MD
C. Must be explored liberally because of high rate
of occult injury
D. All of the above
309 How many lymph node are required to be accurate predictions regarding the occurrence of LEAN ANGELO BACK-UP
dissected in axillary lymph node disection in distant metastasis were possible after resection SILVERIO, MD (TOP 4 - MIDTERM
breast cancer? of 10 or more level 1 and level II axillary LN. AUG 2014 MED EXAM - FEB
A. 8 Schwartz 9th ed page 451 BOARDS; TOPNOTCH 2015
B. 9 MD), MD
C. 10
D. 11
E. 12

310 which of the following is the most common site of the following are the most common site of LEAN ANGELO BACK-UP
metastasis in breast carcinoma? metastasis in breast cancer in descending order: SILVERIO, MD (TOP 4 - MIDTERM
A. lungs bone, lungs, pleura, soft tissue, liver. Schwarts AUG 2014 MED EXAM - FEB
B. bone 9th ed page 443 BOARDS; TOPNOTCH 2015
C. Liver MD), MD
D. brain
E. Spleen

311 which of the following will have the highest Schwartz 9th ed page 36. LEAN ANGELO BACK-UP
Resting energy expenditure ? SILVERIO, MD (TOP 4 - MIDTERM
A. Severe sepsis AUG 2014 MED EXAM - FEB
B. rhabdomyolysis BOARDS; TOPNOTCH 2015
C. Intraabdominal surgery MD), MD
D. Major burns
E. Starvation

312 Tilaux sign is correlated to what abdominal tilaux sign is positive if the abdominal mass is LEAN ANGELO BACK-UP
pathology ? permissive in lateral direction but restricts SILVERIO, MD (TOP 4 - MIDTERM
A. Omental cyst movement in superoinferior direction. This is AUG 2014 MED EXAM - FEB
B. Abdominal wall hematoma highly correlated with mesenteric cyst. BOARDS; TOPNOTCH 2015
C. Mesenteric cyst MD), MD
D. GIST
E. Choledochal cyst

313 Which of the following risk factor will have the the most important risk factor for gallbladder LEAN ANGELO BACK-UP
highest association with gallbladder carcinoma ? carcinoma is the presence of cholelithiasis. Up to SILVERIO, MD (TOP 4 - MIDTERM
A. 1cm polyp 95% of carcinoma have gallstones. Schwarts 9th AUG 2014 MED EXAM - FEB
B. Cholesterol stones ed p1160. ( SIMILAR TO PREVIOUS BOARD BOARDS; TOPNOTCH 2015
C. Porcelain gallbladder EXAM CONCEPT/PRINCIPLE) MD), MD
D. Choledochal cyst
E. Primary sclerosing cholangitis

TOPNOTCH MEDICAL BOARD PREP SURGERY SUPEREXAM Page 42 of 94


For inquiries visit www.topnotchboardprep.com.ph or email us at topnotchmedicalboardprep@gmail.com
TOPNOTCH MEDICAL BOARD PREP SURGERY SUPEREXAM
For inquiries visit www.topnotchboardprep.com.ph or email us at topnotchmedicalboardprep@gmail.com
Item QUESTION EXPLANATION AUTHOR TOPNOTCH
# EXAM
314 A 69 y/o male presents with a history of suddent preoperative CT findings that determine LEAN ANGELO BACK-UP
onset of obstructive jaundice accompanied by unresectability of a pancreatic head mass SILVERIO, MD (TOP 4 - MIDTERM
recurrent abdominal pain and involuntary weight include encasement of the SMA, celiac axis and AUG 2014 MED EXAM - FEB
loss of 10%. CT scan was performed which reveals occlusion of the superior mesenteric vein or BOARDS; TOPNOTCH 2015
3.9 cm mixed hypodensity lesion on the head of portal vein. Loss of plane b/w tumor and the MD), MD
the pancreas. percutaneous biopsy was done vessels is not a contraindication for curative
revealing irregular, anaplastic, highly mitotic cells resection.
with glandular features. which of the following CT
findings will preclude curative resection via
whipples procedure?
A. encasement of gastroduodenal artery
B. lack of pulsation of the superior mesenteric
artery
C. encroachment of the plane between the tumor
and the portal vein
D. multiple matted lymphadenopathies in the
celiac axis
E. malignant refractory ascites.
315 Inadequate supplementation of this nutrient is the metabolism and viability of colonic cells is LEAN ANGELO BACK-UP
responsible for the development of diversion dependent on supply of short chain fatty acids ( SILVERIO, MD (TOP 4 - MIDTERM
colitis? butyrate) AUG 2014 MED EXAM - FEB
A. butyrate BOARDS; TOPNOTCH 2015
B. Glutamine MD), MD
C. arginine
D. Palmitate
E. MCT

316 what is the recommended age for surgical Scwartz 9th ed page 1446. 2 years old is the LEAN ANGELO BACK-UP
repositioning of an undescended testis? recommended age for orchiopexy. SILVERIO, MD (TOP 4 - MIDTERM
A. 1 y/o AUG 2014 MED EXAM - FEB
B. 2y/o BOARDS; TOPNOTCH 2015
C. 3y/o MD), MD
D. 4y/o
E. 5y/o

317 which of the following is not true about hepatic the indication for angiography to control hepatic LEAN ANGELO BACK-UP
injury secondary to blunt trauma? hemorrhage is transfusion of 4 units pRBC in 6 SILVERIO, MD (TOP 4 - MIDTERM
A. Delineating the source of hepatic hemorrhage hours or 6 units of pRBC in 24 hours. Schwartz AUG 2014 MED EXAM - FEB
can be done by pringle manuever 9th ed p 175 BOARDS; TOPNOTCH 2015
B. Transfusion of 4 units of pRBC in 24 hours is MD), MD
an indication for hepatic angiography
C. cholecystectomy is required if the right hepatic
artery is to be ligated
D. left lobe injury is not responsive to extensive
packing
E. none of the above
318 the higher rates of anastomotic failure observed serosal healing is essential for quickly achieving LEAN ANGELO BACK-UP
during repair of duodenum is secondary to which watertight seal from the luminal side of the SILVERIO, MD (TOP 4 - MIDTERM
of the following cause? bowel. The importance of the serosa is AUG 2014 MED EXAM - FEB
A. Deficient closure of the submucosa underscored by the signifantly higher rates of BOARDS; TOPNOTCH 2015
B. Lack of collateral supply anastomotic failure observed clinically in MD), MD
C. Absence of serosa segments of bowel that are extraperitoneal.
D. Undue tension of sutures Schwarts 9th ed p 216
E. B and C

319 what is the correct sequence of the processes imbibition - thin film of plasma separates the LEAN ANGELO BACK-UP
occuring in graft take? graft from the wound bed. Inosculation- the SILVERIO, MD (TOP 4 - MIDTERM
A. Inosculation-imbibition- revascularization process by which new blood vessels either AUG 2014 MED EXAM - FEB
B. Imbibition- revascularization- inosculation directly invade the graft or anastomose to open BOARDS; TOPNOTCH 2015
C. Imbibition-inosculation- revascularization dermal channels and restoring the pink hue of MD), MD
D. None of the above the skin. Schwartz 9th ed p1651

320 which of the following is not true about the most favorable outcome is obtained LEAN ANGELO BACK-UP
esophageal perforation? following primary closure of the perforation SILVERIO, MD (TOP 4 - MIDTERM
A. most common location is left lateral just above within 24 hours resulting in 80-90% survival. AUG 2014 MED EXAM - FEB
the GEJ Schwartz 9th ed p 875 BOARDS; TOPNOTCH 2015
B. the most important prognostic indicator is the MD), MD
timing of primary closure (within 48 hours)
C. non operative management includes
hyperalimentation, and histamine blockers
D. elevated serum amylase is a common finding
E. none of the above
321 The strongest chemotactic factor for fibroblasts DEBBIE ROSE BACK-UP
during the proliferation phase of wound healing: TANENGSY, MD (TOP 5 MIDTERM
A. VEGF - AUG 2014 MED EXAM - FEB
B. PDGF BOARDS; TOPNOTCH 2015
C. TNF MD)
D. IFN-y
E. Il-1

TOPNOTCH MEDICAL BOARD PREP SURGERY SUPEREXAM Page 43 of 94


For inquiries visit www.topnotchboardprep.com.ph or email us at topnotchmedicalboardprep@gmail.com
TOPNOTCH MEDICAL BOARD PREP SURGERY SUPEREXAM
For inquiries visit www.topnotchboardprep.com.ph or email us at topnotchmedicalboardprep@gmail.com
Item QUESTION EXPLANATION AUTHOR TOPNOTCH
# EXAM
322 True in the management of caustic injuries to the NaHCO3 generates CO2, increasing the danger of DEBBIE ROSE BACK-UP
esophagus, except: perforation. TANENGSY, MD (TOP 5 MIDTERM
A. Alkali agents cause liquefactive necrosis. - AUG 2014 MED EXAM - FEB
B. Asymptomatic patients can be observed. BOARDS; TOPNOTCH 2015
C. In patients with mild acid injury, neutralization MD)
of offending agent with NaHCO3 may be
considered.
D. Any degree of injury may result in strictures
and squamous cell carcinoma.
E. Alkali injuries have a worse prognosis.
323 Regarding the blood supply to the stomach, this is DEBBIE ROSE BACK-UP
also known as the coronary vein: TANENGSY, MD (TOP 5 MIDTERM
A. left gastric vein - AUG 2014 MED EXAM - FEB
B. right gastric vein BOARDS; TOPNOTCH 2015
C. portal vein MD)
D. right gastroepiploic vein
E. left gastroepiploic vein

324 the clinical triad of Zollinger-Ellison syndrome ZES clinical triad: hypersecretion of HCl, severe DEBBIE ROSE BACK-UP
does not include: PUD, and gastrinoma TANENGSY, MD (TOP 5 MIDTERM
A. hypersecretion of HCl - AUG 2014 MED EXAM - FEB
B. steatorrhea BOARDS; TOPNOTCH 2015
C. severe peptic ulcer disease MD)
D. gastrinoma
325 Damage to this nerve during mastectomies causes The anterior thoracic nerve innervates the DEBBIE ROSE BACK-UP
numbness in the upper, inner aspect of the arm: pectoralis muscles, and damage to this causes TANENGSY, MD (TOP 5 MIDTERM
A. intercostobrachial nerve atrophy of these muscles. The thoracodorsal - AUG 2014 MED EXAM - FEB
B. anterior thoracic nerve nerve innervates the latissimus dorsi, and BOARDS; TOPNOTCH 2015
C. thoracodorsal nerve damage results in a weak internal rotation and MD)
D. long thoracic nerve abduction of the arm. The serratus anterior is
innervated by the long thoracic nerve. Damage
to this causes the winged scapula.
326 A pancoast tumor is a superior sulcus tumor Horner's syndrome in a patient with Pancoast DEBBIE ROSE BACK-UP
affecting the brachial plexus and sympathetic tumor presents with ptosis, miosis, and TANENGSY, MD (TOP 5 MIDTERM
trunk causing: anhydrosis. - AUG 2014 MED EXAM - FEB
A. ptosis BOARDS; TOPNOTCH 2015
B. mydriasis MD)
C. anhydrosis
D. A & C
E. all of the above

327 The following radiographic features suggest A doubling time of 30-450 days is one of the DEBBIE ROSE BACK-UP
malignancy in a solitary lung nodule, except: radiographic features that suggest malignancy. TANENGSY, MD (TOP 5 MIDTERM
A. corona radiata sign Benign features: smooth calcifications, < 2cm - AUG 2014 MED EXAM - FEB
B. size > 2 cm size, doubling time < 20 days or > 450 days. BOARDS; TOPNOTCH 2015
C. doubling time < 20 days MD)
D. lobulations
E. none of the above

328 True about septic shock: DEBBIE ROSE BACK-UP


A. increased cardiac index TANENGSY, MD (TOP 5 MIDTERM
B. low systemic vascular resistance - AUG 2014 MED EXAM - FEB
C. increased venous capacitance BOARDS; TOPNOTCH 2015
D. all of the above MD)
E. none of the above

329 How much IVF should be administered in a 63 kg 18% (both entire upper extremities) + 18% DEBBIE ROSE BACK-UP
man with third degree burns to both entire upper (anterior torso) = 36% x 4 x 63 kg = 4536 cc for TANENGSY, MD (TOP 5 MIDTERM
extremities, anterior torso in the first 8 hours the first 8 hours post-injury - AUG 2014 MED EXAM - FEB
post-injury? BOARDS; TOPNOTCH 2015
A. 3000 cc LR MD)
B. 3500 cc LR
C. 4000 cc LR
D. 4500 cc LR
E. 5000 cc LR
330 Classic chronologic order of symptoms in acute DEBBIE ROSE BACK-UP
appendicitis: TANENGSY, MD (TOP 5 MIDTERM
A. vomiting - anorexia - abdominal pain - AUG 2014 MED EXAM - FEB
B. anorexia - abdominal pain - vomiting BOARDS; TOPNOTCH 2015
C. anorexia - vomiting - abdominal pain MD)
D. abdominal pain - vomiting - anorexia
E. abdominal pain - anorexia - vomiting

331 Most common type of anorectal abscess: DEBBIE ROSE BACK-UP


A. ischiorectal TANENGSY, MD (TOP 5 MIDTERM
B. perianal - AUG 2014 MED EXAM - FEB
C. intersphincteric BOARDS; TOPNOTCH 2015
D. pelvic MD)

TOPNOTCH MEDICAL BOARD PREP SURGERY SUPEREXAM Page 44 of 94


For inquiries visit www.topnotchboardprep.com.ph or email us at topnotchmedicalboardprep@gmail.com
TOPNOTCH MEDICAL BOARD PREP SURGERY SUPEREXAM
For inquiries visit www.topnotchboardprep.com.ph or email us at topnotchmedicalboardprep@gmail.com
Item QUESTION EXPLANATION AUTHOR TOPNOTCH
# EXAM
332 The following are hard signs of peripheral arterial Operation is mandatory if there are hard signs of DEBBIE ROSE BACK-UP
injury, except: peripheral arterial injury. Further evaluation is TANENGSY, MD (TOP 5 MIDTERM
A. pulsatile hemorrhage indicated in the presence of soft signs, which - AUG 2014 MED EXAM - FEB
B. absent pulses include proximity to vasculature, significant BOARDS; TOPNOTCH 2015
C. acute ischemia hematoma, associated nerve injury, A-A index of MD)
D. associated nerve injury <0.9, and thrill or bruit. Schwartz p.158 table 7-8
E. none of the above

333 The most common presenting complaint in a DEBBIE ROSE BACK-UP


patient with anorectal abscess is: TANENGSY, MD (TOP 5 MIDTERM
A. fever - AUG 2014 MED EXAM - FEB
B. hematochezia BOARDS; TOPNOTCH 2015
C. anal pain MD)
D. draining pus from anus
E. swelling at anal verge

334 The most common cause of a defective sphincter A, B and C define a permanently defective DEBBIE ROSE BACK-UP
is: sphincter.The most common cause of a defective TANENGSY, MD (TOP 5 MIDTERM
A. lower esophageal sphincter with a mean resting sphincter is inadequate abdominal length. - AUG 2014 MED EXAM - FEB
pressure of < 6 mmHg BOARDS; TOPNOTCH 2015
B. overall sphincter length of < 2 cm MD)
C. intraabdominal sphincter length of < 1 cm
D. none of the above
335 The accuracy of a preoperative diagnosis of an DEBBIE ROSE BACK-UP
acute appendicitis ahould be? TANENGSY, MD (TOP 5 MIDTERM
A. 70% - AUG 2014 MED EXAM - FEB
B. 755 BOARDS; TOPNOTCH 2015
C. 80% MD)
D. 85%
E. 90%

336 True of appendicitis in a patient with AIDS/HIV: Appendicitis in patients with HIV/AIDS: a low DEBBIE ROSE BACK-UP
A. CD4 count is unrelated to rupture. CD4 count is associated with a higher chance of TANENGSY, MD (TOP 5 MIDTERM
B. HIV infected patients manifest absolute rupture. These patients manifest relative - AUG 2014 MED EXAM - FEB
leukocytosis. leukocytosis. The presentation is similar to that BOARDS; TOPNOTCH 2015
C. Presentation in HIV patients is strikingly of the general population, and post-operative MD)
different from the general population. morbidity rates are higher. Schwartz pp.1083-
D. Post-op morbidity rates are similar 1084
E. Risk for rupture is increased in HIV patients.
337 The most useful radiologic test for diagnosing The sonographic Murphy's sign: focal DEBBIE ROSE BACK-UP
acute cholecystitis: tenderness over the gallbladder when TANENGSY, MD (TOP 5 MIDTERM
A. ultrasound compressed by the sonographic probe. CT scan - AUG 2014 MED EXAM - FEB
B. CT-scan is a less sensitive test for diagnosing acute BOARDS; TOPNOTCH 2015
C. HIDA scan cholecystitis. A normal HIDA scan excludes acute MD)
D. MRI chole.
338 Absolute contraindications for cholecystectomy Schwartz p.1151 DEBBIE ROSE BACK-UP
include? TANENGSY, MD (TOP 5 MIDTERM
A. uncontrolled coagulopathy - AUG 2014 MED EXAM - FEB
B. end stage liver disease BOARDS; TOPNOTCH 2015
C. both MD)
D. neither
339 In patients with normal preoperative values, blood Schwartz p.79 DEBBIE ROSE BACK-UP
loss of up to ___ can be replaced with crystalloid TANENGSY, MD (TOP 5 MIDTERM
solution: - AUG 2014 MED EXAM - FEB
A. 5% BOARDS; TOPNOTCH 2015
B. 10% MD)
C. 15%
D. 20%
E. 25%

340 True of fresh frozen plasma, except: DEBBIE ROSE BACK-UP


A. It is the only source of factor VIII. TANENGSY, MD (TOP 5 MIDTERM
B. It carries infection risks similar to those of other - AUG 2014 MED EXAM - FEB
component therapies. BOARDS; TOPNOTCH 2015
C. It is a source of Vitamin K dependent factors. MD)
D. none of the above
341 A 23 year old male came in with a 4 cm laceration JESSICA MAE BACK-UP
on his right cheek after engaging in a brawl. Which SANCHEZ, MD (TOP 4 - MIDTERM
of the following sutures is most appropriate for AUG 2014 MED EXAM - FEB
closing this patient’s laceration? BOARDS; TOPNOTCH 2015
A. Vicryl 3-0 MD)
B. Chromic 3-0
C. Silk 4-0
D. Nylon 6-0
342 A 23 year old male came in with a 4 cm laceration Reference: Topnotch Surgery Pearls Handouts JESSICA MAE BACK-UP
on his right cheek after engaging in a brawl. He SANCHEZ, MD (TOP 4 - MIDTERM
was sent home after suturing of his laceration was AUG 2014 MED EXAM - FEB
done. When will you advise him to come back for BOARDS; TOPNOTCH 2015
removal of his sutures? MD)
A. After 3 days
B. After 1 week
C. After 10 days
D. After 2 weeks

TOPNOTCH MEDICAL BOARD PREP SURGERY SUPEREXAM Page 45 of 94


For inquiries visit www.topnotchboardprep.com.ph or email us at topnotchmedicalboardprep@gmail.com
TOPNOTCH MEDICAL BOARD PREP SURGERY SUPEREXAM
For inquiries visit www.topnotchboardprep.com.ph or email us at topnotchmedicalboardprep@gmail.com
Item QUESTION EXPLANATION AUTHOR TOPNOTCH
# EXAM
343 A 17 year old football player sustained a fall on his Immobilization with a sling or a figure of eight JESSICA MAE BACK-UP
outstretched right arm during practice, after splint are equally effective in the treatment of SANCHEZ, MD (TOP 4 - MIDTERM
which there was noted pain and swelling of the uncomplicated clavicle fractures, with a sling AUG 2014 MED EXAM - FEB
right clavicular area. X-ray confirmed a fracture of being preferable because it is more comfortable BOARDS; TOPNOTCH 2015
the middle third of the clavicle. At the ER, what for the patient and may have less complications. MD)
would be the most appropriate initial
management for this type of fracture? Reference:
A. Analgesics only http://www.hopkinsortho.org/claviclefx.html;

B. Ipsilateral sling to immobilize the shoulder


C. Figure of eight splint
D. Send to the OR for open reduction and internal
fixation
344 A 38 year old man consults because of a 4 cm Review algorithm on work-up of solitary thyroid JESSICA MAE BACK-UP
nodule at the right lobe of the thyroid noted 3 nodule SANCHEZ, MD (TOP 4 - MIDTERM
years ago. There are no accompanying symptoms. AUG 2014 MED EXAM - FEB
He denies family history of cancer nor radiation Reference: Schwartz, Principles of Surgery, 9th BOARDS; TOPNOTCH 2015
exposure. On PE, the nodule is hard, moves with ed. p. 1359 MD)
deglutition, without any palpable cervical lymph
nodes. The rest of the examination is
unremarkable. The initial diagnostic test to
perform in evaluating this nodule is:
A. FNAB
B. Radioactive iodine scan
C. CT scan of the neck
D. Ultrasound of the neck
345 A 30 year old woman underwent right lobectomy For minimal papillary thyroid cancers (1 cm or JESSICA MAE BACK-UP
and isthmusectomy for a 1 cm papillary cancer. less), unilateral lobectomy and isthmusectomy is SANCHEZ, MD (TOP 4 - MIDTERM
She will require: adequate. AUG 2014 MED EXAM - FEB
A. Observation only BOARDS; TOPNOTCH 2015
B. Completion thyroidectomy Reference: Schwartz. Principles of Surgery, 9th MD)
C. RAI ablation therapy ed. p. 1363
D. B and C
346 An 85 year old female patient with atrial Severe abdominal pain out of proportion to the JESSICA MAE BACK-UP
fibrillation developed sudden onset of severe PE findings is characteristic of acute mesenteric SANCHEZ, MD (TOP 4 - MIDTERM
abdominal pain. On PE, abdominal findings are ischemia. Because of this patient’s history of AUG 2014 MED EXAM - FEB
minimal. Vital signs show tachycardia with normal atrial fibrillation, the most likely cause of the BOARDS; TOPNOTCH 2015
BP and temperature. The most likely diagnosis is: mesenteric ischemia is an embolus. MD)
A. Non-occlusive mesenteric ischemia
B. Acute mesenteric ischemia due to embolism
C. Mesenteric vascular thrombosis
D. Mesenteric vasculitis
347 A 75 year old male presents with bulging of the Diagnosis: Diastasis recti. This is a separation of JESSICA MAE BACK-UP
midline abdomen especially on straining. On PE, the pillars of rectus abdominis muscles, and may SANCHEZ, MD (TOP 4 - MIDTERM
there is a separation of the midline. Which of the either be congenital or acquired in aetiology. A AUG 2014 MED EXAM - FEB
following statements is true about this condition? Spigelian hernia is not located at the midline and BOARDS; TOPNOTCH 2015
A. It is due to a separation of the external and is not usually mistaken for diastasis recti. MD)
internal oblique aponeurosis
B. It is sometimes mistaken for a Spigelian hernia
C. It is always congenital in aetiology
D.None of the above
348 A 52 year old female underwent breast Based on data from the American College of JESSICA MAE BACK-UP
conservation surgery with adjuvant radiotherapy Surgeons National Cancer Database, the 5 year SANCHEZ, MD (TOP 4 - MIDTERM
for a stage 1 breast cancer. What is her expected 5 survival rate for breast cancer by stage are as AUG 2014 MED EXAM - FEB
year survival rate? follows: BOARDS; TOPNOTCH 2015
A. 100% Stage I - 100% MD)
B. 95% Stage IIA - 92% Stage IIB - 81% Stage IIIA -
C. 80% 67% Stage IIIB - 54%
D. 70%
Reference: Schwartz, Principles of Surgery, 9th
ed. p. 457

349 A 54 year old female with a history of rheumatic Rectus sheath hematoma may occur JESSICA MAE BACK-UP
heart disease and valve replacement surgery spontaneously in patients receiving SANCHEZ, MD (TOP 4 - MIDTERM
suddenly developed severe abdominal pain with anticoagulant therapy, which is probably the AUG 2014 MED EXAM - FEB
some tenderness and a palpable mass around 3 cm case in a patient who has undergone valve BOARDS; TOPNOTCH 2015
in size. There is also a bluish discolouration replacement surgery. MD)
around the umbilicus. The most likely diagnosis is:
A. Desmoid tumor
B. Rectus sheath hematoma
C. Acute appendicitis
D. Acute cholecystitis
350 A 45 year old female was brought to the ER Intra-abdominal adhesions related to prior JESSICA MAE BACK-UP
because of colicky abdominal pain, vomiting, and abdominal surgery account for up to 75% of SANCHEZ, MD (TOP 4 - MIDTERM
abdominal distention. Condition started 3 days cases of small bowel obstruction. AUG 2014 MED EXAM - FEB
PTA as occasional cramps epigastric pain BOARDS; TOPNOTCH 2015
accompanied by vomiting, abdominal distention, Reference: Schwartz, Principles of Surgery, 9th MD)
and obstipation. Past medical history revealed ed. p. 988
appendectomy 1 year ago. On PE, there was high
grade fever and (+) rebound tenderness. The
patient is most likely suffering from:
A. Mechanical intestinal obstruction due to post-
op adhesions
B. Intestinal distention from prolonged ileus
C. Mechanical obstruction due to benign intestinal
tumor
D. Mechanical obstruction due to malignant
TOPNOTCH MEDICAL BOARD PREP SURGERY SUPEREXAM Page 46 of 94
For inquiries visit www.topnotchboardprep.com.ph or email us at topnotchmedicalboardprep@gmail.com
TOPNOTCH MEDICAL BOARD PREP SURGERY SUPEREXAM
For inquiries visit www.topnotchboardprep.com.ph or email us at topnotchmedicalboardprep@gmail.com
Item QUESTION EXPLANATION AUTHOR TOPNOTCH
# EXAM
neoplasm

351 On rectal exam, a 42 year old female is found to JESSICA MAE BACK-UP
have a hard, nodular mass 4 cm from the anal SANCHEZ, MD (TOP 4 - MIDTERM
verge. Initial diagnostic work up should include AUG 2014 MED EXAM - FEB
one of the following: BOARDS; TOPNOTCH 2015
A. Bone scan MD)
B. Scout film of the abdomen
C. Proctosigmoidoscopy
D. CT scan of the abdomen
352 A rare, premalignant condition in which the JESSICA MAE BACK-UP
gallbladder wall becomes calcified: SANCHEZ, MD (TOP 4 - MIDTERM
A. Porcelain gallbladder AUG 2014 MED EXAM - FEB
B. Adenomyomatosis of the gallbladder BOARDS; TOPNOTCH 2015
C. Hydrops of the gallbladder MD)
D. Empyema of the gallbladder
353 A 60 year old male consulted because of dysphagia JESSICA MAE BACK-UP
of 5 months duration, initially to solid foods. At SANCHEZ, MD (TOP 4 - MIDTERM
present, he is having problems swallowing even AUG 2014 MED EXAM - FEB
his own saliva. Other accompanying symptoms BOARDS; TOPNOTCH 2015
include anorexia, weight loss of 50 lbs in 4 months. MD)
Which of the following is the most likely initial
diagnosis?
A. Achalasia
B. Esophageal CA
C. Severe esophagitis from reflux
D. Zenker’s diverticulum
354 The most important prognostic factor for JESSICA MAE BACK-UP
recurrent disease and survival in breast cancer is: SANCHEZ, MD (TOP 4 - MIDTERM
A. Age AUG 2014 MED EXAM - FEB
B. Axillary nodal status BOARDS; TOPNOTCH 2015
C. ER and PR status MD)
D. Histologic grade
355 A 28 year old bus driver with his seatbelt on, In this case of blunt abdominal trauma, patient JESSICA MAE BACK-UP
figured in a vehicular crash vs another bus. At the is hemodynamically stable, and abdominal SANCHEZ, MD (TOP 4 - MIDTERM
ER, 30 minutes post-injury, his VS were noted as findings are equivocal (overt peritonitis is not AUG 2014 MED EXAM - FEB
follows: BP=130/70, PR=100/min, RR=20/min, present), therefore, FAST is warranted. BOARDS; TOPNOTCH 2015
GCS=15. Abdomen is slightly globular, with a MD)
contusion hematoma and direct tenderness at the Review algorithm for evaluation of suspected
RUQ, no rebound tenderness. What is the most abdominal trauma.
appropriate initial diagnostic procedure which Reference: Topnotch Surgery Pearls Handouts
will help you decide on the proper management? Schwartz, Principles of Surgery, 9th ed. p. 155
A. FAST
B. Triple contrast CT scan
C. DPL
D. Laparoscopy
356 A 40 year old female presented with a 5 cm Follicular thyroid cancer is more likely to JESSICA MAE BACK-UP
solitary, firm thyroid nodule as well as a scalp metastasize than the other differentiated SANCHEZ, MD (TOP 4 - MIDTERM
mass on the parieto-occipital area which showed thyroid cancers. AUG 2014 MED EXAM - FEB
lytic changes on X-ray. Biopsy of the skull lesion BOARDS; TOPNOTCH 2015
revealed colloid material, same as that seen on MD)
FNAB of the thyroid nodule. What is the most
likely diagnosis?
A. Anaplastic thyroid cancer
B. Papillary thyroid cancer
C. Follicular thyroid cancer
D. Medullary thyroid cancer
357 The appropriate management for lobular Reference: Topnotch Surgery Handouts JESSICA MAE BACK-UP
carcinoma in situ includes which of the following: Schwartz, Principles of Surgery, 9th ed. p. 454 SANCHEZ, MD (TOP 4 - MIDTERM
A. Close follow-up AUG 2014 MED EXAM - FEB
B. Radiation after excision BOARDS; TOPNOTCH 2015
C. Biopsy of the opposite breast MD)
D. Mastectomy and regional node dissection
358 A 28 year old male presents with a 3 year history JESSICA MAE BACK-UP
of left inguino-scrotal mass. On examination, the SANCHEZ, MD (TOP 4 - MIDTERM
mass measures 5x4 cm, slightly tender, and is AUG 2014 MED EXAM - FEB
reducible. The next step would be: BOARDS; TOPNOTCH 2015
A. Start IV antibiotics MD)
B. Request for ultrasound of the mass
C. Advise elective surgery
D. Admit the patient and perform emergency
surgery

TOPNOTCH MEDICAL BOARD PREP SURGERY SUPEREXAM Page 47 of 94


For inquiries visit www.topnotchboardprep.com.ph or email us at topnotchmedicalboardprep@gmail.com
TOPNOTCH MEDICAL BOARD PREP SURGERY SUPEREXAM
For inquiries visit www.topnotchboardprep.com.ph or email us at topnotchmedicalboardprep@gmail.com
Item QUESTION EXPLANATION AUTHOR TOPNOTCH
# EXAM
359 A 50 year old female consulted because of JESSICA MAE BACK-UP
yellowish discolouration of the skin and sclera. SANCHEZ, MD (TOP 4 - MIDTERM
She was diagnosed to have multiple gallstones 2 AUG 2014 MED EXAM - FEB
years ago based on ultrasound results. On PE, BOARDS; TOPNOTCH 2015
patient is icteric, afebrile, with RUQ tenderness. MD)
The most likely cause of this patient’s jaundice is:
A. Hepatitis
B. Liver abscess
C. Acute cholecystitis
D. Common bile duct stones
360 A 43 year old female who practices reverse JESSICA MAE BACK-UP
smoking presents with a 2.5x2 cm ulcer with SANCHEZ, MD (TOP 4 - MIDTERM
heaped up borders on the right floor of the mouth. AUG 2014 MED EXAM - FEB
PE showed a 1x2 cm submandibular mass on the BOARDS; TOPNOTCH 2015
right together with several enlarged discrete MD)
masses on the right upper jugular area. The most
practical way of establishing the diagnosis is to do:
A. Punch biopsy
B. Fine needle aspiration biopsy
C. Excision biopsy
D. Comprehensive history and physical
examination
361 A 58M with a 3 week history of jaundice is brought The initial treatment of patients with cholangitis MAIRRE JAMES GADDI, BACK-UP
to the ER because of lethargy. Upon seeing the includes IV antibiotics and fluid resuscitation. MD (TOP 4 - AUG 2013 MIDTERM
patient, he is febrile (39.5C) and hypotensive These patients may require intensive care unit MED BOARDS; EXAM - FEB
(80/45) with RUQ pain. The initial management monitoring and vasopressor sup- port. Most TOPNOTCH MD) 2015
for this patient would be? patients will respond to these measures.
A. Broad spectrum antimicrobial therapy However, the obstructed bile duct must be
B. Fluid resuscitation drained as soon as the patient has been
C. Relief of elevated biliary pressure stabilized. Schwartz 10th pg 1323
D. Surgery to remove the gallbladder
E. A and B
362 A 48/M diabetic sought consult at the OPD for iabetic patients with symptomatic gallstones MAIRRE JAMES GADDI, BACK-UP
recurrent abdominal pain occuring during the should have a cholecystec- tomy promptly, as MD (TOP 4 - AUG 2013 MIDTERM
night and sometimes after eating a heavy meal. they are more prone to develop acute MED BOARDS; EXAM - FEB
The pain typically lasts about 3 hours on average cholecystitis that is often severe. Schwartz 10th TOPNOTCH MD) 2015
and resolves on its own. In between episodes, the pg 1320
patient is well. How will you manage this patient?
A. Prompt cholecystectomy
B. Cystectomy during an episode of abdominal
pain
C. Observation
D. Diet modification
E. C and D
363 The Child-Turcott-Pugh scoring is for predicting Each variable score 1, 2 or 3. Bilirubin <2 MAIRRE JAMES GADDI, BACK-UP
surgical risk of intra-abdominal operations on mg/dL, 2–3 mg/dL, >3 mg/dL; albumin >3.5 MD (TOP 4 - AUG 2013 MIDTERM
cirrhotic patients. A 56/M was diagnosed with g/dL, 2.8–3.5 g/dL, <2.8 g/dL, PT INR <1.7, 1.7– MED BOARDS; EXAM - FEB
chronic liver disease from chronic hep B infection. 2.2, >2.2; encephalopathy none, controlled, TOPNOTCH MD) 2015
On PE, he was oriented to three spheres, uncontrolled; ascites none, controlled,
conversant and not in distress and negative for uncontrolled
shifting dullness Labs done showed bilirubin 3 Child A 5-6
mg/dL, albumin 1.5 g/dL, INR 1.6 and abdominal Child B 7-9
UTZ showed minimal intraabdominal fluid. What Child C 10-15 Schwartz 10th pg 1280
is the patient's CTP score?
A. Class A
B. Class B
C. Class C
D. Class D
E. None of the above
364 Which of the following is/are TRUE regarding the Neither fructose or galactose within the MAIRRE JAMES GADDI, BACK-UP
surgical metabolism of carbohydrates? circulation nor exogenous mannitol (for MD (TOP 4 - AUG 2013 MIDTERM
A. The presence of fructose, galactose and neurologic injury) evokes an insu- lin response. MED BOARDS; EXAM - FEB
mannitol in the circulation evokes an insulin The exogenous administration of small amounts TOPNOTCH MD) 2015
response of glucose (approximately 50 g/d) facilitates fat
B. Infusion of 50g per day of glucose facilitates fat entry into the TCA cycle and reduces ketosis.
entry into the TCA cycle and reduces ketosis The administration of insulin, however, has been
C. Insulin has been shown to reverse protein shown to reverse protein catabolism during
catabolism during severe stress by stimulating severe stress by stimulating protein synthesis in
protein synthesis in skeletal muscles and by skeletal muscles and by inhibiting hepato- cyte
inhibiting hepato- cyte protein degradation. protein degradation. Schwartz 10th pg 48
D. B and C
E. All of the above

365 Which of the following is associated with the Class I 1-2%; Class II - 2.1-9.5%; Class II MAIRRE JAMES GADDI, BACK-UP
highest expected infection rates? colorectal - 4-14%; Class III - 3.4-13.2%; Class IV MD (TOP 4 - AUG 2013 MIDTERM
A. Hernia repair - 3.1-12.8% Schwartz 10th pg 148 MED BOARDS; EXAM - FEB
B. Cholecystectomy TOPNOTCH MD) 2015
C. Colorectal surgery
D. Penetrating abdominal trauma
E. Necrotizing soft tissue infections

TOPNOTCH MEDICAL BOARD PREP SURGERY SUPEREXAM Page 48 of 94


For inquiries visit www.topnotchboardprep.com.ph or email us at topnotchmedicalboardprep@gmail.com
TOPNOTCH MEDICAL BOARD PREP SURGERY SUPEREXAM
For inquiries visit www.topnotchboardprep.com.ph or email us at topnotchmedicalboardprep@gmail.com
Item QUESTION EXPLANATION AUTHOR TOPNOTCH
# EXAM
366 The following are current indications for ED Indications MAIRRE JAMES GADDI, BACK-UP
thoracotomy EXCEPT? Salvageable postinjury cardiac arrest: MD (TOP 4 - AUG 2013 MIDTERM
A. Patients sustaining witnessed penetrating Patients sustaining witnessed penetrating MED BOARDS; EXAM - FEB
trauma to the torso with <15 min of prehospital trauma to the torso with <15 min of prehospital TOPNOTCH MD) 2015
CPR CPR
B. Patients sustaining witnessed blunt trauma Patients sustaining witnessed blunt trauma with
with <10 min of prehospital CPR <10 min of prehospital CPR
C. Persistent severe postinjury hypotension (SBP Patients sustaining witnessed penetrating
≤60 mm Hg) due to cardiac tamponade trauma to the neck or extremities with <5 min of
D. Air embolism prehospital CPR
E. No exception Persistent severe postinjury hypotension (SBP
≤60 mm Hg) due to:
Cardiac tamponade
Hemorrhage—intrathoracic, intra-abdominal,
extremity,cervical Air embolism
Contraindications
Penetrating trauma: CPR >15 min and no signs
of life (pupillary response, respiratory effort,
motor activity)
Blunt trauma: CPR >10 min and no signs of life
or asystole without associated tamponade
Schwartz 10th pg 167
367 A 34/M was stabbed in the neck by an unknown Hemodynamically Unstable and/or with Hard MAIRRE JAMES GADDI, BACK-UP
assailant 15 minutes ago. He was immediately signs: massive hemoptysis, rapidly expanding MD (TOP 4 - AUG 2013 MIDTERM
rushed to the ER and upon seeing the patient BP hematoma: Operative exploration regardless of MED BOARDS; EXAM - FEB
was 100/60, HR 98, RR 29, T 37 not in apparent zone Hemodynamically Stable and with soft TOPNOTCH MD) 2015
distress. The wound sustained was at the level of signs: dysphagia, venous bleeding, subcutaneous
the cricoid. He complains of hoarseness and emphysema, hematoma, hoarseness, stridor,
dysphagia. What is the initial step in the odynophagia: Zone 1-2 do CTA neck/chest if
management of the patient? positive operative exploration, Zone 3 if CTA
A. Immediately do operative exploration positive do angioembolization; Asymptomatic:
B. CT angiography Zone 1 and Zone II transcervical GSW do CTA if
C. Observe positive operative exploration, Zone II injuries
D. Local wound exploration (Not transcervical GSW) and Zone III observe
E. Close the wound with nylon 3-0 Schwartz 10th pg 176
368 Which of the following is/are TRUE regarding Patients with acute burn injuries should never MAIRRE JAMES GADDI, BACK-UP
management of burn injuries? receive pro- phylactic antibiotics. This MD (TOP 4 - AUG 2013 MIDTERM
A. Patients with acute burn injuries should be intervention has been clearly demon- strated to MED BOARDS; EXAM - FEB
given prophylactic antibiotics to prevent promote development of fungal infections and TOPNOTCH MD) 2015
overwhelming sepsis resistant organism and was abandoned in the
B. Resuscitation involves giving 3 to 4 mL/kg/% mid-1980s. Schwartz 10th pg 228 and 230
TBSA of lactated Ringer’s, half given during the
first 8 hours after burn and the remaining half
given over the subsequent 16 hours
C. Urine output should be 30 mL/h in adults and 1
to 1.5 mL/kg/h in pediatric patients
D. B and C
E. All of the above
369 MRM involves removal of the following: MRM removes the breast plus nipple areola MAIRRE JAMES GADDI, BACK-UP
A. Level I, II and III lymph nodes complex, Level I and II LNs. Removal of choice A MD (TOP 4 - AUG 2013 MIDTERM
B. Long thoracic, thoracodorsal and lateral to C plus the medial pectoral nerve is done in MED BOARDS; EXAM - FEB
pectoral nerves radical mastectomy.Topnotch TOPNOTCH MD) 2015
C. Pectoralis major and minor
D. A and C
E. None of the above

370 Weight of a normal thyroid gland: Schwartz 10th pg 1523 MAIRRE JAMES GADDI, BACK-UP
A. 10g MD (TOP 4 - AUG 2013 MIDTERM
B. 20g MED BOARDS; EXAM - FEB
C. 30g TOPNOTCH MD) 2015
D. 40g
E. 50g

371 The only drug/s shown to reduce short and long- Adverse cardiac events occur in 2% to 5% of MAIRRE JAMES GADDI, BACK-UP
term perioperative cardiac morbidity and patients undergoing noncardiac surgery and as MD (TOP 4 - AUG 2013 MIDTERM
mortality in non-cardiac surgery: many as 34% of patients undergoing vascular MED BOARDS; EXAM - FEB
A. Nitrates surgery. Certain perioperative cardiac events, TOPNOTCH MD) 2015
B. Anti-platelet agents such as myocardial infarction, are associated
C. Beta-blockers with a mortality rate of 40% to 70% per event,
D. Calcium channel blockers prolonged hospitaliza- tion, and higher costs.
E. ACE inhibitors Appropriately administered β-blockers reduce
perioperative ischemia, especially in at-risk
patients. It has been found that nearly half of the
fatal cardiac events could be preventable with β-
blocker therapy. Schwartz 10th pg 374
372 Which of the following is/are TRUE regarding Choice B refers to inosculation Schwartz 10th pg MAIRRE JAMES GADDI, BACK-UP
graft take? 1833 MD (TOP 4 - AUG 2013 MIDTERM
A. Imbibition refers to the first 24 to 48 hours after MED BOARDS; EXAM - FEB
skin grafting, wherein a thin film of fibrin and TOPNOTCH MD) 2015
plasma separates the graft from the underlying
wound bed
B. Angiogenesis occurs after 48 hours and during
this time, fine vascular network begins to form
within the fibrin layer
C. Graft failure can be caused by infection, shear,
hematoma or seroma formation
TOPNOTCH MEDICAL BOARD PREP SURGERY SUPEREXAM Page 49 of 94
For inquiries visit www.topnotchboardprep.com.ph or email us at topnotchmedicalboardprep@gmail.com
TOPNOTCH MEDICAL BOARD PREP SURGERY SUPEREXAM
For inquiries visit www.topnotchboardprep.com.ph or email us at topnotchmedicalboardprep@gmail.com
Item QUESTION EXPLANATION AUTHOR TOPNOTCH
# EXAM
D. A and C
E. All of the above

373 A 34/M smoker complains of recurrent headache Symptoms associated with nasopharyngeal MAIRRE JAMES GADDI, BACK-UP
and unilateral ear discharge with associated tumors include nasal obstruction, posterior MD (TOP 4 - AUG 2013 MIDTERM
hearing loss. On PE, you noted matted lymph (level V) neck mass, epistaxis, head- ache, serous MED BOARDS; EXAM - FEB
nodes in the posterior neck. What is the standard otitis media with hearing loss, and otalgia. TOPNOTCH MD) 2015
treatment for this disorder? Cranial nerve involvement is indicative of skull
A. Chemoradiation base extension and advanced disease. The
B. Radical resection of the tumor standard treatment for nasopharyngeal
C. Chemotherapy carcinoma is chemo- radiation. Combination
D. Radiotherapy therapy produces superior survival rates for
E. None of the above nasopharyngeal carcinoma in comparison to
radiation alone. Schwartz 10th pg 594
374 Main suspected organism for contact lens users MAIRRE JAMES GADDI, BACK-UP
with microbial keratitis? MD (TOP 4 - AUG 2013 MIDTERM
A. Fungi MED BOARDS; EXAM - FEB
B. Pseudomonas aeruginosa TOPNOTCH MD) 2015
C. Staphylococcus aureus
D. Herpes simplex
E. Adenovirus

375 Elective repair of descending thoracic aneurysms Recommend elective operation in asymptomatic MAIRRE JAMES GADDI, BACK-UP
in asymptomatic patients should be done when? patients when the diameter of an ascending MD (TOP 4 - AUG 2013 MIDTERM
A. diameter > 4.5 cm aortic aneurysm is >5.5 cm, when the diameter MED BOARDS; EXAM - FEB
B. diameter > 5.5 cm of a descending thoracic aortic aneurysm is >6.0 TOPNOTCH MD) 2015
C. diameter > 6 cm cm, or when the rate of dilata- tion is >0.5 cm/y.
D. rate of dilatation < 0.5 cm per year For patients undergoing aortic valve
E. B and C replacement or repair, smaller ascending aortic
aneurysms (>4.5 cm) are considered for
concomitant repair.
376 Treatment modalities available for the treatment Schwartz 10th pg 1223 MAIRRE JAMES GADDI, BACK-UP
of hemorrhoids include the following EXCEPT? MD (TOP 4 - AUG 2013 MIDTERM
A. Rubber band ligation MED BOARDS; EXAM - FEB
B. Infrared photocoagulation TOPNOTCH MD) 2015
C. Sclerotherapy
D. Operative hemorrhoidectomy
E. No exception

377 TRUE of Passaro's triangle EXCEPT: B- the inferolateral point Schwartz 10th pg 1392 MAIRRE JAMES GADDI, BACK-UP
A. Up to 90% of ZES tumors are found within the MD (TOP 4 - AUG 2013 MIDTERM
triangle MED BOARDS; EXAM - FEB
B. The medial point of the triangle is found at the TOPNOTCH MD) 2015
confluence of the 2nd and 3rd part of the
duodenum
C. Tumors located outside the triangle have the
worst prognosis
D. A and B
E. All are true
378 The Amsterdam criteria for clinical diagnosis of The Amsterdam criteria for clinical diagnosis of MAIRRE JAMES GADDI, BACK-UP
HNPCC EXCEPT: HNPCC are three affected relatives with MD (TOP 4 - AUG 2013 MIDTERM
A. Three affected relatives with histologically histologically verified adenocarcinoma of the MED BOARDS; EXAM - FEB
verified adenocarcinoma of the large bowel large bowel (one must be a first-degree relative TOPNOTCH MD) 2015
B. Two successive generations of a family of one of the others) in two successive
C. At least one patient diagnosed before age 50 generations of a family with one patient
D. Presence of other HNPCC-related carcinomas diagnosed before age 50 years. FAP is not an
such as FAP HNPCC related CA Schwartz 10th pg 1207
E. No exception
379 Most common location of aneurysms? Choices arranged from most common to least MAIRRE JAMES GADDI, BACK-UP
A. Anterior communicating artery common with the following frequencies: 30%, MD (TOP 4 - AUG 2013 MIDTERM
B. Posterior communicating artery 25%, 20, 10%, 5% Schwartz 10th pg 1730 MED BOARDS; EXAM - FEB
C. Middle cerebral artery bifurcation TOPNOTCH MD) 2015
D. Basilar artery tip
E. Vertebral artery

380 A 14/M presents with severe local pain and Osgood-Schlatter disease is a common problem MAIRRE JAMES GADDI, BACK-UP
tenderness in the area of the tibial tubercle. The most often seen in athletically active MD (TOP 4 - AUG 2013 MIDTERM
patient was previously healthy and engages in adolescents. This disorder is characterized by MED BOARDS; EXAM - FEB
multiple activities such as football, running and ossification in the distal patellar tendon at the TOPNOTCH MD) 2015
cycling. How will you manage the patient? point of its tibial insertion. This disorder is
A. Activity restriction thought to result from mechanical stress on the
B. NSAIDs tendinous insertion. Radiographs show calcified
C. Surgical correction ossicles within the tendon at its insertion. The
D. Steroids disease presents with severe local pain and
E. None of the above tenderness in the area of the tibial tubercle.
Treatment for the disease is activity restriction.
If the symptoms are improved, athletic

TOPNOTCH MEDICAL BOARD PREP SURGERY SUPEREXAM Page 50 of 94


For inquiries visit www.topnotchboardprep.com.ph or email us at topnotchmedicalboardprep@gmail.com
TOPNOTCH MEDICAL BOARD PREP SURGERY SUPEREXAM
For inquiries visit www.topnotchboardprep.com.ph or email us at topnotchmedicalboardprep@gmail.com
Item QUESTION EXPLANATION AUTHOR TOPNOTCH
# EXAM
participation can be resumed. Schwartz 10th pg
1783

381 Full-thickness rupture of the esophageal wall as a SIMILAR TO PREVIOUS BOARD EXAM SCOTT RILEY ONG, MD BACK-UP
result of forceful vomiting, an entity known as CONCEPT/PRINCIPLE. (TOP 5 - AUG 2014 MIDTERM
Boerhaave syndrome, most commonly occurs in MED BOARDS; EXAM - FEB
which part of the esophagus? TOPNOTCH MD) 2015
A. Right posterolateral wall of the distal esophagus
B. Right posterolateral wall of the mid esophagus
C. Left anterolateral wall of the distal esophagus
D. Left anterolateral wall of the mid esophagus
E. Left posterolateral wall of the distal esophagus

382 A 60-year old male presents with left lower SIMILAR TO PREVIOUS BOARD EXAM SCOTT RILEY ONG, MD BACK-UP
quadrant abdominal pain with direct tenderness, CONCEPT/PRINCIPLE. (TOP 5 - AUG 2014 MIDTERM
fever and leukocytosis. Which of the following MED BOARDS; EXAM - FEB
diagnostic modalities will be your test of choice to TOPNOTCH MD) 2015
confirm your diagnosis and to evaluate this
patient?
A. Barium enema
B. Holoabdominal ultrasound
C. Plain radiograph of the abdomen
D. Contrast-enhanced CT scan
E. Colonoscopy
383 Which of the following structures are spared SIMILAR TO PREVIOUS BOARD EXAM SCOTT RILEY ONG, MD BACK-UP
during Whipple's procedure? CONCEPT/PRINCIPLE. The following structures (TOP 5 - AUG 2014 MIDTERM
A. Gallbladder are removed during a standard Whipple's MED BOARDS; EXAM - FEB
B. Duodenum procedure: pancreatic head, duodenum, TOPNOTCH MD) 2015
C. Common bile duct proximal jejunum, gallbladder and CBD.
D. Distal stomach
E. Proximal jejunum

384 The critical diameter of an abdominal aortic SIMILAR TO PREVIOUS BOARD EXAM SCOTT RILEY ONG, MD BACK-UP
aneurysm where there is significant risk of CONCEPT/PRINCIPLE (TOP 5 - AUG 2014 MIDTERM
rupture is: MED BOARDS; EXAM - FEB
A. 4.5 cm TOPNOTCH MD) 2015
B. 5 cm
C. 5.5 cm
D. 6 cm
E. 7 cm

385 A 40-year old female presented with a 3-cm SIMILAR TO PREVIOUS BOARD EXAM SCOTT RILEY ONG, MD BACK-UP
palpable, non-movable, painless mass on the CONCEPT/PRINCIPLE. You should first request (TOP 5 - AUG 2014 MIDTERM
upper outer quadrant of her right breast. No diagnostic mammography to look for other mass MED BOARDS; EXAM - FEB
axillary nodes are appreciated. Ultrasound showed lesions in the breasts. TOPNOTCH MD) 2015
that the mass is hypoechoic with indistinct
margins. What is the next best step in your
management?
A. Order diagnostic mammography.
B. Perform core-needle biopsy.
C. Excise the mass and send to the laboratory for
biopsy.
D. Do fine-needle biopsy.
E. Admit the patient and perform breast
conservation surgery.
386 A patient with Crohn disease has been sustaining SIMILAR TO PREVIOUS BOARD EXAM SCOTT RILEY ONG, MD BACK-UP
fluid losses due to the presence of colovesical CONCEPT/PRINCIPLE. For fluid losses from the (TOP 5 - AUG 2014 MIDTERM
fistula. Which of the following IV replacement stomach and colon, give NSS. Add KCl if with MED BOARDS; EXAM - FEB
fluids would be the best choice to give to this hypokalemia. For fluid losses from the pancreas TOPNOTCH MD) 2015
patient? and small bowel, give LR.
A. plain NSS
B. Ringer's lactate
C. D5LR
D. Colloid solution
E. Any of the above
387 How many calories would you obtain from 3L of SIMILAR TO PREVIOUS BOARD EXAM SCOTT RILEY ONG, MD BACK-UP
D5LR? CONCEPT/PRINCIPLE. D5LR = 200 kcal/L. D5W (TOP 5 - AUG 2014 MIDTERM
A. 200 kcal = 170 kcal/L. MED BOARDS; EXAM - FEB
B. 300 kcal TOPNOTCH MD) 2015
C. 400 kcal
D. 500 kcal
E. 600 kcal

TOPNOTCH MEDICAL BOARD PREP SURGERY SUPEREXAM Page 51 of 94


For inquiries visit www.topnotchboardprep.com.ph or email us at topnotchmedicalboardprep@gmail.com
TOPNOTCH MEDICAL BOARD PREP SURGERY SUPEREXAM
For inquiries visit www.topnotchboardprep.com.ph or email us at topnotchmedicalboardprep@gmail.com
Item QUESTION EXPLANATION AUTHOR TOPNOTCH
# EXAM
388 Which among the following microorganisms is SCOTT RILEY ONG, MD BACK-UP
most commonly implicated in fatal burn wound (TOP 5 - AUG 2014 MIDTERM
infections? MED BOARDS; EXAM - FEB
A. Staphylococcus aureus TOPNOTCH MD) 2015
B. Pseudomonas aeruginosa
C. Klebsiella granulomatis
D. Acinetobacter baumanii
E. Streptococcus pyogenes

389 Ultrasound is the most common diagnotic SCOTT RILEY ONG, MD BACK-UP
modality requested for initial evaluation of (TOP 5 - AUG 2014 MIDTERM
problems affecting the biliary tree. The common MED BOARDS; EXAM - FEB
bile duct is said to be dilated if its width is more TOPNOTCH MD) 2015
than how many millimeters?
A. >4 mm
B. >6 mm
C. >8 mm
D. >10 mm
E. >12 mm
390 Which of the following CT scan findings would SCOTT RILEY ONG, MD BACK-UP
strongly suggest a diagnosis of hepatocellular (TOP 5 - AUG 2014 MIDTERM
carcinoma? MED BOARDS; EXAM - FEB
A. Strong arterial enhancement and early washout TOPNOTCH MD) 2015
on portal venous phase of a 5-cm hepatic mass
B. Multiple non-enhancing hepatic masses ranging
from 0.5 to 2.0 cm in size
C. Enhancement on both arterial and portal venous
phases with delayed washout of a 4-cm hepatic
mass
D. Initial peripheral enhancement with gradual
centripetal fill-in of a 6.0-cm hepatic mass
E. 3-cm lobulated hepatic mass in the left lobe with
scattered areas of fat attenuation
391 Which of the following ultrasound findings is most SIMILAR TO PREVIOUS BOARD EXAM SCOTT RILEY ONG, MD BACK-UP
specific for chronic calculous cholecystitis? CONCEPT/PRINCIPLE. A thickened gallbladder (TOP 5 - AUG 2014 MIDTERM
A. Thickened gallbladder wall >3mm wall is a non-specific finding of cholecystitis; it MED BOARDS; EXAM - FEB
B. Contracted gallbladder, persistently <2cm may occur in both acute and chronic cases. The TOPNOTCH MD) 2015
despite adequate fasting presence of stones (gravity-dependent
C. Presence of pericholecystic fluid echogenic foci within the lumen) indicates
D. Gallbladder hydrops, >5 cm cholelithiasis, but not necessarily cholecystitis.
E. Visualization of highly echogenic foci with
posterior shadowing within the gallbladder lumen.

392 A 36-year old male is to undergo elective surgery Colorectal and small bowel surgery: cefazolin + SCOTT RILEY ONG, MD BACK-UP
for complicated duodenal ulcer disease. Which of metronidazole. Gastroduodenal surgery: (TOP 5 - AUG 2014 MIDTERM
the following antibiotics will you give as cefoxitin or cefotetan. Biliary tract with active MED BOARDS; EXAM - FEB
prophylaxis? infection: ampicillin-sulbactam. TOPNOTCH MD) 2015
A. Cefazolin
B. Metronidazole
C. Ampicillin-sulbactam
D. Cefoxitin
E. Penicillin G
393 An uncomplicated appendectomy is classified as Class I: hernia repair, breast biopsy. Class II: SCOTT RILEY ONG, MD BACK-UP
what type of surgical wound? appendectomy, elective GI surgery (except those (TOP 5 - AUG 2014 MIDTERM
A. Class I: clean involving the colon), cholecystectomy. Class III: MED BOARDS; EXAM - FEB
B. Class ID: clean with prosthetic device penetrating trauma, enterotomy. Class IV: TOPNOTCH MD) 2015
C. Class II: clean/contaminated perforated diverticulitis, necrotizing infections.
D. Class III: contaminated
E. Class IV: dirty

394 Which of the following statements describes The rest of the statements apply to both keloid SCOTT RILEY ONG, MD BACK-UP
keloid formation but not hypertrophic scars? and hypertrophic scars. (TOP 5 - AUG 2014 MIDTERM
A. Dense accumulation of fibrous tissue MED BOARDS; EXAM - FEB
B. Lesion extending beyond the border of the TOPNOTCH MD) 2015
original wound
C. Thick, raised scars over a previous wound
D. Treatment with intralesional steroid injection
has been effective
E. Due to failure of collagen breakdown
395 A 25-year old female presented to the emergency SIMILAR TO PREVIOUS BOARD EXAM SCOTT RILEY ONG, MD BACK-UP
department due to a 5-cm laceration on her right CONCEPT/PRINCIPLE. Use a 6-0 suture for the (TOP 5 - AUG 2014 MIDTERM
cheek sustained from a broken shard of glass. face. Nylon would be the best choice since it is MED BOARDS; EXAM - FEB
Which of the following sutures would you use for synthetic and would least likely cause allergy TOPNOTCH MD) 2015
this patient? and/or inflammatory reaction, thus promoting
A. Silk 6-0 better aesthetic outcome. Chromic and silk are
B. Chromic 6-0 biological sutures.
C. Nylon 6-0
D. Cotton 4-0
E. Vicryl 4-0

TOPNOTCH MEDICAL BOARD PREP SURGERY SUPEREXAM Page 52 of 94


For inquiries visit www.topnotchboardprep.com.ph or email us at topnotchmedicalboardprep@gmail.com
TOPNOTCH MEDICAL BOARD PREP SURGERY SUPEREXAM
For inquiries visit www.topnotchboardprep.com.ph or email us at topnotchmedicalboardprep@gmail.com
Item QUESTION EXPLANATION AUTHOR TOPNOTCH
# EXAM
396 In relation to the patient described in the Face: 3-5 days. Neck: 5-7 days. Scalp: 7-12 days. SCOTT RILEY ONG, MD BACK-UP
preceding question, when will you advice her to Extremities: 10-14 days. (TOP 5 - AUG 2014 MIDTERM
return for suture removal? MED BOARDS; EXAM - FEB
A. 3-5 days TOPNOTCH MD) 2015
B. 5-7 days
C. 7-12 days
D. 10-14 days
E. 14-28 days

397 The most important reason for requesting a KUB- SCOTT RILEY ONG, MD BACK-UP
IVP in a patient with a stab wound in the left flank (TOP 5 - AUG 2014 MIDTERM
and fross hematuria is to assess which of the MED BOARDS; EXAM - FEB
following? TOPNOTCH MD) 2015
A. Degree of renal injury
B. Location for best surgical approach
C. Site of possible perforation
D. Functional status of the right kidney
E. Viability of the left kidney
398 Which of the following describes the most A: type I. B: type II. C: type III. D: type IV. E: type SCOTT RILEY ONG, MD BACK-UP
common type of Salter-Harris fracture? V. Type II is the most common. Type III, IV and V (TOP 5 - AUG 2014 MIDTERM
A. Fracture across the growth plate without have poorer prognosis. MED BOARDS; EXAM - FEB
epiphyseal and metaphyseal involvement TOPNOTCH MD) 2015
B. Fracture across the growth plate with
metaphyseal involvement but not epiphyseal
involvement
C. Fracture across the growth plate with
epiphyseal involvement but not metaphyseal
involvement
D. Fracture across the growth plate with
metaphyseal and epiphyseal involvement
E. Compression fracture on the growth plate
399 Which of the following is the only extraocular SCOTT RILEY ONG, MD BACK-UP
muscle that does not have any fascial attachment (TOP 5 - AUG 2014 MIDTERM
to any other extraocular muscles? As such, during MED BOARDS; EXAM - FEB
surgery, when it is accidentally disinserted, it TOPNOTCH MD) 2015
becomes difficult to capture and reinsert it in its
proper position.
A. Superior rectus
B. Inferior rectus
C. Lateral rectus
D. Medial rectus
E. Inferior oblique
400 What is the most common intracranial SIMILAR TO PREVIOUS BOARD EXAM SCOTT RILEY ONG, MD BACK-UP
complication of otitis media? CONCEPT/PRINCIPLE. (TOP 5 - AUG 2014 MIDTERM
A. Temporal lobe abscess MED BOARDS; EXAM - FEB
B. Meningitis TOPNOTCH MD) 2015
C. Epidural abscess
D. Subdural effusion
E. Encephalitis

401 An intern without your supervision administered SIMILAR TO PREVIOUS BOARD EXAM JOSE CARLO DIAGNOSTIC
10 mL of 2% Lidocaine into a patient for wound CONCEPT/PRINCIPLE. A 2% Lidocaine MASANGKAY III, MD EXAM - AUG
suture. What dose was administered? preparation is 20mg/mL (10 x 20 = 200mg) (TOP 8 - FEB 2014 2014
A. 20 mg MED BOARDS;
B. 40 mg TOPNOTCH MD)
C. 100 mg
D. 200 mg
E. None of the above

402 A malnourished Colon Adenocarcinoma patient a liter of D5LR provides 170 kcal. (SIMILAR TO JOSE CARLO DIAGNOSTIC
was transfused 2 liters of D5LR, how may PREVIOUS BOARD EXAM CONCEPT/PRINCIPLE) MASANGKAY III, MD EXAM - AUG
kilocalories was provided to the patient? (TOP 8 - FEB 2014 2014
A. 240 kcal MED BOARDS;
B. 340 kcal TOPNOTCH MD)
C. 440 kcal
D. 540 kcal
E. 640 kcal

403 A cytokine secreted by the Macrophages which JOSE CARLO DIAGNOSTIC


was found to be the major inducer of muscle MASANGKAY III, MD EXAM - AUG
catabolism and cachexia during stress. (TOP 8 - FEB 2014 2014
A. IL1 MED BOARDS;
B. IL2 TOPNOTCH MD)
C. IL3
D. TNF-alpha
E. TNF-beta

TOPNOTCH MEDICAL BOARD PREP SURGERY SUPEREXAM Page 53 of 94


For inquiries visit www.topnotchboardprep.com.ph or email us at topnotchmedicalboardprep@gmail.com
TOPNOTCH MEDICAL BOARD PREP SURGERY SUPEREXAM
For inquiries visit www.topnotchboardprep.com.ph or email us at topnotchmedicalboardprep@gmail.com
Item QUESTION EXPLANATION AUTHOR TOPNOTCH
# EXAM
404 A patient with Familial Adenomatous Polyposis Colonic secretions contain the highest JOSE CARLO DIAGNOSTIC
underwent Total Proctocolectomy, which of the Potassium amounting to 35 mEq/L. MASANGKAY III, MD EXAM - AUG
following Electrolytes will most likely be deficient (TOP 8 - FEB 2014 2014
in this patient and thus must be replaced? MED BOARDS;
A. Sodium TOPNOTCH MD)
B. Potassium
C. Chloride
D. Bicarbonate
E. Calcium

405 You managed to suture a lacerated wound in the Sutures done on the face can be removed after 3 JOSE CARLO DIAGNOSTIC
ED on the cheek of a patient after an apparent - 5 days. MASANGKAY III, MD EXAM - AUG
brawl. When would you ask the patient to come (TOP 8 - FEB 2014 2014
back for suture removal? MED BOARDS;
A. 1-2 days TOPNOTCH MD)
B. 3-5 days
C. 5-7 days
D. 7-12 days
E. 10-14 days
406 After an apparent heated argument with a "Siga", a Although the patient is hemodynamically stable, JOSE CARLO DIAGNOSTIC
patient sustained a penetrating Gunshot wound in patient presents with hoarseness which is a sign MASANGKAY III, MD EXAM - AUG
the neck above the cricoid cartilage and the angle that a neck structure was severed, Neck (TOP 8 - FEB 2014 2014
of the mandible, on the right. Patient is exploration should be done immediately. MED BOARDS;
hemodynamically stable with noted hoarseness of TOPNOTCH MD)
the voice. Which of the following would be the
appropriate management?
A. Observe only, the patient is hemodynamically
stable
B. Do a CT-Scan
C. Explore Immediately
D. Do Angiography
E. None of the above
407 A 50 kg burn patient sustained 2nd and 3rd degree 19% TBSA (4x50x19=3800) 3,800 mL JOSE CARLO DIAGNOSTIC
burns on the anterior right thigh, leg and foot, the /2=1,900mL to run for 8 hours, the other half to MASANGKAY III, MD EXAM - AUG
perineum and the whole anterior abdomen, How run for 6 hours. (TOP 8 - FEB 2014 2014
will the IV fluid be initiated? MED BOARDS;
A. 3.2 L PLR fast drip TOPNOTCH MD)
B. 2.35 L PLR fast drip
C. 2.35 L PLR for 8 hours
D. 1.9 PLR fast drip
E. 1.9 PLR for 8 hours

408 A trauma patient with gross hemorrhage from a patien is probably in stage II hypovolemic JOSE CARLO DIAGNOSTIC
large hacking wound in the thigh with the shock/ compensated shock. MASANGKAY III, MD EXAM - AUG
following Vital signs: CR:125 bpm, RR: 22 cpm, BP: (TOP 8 - FEB 2014 2014
110/80, may have an approximate blood loss of MED BOARDS;
around: TOPNOTCH MD)
A. less than 500 mL
B. 500 - 750 mL
C. 750 - 1,500 mL
D. 1,500 - 2,000 mL
E. More than 2,000 mL
409 While watching TV, a family requested help from Kasai Procedure oe Early JOSE CARLO DIAGNOSTIC
"Sagip Kapamilya" for their child to undergo a hepaticoportoenterostomy is a surgical MASANGKAY III, MD EXAM - AUG
Kasai Procedure, you knew that the proper timing procedure for Biliary Atresia which is properly (TOP 8 - FEB 2014 2014
for a Kasai Procedure is: timed prior to 2 months old which is done to MED BOARDS;
A. Immediately at birth lessen the chance for progression to a liver TOPNOTCH MD)
B. Prior to 2 months transplant.
C. At 1 year old
D. before development of speech
E. when the child reached 10 kg

410 A burn patient will undergo reconstructive Split thickness skin graft is used for large JOSE CARLO DIAGNOSTIC
surgery due to a 10% TBSA burn on the right surface area wounds, frequently burns in which MASANGKAY III, MD EXAM - AUG
lower extremity which of the following is a more coverage is more impportant than function. (TOP 8 - FEB 2014 2014
appropriate management? MED BOARDS;
A. Do a Split thickness graft TOPNOTCH MD)
B. Do a Full thickness graft
C. Do a local flap
D. Do a pedicled flap
E. Do a free flap
411 Few years after mastectomy a woman developed Stewart-Treves' Syndrome is the development JOSE CARLO DIAGNOSTIC
Stewart-Treves' Syndrome. Which of the following of Lymphoangiosarcoma of the limb due to the MASANGKAY III, MD EXAM - AUG
is the most appropriate management? absence of lymphatic drainage, amputation of (TOP 8 - FEB 2014 2014
A. Observe only and apply warm compress the affected limb is the management. MED BOARDS;
B. Prescribe antibiotics TOPNOTCH MD)
C. Keep affected arm elevated
D. Do contralateral mastectomy
E. Amputation of the limb

TOPNOTCH MEDICAL BOARD PREP SURGERY SUPEREXAM Page 54 of 94


For inquiries visit www.topnotchboardprep.com.ph or email us at topnotchmedicalboardprep@gmail.com
TOPNOTCH MEDICAL BOARD PREP SURGERY SUPEREXAM
For inquiries visit www.topnotchboardprep.com.ph or email us at topnotchmedicalboardprep@gmail.com
Item QUESTION EXPLANATION AUTHOR TOPNOTCH
# EXAM
412 A patient with a neck mass is suffering from Retropharyngeal abscess presents with the JOSE CARLO DIAGNOSTIC
Ludwig's Angina, What is the primary problem? following symptoms: fever, irritability, drooling, MASANGKAY III, MD EXAM - AUG
A. Peritonsillar Absence nuchal rigidity, dysphagia and airway (TOP 8 - FEB 2014 2014
B. Laryngeal Tumor obstruction (Ludwg's Angina) MED BOARDS;
C. Nasopharyngeal CA TOPNOTCH MD)
D. Adenoid Cystic Carcinoma
E. Retropharyngeal Abscess

413 Following an abdominal operation a patient had Colonic motility is the last to return after a JOSE CARLO DIAGNOSTIC
postoperative ileus, colonic motility will return postoperative ileus, SI motility within the 1st 24 MASANGKAY III, MD EXAM - AUG
after a period of: hours, gastric motility within 48 hours. (TOP 8 - FEB 2014 2014
A. 12 hours MED BOARDS;
B. 24 hours TOPNOTCH MD)
C. 48 hours
D. 3-5 days
E. 4-6 days

414 A patient with Liver Cirrhosis will undergo a Selective shunts such as the Warren shunt or the JOSE CARLO DIAGNOSTIC
surgical shunt to decrease portal hypertension, distal splenorenal shunt has lower incidence of MASANGKAY III, MD EXAM - AUG
which of the following has a lower incidence of hepatic enephalopathy. SIMILAR TO PREVIOUS (TOP 8 - FEB 2014 2014
hepatic encephalopathy? BOARD EXAM CONCEPT/PRINCIPLE MED BOARDS;
A. End-to-side portocaval shunt TOPNOTCH MD)
B. Side-to-side portocaval shunt
C. Drapanas Shunt
D. Distal splenorenal shunt
E. all of the above

415 A young woman with prolonged OCP use was Hepatic Adenoma may transform into a well- JOSE CARLO DIAGNOSTIC
recently diagnosed with Hepatic Adenoma, Which diferrentiated HCC, may also rupture and cause MASANGKAY III, MD EXAM - AUG
of the following statements would you tell the intraperitoneal bleeding (TOP 8 - FEB 2014 2014
patient? MED BOARDS;
A. It is the most common benign lesion found in TOPNOTCH MD)
the liver
B. It usually do not rupture spontaneously
C. It has been there since the patient was born
therefore is congenital
D. It has a risk for malignant transformation
E. It can rupture and cause Tumor Lysis
Syndrome
416 Which of the following compartments is not JOSE CARLO DIAGNOSTIC
included in the abdominal assesment using FAST? MASANGKAY III, MD EXAM - AUG
A. Pericardium (TOP 8 - FEB 2014 2014
B. Hepatorenal Recess MED BOARDS;
C. Splenorenal recess TOPNOTCH MD)
D. Foramen of Winslow
E. Pouch of Douglas

417 A 30 year old gravid patient on her 32nd week of BCS is done in pregnant patients on their 3rd JOSE CARLO DIAGNOSTIC
gestation was diagnosed with Stage IIIB invasive trimester, Chemo and RT should be done at MASANGKAY III, MD EXAM - AUG
ductal CA, what is the most appropriate/practical Puerperium. (TOP 8 - FEB 2014 2014
management? MED BOARDS;
A. Terminate pregnancy at any trimester and do TOPNOTCH MD)
MRM
B. Do lumpectomy only then complete MRM at
Puerperium
C. Initiate chemotherapy, All surgeries and
radiotherapy at Puerperium
D. Do MRM, Chemotherapy then radiation at
puerperium
E. Do Breast Conservation therapy
418 A patient with diverticulitis underwent a CT-scan JOSE CARLO DIAGNOSTIC
of the abdomen, at which Hinchey stage is he in if MASANGKAY III, MD EXAM - AUG
there is noted Large mesenteric Abscess? (TOP 8 - FEB 2014 2014
A. Stage I MED BOARDS;
B. Stage II TOPNOTCH MD)
C. Stage III
D. Stage IV
E. Stage V

419 On a patient with suspected lower GI bleeding, Insertion of an NGT should be done next to rule JOSE CARLO DIAGNOSTIC
after resuscitation and stabilizing the patient, out an Upper GI bleeding first, because the most MASANGKAY III, MD EXAM - AUG
what should be the next step of management? common cause of a LGIB is still UGIB. (TOP 8 - FEB 2014 2014
A. Direct to OR for immediate exploration MED BOARDS;
B. Do Immediate EGD TOPNOTCH MD)
C. Do an urgent Colonoscopy
D. Insert an NGT
E. Administer Procoagulants

TOPNOTCH MEDICAL BOARD PREP SURGERY SUPEREXAM Page 55 of 94


For inquiries visit www.topnotchboardprep.com.ph or email us at topnotchmedicalboardprep@gmail.com
TOPNOTCH MEDICAL BOARD PREP SURGERY SUPEREXAM
For inquiries visit www.topnotchboardprep.com.ph or email us at topnotchmedicalboardprep@gmail.com
Item QUESTION EXPLANATION AUTHOR TOPNOTCH
# EXAM
420 A patient with a Tumor of the GI tract is Imatinib is the Chemotherapeutic Drug of choice JOSE CARLO DIAGNOSTIC
undergoing chemotherapy with Imatinib, a for GIST(Gastrointestinal Stromal tumor) which MASANGKAY III, MD EXAM - AUG
Tyrosine Kinase inhibitor, in this case against c- arises from Interstitial Cells of Cajal (TOP 8 - FEB 2014 2014
KIT mutation, in which cell did the tumor probably MED BOARDS;
arise from? TOPNOTCH MD)
A. Brunner's Gland
B. Interstitial Cells of Cajal
C. Goblet Cells
D. Auerbach's Plexus
E. Mucosa Associated Lymphatic Tissues
421 What is considered to be the earliest sign of an on- WEBSTER ALINDOG, MIDTERM 1
going blood loss? MD (TOP 3 - FEB 2014 EXAM - AUG
A. Hypotension MED BOARDS; 2014
B. Cold clammy extremities TOPNOTCH MD)
C. Tachypnea
D. Tachycardia

422 Air emboli can form after a blunt or penetrating WEBSTER ALINDOG, MIDTERM 1
trauma involves an airway with entry of air into MD (TOP 3 - FEB 2014 EXAM - AUG
damaged pulmonary vein and before finally MED BOARDS; 2014
reaching the left heart. However, the volume of air TOPNOTCH MD)
required that will result in clinically significant
embolus is at least:
A. 50 cc
B. 100 cc
C. 200 cc
D. 500 cc

423 Meckel's diverticulum is associated with what is The rule of 2's: 2% of the population, within 2 WEBSTER ALINDOG, MIDTERM 1
known as the "rule of 2's." One of these includes: feet of the ileocecal valve, 2 inches in length, 2 MD (TOP 3 - FEB 2014 EXAM - AUG
A. Typically found about 2 feet from the ileocecal types of heterotopic Mucosa, and presentation MED BOARDS; 2014
valve before the age of 2. The most common clinical TOPNOTCH MD)
B. Most commonly seen in children older than 2 presentations are: 1) lower gastro-intestinal
years bleeding secondary to an ulceration produced
C. Less than 2 cm long by heterotopic gastric mucosa; 2) intestinal
D. All of the above obstruction; from internal volvulus or
intussusception; 3) and local inflammation with
or without perforation resembling acute
appendicitis.
424 A 47-year old male with long-standing This is a case of aortic dissection which is WEBSTER ALINDOG, MIDTERM 1
uncontrolled hypertension and a history of drug defined as the separation of the layers within MD (TOP 3 - FEB 2014 EXAM - AUG
abuse presents to the emergency department with the aortic wall (intima-media) with blood MED BOARDS; 2014
severe pain described as "tearing" sensation that entering the space leading to propagation of the TOPNOTCH MD)
radiates through his back. Which of the following dissection. Clues here are "uncontrolled
conditions/findings will best serve as an absolute hypertension, hx of drug abuse, and sudden
indication for immediate surgical intervention? onset of a severe "tearing" chest pain (classic).
A. Recurrent shooting chest pain Presence of widened mediastinum with rub
B. Headache with at least 8/10 pain scale score suggests significant propagation with strong
C. Widened mediastinum with rub consideration of developing cardiac tamponade.
D. Urinary retention with rising BUN/creatinine More specifically, emergency surgical
levels intervention is preferred in Stanford type A
(DeBakey type I and II) ascending aortic
dissection and complicated Stanford type B
(DeBakey type III) aortic dissections with
specific clinical or radiologic evidence.
425 Carcinoembryonic antigen (CEA) can be used as a Must know the tumor markers. CEA is found in WEBSTER ALINDOG, MIDTERM 1
tumor marker for: malignancies of the GI (colon), cervix, ovary, MD (TOP 3 - FEB 2014 EXAM - AUG
A. Prostate breast, lung, and urinary tract. AFP for germ cell MED BOARDS; 2014
B. Pancreas tumor and hepatocellular CA. CA 15-3 in breast TOPNOTCH MD)
C. Breast CA. CA 19-9 mainly in pancreatic cancer but also
D. Lung colorectal. CA 125, mainly ovarian. Calcitonin for
medullary thyroid carcinoma. hCG for
gestational trophoblastic disease,
choriocarcinoma, and germ cell tumor. Inhibin
for sex cord-gonadal stromal tumor. PSA for
prostatic cancer. S-100 for melanoma, sarcoma,
astrocytoma. Vimentin in renal cell CA, sarcoma
and endometrial CA.
426 True of extracellular volume excess: WEBSTER ALINDOG, MIDTERM 1
A. Edema is normally prevented by action of the MD (TOP 3 - FEB 2014 EXAM - AUG
lymphatics MED BOARDS; 2014
B. Involves increases in both plasma and TOPNOTCH MD)
interstitial fluid volumes
C. Symptoms are usually restricted to
cardiovascular changes
D. All of the above
E. None of the above

TOPNOTCH MEDICAL BOARD PREP SURGERY SUPEREXAM Page 56 of 94


For inquiries visit www.topnotchboardprep.com.ph or email us at topnotchmedicalboardprep@gmail.com
TOPNOTCH MEDICAL BOARD PREP SURGERY SUPEREXAM
For inquiries visit www.topnotchboardprep.com.ph or email us at topnotchmedicalboardprep@gmail.com
Item QUESTION EXPLANATION AUTHOR TOPNOTCH
# EXAM
427 A 25-year old motorcycle rider was involved in a WEBSTER ALINDOG, MIDTERM 1
vehicular accident. He was rushed to the trauma MD (TOP 3 - FEB 2014 EXAM - AUG
section and blunt abdominal trauma is suspected. MED BOARDS; 2014
He has BP of 120/80, HR 102, good peripheral TOPNOTCH MD)
pulses, soft abdomen with no signs of peritonitis;
no other significant findings. FAST is negative.
What is the next best step to perform?
A. Patient is cleared for discharge.
B. Repeat FAST in 30 minutes.
C. A deep peritoneal aspirate is needed.
D. Laparotomy is warranted.
428 What is the most common type of skin cancer? WEBSTER ALINDOG, MIDTERM 1
A. Basal cell carcinoma MD (TOP 3 - FEB 2014 EXAM - AUG
B. Melanoma MED BOARDS; 2014
C. Squamous cell carcinoma TOPNOTCH MD)
D. Actinic keratosis

429 Which of the following is a risk factor for breast All are risk factors. WEBSTER ALINDOG, MIDTERM 1
cancer? MD (TOP 3 - FEB 2014 EXAM - AUG
A. Nulliparity MED BOARDS; 2014
B. Obesity TOPNOTCH MD)
C. Smoking
D. HRT
E. All of the above

430 This is considered to be the most common WEBSTER ALINDOG, MIDTERM 1


complication of modified radical mastectomy: MD (TOP 3 - FEB 2014 EXAM - AUG
A. Injury to the long thoracic nerve resulting in MED BOARDS; 2014
winging of the scapula TOPNOTCH MD)
B. Hypesthesia of the upper inner aspect of the
ipsilateral arm
C. Painless, gradual swelling of the involved arm
D. Formation of seromas beneath the flaps or
axilla
431 A 59-year old male who is a chronic cigarette Carcinoma of the lower alveolar ridge spreads to WEBSTER ALINDOG, MIDTERM 1
smoker and who is fond of betel nut chewing the buccal mucosa and floor of the mouth, and MD (TOP 3 - FEB 2014 EXAM - AUG
noted a slowly enlarging non-tender mass on his can invade the periosteum of the mandible. And MED BOARDS; 2014
lower gingiva. This has extended to the floor of his spread within medullary bone is best seen on TOPNOTCH MD)
mouth, significantly affecting movements of his MRI, where intermediate signal intensity on T1-
tongue. To demonstrate involvement of the weighted images is seen replacing the normal
medullary cavity of the alveolar bones, one should high T1-weighted signal intensity fatty marrow.
order for:
A. CT
B. MRI
C. Panorex
D. UTZ

432 A 45-year old diabetic patient is complaining of WEBSTER ALINDOG, MIDTERM 1


recurrent epigastric pain for 2 months now. She MD (TOP 3 - FEB 2014 EXAM - AUG
notes that episodes are aggravated by reclining MED BOARDS; 2014
and eating of spicy foods, with frequent TOPNOTCH MD)
regurgitation of some of the recently ingested
foods. To make an accurate diagnosis, this test
must be performed:
A. Endoscopy
B. Urea breath test
C. 24-hour pH monitoring
D. Gastric manometry
433 What is the most common malignant liver tumor? Metastatic hepatic tumor is the most common WEBSTER ALINDOG, MIDTERM 1
A. Hemangiocarcinoma tumor of the liver, while HCC is the most MD (TOP 3 - FEB 2014 EXAM - AUG
B. Hepatocellular carcinoma common PRIMARY tumor of the liver. MED BOARDS; 2014
C. Cholangiocarcinoma TOPNOTCH MD)
D. Metastatic liver tumor

434 Diverticulitis is diagnosed by CT imaging. Stage I: colonic inflammation with pericolic WEBSTER ALINDOG, MIDTERM 1
Complicated cases are staged by what is called as abscess; Stage III: purulent peritonitis; Stage IV: MD (TOP 3 - FEB 2014 EXAM - AUG
the Hinchey staging. At what stage of diverticulitis fecal peritonitis. Abscesses measuring <2cm MED BOARDS; 2014
is colonic inflammation with retroperitoneal or diameter can be treated with parenteral TOPNOTCH MD)
pelvic abscess is expected? antibiotics, whereas larger ones are best treated
A. Stage I with CT-guided percutaneous drainage
B. Stage II
C. Stage III
D. Stage IV

435 The most common cause of appendicitis among Fecalith on the other hand, is said to be the most WEBSTER ALINDOG, MIDTERM 1
pediatric patients: common cause of appendicitis in adults. MD (TOP 3 - FEB 2014 EXAM - AUG
A. Fecalith MED BOARDS; 2014
B. Lymphoid hyperplasia TOPNOTCH MD)
C. Obstructing food remnants
D. Tumors

TOPNOTCH MEDICAL BOARD PREP SURGERY SUPEREXAM Page 57 of 94


For inquiries visit www.topnotchboardprep.com.ph or email us at topnotchmedicalboardprep@gmail.com
TOPNOTCH MEDICAL BOARD PREP SURGERY SUPEREXAM
For inquiries visit www.topnotchboardprep.com.ph or email us at topnotchmedicalboardprep@gmail.com
Item QUESTION EXPLANATION AUTHOR TOPNOTCH
# EXAM
436 Which of the following is true about the hormonal The hyperglycemia is brought about by the WEBSTER ALINDOG, MIDTERM 1
response to an injury? relative insulin resistance as well as the actions MD (TOP 3 - FEB 2014 EXAM - AUG
A. Hyperglycemia is a hallmark of critical illness. of the counter-regulatory hormones (GH, MED BOARDS; 2014
B. Catecholamines may elevate up to 4x the catecholamines and cortisol). The high levels of TOPNOTCH MD)
normal levels lasting for 1-2 days before retruning sugars render the body in an
to normal. immunosuppressive state and hence significant
C. Burn patients may exhibit elevated cortisol for injuries also make patients susceptible to
up to 4 weeks. infections. B and C are also true.
D. All of the above
E. None of the above
437 During severe stress, the basal energy expenditure *Must memorize some values. Simple WEBSTER ALINDOG, MIDTERM 1
of the body is expected to be at: calculations during exams are common, MD (TOP 3 - FEB 2014 EXAM - AUG
A. 25-30 kcal/kg/day including in biochem and physio. Most MED BOARDS; 2014
B. 30-35 kcal/kg/day frequently, they ask to calculate for adjustments TOPNOTCH MD)
C. 35-40 kcal/kg/day in basal metabolic rates, energy expenditures,
D. >50 kcal/kg/day etc.

438 Which of the following is not true in the ED thoracotomy must be performed if SBP is WEBSTER ALINDOG, MIDTERM 1
management of cardiac tamponade? <70 mmHg. It is best accomplished using a left MD (TOP 3 - FEB 2014 EXAM - AUG
A. Beck's triad is not often observed. anterolateral approach. Pericardiocentesis is MED BOARDS; 2014
B. SBP <90 mmHg warrants an ED thoracotomy successful in correcting the tamponade in about TOPNOTCH MD)
C. Pericardiocentesis has a high rate of success as 80% of cases. Beck's triad include dilated neck
treatment modality for tamponade. veins, muffled heart sounds and low BP.
D. All of the above
E. None of the above

439 The technique used to surgically treat laryngeal WEBSTER ALINDOG, MIDTERM 1
malignancies which involves removal of neck MD (TOP 3 - FEB 2014 EXAM - AUG
lymph nodes in levels II through IV only is: MED BOARDS; 2014
A. Posterolateral neck dissection TOPNOTCH MD)
B. Lateral neck dissection
C. Crile method
D. Supraomohyoid neck dissection

440 A salivary gland tumor patient underwent left WEBSTER ALINDOG, MIDTERM 1
parotidectomy. He then complained of numbness MD (TOP 3 - FEB 2014 EXAM - AUG
on the lower portion of his left auricle and MED BOARDS; 2014
periauricular skin. Based on this presentation and TOPNOTCH MD)
on the rate of incidence of such complication, the
nerve most likely transected during the procedure
is:
A. Facial nerve
B. Auriculotemporal nerve
C. Greater auricular nerve
D. Occipital nerve
441 A 45 year-old man presents with diaphoresis, Some benign adrenal tumors can be extremely JULIET KRISTINE MIDTERM 2
headache and palpitations. Imaging studies were harmful. Approximately 10% of adrenal masses EVANGELISTA, MD EXAM - AUG
done which revealed a 3.5cm mass in his left secrete an excess of hormones that deleteriously (TOP 9 - FEB 2014 2014
adrenal gland. Laboratory examination revealed affect the body. For example, hormonal MED BOARDS;
elevated urine metanephrines. The treatment of hypersecretion can lead to such conditions as TOPNOTCH MD)
choice for this patient is: Cushing’s Syndrome, Conn’s Syndrome, and
A. chemoradiation Pheochromocytoma. In these cases,
B. tumor embolization adrenalectomy is the only surgical approach,
C. adrenalectomy open or laparoscopically.
D. radiotherapy
E. combined surgery and radiotherapy
442 It is a structure which demarcates the node levels Pectoralis minor demarcates the levels of the JULIET KRISTINE MIDTERM 2
in breast cancer: nodes in breast cancer. Level I nodes are located EVANGELISTA, MD EXAM - AUG
A. Pectoralis minor below and lateral to the muscle. Level II nodes (TOP 9 - FEB 2014 2014
B. Pectoralis major are at or behind pectoralis minor and level III MED BOARDS;
C. Latissimus dorsi nodes are medial and superior to the muscle. TOPNOTCH MD)
D. Serratus anterior
E. Axillary artery

443 A 75 year-old woman was going downstairs when Intracapsular fractures (femoral neck fractures) JULIET KRISTINE MIDTERM 2
she accidentally slipped and fell. She experienced occurs between the edge of the femoral head EVANGELISTA, MD EXAM - AUG
severe pain on her right buttocks and was not able and insertion of the capsule of the hip joint. (TOP 9 - FEB 2014 2014
to walk. She was rushed to the Emergency Room. Fractures of the femoral neck are more common MED BOARDS;
Xrays of the pelvis and femur were requested and in the elderly but fractures of the femoral shaft TOPNOTCH MD)
done revealing femoral fracture. The most and supracondylar fractures are usually caused
common part of femur fractured in this age group by violent trauma and most often occur in
is: adolescents and young adults. Extracapsular
A. Supracondylar fractures occur in between the insertion of the
B. Intracapsular capsule of the hip joint and a line 5 cm below the
C. Extracapsular lesser trochanter. Trochanteric fractures are
D. Intertrochanteric extracapsular fractures that include
E. Subtrochanteric intertrochanteric or pertrochanteric and reverse
oblique fractures. Subtrochanteric fractures are
extracapsular fractures where the fracture
occurs below the lesser trochanter.

TOPNOTCH MEDICAL BOARD PREP SURGERY SUPEREXAM Page 58 of 94


For inquiries visit www.topnotchboardprep.com.ph or email us at topnotchmedicalboardprep@gmail.com
TOPNOTCH MEDICAL BOARD PREP SURGERY SUPEREXAM
For inquiries visit www.topnotchboardprep.com.ph or email us at topnotchmedicalboardprep@gmail.com
Item QUESTION EXPLANATION AUTHOR TOPNOTCH
# EXAM
444 A neonate was born with a cleft palate. A surgical Cleft palate surgery is typically performed on JULIET KRISTINE MIDTERM 2
correction is advised. The main purpose for the infants that are between 6 and 18 months of age. EVANGELISTA, MD EXAM - AUG
surgical corection is the relief of: Since the primary goal of repairing the palate is (TOP 9 - FEB 2014 2014
A. Dysphagia to avoid any abnormal speech development, the MED BOARDS;
B. Dyspnea surgery is scheduled earlier before the child TOPNOTCH MD)
C. Speech abnormality develops much speech.
D. Dental malocclusion
E. Unexposed deformity

445 A 12 year-old boy, weighing 40 kg will undergo Maximum dosage of lidocaine is at 5mg/kg, 40kg JULIET KRISTINE MIDTERM 2
circumcision. The maximum dosage of lidocaine X 5mg=200mg EVANGELISTA, MD EXAM - AUG
that can be administered is: (TOP 9 - FEB 2014 2014
A. 50 mg MED BOARDS;
B. 100 mg TOPNOTCH MD)
C. 150 mg
D. 200 mg
E. 250 mg

446 A 25 year-old male was invloved in a vehicular Intraperitoneal bladder ruptures are described JULIET KRISTINE MIDTERM 2
accident. He sustained a blunt injury to the as large horizontal tears in the dome of the EVANGELISTA, MD EXAM - AUG
bladder. It was noted that the urine extravasated bladder. The dome is the least supported area (TOP 9 - FEB 2014 2014
to the perivesical space but not to the peritoneal and the only portion of the adult bladder MED BOARDS;
space. The most probable part of urinary bladder covered by peritoneum. This is the weakest part TOPNOTCH MD)
that is perforated is: of the bladder, since its muscle fibers are most
A. dome widely separated. This type of injury is common
B. anterior among patients diagnosed with alcoholism or
C. lateral those sustaining a seatbelt or steering wheel
D. trigone injury. The classic cystographic finding is
E. posterior contrast extravasation around the base of the
bladder confined to the perivesical space. The
bladder may assume a teardrop shape from
compression by a pelvic hematoma.
447 A 10 year-old child suffered fractures on his left Once the spleen is injured, there are two options JULIET KRISTINE MIDTERM 2
10th and 11th ribs after a fall. He was noted of available: splenectomy or splenic conservation. EVANGELISTA, MD EXAM - AUG
signs of intraabdominal bleeding. An emergency Preservation of the spleen is clearly possible, (TOP 9 - FEB 2014 2014
explore laparotomy was done revealing laceration particularly with minor capsular tears which MED BOARDS;
of the lower pole of the spleen. The best constitute the commonest type of splenic injury. TOPNOTCH MD)
management for the above injury to the spleen is Splenorrhaphy has also been demonstrated to
to: be effective in severely damaged spleens.
A. perform splenectomy
B. perform repair of the laceration
C. cauterize the spleen
D. perform partial splenectomy
E. ligate the splenic artery and do splenectomy
448 A 40 year-old patient with jaundice was suspected Pancreatic cancer is a malignant neoplasm JULIET KRISTINE MIDTERM 2
with a carcinoma through the "inverted 3 sign" originating from transformed cells arising in EVANGELISTA, MD EXAM - AUG
seen on imaging. This malignancy is usually tissues forming the pancreas. The most common (TOP 9 - FEB 2014 2014
aymptomatic until in advanced stages: type of pancreatic cancer, accounting for 95% of MED BOARDS;
A. Carcinoma of the duodenum these tumors, is adenocarcinoma. Initially, TOPNOTCH MD)
B. Carcinoma of the common bile duct pancreatic cancer of the head often tends to be
C. Carcinoma of the ampulla of Vater silent and painless as it grows, and the later
D. Carcinoma of the head of pancreas symptoms are usually nonspecific and varied.
E. Carcinoma of the body and tail of pancreas Frostburg's inverted 3 sign is a sign seen on a
Barium examination where there is effacement
and distortion of the mucosal pattern on the
medial wall of the second part of the duodenum.
It is suggestive of carcinoma of the head of the
pancreas.
449 A 45 year-old diabetic woman underwent routine Most patients with gallstones remain JULIET KRISTINE MIDTERM 2
GI series. A presence of gallstone is an incidental asymptomatoc. However, even asymptomatic, EVANGELISTA, MD EXAM - AUG
finding but the patient is asymptomatic. The best elective cholecystectomy is indicated to elderly (TOP 9 - FEB 2014 2014
management is: with diabetes, those who are in isolation from MED BOARDS;
A. Emergency cholecystectomy medical care for extended periods (seamen), TOPNOTCH MD)
B. Elective cholecystectomy and those with increased risk of GB cancer.
C. Low cholesterol diet
D. Regular follow-up
E. No treatment needed
450 A 54 year-old businessman had a solitary lesion on Hepatocellular carcinoma is the most common JULIET KRISTINE MIDTERM 2
the right lobe of the liver on ultrasound. There was type of primary liver cancer. Most cases are EVANGELISTA, MD EXAM - AUG
no jaundice or RUQ pain noted. Alkaline secondary to either a viral hepatitis infection (TOP 9 - FEB 2014 2014
phosphatase was elevated but serum bilirubin is (hepatitis B or C) or cirrhosis. HCC may present MED BOARDS;
normal. Tha patient most probably has: with jaundice, bloating, easy bruising, loss of TOPNOTCH MD)
A. Amoebic liver abscess appetite, weight loss, RUQ abdominal pain,
B. Hepatic adenoma nausea, emesis, or fatigue. Total bilirubin, AST,
C. Metastatic liver cancer alkaline phosphatase, albumin, and prothrombin
D. Pyogenic liver abscess time show results consistent with cirrhosis. AFP
E. Primary hepatocellular Ca is elevated in 75% of cases.
451 A 62 year-old patient underwent Abdomino- An abdominoperineal resection or the Miles JULIET KRISTINE MIDTERM 2
perineal resection for rectal tumor. The principal operation is principally indicated for resection EVANGELISTA, MD EXAM - AUG
drawback to APR is: of a rectal carcinoma situated in the distal one- (TOP 9 - FEB 2014 2014
A. High mortality third of the rectum or anal cancer. The principal MED BOARDS;
B. Potential leak in the anastomosis drawback of this procedure is permanent TOPNOTCH MD)
C. Permanent colostomy colostomy as it involves removal of the anus, the
D. Incontinence rectum and part of the sigmoid colon along with
E. Impotence the associated regional lymph nodes, through
incisions made in the abdomen and perineum.
TOPNOTCH MEDICAL BOARD PREP SURGERY SUPEREXAM Page 59 of 94
For inquiries visit www.topnotchboardprep.com.ph or email us at topnotchmedicalboardprep@gmail.com
TOPNOTCH MEDICAL BOARD PREP SURGERY SUPEREXAM
For inquiries visit www.topnotchboardprep.com.ph or email us at topnotchmedicalboardprep@gmail.com
Item QUESTION EXPLANATION AUTHOR TOPNOTCH
# EXAM
During lateral mobilization of the rectum,
pudendal nerve may be injured resulting to
impotence in male patients.

452 A space that is also known as the Morrison's Morrison's pouch is also known as hepatorenal JULIET KRISTINE MIDTERM 2
pouch: or subhepatic space. EVANGELISTA, MD EXAM - AUG
A. Subxiphoid (TOP 9 - FEB 2014 2014
B. Hepatorenal MED BOARDS;
C. Splenorenal TOPNOTCH MD)
D. Retrocecal
E. Cul-de-sac

453 A young woman was rushed to the Emergency Thoracostomy tube is indicated to patients JULIET KRISTINE MIDTERM 2
room who suffered from a blunt chest trauma in a suffering from a blunt chest trauma. It is done to EVANGELISTA, MD EXAM - AUG
vehicular crash. She is dyspneic, RR is 40 breaths decompress the chest with pertinent physical (TOP 9 - FEB 2014 2014
per minute. Breath sounds markedly diminished findings and hemothorax. The lowermost limit MED BOARDS;
on the right side. In inserting a thoracostomy tube, of pleura in midclavicular line is 8th rib, in TOPNOTCH MD)
the lowermost limit of the parietal pleura at the midaxillary line is 10th rib, and at the lateral
lateral border of erector spinae is: border of erector spinae is 12th rib.
A. 6th rib
B. 8th rin
C. 9th rib
D. 10th rib
E. 12th rib
454 A 53 year-old woman comes in with a 9 cm X 8 cm Phyllodes tumor are most often benign, but it is JULIET KRISTINE MIDTERM 2
mobile mass in her right breast for the past 4 still critical to remove the entire tumor. Even if EVANGELISTA, MD EXAM - AUG
years. Incision biopsy revealed a highly cellular one cell is left behind, it will grow back. (TOP 9 - FEB 2014 2014
fibrous stroma with cytsic areas and a leafy Treatment involves excision of both the tumor MED BOARDS;
appearance on cut section. The best management and a wide margin of healthy tissue surrounding TOPNOTCH MD)
for the above findings is: the tumor. Treatment is wide local excision with
A. Radical mastectomy a 1 cm margin of normal tissue is recommended.
B. Lumpectomy Mastectomy is indicated for patients with large
C. Simple mastectomy lesions.
D. Modified radical mastectomy
E. MRM with radiotherapy
455 A 40 year-old male presents with a painless A hordeolum is a common disorder of the eyelid, JULIET KRISTINE MIDTERM 2
swelling of the gland on the upper portion of his an acute focal infection (usually staphylococcal) EVANGELISTA, MD EXAM - AUG
upper eyelid without external signs of involving either the glands of Zeis (external (TOP 9 - FEB 2014 2014
inflammation, a chronic inflammatory hordeola, or styes) or, less frequently, the MED BOARDS;
lipogranuloma of a meibomian gland. Diagnosis is: meibomian glands (internal hordeola). TOPNOTCH MD)
A. External hordeolum Essentially, a hordeolum represents an acute
B. Internal hordeolum focal infectious process. A chalazion
C. Chalazion represents a chronic, noninfectious
D. Meibomianitis granulomatous reaction. However, chalazion
E. Blepharitis often evolve from internal hordeolum.
456 A 30 year-old female patient noted redness on her A subconjunctival hemorrhage is is bleeding JULIET KRISTINE MIDTERM 2
left eye. She went to an ophthalmologist who underneath the conjunctiva. The conjunctiva EVANGELISTA, MD EXAM - AUG
found that she had subconjunctival hemorrage. contains many small, fragile blood vessels that (TOP 9 - FEB 2014 2014
Management for the finding is: are easily ruptured or broken. It is is typically a MED BOARDS;
A. Trabeculectomy of the affected eye self-limiting condition that requires no TOPNOTCH MD)
B. Put patient on high back rest with patch on treatment in the absence of infection or
affected eye significant trauma.
C. Topical antibiotic for 5 days and atropine
ophthalmic drops BID
D. Reassure patient that the condition will
resolve after some time
E. Surgical drainage of blood to prevent further
complication
457 A 40 year-old carpenter accidentally hit his right Hyphema is the collection of blood in the JULIET KRISTINE MIDTERM 2
half of his face with a hammer resulting to a anterior chamber of the eye. It may appear as a EVANGELISTA, MD EXAM - AUG
hyphema. It is characterized as: reddish tinge, or it may appear as a small pool of (TOP 9 - FEB 2014 2014
A. An angioma of the face blood at the bottom of the iris or in the cornea. MED BOARDS;
B. Blood in the anterior chamber of the eye The most common causes of hyphema are blunt TOPNOTCH MD)
C. Hemorrhage in the maxillary sinus trauma, intraocular surgery, and lacerating
D. Lymphedema of the face trauma that my partially or completely block
E. Excessive lacrimation vision.

458 Malignant otitis externa is usually asociated with Malignant otitis externa is caused by the spread JULIET KRISTINE MIDTERM 2
infection by: of an otitis externa, also called swimmer's ear. EVANGELISTA, MD EXAM - AUG
A. Streptococcus sp. The majority of cases are due to Pseudomonas (TOP 9 - FEB 2014 2014
B. Staphylococcus sp. aeruginosa, followed by a great number of other MED BOARDS;
C. Pseudomonas sp. gram-positive and gram-negative species. TOPNOTCH MD)
D. Pneumococcus sp.
E. H. influenza

TOPNOTCH MEDICAL BOARD PREP SURGERY SUPEREXAM Page 60 of 94


For inquiries visit www.topnotchboardprep.com.ph or email us at topnotchmedicalboardprep@gmail.com
TOPNOTCH MEDICAL BOARD PREP SURGERY SUPEREXAM
For inquiries visit www.topnotchboardprep.com.ph or email us at topnotchmedicalboardprep@gmail.com
Item QUESTION EXPLANATION AUTHOR TOPNOTCH
# EXAM
459 A 50 year-old patient came in with episodic Meniere's disease or endolymphatic hydrops, is JULIET KRISTINE MIDTERM 2
vertigo, tinnitus and hearing loss. Upon a disorder of the inner ear that can affect EVANGELISTA, MD EXAM - AUG
examination, he was noted of sensorineural hearing and balance to a varying degree. It is (TOP 9 - FEB 2014 2014
hearing loss with proportionate loss of characterized by episodes of vertigo, low- MED BOARDS;
discrimination, noise intolerance and reduced pitched tinnitus, and fluctuating hearing loss. TOPNOTCH MD)
vestibular function. The probable diagnosis is:
A. Vestibular neuronitis
B. Acoustic neuroma
C. Otogenic labyrinthitis
D. Chronic labyrinthitis
E. Meniere's disease
460 In an extensive maxillofacial trauma, adequate ABC's. Administer oxygen and maintain a patent JULIET KRISTINE MIDTERM 2
initial care should be directed to: airway. Maintain an immobilized cervical spine EVANGELISTA, MD EXAM - AUG
A. Adequate prevention of infection at all times. Clear the mouth of any foreign body (TOP 9 - FEB 2014 2014
B. Proper analgesia and anesthesia or debris, and suction any blood present. MED BOARDS;
C. Maintenance of patent airway TOPNOTCH MD)
D. Meticulous soft tissues repair
E. Proper skeletal reduction and fixation

461 Sutures are removed after 3-5 days in: Timing of suture removal are as follows: face - 3 LUISA SARANILLO, MD BACK-UP
A. legs to 5days; neck - 5 to 7 days; scalp - 7 to 12 days; (TOP 6 - FEB 2014 MIDTERM
B. scalp upper extremity and trunk - 10 to 14 days; MED BOARDS; EXAM AUG
C. arm lower extremity - 14 to 28 days; soles, palms, TOPNOTCH MD) 2014 - FOR
D. face back or over joints - 10 days INCLUSION IN
E. Abdomen THE SAMPLEX

462 A 70 year old male farmer was diagnosed with Clark level: I- superficial to basement LUISA SARANILLO, MD BACK-UP
malignant melanoma extending to the reticular membrane; II - papillary dermis; III - papillary- (TOP 6 - FEB 2014 MIDTERM
dermis. What is the Clark level? reticular dermal junction; IV - reticular dermis; MED BOARDS; EXAM AUG
A. V V - subcutaneous fat TOPNOTCH MD) 2014 - FOR
B. IV INCLUSION IN
C. III THE SAMPLEX
D. II
E. I

463 A 35 year old pegnant was alarmed when she Internal hemorrhoids present with bleeding and LUISA SARANILLO, MD BACK-UP
noticed a blood dripping when she defecates. She non-painful anal mass. It is classified into 4: 1st (TOP 6 - FEB 2014 MIDTERM
also noted a non-painful anal mass that reduces degree - bleeding with no protrusion; 2nd MED BOARDS; EXAM AUG
spontaneously. What is your first consideration? degree - bleeding with prolapse but reduces TOPNOTCH MD) 2014 - FOR
A. 1st degree internal hemorrhoids spontaneously; 3rd degree - bleeding with INCLUSION IN
B. 2nd degree internal hemorrhoids prolapse that is reduced manually; 4th degree - THE SAMPLEX
C. 3rd degree internal hemorrhoids bleeding with prolapse that cannot be reduced.
D. 4th degree internal hemorrhoids External hemorrhoids present with pain.
E. 1st degree external hemorrhoids

464 What is the stage of pressure ulcer when there is Stage I - nonblanchable erythema of intact skin; LUISA SARANILLO, MD BACK-UP
full thickness skin loss but not through the fascia? Stage II - partial thickness skin loss; Stage III - (TOP 6 - FEB 2014 MIDTERM
A. Stage I full thickness skin loss but not thru the fascia; MED BOARDS; EXAM AUG
B. Stage II Stage IV - full thickness skin loss + muscle and TOPNOTCH MD) 2014 - FOR
C. Stage III bone involvement. There is no stage V. INCLUSION IN
D. Stage IV THE SAMPLEX
E. Stage V

465 A 48 year old female had a breast mass which This is a case of benign phyllodes tumor. It is LUISA SARANILLO, MD BACK-UP
turned out to be a phyllodes tumor with less than malignant if there is more than 10 mitosis / 50 (TOP 6 - FEB 2014 MIDTERM
10 mitosis / 50 HPF. What is the appropriate HPF. The treatment is excision with 1cm margin MED BOARDS; EXAM AUG
treatment? without the need of axillary node dissection. It TOPNOTCH MD) 2014 - FOR
A. Excision with a 1cm margin with axillary node may require mastectomy. INCLUSION IN
dissection THE SAMPLEX
B. Excision with a 1 cm margin without axillary
node dissection
C. excision with a 5cm margin with axillary node
dissection
D. excision with a 5 cm margin without axillary
node dissection
E. MRM + Chemo + radiotherapy
466 A 50 year old male with a malignant thyroid mass For thyroid malignancies, posterolaternal neck LUISA SARANILLO, MD BACK-UP
underwent total thyroidectomy with selective dissection is removed which includes cervical (TOP 6 - FEB 2014 MIDTERM
neck dissection. One of the levels of cervical lymph lymph node levels - II, III, IV, and V. MED BOARDS; EXAM AUG
node is not removed. TOPNOTCH MD) 2014 - FOR
A. I INCLUSION IN
B. II THE SAMPLEX
C. III
D. IV
E. V

TOPNOTCH MEDICAL BOARD PREP SURGERY SUPEREXAM Page 61 of 94


For inquiries visit www.topnotchboardprep.com.ph or email us at topnotchmedicalboardprep@gmail.com
TOPNOTCH MEDICAL BOARD PREP SURGERY SUPEREXAM
For inquiries visit www.topnotchboardprep.com.ph or email us at topnotchmedicalboardprep@gmail.com
Item QUESTION EXPLANATION AUTHOR TOPNOTCH
# EXAM
467 Gastroesophageal reflux disease (GERD) is the LUISA SARANILLO, MD BACK-UP
most common esophageal pathology. The possible (TOP 6 - FEB 2014 MIDTERM
complications of GERD are the following except: MED BOARDS; EXAM AUG
A. esophagitis TOPNOTCH MD) 2014 - FOR
B. stricture INCLUSION IN
C. aspiration THE SAMPLEX
D. Esophageal adenocarcinoma
E. No exception

468 A 50 year old male presented with abdominal pain Upright chest x-ray is a non invasive procedure LUISA SARANILLO, MD BACK-UP
and melena, with normal vital signs. He said that to check for pneumoperitoneum in 80%. (TOP 6 - FEB 2014 MIDTERM
for the past few months, he experienced episodes MED BOARDS; EXAM AUG
of abdominal pain that awakens him at around 3- TOPNOTCH MD) 2014 - FOR
4am. What is a non invasive procedure performed INCLUSION IN
to check for pneumoperitoneum? THE SAMPLEX
A. upper endoscopy
B. barium enema
C. upright chest x-ray
D. Exploratory laparotomy
E. abdominal ultrasound
469 You performed an appendectomy during your first LUISA SARANILLO, MD BACK-UP
day in surgery. While doing the procedure, you (TOP 6 - FEB 2014 MIDTERM
had difficulty in looking for the base of the MED BOARDS; EXAM AUG
appendix. What is the landmark used to identify TOPNOTCH MD) 2014 - FOR
the location of the appendix? INCLUSION IN
A. anterior taenia coli THE SAMPLEX
B. appendiceal artery
C. posterior taenia coli
D. posterior cecal artery
E. colon
470 A 70 year old male complained of rectal bleeding this is a case of diverticulitis. CT scan of the LUISA SARANILLO, MD BACK-UP
and altered bowel habits. He had left lower abdomen is the gold standard for diagnosis. (TOP 6 - FEB 2014 MIDTERM
quadrant pain and tenderness on the LLQ upon MED BOARDS; EXAM AUG
palpation. What is the diagnostic of choice, and TOPNOTCH MD) 2014 - FOR
considered as the gold standard? INCLUSION IN
A. contrast enema THE SAMPLEX
B. abdominal ultrasound
C. computed tomography of the abdomen
D. rectal biopsy
E. x-ray of the abdomen
471 The gene found in chromosome 5 that is mutated LUISA SARANILLO, MD BACK-UP
in colorectal cancer. It is also the earliest mutation (TOP 6 - FEB 2014 MIDTERM
in colorectal Cancer. MED BOARDS; EXAM AUG
A. K-ras TOPNOTCH MD) 2014 - FOR
B. APC INCLUSION IN
C. p53 THE SAMPLEX
D. DCC
E. BRCA

472 A 65 year old alcoholic complained of distended LUISA SARANILLO, MD BACK-UP


abdomen, pruritus and jaundice. He also had (TOP 6 - FEB 2014 MIDTERM
hematemesis. Upon further examination, there MED BOARDS; EXAM AUG
was splenomegaly and ano-rectal varices. What is TOPNOTCH MD) 2014 - FOR
the most accurate method of determining portal INCLUSION IN
hypertension? THE SAMPLEX
A. hepatic venography
B. upper endoscopy
C. ultrasound of whole abdomen
D. CT-scan of the abdomen
E. MRI of the abdomen
473 Rotter's node is at what level of axillary lymph There are 3 groups of axillary lymph nodes: LUISA SARANILLO, MD BACK-UP
node? Rotter's node is the interpectoral group of node (TOP 6 - FEB 2014 MIDTERM
A. Level I at level II. MED BOARDS; EXAM AUG
B. Level II TOPNOTCH MD) 2014 - FOR
C. Level III INCLUSION IN
D. Level IV THE SAMPLEX
E. Level V

474 While playing around the kitchen, a 5 year old Superficial burn should not be included in the LUISA SARANILLO, MD BACK-UP
child accidentally bumps off to her mother calculating the % TBSA. In a child, using the rule (TOP 6 - FEB 2014 MIDTERM
carrying a cup of hot coffee, spilling it over her of 9's, this is the distribution: face - 18%, MED BOARDS; EXAM AUG
body sustaining burn injury as follows: anterior anterior trunk - 18%, posterior trunk - 18%, TOPNOTCH MD) 2014 - FOR
trunk with partial thickness burn, right arm with each arm - 9%, each leg - 14 %. Therefore in this INCLUSION IN
full thickness burn and right leg with superficial case, anterior trunk 18% + right arm 9% = 27%. THE SAMPLEX
burn. What is the burn size?
A. 27% TBSA
B. 41% TBSA
C. 36% TBSA
D. 18% TBSA
E. 30% TBSA

TOPNOTCH MEDICAL BOARD PREP SURGERY SUPEREXAM Page 62 of 94


For inquiries visit www.topnotchboardprep.com.ph or email us at topnotchmedicalboardprep@gmail.com
TOPNOTCH MEDICAL BOARD PREP SURGERY SUPEREXAM
For inquiries visit www.topnotchboardprep.com.ph or email us at topnotchmedicalboardprep@gmail.com
Item QUESTION EXPLANATION AUTHOR TOPNOTCH
# EXAM
475 A 24 year old male sustained a stab wound on the In managing esophageal perforation, the most LUISA SARANILLO, MD BACK-UP
chest perforating the esophagus. If he was brought favorable outcome is obtained following (TOP 6 - FEB 2014 MIDTERM
in the ER 10 hours after the injury with BP of primary closure of the perforation within 24 MED BOARDS; EXAM AUG
90/60, pulse rate of 90, respiratory rate of 20, hours resulting in 80-90%. TOPNOTCH MD) 2014 - FOR
what will you do? INCLUSION IN
A. Repair by primary closure THE SAMPLEX
B. Repair by secondary intention
C. esophagectomy with gastric pull up
D. observe
E. transfuse blood as soon as possible
476 A patient with previous cesarian section 2 years This is a case of bowel obstruction. The LUISA SARANILLO, MD BACK-UP
ago returns with colicky abdominal pain, nausea, confirmatory test for this is abdominal series (TOP 6 - FEB 2014 MIDTERM
and obstipation. On physical examination, there consisting of radiograph of the abdomen in MED BOARDS; EXAM AUG
was abdominal distention and hypoactive bowel upright and supine position, and chest x-ray in TOPNOTCH MD) 2014 - FOR
sounds. If you want to confirm your diagnosis, upright position. INCLUSION IN
what test will you do? THE SAMPLEX
A. CT scan of the abdomen
B. ultrasound of the abdomen
C. MRI of the abdomen
D. abdominal series
E. barium enema
477 A 50 year old male had vehicular accident Massive hemothorax in adult is defined as more LUISA SARANILLO, MD BACK-UP
sustaining multiple abrasions and lacerations in than 1.5 liters of blood in the pleural space, (TOP 6 - FEB 2014 MIDTERM
the arms, face, and chest. X-ray of the chest while in pediatrics, it is defined as more than MED BOARDS; EXAM AUG
revealed a multiple rib fracture and massive 1/3 of the blood volume in the pleural space. TOPNOTCH MD) 2014 - FOR
hemothorax. In adult, massive hemothorax is INCLUSION IN
defined as: THE SAMPLEX
A. more than 1/3 of the blood volume in the
pleural space
B. more than 1.5 liters of blood in the pleural
space
C. more than 1/2 of blood volume in the pleural
space
D. more than 2 liters of blood in the pleural space
E. more than 2.5 liters of blood in the pleural
space
478 Patient had excruciating abdominal pain and tachycardia is the earliest sign of ongoing blood LUISA SARANILLO, MD BACK-UP
dizziness after sustaining a blunt abdominal loss, while bradycardia is an ominous sign (TOP 6 - FEB 2014 MIDTERM
trauma 1 day ago. He was brought to the heralding impending cardiovascular collapse. MED BOARDS; EXAM AUG
emergency department with BP of 90/60mmHg, TOPNOTCH MD) 2014 - FOR
respiratory rate of 26 cpm, and heart rate of INCLUSION IN
130bpm. What is the earliest sign of ongoing blood THE SAMPLEX
loss.
A. tachypnea
B. hypotension
C. bradycardia
D. tachycardia
E. all of the choices
479 In a post gastrectomy patient receiving a prenteral Vitamin K is not part of any commercially LUISA SARANILLO, MD BACK-UP
nutrition, what vitamin should be supplemented prepared vitamin solution, so it should be (TOP 6 - FEB 2014 MIDTERM
on a weekly basis because it is not part of supplemented on a weekly basis. MED BOARDS; EXAM AUG
commercially prepared vitamin solution. TOPNOTCH MD) 2014 - FOR
A. Vitamin A INCLUSION IN
B. Vitamin D THE SAMPLEX
C. Vitamin K
D. Vitamin B
E. Vitamin E

480 A 40 year old female obese patient came in due to Cholesterol stone accounts for 80% of gallstone, LUISA SARANILLO, MD BACK-UP
right upper quadrant abdominal pain. Ultrasound while pigment stone accounts for 15-20% of (TOP 6 - FEB 2014 MIDTERM
of the hepatobiliary tree shows hyperechoic gallstones. MED BOARDS; EXAM AUG
transluminal focus with shadowing. Your primary TOPNOTCH MD) 2014 - FOR
consideration is gallstone. What is the most INCLUSION IN
common type of stone? THE SAMPLEX
A. cholesterol stone
B. black pigment stone
C. brown pigment stone
D. bile sludge
E. pigment stones
481 The maximum dose of lidocaine for a 70-kg man is: There were several computations on lidocaine ANGELIS ANDREA FINAL EXAM -
A. 210 mg (not mentioned whether with or without epi) COCOS, MD (TOP 1 - AUG 2014
B. 300 mg during our board exam. The maximum dose for FEB 2014 MED
C. 490 mg lidocaine without epinephrine is 3-5mg/kg. The BOARDS; TOPNOTCH
D. 700 mg maximum dose for lidocaine with epinephrine is MD)
7mg/kg. 70kg x 7mg/kg= 490 mg.
482 A patient presents at the emergency room Opens eyes in response to pain (2) + makes ANGELIS ANDREA FINAL EXAM -
obtunded. On sternal rub, his eyes opened and he nonpurposeful movement in response to pain COCOS, MD (TOP 1 - AUG 2014
makes nonpurposeful movements. He moans (4) + makes incomprehensible words (2) = 8. FEB 2014 MED
when being asked something. What is his Glasgow SIMILAR TO PREVIOUS BOARD EXAM BOARDS; TOPNOTCH
coma scale? CONCEPT/PRINCIPLE. MD)
A. 7
B. 8
C. 9
D. 10

TOPNOTCH MEDICAL BOARD PREP SURGERY SUPEREXAM Page 63 of 94


For inquiries visit www.topnotchboardprep.com.ph or email us at topnotchmedicalboardprep@gmail.com
TOPNOTCH MEDICAL BOARD PREP SURGERY SUPEREXAM
For inquiries visit www.topnotchboardprep.com.ph or email us at topnotchmedicalboardprep@gmail.com
Item QUESTION EXPLANATION AUTHOR TOPNOTCH
# EXAM
483 With regards normal homeostasis and platelet SIMILAR TO PREVIOUS BOARD EXAM ANGELIS ANDREA FINAL EXAM -
function, which of the following is a CONCEPT/PRINCIPLE. pGE1 and histamine COCOS, MD (TOP 1 - AUG 2014
vasoconstrictor? causes vasodilation. FEB 2014 MED
A. thromboxane BOARDS; TOPNOTCH
B. PGE1 MD)
C. histamine
D. serotonin

484 For a normal adult female who weighs 60 kg, what The total body water of females is said to be 50- ANGELIS ANDREA FINAL EXAM -
is the total body water in liters? 55%, while for males it is 60%. 60kg x 0.55= 33 COCOS, MD (TOP 1 - AUG 2014
A. 24 L L FEB 2014 MED
B. 36 L BOARDS; TOPNOTCH
C. 48 L MD)
D. 33 L
485 Where is the common location of carcinoma of the Questions regarding "most commons" were ANGELIS ANDREA FINAL EXAM -
stomach? frequently asked during our exam. Gastric COCOS, MD (TOP 1 - AUG 2014
A. Along the lesser curvature carcinoma occurs 50% in the pylorus, 25% FEB 2014 MED
B. At the cardia along the lesser curvature, 5% along the greater BOARDS; TOPNOTCH
C. Along the greater curvature curvature and 10 % the cardia. MD)
D. In the pylorus
486 In orthopedics, "no man's land" refers to tendon Zone 1: distal to the insertion of the sublimis ANGELIS ANDREA FINAL EXAM -
injuries in what zone of the hand? tendon. Zone 2: between the sublimis insertion COCOS, MD (TOP 1 - AUG 2014
A. Zone 1 and the proximal end of the flexor sheath. Zone FEB 2014 MED
B. Zone 2 3: midpalm. Zone 4: carpal tunnel. Zone 5: distal BOARDS; TOPNOTCH
C. Zone 3 forearm. Zone 2 is called no man's land because MD)
D. Zone 4 two flexor tendons run through the flexor
sheath in this area and the prognosis after a
flexor tendon injury in this area is worse than
the other areas.
487 The most common type of fistula-in-ano is: Intersphincteric fistulas compose almost 40- ANGELIS ANDREA FINAL EXAM -
A. intersphincteric 45% of fistulas-in-ano. Second most common COCOS, MD (TOP 1 - AUG 2014
B. transsphincteric would be transsphincteric fistulas, composing FEB 2014 MED
C. suprasphincteric around 30%. BOARDS; TOPNOTCH
D. Extrasphincteric MD)

488 A 60-year-old woman is admitted for A colonic mass is a common cause of anemia and ANGELIS ANDREA FINAL EXAM -
hematochezia, severe enough to decrease her hematochezia in the elderly. COCOS, MD (TOP 1 - AUG 2014
hemoglobin to 8 g/dl. Which of the following is the FEB 2014 MED
best diagnostic procedure to use? BOARDS; TOPNOTCH
A. esophagogastroduodenoscopy MD)
B. Double contrast barium enema
C. colonoscopy
D. selective angiography
489 A 3-year-old child presents at your office for The diagnosis of Hirschsprung disease should be ANGELIS ANDREA FINAL EXAM -
chronic constipation. The mother noted that since suspected in a child with intractable chronic COCOS, MD (TOP 1 - AUG 2014
birth, the child has only one stool per week. He constipation. A neonatal history of delayed FEB 2014 MED
was born term without complications but he did passage of meconium is often obtained. Surgery BOARDS; TOPNOTCH
not pass his stool for the first 48 hours. What is indicated as soon as diagnosis is made MD)
should be the initial management for this child? through barium enema and rectal manometry. A
A. plain film of the abdomen similar question came up during our board
B. a child psychiatry evaluation exam.
C. begin oral antispasmodic medication
D. barium enema and rectal manometry
490 The most common histologic type of urinary The most common type of bladder cancer ANGELIS ANDREA FINAL EXAM -
bladder malignancy is: recapitulates the normal histology of the COCOS, MD (TOP 1 - AUG 2014
A. adenocarcinoma urothelium--transitional cell carcinoma or more FEB 2014 MED
B. Urothelial carcinoma properly urothelial cell carcinoma. BOARDS; TOPNOTCH
C. Squamous cell carcinoma MD)
D. Rhabdomyosarcoma
491 A 52-year-old man is diagnosed to have carcinoma This would require an APR because the lesion is ANGELIS ANDREA FINAL EXAM -
of the rectum about 3 cm from the anal verge. too close to the anus. COCOS, MD (TOP 1 - AUG 2014
Which procedure should be done assuming FEB 2014 MED
curative surgery is feasible? BOARDS; TOPNOTCH
A. A hartmann procedure MD)
B. A low anterior resection plus total
mesorectum excision
C. An abdomino-perineal resection
D. A left hemicolectomy
492 A patient suffered a stab wound at the left Beck's triad: muffled heart sounds, hypotension ANGELIS ANDREA FINAL EXAM -
parasternal line at the level of 4th ICS. He was and distended neck point to the condition of COCOS, MD (TOP 1 - AUG 2014
brought to the ER and was found to have a BP of acute cardiac tamponade. There were a few FEB 2014 MED
80/60. His heart sounds of 100 per minute are trauma questions during our exam. BOARDS; TOPNOTCH
barely audible and his neck veins are distended. MD)
He is likely suffering from:
A. hemorrhagic shock
B. hemothorax
C. cardiac tamponade
D. tension pneumothorax

TOPNOTCH MEDICAL BOARD PREP SURGERY SUPEREXAM Page 64 of 94


For inquiries visit www.topnotchboardprep.com.ph or email us at topnotchmedicalboardprep@gmail.com
TOPNOTCH MEDICAL BOARD PREP SURGERY SUPEREXAM
For inquiries visit www.topnotchboardprep.com.ph or email us at topnotchmedicalboardprep@gmail.com
Item QUESTION EXPLANATION AUTHOR TOPNOTCH
# EXAM
493 According to the Clark Level, the invasion of a SIMILAR TO PREVIOUS BOARD EXAM ANGELIS ANDREA FINAL EXAM -
melanoma into the reticular dermis is considered: CONCEPT/PRINCIPLE. Level I: confined to COCOS, MD (TOP 1 - AUG 2014
A. Level II epidermis, II: papillary dermis, III: junction of FEB 2014 MED
B. Level III papillary dermis and reticular dermis, IV: BOARDS; TOPNOTCH
C. Level IV reticular dermis, and V: deep, subcutaneous MD)
D. Level V tissue

494 A 45-year old woman occasionally notices bloody The most common cause of bloody nipple ANGELIS ANDREA FINAL EXAM -
discharge from her left nipple. In this patient: discharge is benign intraductal papilloma which COCOS, MD (TOP 1 - AUG 2014
A. An excision of the areola-nipple complex has good prognosis. FEB 2014 MED
should be done BOARDS; TOPNOTCH
B. The cause is likely to be ductal carcinoma in MD)
situ
C. The most likely cause has a good prognosis
D. A modified radical mastectomy should be
done
495 Metastasis to regional lymph nodes are common Unlike squamous cell CA, basal cell carcinoma ANGELIS ANDREA FINAL EXAM -
with all the following cancers, EXCEPT: which is the most common type of skin cancer is COCOS, MD (TOP 1 - AUG 2014
A. Papillary thyroid carcinoma known for its local invasion rather than FEB 2014 MED
B. Squamous cell carcinoma metastasis. BOARDS; TOPNOTCH
C. Cervical carcinoma MD)
D. Basal cell carcinoma

496 A 45-year-old male patient presents at the ER with Open-angle glaucoma usually is painless and ANGELIS ANDREA FINAL EXAM -
very painful, sudden vision loss, and halos around does not have an acute presentation. Choices C COCOS, MD (TOP 1 - AUG 2014
the light. Which is the most likely diagnosis? and D are painful but the symptoms of sudden FEB 2014 MED
A. Open-angle glaucoma vision loss and appearance of halos are BOARDS; TOPNOTCH
B. Closed-angle glaucoma characteristic of closed angle glaucoma. MD)
C. Endophthalmitis
D. Uveitis

497 The liver is functionally divided into how many Couinaud proposed the liver to be divided into 8 ANGELIS ANDREA FINAL EXAM -
segments according to Couinaud? segments in relation to the hepatic venous COCOS, MD (TOP 1 - AUG 2014
A. 4 drainage. FEB 2014 MED
B. 6 BOARDS; TOPNOTCH
C. 8 MD)
D. 10
498 A 43-year-old unmarried female with a BMI of 32, Generally speaking, up to 90 percent of ANGELIS ANDREA FINAL EXAM -
complained of RUQ pain and low-grade fever. On gallstones are cholesterol stones. Pure COCOS, MD (TOP 1 - AUG 2014
PE, she exhibited a positive Murphy's sign. What cholesterol (>80% cholesterol) and mixed FEB 2014 MED
type of gallstone is the most common etiology for cholesterol (>50% cholesterol) are the most BOARDS; TOPNOTCH
acute cholecystitis? common causes of acute cholecystitis MD)
A. pure cholesterol gallstones worldwide. SIMILAR TO PREVIOUS BOARD
B. mixed cholesterol gallstones EXAM CONCEPT/PRINCIPLE.
C. brown pigment gallstones
D. black pigment gallstones
499 An otorhinolaryngologist decided to perform a The rest of the choices are sacrificed during the ANGELIS ANDREA FINAL EXAM -
radical neck dissection for an oral cavity mass. standard radical neck dissection. COCOS, MD (TOP 1 - AUG 2014
Which of the following structures is NOT FEB 2014 MED
removed? BOARDS; TOPNOTCH
A. Hypoglossal nerve MD)
B. Spinal accessory nerve
C. Internal jugular vein
D. Sternocleidomastoid
500 The esophagus is more prone to full-thickness SIMILAR TO PREVIOUS BOARD EXAM ANGELIS ANDREA FINAL EXAM -
tears than other gastrointestinal organs mainly CONCEPT/PRINCIPLE. COCOS, MD (TOP 1 - AUG 2014
because of its lack of which layer? FEB 2014 MED
A. mucosa BOARDS; TOPNOTCH
B. submucosa MD)
C. muscularis propria
D. Serosa

501 What is the maximum amount of 1% lidocaine There were 3 board questions of this type in my JAN CHARMAINE BACK-UP
solution that you can give a 70kg man? time. toxic dose of lidocaine=5mg/kg, for any PALOMAR, MD (TOP 9 - MIDTERM
A. 35 mL drug or solution, 1%=10mg/mL, for a 70kg FEB 2014 MED EXAM AUG
B. 45 mL patient: 70kgx 5mg/kg=350mg toxic dose; if 1% BOARDS; TOPNOTCH 2014
C. 55 mL solution is used: 350mg/10mg/mL=35mL MD)
D. 65 mL
E. 75 mL

502 A 39 year old man was involved in a shooting Immediate management of patient with tension JAN CHARMAINE BACK-UP
incident and was rushed to the ER for the multiple pneumothorax is needle thoracostomy (2nd ICS PALOMAR, MD (TOP 9 - MIDTERM
gun shot wounds to the abdomen and chest. MCL); Definitive management is CTT FEB 2014 MED EXAM AUG
Physical exam revealed patient to have BP of BOARDS; TOPNOTCH 2014
60/40mmHg, with labored breathing, cold clammy MD)
skin and tachycardia at 160bpm.Right side of chest
is hyperresonant to percussion with absence of
breath sounds. Trachea is deviated to the
right.What should you do immediately?
A. Do Chest x-ray
B. Exploratory laparotomy
C. Do needle thoracostomy
D. Observe
E. Insert chest tube now
TOPNOTCH MEDICAL BOARD PREP SURGERY SUPEREXAM Page 65 of 94
For inquiries visit www.topnotchboardprep.com.ph or email us at topnotchmedicalboardprep@gmail.com
TOPNOTCH MEDICAL BOARD PREP SURGERY SUPEREXAM
For inquiries visit www.topnotchboardprep.com.ph or email us at topnotchmedicalboardprep@gmail.com
Item QUESTION EXPLANATION AUTHOR TOPNOTCH
# EXAM
503 What do you call the impaction of a stone in the A. Mittelschmerz- lower quadrant pain due to JAN CHARMAINE BACK-UP
infundibulum of the gallbladder that may ovulation; Mirrizi's syndrome-extrinsic PALOMAR, MD (TOP 9 - MIDTERM
mechanically obstruct the bile duct resulting to obstruction of the common bile duct from a FEB 2014 MED EXAM AUG
jaundice? cystic duct gallstone; C. Miller Fisher BOARDS; TOPNOTCH 2014
A. Mittelschmerz syndrome-variant of Guillain-Barre where MD)
B. Mirrizi syndrome cranial nerves are affected, leading to triad of
C. Miller Fisher syndrome ataxia, areflexia and ophthalmoplegia; D.
D. Meig's syndrome Meig's syndrome- unuasual combination of
E. None of the above hydrothorax, ascites, and ovarian fibroma
504 A 63 year old male was brought in your clinic In the absence of gallstones, malignant JAN CHARMAINE BACK-UP
because of complaints of vague epigastric pain obstruction of the bile duct is likely, and a CT PALOMAR, MD (TOP 9 - MIDTERM
that penetrates to the back for several months scan rather than ERCP would be the next logical FEB 2014 MED EXAM AUG
now with associated jaundice and weight loss. On step. The current diagnostic and staging test of BOARDS; TOPNOTCH 2014
physical exam, skin is icteric, gallbladder was choice for pancreatic cancer is a multidetector, MD)
distended and palpable.Abdominal ultrasound dynamic, contrast-enhanced CT-scan. Schwartz,
revealed no gall stones. What would you do next? 9th ed., 1222.
A. Reassure patient.
B. Do ERCP to clear the bile duct.
C. Do a CT scan.
D. Do scout film of the abdomen.
E. Do exploratory laparotomy.

505 Reynolds pentad of cholangitis include the also: mental status changes JAN CHARMAINE BACK-UP
following signs and symptoms, except: PALOMAR, MD (TOP 9 - MIDTERM
A. fever FEB 2014 MED EXAM AUG
B. jaundice BOARDS; TOPNOTCH 2014
C. septic shock MD)
D. right upper quadrant pain
E. all of the above

506 What causes the initial vague, dull, diffuse Schwatz 9th ed., 1075 JAN CHARMAINE BACK-UP
epigastric pain in acute appendicitis? PALOMAR, MD (TOP 9 - MIDTERM
A. It is caused by the inflammatory process that FEB 2014 MED EXAM AUG
involves the serosa of the appendix and in turn the BOARDS; TOPNOTCH 2014
parietal peritoneum in the region. MD)
B. Distention of the appendix stimulates the
nerve endings of visceral afferent stretch fibers
C. Appendiceal perforation
D. All of the above
E. None of the above
507 A 6 year old little girl was brought to ER because Schwartz 9th ed., 1080 JAN CHARMAINE BACK-UP
of abdominal pain which started 12 hours prior to PALOMAR, MD (TOP 9 - MIDTERM
consult. History revealed that patient has just FEB 2014 MED EXAM AUG
recovered from acute nasopharyngitis. Physical BOARDS; TOPNOTCH 2014
examination showed stable vital signs with MD)
presence of voluntary guarding with no true
rigidity and diffuse abdominal tenderness rated as
5/10 in greatest intensity. What is the disease
most often confused with acute appendicitis in
children?
A. Meckel's diverticulitis
B. Crohn's enteritis
C. Urinary tract infection
D. Acute mesenteric adenitis
E. Henoch-Schonlein purpura
508 A 26 year old man was brought to the ER because 1 point each: migration of pain, anorexia, nausea JAN CHARMAINE BACK-UP
of severe right lower quadrant pain associated and miting, rebound tenderness, elevated temp, PALOMAR, MD (TOP 9 - MIDTERM
with fever, nausea and vomiting and loss of leukocytosis, left shift in leukocyte count; 2 FEB 2014 MED EXAM AUG
appetite which started the night before ER consult. points= RLQ tenderness and leukocytosis BOARDS; TOPNOTCH 2014
Physical examination revealed BP of MD)
130/90mmHg, HR=108bpm, RR=18rpm,
Temperature=38.9C. There is direct right lower
quadrant and rebound tenderness with positive
Psoas sign. CBC showed WBC count of
17,000cells/mm3 with predominance of
neutrophils. what is the Alvarado score?
A. 12
B. 11
C. 8
D. 10
E. 9
509 What is currently the most accurate and most Schwatz 9th ed., 1045 JAN CHARMAINE BACK-UP
complete method for examining the large bowel PALOMAR, MD (TOP 9 - MIDTERM
that is highly sensitive for detecting even small FEB 2014 MED EXAM AUG
polyps and allows biopsy, polypectomy, control of BOARDS; TOPNOTCH 2014
hemorrhage, and dilation of strictures? MD)
A. Colonoscopy
B. Air contrast enema
C. Flexible sigmoidoscopy
D. Digital rectal exam
E. Whole abdominal ultrasound

TOPNOTCH MEDICAL BOARD PREP SURGERY SUPEREXAM Page 66 of 94


For inquiries visit www.topnotchboardprep.com.ph or email us at topnotchmedicalboardprep@gmail.com
TOPNOTCH MEDICAL BOARD PREP SURGERY SUPEREXAM
For inquiries visit www.topnotchboardprep.com.ph or email us at topnotchmedicalboardprep@gmail.com
Item QUESTION EXPLANATION AUTHOR TOPNOTCH
# EXAM
510 A 56 year old man was recently diagnosed of Ileocolic resection- removes disease in the JAN CHARMAINE BACK-UP
colonic adenocarcinoma located in the hepatic terminal ileum, cecum and appendix; right PALOMAR, MD (TOP 9 - MIDTERM
flexure. What is is most appropriate colectomy to colectomy- for proximal colon carcinoma; FEB 2014 MED EXAM AUG
perform in this patient? extended right colectomy- lesions located at the BOARDS; TOPNOTCH 2014
A. Ileocolic resection hepatic flexure or proximal transverse colon; MD)
B. Right colectomy transverse colectomy- for lesions in the mid and
C. Extended right colectomy distal transverse colon -Schwartz 9th ed., 1026
D. Transverse colectomy
E. None of the above
511 Which of the following hormones is expected to Gonadotropin and Sex hormones, Prolactin in JAN CHARMAINE BACK-UP
decrease following stress or injury? children, Insulin and insulin-like growth factors PALOMAR, MD (TOP 9 - MIDTERM
A. Cortisol and Thyroid hormones, decrease following FEB 2014 MED EXAM AUG
B. Thyroxine trauma or injury (SIT down). The rest of the BOARDS; TOPNOTCH 2014
C. Glucagon hormones increase. MD)
D. Epinephrine
E. Growth hormone

512 The following body fluids have similar electrolyte SIMILAR TO PREVIOUS BOARD EXAM JAN CHARMAINE BACK-UP
composition to plasma, except: CONCEPT/PRINCIPLE: Pearls PALOMAR, MD (TOP 9 - MIDTERM
A. Duodenum FEB 2014 MED EXAM AUG
B. Bile BOARDS; TOPNOTCH 2014
C. Ileum MD)
D. Colon
E. Pancreas

513 What is the most important treatment in Pearls JAN CHARMAINE BACK-UP
metabolic acidosis? PALOMAR, MD (TOP 9 - MIDTERM
A. Fluid resuscitation to restore perfusion FEB 2014 MED EXAM AUG
B. Bicarbonate infusion BOARDS; TOPNOTCH 2014
C. Alkalinization of urine MD)
D. Antibiotics administration
E. None of the above

514 The following constitutes a positive diagnostic In an anterior abdominal stab wound an RBC JAN CHARMAINE BACK-UP
peritoneal lavage in a patient who sustained an count of >100,000/mL constitute a (+) DPL; PALOMAR, MD (TOP 9 - MIDTERM
anterior abdominal wall stab wound, except: >10,000/mL if thoracoabdominal stab wounds. FEB 2014 MED EXAM AUG
A. Red cell count >10, 000/mL The rest of the choices are correct for both BOARDS; TOPNOTCH 2014
B. WBC count >500/mL anterior abomen and thoracoabdominal stab MD)
C. Amylase >19 IU/L wounds. Schwartz 9th ed., 155
D. Alkaline phosphatase >2 IU/L
E. Bilirubin >0.01mg/dL

515 Which of the following patients would not require Indications for operative treatment of thoracic JAN CHARMAINE BACK-UP
thoracotomy? injuries: 1. initial tube thoracostomy drainage of PALOMAR, MD (TOP 9 - MIDTERM
A. A patient sustaining stab wound in the chest >1000mL (penetrating injury) or >1500 mL FEB 2014 MED EXAM AUG
with an initial tube thoracostomy drainage of (Blunt injury) 2. Ongoing tube thoracostomy BOARDS; TOPNOTCH 2014
>800mL drainage of >200mL/h for 3 consecutive hours MD)
B. A patient with ongoing tube thoracostomy in non-coagulopathic patients 3. Caked
drainage of 800mL in a span of 3 hours hemothorax despite placement of 2 chest tubes
C. A patient with 2 chest tubes with caked 4. selected descending torn aortas 5. Great
hemothorax vessel injury 6. pericardial tamponade 7. cardiac
D. A patient with open pneumothorax herniation 8. massive air leak from the chest
E. All of the above patients require thoracotomy tube with inadequate ventilation 9. tracheal or
main stem bronchial injury diagnosed by
endoscopy or imaging 10. open pneumothorax
11. esophageal perforation -Schwartz 9th ed.,
172
516 Quinke's triad of hemobilia include the following, Schwartz 9th ed., 177 JAN CHARMAINE BACK-UP
except: PALOMAR, MD (TOP 9 - MIDTERM
A. Right upper quadrant pain FEB 2014 MED EXAM AUG
B. Upper GI bleeding BOARDS; TOPNOTCH 2014
C. Jaundice MD)
D. Fever
E. None of the above

517 The following statements reagrding Peyer's patches are located in the ileum. JAN CHARMAINE BACK-UP
intussusception is true, except: PALOMAR, MD (TOP 9 - MIDTERM
A. Intussusception is the leading cause of FEB 2014 MED EXAM AUG
intestinal obstruction in the young child. BOARDS; TOPNOTCH 2014
B. Hypertrophy of Peyer's patches in the MD)
duodenum from an antecedent viral infection acts
as a lead point.
C. Peristaltic action of the intestine then causes
the bowel distal to the lead poin to invaginate into
itself.
D. Between paroxysms of crampy abdominal
pain and intermittent vomiting, the infacnt may
act normally.
E. Positive Dance's sign show elongated mass in
the right upper quadrant with an absence of bowel
in the right lower quadrant.

TOPNOTCH MEDICAL BOARD PREP SURGERY SUPEREXAM Page 67 of 94


For inquiries visit www.topnotchboardprep.com.ph or email us at topnotchmedicalboardprep@gmail.com
TOPNOTCH MEDICAL BOARD PREP SURGERY SUPEREXAM
For inquiries visit www.topnotchboardprep.com.ph or email us at topnotchmedicalboardprep@gmail.com
Item QUESTION EXPLANATION AUTHOR TOPNOTCH
# EXAM
518 A 43 year old man was rushed to the ER because of Patient is bleeding inside. UGIB is still more JAN CHARMAINE BACK-UP
weakness and dizziness. History revealed 3 common than LGIB, especially in young adults. PALOMAR, MD (TOP 9 - MIDTERM
episodes of drak red bloody bowel movement, the Place NG tube and aspirate to define the area FEB 2014 MED EXAM AUG
last episode occuring 15 minutes ago. BP is from which he is bleeding. Do endoscopy if NG BOARDS; TOPNOTCH 2014
90/70mmHg and HR=118bpm, skin is cold and tube returns significant amount of bright red MD)
clammy. What should you do next? blood.
A. Do upper GI endoscopy
B. Do emergency colonoscopy
C. Insert NGT and aspirate
D. Transfuse type specific packed RBC
immediately
E. Reaasure but do close observation of patient
519 A 55 year old man falls on his outstretched hand. Snell 8th ed., 480 B. Distal fragment is JAN CHARMAINE BACK-UP
He comes to the ER with a deformed wrist that displaced anteriorly-Smith fracture; C. Shaft of PALOMAR, MD (TOP 9 - MIDTERM
looks like a fork. How would his X-ray show if he ulna is fractured- Monteggia's fracture FEB 2014 MED EXAM AUG
has Colles fracture? BOARDS; TOPNOTCH 2014
A. Dorsally displaced distal fragment MD)
B. Distal fragment is displaced anteriorly
C. Shaft of ulna is fractured
D. Dislocation of the lunate bone
E. None of the above

520 Where is the most frequent site of peptic ulcer? The Absite Review 3rd edition, 222. JAN CHARMAINE BACK-UP
A. 1st part of duodenum, posteriorly PALOMAR, MD (TOP 9 - MIDTERM
B. 1st part of duodenum, anteriorly FEB 2014 MED EXAM AUG
C. Antrum of stomach BOARDS; TOPNOTCH 2014
D. Body of stomach MD)
E. None of the above

521 A 37 year old man comes to the physician 12 MIGUEL RAFAEL MIDTERM 1
hours after the onset of vomiting and abdominal RAMOS, MD (TOP 3 - EXAM - FEB
cramps and swelling. He has had constipation for FEB 2012 MED 2013
the past 4 days. He was diagnosed with Crohn BOARDS; TOPNOTCH
disease 7 years ago. Vital signs are stable. MD)
Examination shows a diffusely distended,
tympanitic abdomen and visible peristalsis; high
pitched bowel sounds are heard. Rectal
examination shows no stool in the rectal vault. An
x-ray of the abdomen shows a small-bowel
obstruction. Which of the following is the most
likely cause of these findings?
A) Colon cancer
B) Ilocecal fistula
C) Small bowel adhesions
D) Small bowl fibrotic stricture
522 Three days after undergoing colectomy for colon MIGUEL RAFAEL MIDTERM 1
cancer, 77 year old women complains of being RAMOS, MD (TOP 3 - EXAM - FEB
unable to catch her breath. Current medications FEB 2012 MED 2013
include subcutaneous heparin and patient- BOARDS; TOPNOTCH
controlled morphine. Her temperature is 37 C, MD)
pulse is 140 bpm, respirations are 28 cpm, and
blood pressure is 110/60 mmHg. Pulse oximetry
on room air sows an oxygen saturation of 89%.
Arterial blood gas analysis reveals pH 7.38 pCO2
23 pO2 55. An ECG shows nonspecific ST-T wave
changes. In addition to oxygen therapy, which of
the following is the most appropriate next step in
management?
A) Spiral CT scan of the chest
B) Anti-embolic stockings
C) Intravenous administration of a recombinant
tissue plasminogen activator
D) Pulmonary angiography
523 A 37 year old woman comes to the physician MIGUEL RAFAEL MIDTERM 1
because of increasingly severe pain and masses in RAMOS, MD (TOP 3 - EXAM - FEB
both breasts over the past 4 months. She has a 12 FEB 2012 MED 2013
year history of similar episodes that were not as BOARDS; TOPNOTCH
severe. The masses vary in size with her menstrual MD)
cycles. She has used an oral contraceptive for 16
years. Which of the following is the most likely
diagnosis?
A) Oral contraceptive induced breast changes
B) Ductal papilloma
C) Fibroadenoma
D) Fibrocystic changes

TOPNOTCH MEDICAL BOARD PREP SURGERY SUPEREXAM Page 68 of 94


For inquiries visit www.topnotchboardprep.com.ph or email us at topnotchmedicalboardprep@gmail.com
TOPNOTCH MEDICAL BOARD PREP SURGERY SUPEREXAM
For inquiries visit www.topnotchboardprep.com.ph or email us at topnotchmedicalboardprep@gmail.com
Item QUESTION EXPLANATION AUTHOR TOPNOTCH
# EXAM
524 A 70-year-old woman has had increasing MIGUEL RAFAEL MIDTERM 1
abdominal pain over the past 2 days. She has renal RAMOS, MD (TOP 3 - EXAM - FEB
failure and has been receiving peritoneal dialysis FEB 2012 MED 2013
for 18 months; her last treatment was 2 hours ago. BOARDS; TOPNOTCH
She appears toxic. Her temperature is 39 C (102.2 MD)
F), and blood pressure is 140/90 mm Hg. Her
abdomen is distended and diffusely tender to deep
palpation with rebound tenderness. Leukocyte
count is 18,000/mm3. Which of the following is
the most appropriate next step?
A. Comparison of abdominal fluid amylase with
serum amylase activity
B. Gram's stain of abdominal fluid
C. Ultrasonography of the abdomen
D. CT scan of the abdomen and pelvis
525 A previously healthy 62-year-old man comes to MIGUEL RAFAEL MIDTERM 1
the emergency department because of abdominal RAMOS, MD (TOP 3 - EXAM - FEB
pain for 48 hours. His temperature is 38.6 C (101.5 FEB 2012 MED 2013
F), blood pressure is 130/80 mm Hg, pulse is BOARDS; TOPNOTCH
110/min, and respirations are 15/min. Abdominal MD)
examination shows diffuse left lower quadrant
tenderness with no peritoneal signs. Rectal
examination shows no abnormalities; test of the
stool for occult blood is negative. His leukocyte
count is 14,700/mm3. Which of the following is
the most appropriate next step in diagnosis?
A) Exploratory laparotomy
B) CT scan of the abdomen
C) Colonoscopy
D) Cystoscopy
526 A 77-year-old woman comes to the physician MIGUEL RAFAEL MIDTERM 1
because of a 2-day history of cramping abdominal RAMOS, MD (TOP 3 - EXAM - FEB
pain and distention accompanied by nausea and FEB 2012 MED 2013
vomiting. She is otherwise healthy and has no BOARDS; TOPNOTCH
history of abdominal operations. Her temperature MD)
is 37.4 C (99.4 F), blood pressure is 110/86 mm
Hg, pulse is 112/min, and respirations are 24/min.
Cardiopulmonary examination shows no
abnormalities. Examination of the abdomen shows
distention and mild diffuse tenderness; bowel
sounds are high-pitched. An x-ray film of the
abdomen shows air-fluid levels throughout the
small bowel and air in the liver; there is no gas in
the colon or free air. Which of the following is the
most likely diagnosis?
A) Adhesive small-bowel obstruction
B) Cecal cancer
C) Gallstone ileus
D) Mesenteric infarction
527 An 18-year-old man comes for an examination MIGUEL RAFAEL MIDTERM 1
prior to participation in school sports. He states RAMOS, MD (TOP 3 - EXAM - FEB
that he has had a dull ache in the scrotum since FEB 2012 MED 2013
being hit in that area during a basketball game 2 BOARDS; TOPNOTCH
months ago. Examination shows a 2-cm, hard, MD)
nontender mass in the right testicle. The mass
does not transilluminate or change in size when
the patient is placed in the supine position. Which
of the following is the most likely cause?
A) Cystic dilations of the efferent ductules
B) Dilated pampiniform venous plexus
C) Fluid accumulation within the tunica vaginalis
testis
D) Germinal cell tumor
528 A 64-year-old woman has moderately severe Patient-controlled analgesia is the treatment of MIGUEL RAFAEL MIDTERM 1
postoperative pain 1 day after a total abdominal choice for moderate to severe postop pain RAMOS, MD (TOP 3 - EXAM - FEB
hysterectomy and bilateral salpingo- FEB 2012 MED 2013
oophorectomy. Which of the following is the most BOARDS; TOPNOTCH
appropriate analgesic pharmacotherapy? MD)
A) Patient-controlled intravenous morphine
B) Oral diazepam
C) Oral ibuprofen
D) Intermittent intravenous naloxone

TOPNOTCH MEDICAL BOARD PREP SURGERY SUPEREXAM Page 69 of 94


For inquiries visit www.topnotchboardprep.com.ph or email us at topnotchmedicalboardprep@gmail.com
TOPNOTCH MEDICAL BOARD PREP SURGERY SUPEREXAM
For inquiries visit www.topnotchboardprep.com.ph or email us at topnotchmedicalboardprep@gmail.com
Item QUESTION EXPLANATION AUTHOR TOPNOTCH
# EXAM
529 A 67-year-old woman is brought to the emergency MIGUEL RAFAEL MIDTERM 1
department because of severe chest pain 4 hours RAMOS, MD (TOP 3 - EXAM - FEB
after undergoing outpatient endoscopy and FEB 2012 MED 2013
dilatation of an esophageal stricture caused by BOARDS; TOPNOTCH
reflux. At discharge, she reported no chest pain. MD)
Three hours later, she vomited a small amount of
blood and had severe pain. She is pale. Her
temperature is 38 C (100.4 F), blood pressure is
140/85 mm Hg, pulse is 125/min, and respirations
are 22/min. Examination shows crepitus in the
neck and moderate epigastric tenderness. The
lungs are clear to auscultation, and breath sounds
are equal bilaterally. Rectal examination shows no
masses; test of the stool for occult blood is
positive. Which of the following is the most likely
cause of these symptoms?
A) Bleeding from erosive esophagitis
B) Esophageal perforation
C) Mallory-Weiss syndrome
D) Perforated gastric ulcer
530 A 67-year-old woman comes to the physician MIGUEL RAFAEL MIDTERM 1
because she has had a lesion on her right eyelid for RAMOS, MD (TOP 3 - EXAM - FEB
3 months. She has no history of similar lesions. FEB 2012 MED 2013
Examination shows a 2 x 1.5-cm lesion on the BOARDS; TOPNOTCH
lower lid of the right eye. A biopsy specimen of the MD)
lesion shows basal cell carcinoma. Which of the
following is the most appropriate step in
management?
A) Wide (1-cm) excision
B) Mohs micrographic surgery
C) Radiation therapy
D) Chemotherapy
531 A 37-year-old man is brought to the emergency MIGUEL RAFAEL MIDTERM 1
department 6 hours after the onset of constant, RAMOS, MD (TOP 3 - EXAM - FEB
increasingly severe abdominal pain and nausea. FEB 2012 MED 2013
His symptoms awoke him from sleep, and he has BOARDS; TOPNOTCH
vomited once since that time. He is in acute MD)
distress and lying in the fetal position. Any
movement exacerbates the pain. His temperature
is 37.8 C (100 F), blood pressure is 108/68 mm
Hg, pulse is 112/min, and respirations are 24/min.
Examination shows a rigid abdomen; bowel
sounds are absent. Laboratory studies show:

Hemoglobin 14 g/dL
Leukocyte count 18,200/mm3
Platelet count 150,000/mm3
Urea nitrogen (BUN) 34 mg/dL
Creatinine 1.9 mg/dL
Total bilirubin 1.2 mg/dL

An x-ray film of the chest shows a small amount of
free air under the left diaphragm. Administration
of antibiotics and fluids is begun. Which of the
following is the most appropriate next step in
management?
A) Barium swallow
B) CT scan of the abdomen
C) Intravenous administration of an H2-receptor
blocking agent
D) Laparotomy
532 A 19-year-old man is brought to the emergency MIGUEL RAFAEL MIDTERM 1
department 45 minutes after sustaining a single, RAMOS, MD (TOP 3 - EXAM - FEB
large stab wound to the right upper quadrant of FEB 2012 MED 2013
the abdomen. He is obtunded. His blood pressure BOARDS; TOPNOTCH
is 60/palpable mm Hg, and pulse is 148/min. MD)
Breath sounds are equal bilaterally. Examination
shows a 4-cm laceration in the right upper
quadrant in the midclavicular line. The abdomen is
distended. Which of the following is the most
appropriate next step in management?
A) X-ray films of the abdomen and pelvis
B) CT scan of the abdomen
C) Peritoneal lavage
D) Laparotomy

TOPNOTCH MEDICAL BOARD PREP SURGERY SUPEREXAM Page 70 of 94


For inquiries visit www.topnotchboardprep.com.ph or email us at topnotchmedicalboardprep@gmail.com
TOPNOTCH MEDICAL BOARD PREP SURGERY SUPEREXAM
For inquiries visit www.topnotchboardprep.com.ph or email us at topnotchmedicalboardprep@gmail.com
Item QUESTION EXPLANATION AUTHOR TOPNOTCH
# EXAM
533 A 70-year-old nursing home resident is admitted MIGUEL RAFAEL MIDTERM 1
to the hospital because of progressive obtundation RAMOS, MD (TOP 3 - EXAM - FEB
over the past 2 days. He has tachycardia, FEB 2012 MED 2013
tachypnea, and hypotension. Bilateral basilar BOARDS; TOPNOTCH
crackles and an S3 gallop are heard on MD)
auscultation.
Examination shows jugular venous distention and
peripheral edema. Swan-Ganzcatheterization
shows a cardiac index of 1.8 L/min/m2 (N=2.5–
4.2), a mean Pulmonary capillary wedge pressure
of 23 mm Hg (N=1–10), and markedly
increased systemic vascular resistance. Which of
the following is the most likely diagnosis?
A) Cardiogenic shock
B) Hypovolemic shock
C) Neurogenic shock
D) Septic shock
534 Six weeks after spontaneous drainage of an anal MIGUEL RAFAEL MIDTERM 1
abscess, a 32-year-old man has persistent blood- RAMOS, MD (TOP 3 - EXAM - FEB
stained purulent FEB 2012 MED 2013
fluid on his underwear. He has not had significant BOARDS; TOPNOTCH
anal pain since drainage of the boil. Bowel MD)
movements are normal. Which of the following is
the most likely diagnosis?
A) Anal fissure
B) Fistula in ano
C) Pruritus ani
D) Thrombosed external hemorrhoids
535 A previously healthy 16-year-old boy comes to the Elevating the left testicle relieves the pain >> MIGUEL RAFAEL MIDTERM 1
physician because of persistent pain in his left Epididymitis RAMOS, MD (TOP 3 - EXAM - FEB
testicle for 24hours. He has not had any penile FEB 2012 MED 2013
discharge. There is no history of trauma, but he BOARDS; TOPNOTCH
plays soccer every day. Two months ago, he had MD)
sexual intercourse for the first time, and he used a
condom. Examination shows an edematous,
erythematous, exquisitely tender left scrotum that
is lower than the right. Elevating the left testicle
relieves the pain. The cremasteric reflex is present.
Urinalysis shows 10 leukocytes/hpf and 1+
leukocyte esterase. A technetium 99m scan shows
increased uptake in the left testicle. Which of the
following is the most likely cause of the pain?
A) Cystitis
B) Epididymitis
C) Spermatocele
D) Testicular torsion
536 A 32-year-old woman has had a lump in her neck MIGUEL RAFAEL MIDTERM 1
for 6 months. She has a 1.2-cm solitary left thyroid RAMOS, MD (TOP 3 - EXAM - FEB
nodule. Fine-needle aspiration cytology of the FEB 2012 MED 2013
mass is consistent with a low-grade papillary BOARDS; TOPNOTCH
malignancy. Which of the following is the most MD)
appropriate next step in management?
A) Thyroid scan
B) 131I therapy
C) Propylthiouracil therapy
D) Left thyroid lobectomy

537 A 75-year-old man has had hypertension for 25 MIGUEL RAFAEL MIDTERM 1
years. There is an unusually prominent pulsation RAMOS, MD (TOP 3 - EXAM - FEB
of the abdominal aorta in the upper midabdomen. FEB 2012 MED 2013
A systolic bruit is heard at this site. Femoral, BOARDS; TOPNOTCH
popliteal, and pedal pulses are present. Which of MD)
the following is the most appropriate initial
diagnostic study?
A) X-ray film of the abdomen
B) Abdominal ultrasonography
C) Doppler ultrasonography of the arteries of the
legs
D) Abdominal aortography
538 A 77-year-old woman is brought to the emergency Most common cause of hematochezia in elderly MIGUEL RAFAEL MIDTERM 1
department after collapsing at home. Six hours >> diverticuLOSIS RAMOS, MD (TOP 3 - EXAM - FEB
ago, she had the sudden onset of massive bright FEB 2012 MED 2013
red rectal bleeding. On arrival, her blood pressure BOARDS; TOPNOTCH
is 90/60 mm Hg, and pulse is 120/min. Abdominal MD)
examination shows no abnormalities. Insertion of
a nasogastric tube yields clear aspirate. Her
hematocrit is 28%. Which of the following is the
most likely diagnosis?
A) Colon cancer
B) Diverticulosis
C) Duodenal ulcer
D) Diverticulitis

TOPNOTCH MEDICAL BOARD PREP SURGERY SUPEREXAM Page 71 of 94


For inquiries visit www.topnotchboardprep.com.ph or email us at topnotchmedicalboardprep@gmail.com
TOPNOTCH MEDICAL BOARD PREP SURGERY SUPEREXAM
For inquiries visit www.topnotchboardprep.com.ph or email us at topnotchmedicalboardprep@gmail.com
Item QUESTION EXPLANATION AUTHOR TOPNOTCH
# EXAM
539 A previously healthy 85-year-old man has had Surgery to relieve obstruction MIGUEL RAFAEL MIDTERM 1
abdominal distention, decreased caliber of stools, RAMOS, MD (TOP 3 - EXAM - FEB
and decreased appetite over the past 2 weeks and FEB 2012 MED 2013
a 9-kg (20-lb) weight loss over the past 3 months. BOARDS; TOPNOTCH
On sigmoidoscopy, he is found to have a MD)
constricting adenocarcinoma of the sigmoid colon;
imaging studies show three 1-cmnmetastases to
the liver. Which of the following is the most
appropriate next step in management?
A) Radiation therapy
B) Chemotherapy
C) Combination radiation therapy and
chemotherapy
D) Resection of the colon tumor
540 A 32-year-old man is brought to the emergency tension pneumothorax >> decreased venous MIGUEL RAFAEL MIDTERM 1
department 30 minutes after being involved in a return RAMOS, MD (TOP 3 - EXAM - FEB
motor vehicle collision. He was the restrained FEB 2012 MED 2013
driver. On arrival, he is alert and has shortness of BOARDS; TOPNOTCH
breath. His blood pressure is 80/50 mm Hg, pulse MD)
is 130/min, and respirations are 30/min.
Examination shows jugular venous distention and
abrasions
over the left hemithorax. The trachea is deviated
to the right. Breath sounds are absent on the left.
Which of the following is the most likely cause of
the hypotension?
A) Cardiogenic shock
B) Congestive heart failure
C) Decreased systemic vascular resistance
D) Decreased venous return
541 A 48/F, known case of gallstone disease, is ABDELSIMAR OMAR II, FINAL EXAM -
scheduled for elective open cholecystectomy. MD (TOP 2 - AUG 2013 FEB 2014
Obese and a known hypertensive and diabetic, she MED BOARDS;
is being maintained on Aspirin by her internist to TOPNOTCH MD - 200
reduce risk for cardiovascular events. How long QUESTIONS) AND
should one stop aspirin prior to surgery? MARC DENVER
A. 14 days TIONGSON, MD (40
B. 12 days QUESTIONS)
C. 7 days
D. 3 days
E. 1 day
542 A 22/F comes in with a straight facial laceration. The subcuticular running suture is often used by ABDELSIMAR OMAR II, FINAL EXAM -
To achieve good cosmetic outcome, which plastic surgeons to close straight lacerations on MD (TOP 2 - AUG 2013 FEB 2014
suturing technique and which suture type would the face. An absorbable suture, such as Monocryl MED BOARDS;
be most appropriate? or TOPNOTCH MD - 200
A. Simple interrupted, Vicryl Vicryl, is used. QUESTIONS) AND
B. Subcuticular running suture, Nylon MARC DENVER
C. Subcuticular running suture, Vicryl TIONGSON, MD (40
D. Vertical mattress, Vicryl QUESTIONS)
E. Horizontal mattress, Nylon
543 For patients on chronic corticosteroids, intake of Supplemental vitamin A can reverse the ABDELSIMAR OMAR II, FINAL EXAM -
this vitamin can reverse the inhibitory effects of inhibitory effects of corticosteroids on wound MD (TOP 2 - AUG 2013 FEB 2014
corticorsteroids on wound healing. Supplemental healing. Vitamin A also can restore wound MED BOARDS;
doses are thus recommended during the pre- and healing that has been impaired by diabetes, TOPNOTCH MD - 200
post-operative periods. tumor formation, cyclophosphamide, and QUESTIONS) AND
A. Vitamin A radiation. MARC DENVER
B. Pyridoxine TIONGSON, MD (40
C. Thiamine QUESTIONS)
D. Vitamin C
E. Vitamin D
544 For the multiply injured patient who sustained Remember the 'big three' radiographic exams ABDELSIMAR OMAR II, FINAL EXAM -
severe blunt trauma, the following diagnostic for severe blunt trauma. MD (TOP 2 - AUG 2013 FEB 2014
examinations are routinely ordered, except: MED BOARDS;
A. Lateral cervical spine radiograph TOPNOTCH MD - 200
B. Cranial CT scan QUESTIONS) AND
C. Chest X-ray MARC DENVER
D. Pelvic radiograph TIONGSON, MD (40
E. None of the above QUESTIONS)

545 A 28/M who crashed his car into another vehicle Patient with peritoneal signs? Do exploratory ABDELSIMAR OMAR II, FINAL EXAM -
was brought into the emergency room. Vital signs lap. MD (TOP 2 - AUG 2013 FEB 2014
on admission are as follows: BP 90/60, HR 118, RR MED BOARDS;
22, T 37.5 C. On PE, you note direct and rebound TOPNOTCH MD - 200
tenderness on all quadrants with guarding. Which QUESTIONS) AND
of the following would be your next step in MARC DENVER
management? TIONGSON, MD (40
A. Perform a Focused Abdominal Sonography for QUESTIONS)
Trauma (FAST)
B. Perform an abdominal CT scan
C. Send the patient to Radiology for chest, pelvic
and cervical spinal radiographs
D. Perform a diagnostic peritoneal lavage.
E. Perform an exploratory laparotomy

TOPNOTCH MEDICAL BOARD PREP SURGERY SUPEREXAM Page 72 of 94


For inquiries visit www.topnotchboardprep.com.ph or email us at topnotchmedicalboardprep@gmail.com
TOPNOTCH MEDICAL BOARD PREP SURGERY SUPEREXAM
For inquiries visit www.topnotchboardprep.com.ph or email us at topnotchmedicalboardprep@gmail.com
Item QUESTION EXPLANATION AUTHOR TOPNOTCH
# EXAM
546 A 45/M crashed his car into another vehicle; and Patient has pulmonary contusion associated ABDELSIMAR OMAR II, FINAL EXAM -
was brought to the ER. On PE, patient has multiple with a flail chest. Positive airway pressure and MD (TOP 2 - AUG 2013 FEB 2014
bruises over the chest. CXR showed multiple rib respiratory support are key to management. MED BOARDS;
fractures, no pneumohemothorax, and clear lung TOPNOTCH MD - 200
parenchyma. After two days in the ER, however, QUESTIONS) AND
patient started having respiratory distress. Repeat MARC DENVER
CXR showed "white out" of bilateral lungs. Which TIONGSON, MD (40
of the following steps is most important? QUESTIONS)
A. Insertion of a chest tube
B. Use of diuretics
C. Close observation
D. Fluid restriction
E. Intubation and mechanical ventilation with
positive pressure
547 A 38/F came in to your clinic for a firm, well- From the PCS CPG on breast masses: In patients ABDELSIMAR OMAR II, FINAL EXAM -
demarcated, movable 2 x 2 cm mass in the upper with a palpable breast mass in which cancer is MD (TOP 2 - AUG 2013 FEB 2014
outer quadrant of the left breast. Which of the suspected, BIOPSY is mandatory. ( Level I, MED BOARDS;
following would be the most appropriate next Category A ) Fine needle aspiration cytology ( TOPNOTCH MD - 200
step? FNAC ) is the initial diagnostic procedure in QUESTIONS) AND
A. Reassure the patient that this is a benign patients with a palpable breast mass in which MARC DENVER
condition and is not a cause for concern. cancer is suspected. TIONGSON, MD (40
B. Do fine needle aspiration for cytology. QUESTIONS)
C. Do core needle biopsy.
D. Do excisional biopsy of the mass.
E. Do a mammogram on both breasts.
548 Which of the following benign breast disorders Sclerosing adenosis and intraductal papilloma ABDELSIMAR OMAR II, FINAL EXAM -
does NOT confer an increased cancer risk? does NOT confer cancer risk. MD (TOP 2 - AUG 2013 FEB 2014
A. Lobular carcinoma in situ MED BOARDS;
B. Atypical ductal hyperplasia TOPNOTCH MD - 200
C. Sclerosing adenosis QUESTIONS) AND
D. Florid hyperplasia MARC DENVER
E. None of the above TIONGSON, MD (40
QUESTIONS)

549 A 55/F with a long-standing history of burning Barrett's esophagus is a premalignant condition, ABDELSIMAR OMAR II, FINAL EXAM -
retrosternal pain and heartburn, underwent necessitating antireflux surgery. MD (TOP 2 - AUG 2013 FEB 2014
endoscopy, which revealed peptic esophagitis and MED BOARDS;
Barett's esophagus. She has been taking PPIs with TOPNOTCH MD - 200
some relief of symptoms. What is the most QUESTIONS) AND
appropriate next step in therapy? MARC DENVER
A. Advise dietary modifications to reduce acid TIONGSON, MD (40
reflux. QUESTIONS)
B. Increase doses of proton pump inhibitors.
C. Do Nissen fundoplication.
D. Do esophagectomy.
E. Do repeat biopsy regularly to screen for cancer.
550 A 50/F presents with a year long history of Laparoscopic Heller cardiomyotomy remains ABDELSIMAR OMAR II, FINAL EXAM -
heartburn and chest pain, unrelated to food intake. the MOST effective treatment for achalasia. MD (TOP 2 - AUG 2013 FEB 2014
A few weeks prior to consult, he has developed MED BOARDS;
intermittent dysphagia to both solids and liquids, TOPNOTCH MD - 200
and as a result lost weight. Barium swallow done QUESTIONS) AND
revealed a dilated esophagus that tapers smoothly MARC DENVER
to a beak-like narrowing at the gastroesophageal TIONGSON, MD (40
junction. What is considered to be the MOST QUESTIONS)
effective treatment for this condition?
A. Calcium channel blockers and nitrates
B. Intrasphincteric injection of botulinum toxin
C. Pneumatic dilatation
D. Laparoscopic myotomy
E. Peroral endoscopic myotomy
551 A patient who underwent MRM (modified radical ABDELSIMAR OMAR II, FINAL EXAM -
mastectomy) for breast cancer complains of loss MD (TOP 2 - AUG 2013 FEB 2014
of sensation on medial aspect of arm. Patient MED BOARDS;
denies loss of motor movement. TOPNOTCH MD - 200
A. injury to the thoracodorsal nerve QUESTIONS) AND
B. injury to long thoracic nerve MARC DENVER
C. injury to interchostobranchial nerve TIONGSON, MD (40
D. hyperextension of arm during surgery QUESTIONS)

552 A 35 year old patient on her 10th week AOG ABDELSIMAR OMAR II, FINAL EXAM -
presents with a 3 cm breast mass right. On core MD (TOP 2 - AUG 2013 FEB 2014
needle biopsy, it showed invasive ductal MED BOARDS;
carcinoma, ER(+) PR (+). The most appropriate TOPNOTCH MD - 200
management would be: QUESTIONS) AND
A. modified radical mastectomy (MRM) + adjuvant MARC DENVER
chemotherapy TIONGSON, MD (40
B. breast conservation + adjuvant chemotherapy QUESTIONS)
C. neoadjuvant chemotherapy then MRM post-
partum
D. MRM and bilateral oopherectomy
E. Watchful waiting

TOPNOTCH MEDICAL BOARD PREP SURGERY SUPEREXAM Page 73 of 94


For inquiries visit www.topnotchboardprep.com.ph or email us at topnotchmedicalboardprep@gmail.com
TOPNOTCH MEDICAL BOARD PREP SURGERY SUPEREXAM
For inquiries visit www.topnotchboardprep.com.ph or email us at topnotchmedicalboardprep@gmail.com
Item QUESTION EXPLANATION AUTHOR TOPNOTCH
# EXAM
553 A 28 year old Male was referred to GS 1 for orphan annie ABDELSIMAR OMAR II, FINAL EXAM -
evaluation of ANM. On histological examination of MD (TOP 2 - AUG 2013 FEB 2014
the thyroid gland revealed large nuclei with pale MED BOARDS;
staining, ground glass appearance. What is the TOPNOTCH MD - 200
diagnosis? QUESTIONS) AND
A. Colloid adenomatous goiter MARC DENVER
B. Papillary thyroid carcinoma TIONGSON, MD (40
C. Follicular thyroid carcinoma QUESTIONS)
D. Medullary thyroid carcinoma
E. Anaplastic thyroid carcionoma

554 48/M blacksmith had an ifected wound over a cut ABDELSIMAR OMAR II, FINAL EXAM -
on the right arm. It presented with purulent MD (TOP 2 - AUG 2013 FEB 2014
discharge and some granulation. What would be MED BOARDS;
the best management? TOPNOTCH MD - 200
A. Close the wound primarily QUESTIONS) AND
B. Advise the patient on wound care and close the MARC DENVER
wound at a later date TIONGSON, MD (40
C. Advise patient on daily wound cleaning and let QUESTIONS)
the wound heal by secondary intention
D. Wash and clean the wound well and close
primarily
E. Do debridement
555 A patient with RUQ abdominal pain and fever was ABDELSIMAR OMAR II, FINAL EXAM -
brought to the MD (TOP 2 - AUG 2013 FEB 2014
OPD. On PE, there was a note of tenderness on the MED BOARDS;
RUQ but no jaundice. TOPNOTCH MD - 200
He brought an UTZ done outside with note of QUESTIONS) AND
predominantly cystic mass MARC DENVER
at the right lobe. the most probable diagnosis is: TIONGSON, MD (40
A. hepatic cyst QUESTIONS)
B. amoebic abscess
C. pyogenic abscess
D. hepatic adenoma
E. hepatocellular carcinoma

556 The most common location of the superior ABDELSIMAR OMAR II, FINAL EXAM -
parathyroid gland is: MD (TOP 2 - AUG 2013 FEB 2014
A. Dorsal to the recurrent laryngeal nerve, within MED BOARDS;
1 cm of the junction of the RLN and inferior TOPNOTCH MD - 200
thyroid artery QUESTIONS) AND
B. Ventral to the RLN, within within 1 cm of the MARC DENVER
junction of the RLN and inferior thyroid artery TIONGSON, MD (40
C. Dorsal to the recurrent laryngeal nerve, within 3 QUESTIONS)
cm of the junction of the RLN and inferior thyroid
artery
D. Ventral to the RLN, within within 3 cm of the
junction of the RLN and inferior thyroid artery
557 The most common ectopic mucosa present in ABDELSIMAR OMAR II, FINAL EXAM -
Meckel’s diverticulum is: MD (TOP 2 - AUG 2013 FEB 2014
A. Colonic mucosa MED BOARDS;
B. Pancreatic mucosa TOPNOTCH MD - 200
C. Ovarian mucosa QUESTIONS) AND
D. Gastric mucosa MARC DENVER
TIONGSON, MD (40
QUESTIONS)
558 Which of the following is the most significant risk ABDELSIMAR OMAR II, FINAL EXAM -
factor for invasive breast cancer when screening a MD (TOP 2 - AUG 2013 FEB 2014
patient for risk? MED BOARDS;
A. >2 first-degree relatives with breast cancer TOPNOTCH MD - 200
B. 2 previous breast biopsies in a patient <50 QUESTIONS) AND
years of age MARC DENVER
C. Age <12 at menarche TIONGSON, MD (40
D. Atypical hyperplasia in a previous breast biopsy QUESTIONS)
559 A 34 y.o. male comes into the ER with the ABDELSIMAR OMAR II, FINAL EXAM -
following vital signs HR 120s BP 90/60 RR 30. On MD (TOP 2 - AUG 2013 FEB 2014
initial survey, patient was noted to have multiple MED BOARDS;
stab wounds in L lateral neck (gurgling), 4th ICS R TOPNOTCH MD - 200
midclavicular (profusely bleeding), RLQ with QUESTIONS) AND
intestinal evisceration (non-bleeding) and L groin MARC DENVER
with pulsatile bleeding. The appropriate step in TIONGSON, MD (40
the management of this patient is: QUESTIONS)
A. Vaselinized gauze over L neck
B. Vaselinized gauze over R chest
C. Make sure the patient has a patent airway
D. Pressure over the L groin

TOPNOTCH MEDICAL BOARD PREP SURGERY SUPEREXAM Page 74 of 94


For inquiries visit www.topnotchboardprep.com.ph or email us at topnotchmedicalboardprep@gmail.com
TOPNOTCH MEDICAL BOARD PREP SURGERY SUPEREXAM
For inquiries visit www.topnotchboardprep.com.ph or email us at topnotchmedicalboardprep@gmail.com
Item QUESTION EXPLANATION AUTHOR TOPNOTCH
# EXAM
560 A 72 yo male complains to have decreased stool ABDELSIMAR OMAR II, FINAL EXAM -
caliber with blood streaks and weight loss (~15% MD (TOP 2 - AUG 2013 FEB 2014
over 3 months). The patient is suffering from: MED BOARDS;
A. Adenocarcinoma found on flexible TOPNOTCH MD - 200
sigmoidoscopy, 22 cm from anal verge QUESTIONS) AND
B. Squamous cell carcinoma at anal verge MARC DENVER
C. Adenocarcinoma found on TIONGSON, MD (40
esophagogastroduodenoscopy, 3 cm distal from QUESTIONS)
esophagogastric junction in lesser curvature
D. Adenocarcinoma found on colonoscopy, at
cecum.
561 The surgical margins of a patient diagnosed with 2 cm is the margin for melanoma with 2-4mm in BLAKE WARREN ANG, MIDTERM 2
melanoma staged at T3N0M0 is: depth (T3) MD (TOP 1 - AUG 2013 EXAM - FEB
a. 1cm MED BOARDS; 2014
b. 2cm TOPNOTCH MD)
c. 3cm
d. 4cm

562 A 24 year old female, 5'0'' weighing 140lbs, 24 hour pH monitoring to document GERD. BLAKE WARREN ANG, MIDTERM 2
complained of recurrent chest tightness with MD (TOP 1 - AUG 2013 EXAM - FEB
ascending burning pain came to your clinic. the MED BOARDS; 2014
best method to confirm your impression is: TOPNOTCH MD)
a. esophageal manometry
b. 24 hour pH monitoring
c. esophagoduodenoscopy
d. upper GI series

563 A patient presents to you with a left preauricular Adenoid cystic Ca is notable for its perineural BLAKE WARREN ANG, MIDTERM 2
mass that was irregular, and firm in consistency invasion MD (TOP 1 - AUG 2013 EXAM - FEB
that has persisted for more than a year. the mass MED BOARDS; 2014
was characteristically noted to be painful. the TOPNOTCH MD)
tumor most likely present
in this patient is:
a. pleomorphic adenoma
b. mucoepidermoid carcinoma
c. adenoid cystic carcinoma
d. acinic cell carcinoma

564 The nerve in close proximity to the submandibular BLAKE WARREN ANG, MIDTERM 2
duct that needs to be identified during resection of MD (TOP 1 - AUG 2013 EXAM - FEB
the gland is the: MED BOARDS; 2014
a. greater auricular nerve TOPNOTCH MD)
b. lingual nerve
c. glossopharyngeal nerve
d. anterior jugular nerve

565 The anterior belly of the digastric muscle, the BLAKE WARREN ANG, MIDTERM 2
hyoid bone, and the midline of the neck delineate MD (TOP 1 - AUG 2013 EXAM - FEB
what triangle of the neck? MED BOARDS; 2014
a. submental TOPNOTCH MD)
b. muscular
c. carotid
d. digastric

566 A true anatomic precursor of breast cancer: : DCIS. BLAKE WARREN ANG, MIDTERM 2
a. LCIS LCIS is only a marker of increased risk since MD (TOP 1 - AUG 2013 EXAM - FEB
b. DCIS Invasive ductal Ca develops more than Invasive MED BOARDS; 2014
c. Intraductal papilloma lobular Ca in a patient who had history of LCIS. TOPNOTCH MD)
d. Sclerosing adenosis

567 The triad of right upper quadrant pain, palpable Choledochal cyst; Ascending cholangitis BLAKE WARREN ANG, MIDTERM 2
mass, and jaundice describe this condition: characteristically presents with Charcot’s triad MD (TOP 1 - AUG 2013 EXAM - FEB
a. ascending cholangitis of pain fever and jaundice. MED BOARDS; 2014
b. acute cholecystitis TOPNOTCH MD)
c. choledochal cyst
d. hepatic adenoma

568 The most common cause of acute appendicitis in BLAKE WARREN ANG, MIDTERM 2
children is: MD (TOP 1 - AUG 2013 EXAM - FEB
a. fecalith MED BOARDS; 2014
b. Enterobius vermicularis TOPNOTCH MD)
c. typhoid ileitis
d. lymphoid hyperplasia

TOPNOTCH MEDICAL BOARD PREP SURGERY SUPEREXAM Page 75 of 94


For inquiries visit www.topnotchboardprep.com.ph or email us at topnotchmedicalboardprep@gmail.com
TOPNOTCH MEDICAL BOARD PREP SURGERY SUPEREXAM
For inquiries visit www.topnotchboardprep.com.ph or email us at topnotchmedicalboardprep@gmail.com
Item QUESTION EXPLANATION AUTHOR TOPNOTCH
# EXAM
569 The most common location of the appendix is: BLAKE WARREN ANG, MIDTERM 2
a. retrocecal MD (TOP 1 - AUG 2013 EXAM - FEB
b. pelvic MED BOARDS; 2014
c. femoral TOPNOTCH MD)
d. obturator

570 The layer posterior to the rectus abdominis Below the arcuate line, the posterior layer BLAKE WARREN ANG, MIDTERM 2
muscle at the level 2cm above the symphisis pubis beneath the rectus muscle is derived from the MD (TOP 1 - AUG 2013 EXAM - FEB
comes from transversalis MED BOARDS; 2014
a. aponeurosis of external oblique TOPNOTCH MD)
b. aponeurosis of internal oblique
c. transversus abdominis
d. transversalis

571 Strangulated hernias are differentiated from A bulge felt above the inguinal ligament BLAKE WARREN ANG, MIDTERM 2
incarcerated hernias by the following except: MD (TOP 1 - AUG 2013 EXAM - FEB
a. Pain out of proportion to examination findings MED BOARDS; 2014
b. Fever or toxic appearance TOPNOTCH MD)
c. Pain that persists after reduction of hernia
d. A bulge felt above the inguinal ligament

572 Modified radical neck dissection spares the MRND spares the IJA, SCM, and CN XI BLAKE WARREN ANG, MIDTERM 2
following structures except: MD (TOP 1 - AUG 2013 EXAM - FEB
a. Internal jugular vein MED BOARDS; 2014
b. Spinal accessory nerve TOPNOTCH MD)
c. Greater auricular nerve
d. Sternocleidomastoid muscle

573 A 17 year old sexually active male patient comes Answer: Any solid, firm mass within the testis BLAKE WARREN ANG, MIDTERM 2
to your clinic for a palpable nontender mass of the should be considered testicular cancer until MD (TOP 1 - AUG 2013 EXAM - FEB
right testicle, your primary impression is: proven otherwise. Prompt diagnosis and early MED BOARDS; 2014
a. testicular cancer treatment are required for cure. TOPNOTCH MD)
b. TB of the testis
c. secondary syphilis
d. Gonococcal epidydimitis

574 Anal fissure most often occurs: Answer: Most anal fissures occur in the BLAKE WARREN ANG, MIDTERM 2
a. anterior midline posterior midline, with the remainder occurring MD (TOP 1 - AUG 2013 EXAM - FEB
b. posterior midline in the anterior midline (99% of men, 90% of MED BOARDS; 2014
c. anterolateral women). Two percent of patients have anterior TOPNOTCH MD)
d. posterolateral and posterior fissures. Fissures occurring off the
midline should raise the possibility of other
etiologies (eg, Crohn disease), an infectious
etiology (eg, sexually transmitted disease,
acquired immunodeficiency syndrome or
cancer.
575 In estimating total body surface area (TBSA) BLAKE WARREN ANG, MIDTERM 2
involved, what depth or thickness of burn injuries MD (TOP 1 - AUG 2013 EXAM - FEB
should be considered? MED BOARDS; 2014
a. Full TOPNOTCH MD)
b. Full and superficial
c. Full and partial
d. Full, partial and superficial

576 If you suspect carbon monoxide poisoning in a establish airway via ET tube since the patient is BLAKE WARREN ANG, MIDTERM 2
semi-conscious patient, what is the best action to already semi to unconscious and unable to MD (TOP 1 - AUG 2013 EXAM - FEB
perform? protect his/her airway, subsequently, give 100% MED BOARDS; 2014
a. Bronchoscopy oxygen. TOPNOTCH MD)
b. Administer methylene blue
c. Intubate the patient and give 100% oxygen
d. Provide 100% supplemental oxygen via
facemask

577 Which of the following is the best radiographic BLAKE WARREN ANG, MIDTERM 2
view to check the fracture of the forearm? MD (TOP 1 - AUG 2013 EXAM - FEB
a. Forearm AP-Oblique MED BOARDS; 2014
b. Forearm AP-Lateral TOPNOTCH MD)
c. Forearm pronation-lateral
d. Forearm pronation-supination

TOPNOTCH MEDICAL BOARD PREP SURGERY SUPEREXAM Page 76 of 94


For inquiries visit www.topnotchboardprep.com.ph or email us at topnotchmedicalboardprep@gmail.com
TOPNOTCH MEDICAL BOARD PREP SURGERY SUPEREXAM
For inquiries visit www.topnotchboardprep.com.ph or email us at topnotchmedicalboardprep@gmail.com
Item QUESTION EXPLANATION AUTHOR TOPNOTCH
# EXAM
578 Compartment syndrome should always be ruled Pain, weakness, increased tenseness, BLAKE WARREN ANG, MIDTERM 2
out in any severe injuries of the forearm. Which of hypoesthesia MD (TOP 1 - AUG 2013 EXAM - FEB
the following is the typical sequence of symptoms? MED BOARDS; 2014
a. Pain, weakness, increased tenseness, TOPNOTCH MD)
hypoesthesia
b. Pain, hypoesthesia, weakness, increased
tenseness
c. Pain, increased tenseness, hypoesthesia,
weakness
d. Pain, weakness, hypoesthesia, increased
tenseness
579 A 50 year old female consulted at the OPD clinic Answer: Cosmesis. Colloid goiter has no BLAKE WARREN ANG, MIDTERM 2
due to an anterior neck mass. On physical malignancy potential, and does not usually cause MD (TOP 1 - AUG 2013 EXAM - FEB
examination, there was a 3 x 4 cm diffuse, firm, compression of underlying structures (unlike MED BOARDS; 2014
non-tender mass at the right neck which moves Ca). TOPNOTCH MD)
during swallowing. No cervical lymphadenopathy
was noted. TSH and FT4 were normal.
If the biopsy turned out to be colloid
adenomatous goiter (CAG), the most likely
indication for surgery is:
a. Cosmesis
b. Potential for malignancy
c. Prevention of recurrence
d. Compression of surrounding structures

580 Which of the following is an easy and reliable way BLAKE WARREN ANG, MIDTERM 2
to determine if the tympanic membrane is intact MD (TOP 1 - AUG 2013 EXAM - FEB
on otoscopy? MED BOARDS; 2014
a. Calorics testing TOPNOTCH MD)
b. Pneumatoscopy
c. Tuning fork test
d. Tympanometry

581 What is the normal weight of the thyroid gland? The normal thyroid has a weight of 20 g. TIMOTHY TANG LEE MIDTERM 1
A. 10 g SAY, MD (TOP 4 - AUG EXAM - FEB
B. 20 g 2013 MED BOARDS; 2014
C. 30 g TOPNOTCH MD)
D. 40 g
E. 50 g

582 Which interleukin is responsible for IL 2 is responsible for lyphocyte proliferation TIMOTHY TANG LEE MIDTERM 1
immunoglobulin production? and Ig production SAY, MD (TOP 4 - AUG EXAM - FEB
A. IL-1 2013 MED BOARDS; 2014
B. IL-2 TOPNOTCH MD)
C. IL-3
D. IL-6
E. IL-12

583 Total body water in a young adult female A young female has 50% of body weight since TIMOTHY TANG LEE MIDTERM 1
comprised how many percent of total body they have more fat per kg of body weight than SAY, MD (TOP 4 - AUG EXAM - FEB
weight? males. 2013 MED BOARDS; 2014
A. 50% TOPNOTCH MD)
B. 55%
C. 60%
D. 70%
E. 75%

584 What is the important test in determining a A directed bleeding history is the most TIMOTHY TANG LEE MIDTERM 1
patient perioperative risk for bleeding? appropriate test in determining risk of bleeding. SAY, MD (TOP 4 - AUG EXAM - FEB
A. Platelet count 2013 MED BOARDS; 2014
B. PT TOPNOTCH MD)
C. aPTT
D. Hemoglobin
E. History taking

585 Patient has been scheduled to undergo Class II is the classification for wounds that TIMOTHY TANG LEE MIDTERM 1
laparoscopic cholecystectomy. What is the surgical violate the integrity of the GI or GU tract under SAY, MD (TOP 4 - AUG EXAM - FEB
wound classification? controlled circumstances (antibiotic 2013 MED BOARDS; 2014
A. Class I prophylaxis). TOPNOTCH MD)
B. Class ID
C. Class II
D. Class III
E. Class IV

586 A 24 year old was immediately rush to the ER due The first step is evaluation of the ABCs and TIMOTHY TANG LEE MIDTERM 1
to a gunshot wound to the abdomen. On initial altered mental status is the most common SAY, MD (TOP 4 - AUG EXAM - FEB
examination, patient was noted to be drowsy with indication for intubation. 2013 MED BOARDS; 2014
pallor, blood continuously gush out from the TOPNOTCH MD)
wound. The next step in management is?
A. Insert large bore needles and start fluid
resuscitation
B. Control bleeding from the wound
C. Intubate the patient
D. Do diagnostic periteoneal lavage
E. Immediately wheel the patient to the OR
TOPNOTCH MEDICAL BOARD PREP SURGERY SUPEREXAM Page 77 of 94
For inquiries visit www.topnotchboardprep.com.ph or email us at topnotchmedicalboardprep@gmail.com
TOPNOTCH MEDICAL BOARD PREP SURGERY SUPEREXAM
For inquiries visit www.topnotchboardprep.com.ph or email us at topnotchmedicalboardprep@gmail.com
Item QUESTION EXPLANATION AUTHOR TOPNOTCH
# EXAM
587 A 30 year old woman suffered abdominal trauma The 2 paracolic gutters are not included in the TIMOTHY TANG LEE MIDTERM 1
after being assualted by a thief. In the ER, the evaluation. SAY, MD (TOP 4 - AUG EXAM - FEB
surgical resident decided to perform FAST. Which 2013 MED BOARDS; 2014
of the following is NOT a location evaluated by the TOPNOTCH MD)
procedure?
A. Subxiphoid
B. Morrison's pouch
C. Perisplenic recess
D. Paracolic gutter
E. Pelvis
588 How long before wounds usually achieve their Wounds never achieve their normal tensile TIMOTHY TANG LEE MIDTERM 1
normal (pre-injury) tensile strength? strength before injury. SAY, MD (TOP 4 - AUG EXAM - FEB
A. 4 weeks 2013 MED BOARDS; 2014
B. 8 weeks TOPNOTCH MD)
C. 6 months
D. 1 year
E. Never

589 Which of the following sutures is used for ligating For ligation of blood vessels, non absorbable TIMOTHY TANG LEE MIDTERM 1
bleeders? sutures like silk or cotton should be used. SAY, MD (TOP 4 - AUG EXAM - FEB
A. Silk 2013 MED BOARDS; 2014
B. Vicryl TOPNOTCH MD)
C. PDS
D. Monocryl
E. Chromic

590 A 50 year old farmer has a 10 year history of a 2x2 For SCC, therapy is 1 cm tumor free margins. For TIMOTHY TANG LEE MIDTERM 1
cm wound in the dorsum of his right foot. Biopsy high risk lesions such as chronic wounds, SAY, MD (TOP 4 - AUG EXAM - FEB
revealed nest of tumor cells with keratin pearls. prophylactic lymph node dissection should be 2013 MED BOARDS; 2014
Definitive management would include? done TOPNOTCH MD)
A. Below the knee amputation
B. Excision with 1 cm margin of normal tissue
and prophylactic inguinal lymph node dissection
C. Excision with 0.5-1 cm tumor free margins
plus radiation
D. Excision with 0.5-1 cm tumor free margins
with adjuvant radiation and chemothrapy
E. Excision with 1 cm margin of normal tissue
and radical lymph node dissection
591 A 29 year old real estate agent noticed a lump in BIRADS 2 is a benign finding so you just need to TIMOTHY TANG LEE MIDTERM 1
the upper outer quadrant of her right breast while reassure the patient with routine screening. SAY, MD (TOP 4 - AUG EXAM - FEB
taking a bath. You examined the patient and BIRADS 3 would need follow-up. 4-5 may need 2013 MED BOARDS; 2014
ordered an ultrasound of both breasts. Results to do biopsy. TOPNOTCH MD)
showed BIRADS 2. The patient is reluctant to
undergo surgery. What is the next step?
A. Do core-needle biopsy as well as
immunohistochemical stains (ER/PR/Her2neu)
B. Excise the lesion and send to histopath
C. Do local irradiation and chemotherapy
D. Observation with short interval follow-up
E. Reassure patient with routine screening
592 A 32 year old female complains of an enlarging Based on the algorhythm, no matter whether the TIMOTHY TANG LEE MIDTERM 1
anterior neck mass that moves with degluttition. patient is clinically euthyroid or not, a SAY, MD (TOP 4 - AUG EXAM - FEB
Patient is clinically euthyroid and has no other biochemical documentation is needed so one 2013 MED BOARDS; 2014
subjective complaints. Work-up revealed Hgb of must order TSH first. TOPNOTCH MD)
10.0 g/dL; WBC 5.5; Platelet 200. What is the next
step in the algorhythm?
A. Order TSH
B. Do FNAB
C. Do FNAB with Cervical UTZ
D. Order Thyroid Scan
E. Observe with close follow-up
593 On subsequent work-ups of this same patient, Age is the most important prognosticating factor TIMOTHY TANG LEE MIDTERM 1
biopsy revealed that it is follicular carcinoma in thyroid papillary and follicular carcinoma. SAY, MD (TOP 4 - AUG EXAM - FEB
measuring 4x4cm by ultrasound with metastasis Even with meatastasis, in a patient under 45 2013 MED BOARDS; 2014
noted to be present in both adrenals and lumbar years old, it is still Stage II. TOPNOTCH MD)
vertebra resulting to a poorer prognosis. What is
the Stage based on TNM?
A. Stage 0
B. Stage I
C. Stage II
D. Stage III
E. Stage IV
594 Which of the following is NOT a principle of anti- There should be placement of an adequate TIMOTHY TANG LEE MIDTERM 1
reflux surgery? length in the positive pressure environment of SAY, MD (TOP 4 - AUG EXAM - FEB
A. Repair can be placed in the abdomen without the abdomen. 2013 MED BOARDS; 2014
undue tension. TOPNOTCH MD)
B. The fundoplication should not increase the
resistance of the LES to a level greater than the
peristaltic power of the esophageal body.
C. Restore LES pressure to a level twice the
resting gastric pressure and its length to at least 3
cm.
D. Minimize placement of the LES in the positive
pressure environment of the abdomen.

TOPNOTCH MEDICAL BOARD PREP SURGERY SUPEREXAM Page 78 of 94


For inquiries visit www.topnotchboardprep.com.ph or email us at topnotchmedicalboardprep@gmail.com
TOPNOTCH MEDICAL BOARD PREP SURGERY SUPEREXAM
For inquiries visit www.topnotchboardprep.com.ph or email us at topnotchmedicalboardprep@gmail.com
Item QUESTION EXPLANATION AUTHOR TOPNOTCH
# EXAM
E. Allow the the reconstructed cardia to relax on
deglutition.

595 Appendectomy was done to a 20 year old college Lesions of 0.5 cm of carcinoid tumor, the TIMOTHY TANG LEE MIDTERM 1
student for which histopathology showed a 0.5 cm treatment is only appendectomy. SAY, MD (TOP 4 - AUG EXAM - FEB
focus of carcinoid tumor located in the base of the 2013 MED BOARDS; 2014
appendix with clear margins. As the attending TOPNOTCH MD)
surgeon, what is your next course of action?
A. Re-explore the patient for evaluation of stump.
B. Plan for an elective right hemicolectomy.
C. Do chemotherapy.
D. Order a abdominal CT scan and do a
metastatic work-up
E. Reassure patient.
596 Which of the following criteria is NOT included in Presence of ascites and nutritional status are the TIMOTHY TANG LEE MIDTERM 1
the Child-Pugh scoring of hepatic reserve? other criteria included and not serum AST, ALT. SAY, MD (TOP 4 - AUG EXAM - FEB
A. Hepatic encephalopathy 2013 MED BOARDS; 2014
B. Serum bilirubin TOPNOTCH MD)
C. Serum AST, ALT
D. Serum albumin
E. Prothrombin time

597 The presence of hernia are more common in males Indirect inguinal hernia still outnumbers TIMOTHY TANG LEE MIDTERM 1
than in females especially hernias associated with femoral hernia in incidence even in the female SAY, MD (TOP 4 - AUG EXAM - FEB
a patent processus vaginalis. What is the most population. 2013 MED BOARDS; 2014
common hernia in the female population? TOPNOTCH MD)
A. Indirect Inguinal hernia
B. Direct inguinal hernia
C. Femoral hernia
D. Obturator hernia
E. Spigelian hernia

598 A 50 year old smoker has a 2x2 cm erythematous For lesions with palpable lymph nodes, one TIMOTHY TANG LEE MIDTERM 1
lesion in the lateral border of the tongue with a should do a formal unilateral MRND and not a SAY, MD (TOP 4 - AUG EXAM - FEB
1x2 cm palpable lymph node in the right neck prophylactic supraomohyoid dissection. 2013 MED BOARDS; 2014
area. What is the definitive management? Excision is the same as parital glossectomy. TOPNOTCH MD)
A. Partial glossectomy with wide margins (>1
cm)
B. Excision plus supraomohyoid dissection in the
right
C. Partial glossectomy and mandibulectomy plus
unilateral supraomohyoid dissection
D. Excision plus unilateral modified radical neck
dissection
E. Excision plus bilateral modified radical neck
dissection
599 In the ER, a 70 year old post-TURP male patient Drain urine slowly to avoid hemorrhagic cystitis. TIMOTHY TANG LEE MIDTERM 1
complain of difficulty in urination. What is the SAY, MD (TOP 4 - AUG EXAM - FEB
initial mangement? 2013 MED BOARDS; 2014
A. Do suprapubic catheterization TOPNOTCH MD)
B. Insert a foley catheter and drain all urine
C. Insert a foley catheter and drain 500 cc every
hour
D. Give tamsulosin
E. Refer to urologist for decompression and DJ
stent insertion
600 In the immediate post-operative period after Increased IOP is common after vitrectomy. TIMOTHY TANG LEE MIDTERM 1
vitrectomy, which of the following findings is most SAY, MD (TOP 4 - AUG EXAM - FEB
commonly observed? 2013 MED BOARDS; 2014
A. Elevation of intraocular pressure TOPNOTCH MD)
B. Presence of subconjunctival hemorrhages
C. Acceleration of cataract development
D. Hyphema
E. Retinal detachment

601 During thyroidectomy, the surgeon accidentally The recurrent laryngeal nerve goes along the RACHELLE MENDOZA, FINAL EXAM -
ligated a nerve located along the right tracheoesophageal groove. Unilateral injury to MD (TOP 9 - AUG 2012 FEB 2013
tracheoesophageal groove. Post-operatively, the the RLN results to hoarseness. MED BOARDS;
patient is expected to experience: TOPNOTCH MD)
A. inability to reach high notes
B. hoarseness
C. stridor
D. voice fatigue
E. no symptom

TOPNOTCH MEDICAL BOARD PREP SURGERY SUPEREXAM Page 79 of 94


For inquiries visit www.topnotchboardprep.com.ph or email us at topnotchmedicalboardprep@gmail.com
TOPNOTCH MEDICAL BOARD PREP SURGERY SUPEREXAM
For inquiries visit www.topnotchboardprep.com.ph or email us at topnotchmedicalboardprep@gmail.com
Item QUESTION EXPLANATION AUTHOR TOPNOTCH
# EXAM
602 Pulmonary risk reduction in perioperative care is Pulmonary risk reduction is achieved through RACHELLE MENDOZA, FINAL EXAM -
achieved through the following, EXCEPT: smoking cessation, pre- and postoperative chest MD (TOP 9 - AUG 2012 FEB 2013
A. Smoking cessation physiotherapy, laparoscopic approach (when MED BOARDS;
B. Pre- and postoperative chest physiotherapy possible), and appropriate pain control. TOPNOTCH MD)
C. Laparoscopic approach (when possible) Additionally, obese patients (body mass index
D. Appropriate pain control [BMI] > 40 kg/m2) benefit from perioperative
E. None of the above CPAP and intraoperative use of ↑ positive end-
expiratory pressure (PEEP; 10 cm H2O)

603 An 18-year old male was hit by a wooden paddle In trauma involving blunt objects, fracture is RACHELLE MENDOZA, FINAL EXAM -
on his left eye during rowing practice in Pasig expected to occur in the area of least resistance. MD (TOP 9 - AUG 2012 FEB 2013
river. Blow out fracture will most likely be noted In the case of the orbit, it is the floor which MED BOARDS;
in: offers least resistance. TOPNOTCH MD)
A. lamina papyracea
B. lacrimal bone
C. floor of the orbit
D. zygomatic bone
E. frontal bone
604 Auscultation of a 25-year old female patient The patient has mitral valve stenosis most RACHELLE MENDOZA, FINAL EXAM -
revealed mid-diastolic murmur with opening click probably due to RHD. The management MD (TOP 9 - AUG 2012 FEB 2013
at the apical area, grade 3/5. The patient has been depends upon the severity of valve damage. MED BOARDS;
experiencing worsening shortness of breath, Percutaneous valvuloplasty is the most TOPNOTCH MD)
orthopnea and occasional constricting chest pain appropriate initial management. If this fails, or
for 6 months. She had a history of repeated if the valve is too damages, valve replacement
episodes of untreated pharyngitis in the past. should be done.
What is the most appropriate management for this
patient?
A. Percutaneous mitral valvuloplasty
B. Valve replacement
C. Penicillin G monthly for life
D. VSD closure
E. Observe
605 Following an MI, a 65-year old male patient had an The patient is described to be having an acute RACHELLE MENDOZA, FINAL EXAM -
acute heart failure, with an associated acute mitral regurgitation (as characterized by the MD (TOP 9 - AUG 2012 FEB 2013
occurrence of holosystolic blowing murmur at the murmur). The most common cause of new MED BOARDS;
apical area. What is the mechanism? onset MR in post-MI patient is rupture of mitral TOPNOTCH MD)
A. Reinfarction, compromising left circumflex valve papillary muscle due to necrosis.
coronary artery
B. Ventricular wall rupture
C. Ruptured mitral valve papillary muscle
D. Ventricular septal wall rupture
E. Pericarditis
606 A 53-year-old woman has a right-sided lung mass Mediastinoscopy and lymph node biopsy should RACHELLE MENDOZA, FINAL EXAM -
noted on CXR. She is a smoker with no cancer be done to determine presence of metastatic MD (TOP 9 - AUG 2012 FEB 2013
history. Chest CT confirms an irregular 4-cm mass lesions from other organs. MED BOARDS;
and a right paratracheal node measuring 1.9 cm. TOPNOTCH MD)
Percutaneous biopsy of the lung mass confirms a
bronchogenic adenocarcinoma. What must be
done before choosing further treatment?
A. PET scan
B. MediastinoscopyLymph node biopsy
C. Neo-adjuvant radiotherapy
D. MRI angiography
E. No further diagnostics needed

607 A 1 1/2 month-old baby girl was brought to your Laryngomalacia is the most common cause of RACHELLE MENDOZA, FINAL EXAM -
clinic because of stridor. There was no associated stridor among infants. The stridor is releived by MD (TOP 9 - AUG 2012 FEB 2013
cough. The patient was delivered to a G9P8 37 having the baby lie on his/her tummy. MED BOARDS;
year-old woman via NSD. Birthweight was 2.8kg TOPNOTCH MD)
with APGAR score of 8 and 9. She was sent home
after 2 days with no perinatal complications. She
had a good suck with normal swallowing, good cry
and weight gain. Stridor is slightly relieved by
putting her in a prone position. T=37.2 C, RR=
40/min. Your initial impression is:
A. Congenital subglottic stenosis
B. Congenital laryngeal web
C. Laryngocoele
D. Laryngomalacia
E. Congenital cyst
608 Epidural hematoma is characterized by: Epidural hematoma involves tear in middle RACHELLE MENDOZA, FINAL EXAM -
A. Convex hyperdensity in CT meningeal artery. CT will show lens-shaped MD (TOP 9 - AUG 2012 FEB 2013
B. May cross midline but not suture lines (convex) hyperdensity. May cross midline but MED BOARDS;
C. Lucid interval in 20% of cases not suture lines. Lucid interval is noted to some TOPNOTCH MD)
D. All are correct patients. Management is emergent craniotomy
E. A and C only and evacuation of hematoma.

TOPNOTCH MEDICAL BOARD PREP SURGERY SUPEREXAM Page 80 of 94


For inquiries visit www.topnotchboardprep.com.ph or email us at topnotchmedicalboardprep@gmail.com
TOPNOTCH MEDICAL BOARD PREP SURGERY SUPEREXAM
For inquiries visit www.topnotchboardprep.com.ph or email us at topnotchmedicalboardprep@gmail.com
Item QUESTION EXPLANATION AUTHOR TOPNOTCH
# EXAM
609 A 25-year old male (BW = 55kg) wasimmediately Using the rule of 9s, the following is the RACHELLE MENDOZA, FINAL EXAM -
brought to the ER just 10 minutes after sustaining equivalent computation of the TBSA affected: MD (TOP 9 - AUG 2012 FEB 2013
full thickness burns in several parts of his body. anterior surfaces of both arms - 9, ant and post MED BOARDS;
Affected areas include anterior surfaces of thigh - 9, ant and post leg - 9, genitals 1. Using TOPNOTCH MD)
bilateral arms and forearms, anterior and the parkland formula, the following: 4 ml/kg x
posterior right thigh, anterior and posterior right 55 kg x 28 = 6, 160 mL for the first 8 hours, or
leg and his genitals. Using the Parkland formula, 770ml/hr for 8 hours.
what should be the initial fluid resuscitation for
this patient?
A. 742 ml/hr for the first 8 hours
B. 1017 ml/hr for the first 8 hours
C. 770 ml/hr for the first 8 hours
D. 990 ml/hr for the first 8 hours
E. 687 ml/hr for the first 8 hours
610 After an uncomplicated abdominal surgery Following most abdominal operations or RACHELLE MENDOZA, FINAL EXAM -
(appendectomy), the 35-year old patient is still on injuries, the motility of the gastrointestinal tract MD (TOP 9 - AUG 2012 FEB 2013
NPO during his 5th hospital stay due to inability to is transiently impaired. Among the proposed MED BOARDS;
tolerate feeding. After how many hours/days mechanisms responsible for this dysmotility are TOPNOTCH MD)
should normal intestinal motility be observed in surgical stress-induced sympathetic reflexes,
its earliest post-operatively? inflammatory response-mediator release, and
A. 12 hours anesthetic/analgesic effects; each of which can
B. 24 hours inhibit intestinal motility. The return of normal
C. 48 hours motility generally follows a characteristic
D. 72 hours temporal sequence, with small-intestinal
E. 4 days motility returning to normal within the first 24
hours after laparotomy and gastric and colonic
motility returning to normal by 48 hours and 3
to 5 days, respectively. Resolution of ileus may
be delayed in the presence of other factors
capable of inciting ileus such as the presence of
intra-abdominal abscesses or electrolyte
abnormalities.
611 A 70 y/o male was seen in the ER because of Board like abdomen with a history of chronic RACHELLE MENDOZA, FINAL EXAM -
severe abdominal pain of more than 24 hrs. intake of NSAIDs indicate that the patient may MD (TOP 9 - AUG 2012 FEB 2013
History revealed chronic intake of NSAID due to be having perforated peptic ulcer. Since the MED BOARDS;
osteoarthritis. He is tachycardic, tachypneic, patient has a poor risk for surgery (70 years old, TOPNOTCH MD)
febrile with a BP of 80/50. PE revealed board-like hypotensive), the most appropriate surgical
rigidity of the abdomen. What is the most procedure is just graham patching.
appropriate treatment for this patient?
A. Graham patching
B. Vagotomy with antrectomy
C. Vagotomy
D. Partial gastrectomy
E. A and B
612 A 45-year old female was suffering from GERD for Fundoplication is the preferred surgical RACHELLE MENDOZA, FINAL EXAM -
10 years now, unrelieved by any medical treatment for GERD. MD (TOP 9 - AUG 2012 FEB 2013
treatment. What is the most appropriate option MED BOARDS;
for this patient? TOPNOTCH MD)
A. Kocher's maneuver
B. Gastric pull-up
C. Fundoplication
D. Esophagectomy with gastric pull-up
E. Vagotomy
613 A TRUE statement regarding chemical burns of the Alkali burns cause more extensive damage than RACHELLE MENDOZA, FINAL EXAM -
eye: acid burns. Neutralization should never be MD (TOP 9 - AUG 2012 FEB 2013
A. Acid burns cause more extensive damage than attempted. Copious irrigation of the eye even MED BOARDS;
alkali burns with a nonsterile water should be the first thing TOPNOTCH MD)
B. The first thing to do in cases of chemical burns to be done.
is to check the visual acuity
C. Neutralization should be achieved, so that an
alkali burn can be treated by instilling an acidic
substance and vice versa
D. In copious irrigation of the eye, the fluid used
need not be sterile, provided the chemical is
diluted properly
614 The most common pediatric malignant lesion in Hepatoblastoma is the most common liver RACHELLE MENDOZA, FINAL EXAM -
the liver is: tumor among pediatric patients. MD (TOP 9 - AUG 2012 FEB 2013
A. Hepatocarcinoma MED BOARDS;
B. Malignant teratoma TOPNOTCH MD)
C. Hepatoblastoma
D. Metastatic lesion from neuroblastoma
E. Hemangioma

615 Which of the following statement is TRUE Vessel constriction is due to release of local RACHELLE MENDOZA, FINAL EXAM -
regarding normal hemostasis and platelet endothelial factors such as adenosine as a result MD (TOP 9 - AUG 2012 FEB 2013
function? of endothelial damage. The vascular endothelial MED BOARDS;
A. Vascular disruption is followed by vessel wall is normally remained smooth so as not to TOPNOTCH MD)
constriction mediated by vasoactive substances promote platelet adhesion and clot formation.
released by platelets. Platelet adhesion occurs via von willebrand
B. Platelets adhesion depends on the preformation factors that are exposed on damaged
of fibrin monomers. endothelium. Platelet plug formation occurs
C. The endothelial surface supports platelet first before formation of fibrin monomers.
adhesion and thrombus formation fibrinogen is needed in promoting platelet
D. A prolonged bleeding time may be due to aggregation.
thrombocytopenia or reduced amounts of von
Willebrand factor

TOPNOTCH MEDICAL BOARD PREP SURGERY SUPEREXAM Page 81 of 94


For inquiries visit www.topnotchboardprep.com.ph or email us at topnotchmedicalboardprep@gmail.com
TOPNOTCH MEDICAL BOARD PREP SURGERY SUPEREXAM
For inquiries visit www.topnotchboardprep.com.ph or email us at topnotchmedicalboardprep@gmail.com
Item QUESTION EXPLANATION AUTHOR TOPNOTCH
# EXAM
E. None of the above

616 A 42-year old female underwent cholecystectomy. Cause of fever within 24 hours post-op is most RACHELLE MENDOZA, FINAL EXAM -
Sixteen hours after the operation, she developed likely due to atelectasis (wind). Followed by UTI MD (TOP 9 - AUG 2012 FEB 2013
fever. The most probable cause is: (water), wound infection (wound), DVT (walk). MED BOARDS;
A. Pneumonia TOPNOTCH MD)
B. UTI
C. Atelectasis
D. Wound infection
E. Thrombophlebitis

617 A 42-year old female sought consult because of a The most common odontogenic tumor is RACHELLE MENDOZA, FINAL EXAM -
slow-growing, painless swelling in her right ameloblastoma, presenting as multiloculated MD (TOP 9 - AUG 2012 FEB 2013
mandible. X-ray showed a multiloculated mass usually located in the body of the MED BOARDS;
radiolucency in the body of the mandible with an mandible. TOPNOTCH MD)
impacted tooth clearly visualized. This condition
is suggestive of:
A. Torus mandibularis
B. Ameloblastoma
C. Gorlin’s cyst
D. Osteosarcoma
E. None of the above
618 TRUE statement/s about gallstone disease: Asymptomatic patients with incidental finding RACHELLE MENDOZA, FINAL EXAM -
A. All asymptomatics gallstones requires of a gallstone will not require surgery unless MD (TOP 9 - AUG 2012 FEB 2013
cholecystectomy to prevent complications porcelain gallbladder was also observed. MED BOARDS;
B. Acute cholecystitis is associated with cystic duct Gallstone pancreatitis is caused by obstruction TOPNOTCH MD)
obstruction usually due to gallstone impaction in of a stone at the ampulla of vater, allowing
the cystic duct or infundibulum backflow of pancreatic secretions.
C. Gallstone pancreatitis is caused by stones Choelcystolithiasis is far more common than
obstruction the second portion of the duodenum choledocholithiasis.
D. Choledocholithiasis occur more commonly than
cholecystolithiasis
E. All of the above
619 Relative contraindications to lung surgery due to Perform preoperative risk stratification for RACHELLE MENDOZA, FINAL EXAM -
high morbidity include the following, EXCEPT: surgical candidates. Relative MD (TOP 9 - AUG 2012 FEB 2013
A. Preoperative FEV1 < 0.8 L contraindications to lung surgery due to high MED BOARDS;
B. Predicted postoperative FEV1 < 0.8 L morbidity include preoperative FEV1 < 0.8 L, TOPNOTCH MD)
C. PaCO2 > 45 mm Hg predicted postoperative FEV1 < 0.8 L, PaCO2 >
D. PaO2 < 50 mm Hg 45 mm Hg, PaO2 < 50 mm Hg, and maximum
E. Maximum voluntary ventilation > 50% voluntary ventilation < 50%.

620 A 33-year old female was diagnosed with Lateral (humeral) axillary lymph node drains RACHELLE MENDOZA, FINAL EXAM -
carbuncle on the volar surface of the right the arm, posterior (scapular) group drains the MD (TOP 9 - AUG 2012 FEB 2013
forearm. Which of the following group of axillary upper back, anterior (pectoral) drains the MED BOARDS;
lymph nodes will first drain the lesion? breast. The central group of nodes receives TOPNOTCH MD)
A. Anterior lymph from these three groups. There is no
B. Posterior medial group of axillary lymph nodes.
C. Lateral
D. Central
E. Medial

621 The appropriate duration of antibiotic therapy for 3-5 days for perforated or gangrenous AP, 5-7 VON ANDRE MEDINA, DIAGNOSTIC
most patients with bacterial peritonitis from days for treatment for peritoneal soilage due to MD (TOP 4 - FEB 2012 EXAM - AUG
perforated appendicitis is: perforated viscus with moderate degrees of MED BOARDS; 2012
A. 3-5 days contamination, 7-14 days extensive peritoneal TOPNOTCH MD)
B. 7-10 days soilage or occuring in immunosuppressed host
C. 14-21 days
D. >21 days
E. 24 hrs

622 Which of the following is NOT a risk factor for VON ANDRE MEDINA, DIAGNOSTIC
developing a surgical site infection? MD (TOP 4 - FEB 2012 EXAM - AUG
A. Radiation exposure MED BOARDS; 2012
B. Recent surgery TOPNOTCH MD)
C. Prolonged hospitalization
D. Malnutrition
E. Infancy

TOPNOTCH MEDICAL BOARD PREP SURGERY SUPEREXAM Page 82 of 94


For inquiries visit www.topnotchboardprep.com.ph or email us at topnotchmedicalboardprep@gmail.com
TOPNOTCH MEDICAL BOARD PREP SURGERY SUPEREXAM
For inquiries visit www.topnotchboardprep.com.ph or email us at topnotchmedicalboardprep@gmail.com
Item QUESTION EXPLANATION AUTHOR TOPNOTCH
# EXAM
623 A 20 year old young man presents with a left An RBC count of > 10,000/uL is considered VON ANDRE MEDINA, DIAGNOSTIC
anterior 8th intercostal space stab wound. He is in positive finding and an indication for MD (TOP 4 - FEB 2012 EXAM - AUG
no distress and a chest xray is normal. A diagnostic laparotomy; patients with DPL RBC count MED BOARDS; 2012
peritoneal lavage is performed and has a RBC between 1000/uL and 10,000/uL should TOPNOTCH MD)
count of 8,000/uL and a WBC count of 300/uL. undergo laparoscopy or thoracoscopy (See
Which of the following is the best treatment for schwartz 9th edition, pp 153-155)
this patient?
A. Observation only
B. CT Scan
C. Laparoscopy or Thoracoscopy
D. Exploratory laparotomy
E. Any of the above choices
624 Which of the following patients should be All patients with a partial thickness burns of 10 VON ANDRE MEDINA, DIAGNOSTIC
immediately reffered to a burn center? % and above should be transferred to a burn MD (TOP 4 - FEB 2012 EXAM - AUG
A. A 20 year old with 12 % partial thickness burn center. A patient with a burn and another major MED BOARDS; 2012
B. A 30 year old with major liver injury and a 15 TRAUMA can be treated and stabilized in the TOPNOTCH MD)
% partial thickness burn trauma center first.
C. A 2 % TBSA partial thickness burn to the
anterior leg crossing the knee
D. A 10 year old with a 7% partial thickness burn
E.
625 The ideal time to administer prophylactic Antibiotic prophylaxis is most effective when VON ANDRE MEDINA, DIAGNOSTIC
antibiotics to a patient undergoing a colon adequate concentrations of antibiotic are MD (TOP 4 - FEB 2012 EXAM - AUG
resection is: present in the tissues at the time of incision and MED BOARDS; 2012
A. 8 hours before surgery with a dose repeated at assurance of adequate preoperative antibiotic TOPNOTCH MD)
the time of incision dosing, and timing has become a significant
B. 2 hours prior to surgery with a dose repeated hospital performance measure. Addition of
at the time of incision antibiotics after operative contamination has
C. 1 hour before surgery occured is clearly ineffective in preventing
D. at the time of incision postoperative wound infections. (Schwartz 9th
E. 30 minutes after the incision ed, pp 222)
626 Malignant cells are LEAST likely to be in which of malignant cells are cells that do not enter the Go VON ANDRE MEDINA, DIAGNOSTIC
the following stages of the cell cycle? stage after proliferation. MD (TOP 4 - FEB 2012 EXAM - AUG
A. S phase MED BOARDS; 2012
B. Go phase TOPNOTCH MD)
C. G1 phase
D. M phase
E. Any of the above

627 A 35 year old woman with a BRAC1 gene mutation VON ANDRE MEDINA, DIAGNOSTIC
seeks your advice about her known increased risk MD (TOP 4 - FEB 2012 EXAM - AUG
of breast cancer. You should recommend? MED BOARDS; 2012
A. mammograms and physical examination every TOPNOTCH MD)
6 months until she is 50, then bilateral
prophylactic mastectomy
B. mammograms and physical examination
every 6 months + tamoxifen
C. prophylactic bilateral mastectomy and, if she
has completed childbearing , prophylactic bilateral
oophorectomy
D. None of the above
628 The most common causative organism in swimmer's ear- Mc is Pseudomonas aeruginosa VON ANDRE MEDINA, DIAGNOSTIC
malignant otitis externa is? MD (TOP 4 - FEB 2012 EXAM - AUG
A. Staphylococcus aureus MED BOARDS; 2012
B. Pseudomonas aeruginosa TOPNOTCH MD)
C. Streptococcus pneumonia
D. Herpes simplex type 1
E. Proteus Vulgaris

629 Which of the following is an indication for Tonsillectomy and adenoidectomy are indicated VON ANDRE MEDINA, DIAGNOSTIC
tonsillectomy in children? for chronic or recurrent acute infection and for MD (TOP 4 - FEB 2012 EXAM - AUG
A. > 5 infections obstructive hypertrophy. The american academy MED BOARDS; 2012
B. > 3 infections with strong family history of otlaryngology -head and neck surgery TOPNOTCH MD)
C. > 3 infections in one year suggests tonsillectomy after three or more
D. > 1 week missed from school in a year due to infections per year despite adequate medical
tonsillar infections therapy.
E. Any of the above
630 A 65 year old who has smoked 2 packs a day for Transthoracic FNAB can accurately identify the VON ANDRE MEDINA, DIAGNOSTIC
45 years is found to have a 2cm solitary nodule 1 status of the peripheral pulmonary lesions I up MD (TOP 4 - FEB 2012 EXAM - AUG
cm from the surface of the superior segment of the to 95 % of cases. It is the best initial diagnostic MED BOARDS; 2012
right lower lobe. The best initial diagnostic procedure. (Schwartz 9th ed pp 527-528) TOPNOTCH MD)
procedure is?
A. Observation with biopsy if this increases size
over 3-6 months
B. Bronchoscopy
C. Transthoracic Fine needle aspiration
D. Open thoracotomy for excisional biopsy
E. Any of the above choices

TOPNOTCH MEDICAL BOARD PREP SURGERY SUPEREXAM Page 83 of 94


For inquiries visit www.topnotchboardprep.com.ph or email us at topnotchmedicalboardprep@gmail.com
TOPNOTCH MEDICAL BOARD PREP SURGERY SUPEREXAM
For inquiries visit www.topnotchboardprep.com.ph or email us at topnotchmedicalboardprep@gmail.com
Item QUESTION EXPLANATION AUTHOR TOPNOTCH
# EXAM
631 The components that contribute to the function of VON ANDRE MEDINA, DIAGNOSTIC
Lower esophageal sphincter (LES) include all of MD (TOP 4 - FEB 2012 EXAM - AUG
the following EXCEPT. MED BOARDS; 2012
A. Length of intra-abdominal esophagus TOPNOTCH MD)
B. Width of the diphragmatic hiatus
C. Resting pressure in the lower esophageal
muscle
D. length of the area of increased tone in the
lower esophageal muscle.
E. None of the above
632 Which of the following preocedures for peptic VON ANDRE MEDINA, DIAGNOSTIC
ulcer disease has the highest incidence of post- MD (TOP 4 - FEB 2012 EXAM - AUG
operative diarrhea? MED BOARDS; 2012
A. Graham patch TOPNOTCH MD)
B. Parietal cell vagotomy
C. Truncal vagotomy and pyloroplasty
D. Distal gastrectomy without vagotomy
E. Nissen Fundoplication

633 The most common premalignant lesion in the VON ANDRE MEDINA, DIAGNOSTIC
stomach is? MD (TOP 4 - FEB 2012 EXAM - AUG
A. Adenoma MED BOARDS; 2012
B. Chronic gastric ulcer TOPNOTCH MD)
C. Atrophic gastritis
D. Verrucous gastritis
E. APUD

634 A patient with an internal hemorrhoid that 1st degree hemorrhoid bulge into the anal canal VON ANDRE MEDINA, DIAGNOSTIC
prolapses past the dentate line with straining is a ? and may prolapse beyond dentate line on MD (TOP 4 - FEB 2012 EXAM - AUG
A. 1st degree hemorrhoid straining. MED BOARDS; 2012
B. 2nd degree hemorrhoid TOPNOTCH MD)
C. 3rd degree hemorrhoid
D. 4th degree hemorrhoid
E. None of the above

635 A patient with a 1.5 cm carcinoid tumor of the mid because this tumor < 2 cm in size in the mid VON ANDRE MEDINA, DIAGNOSTIC
appendix should undergo? appendix, an appendectomy is an adequate MD (TOP 4 - FEB 2012 EXAM - AUG
A. Appendectomy only treatment. MED BOARDS; 2012
B. Partial cecectomy and lymph node sampling to TOPNOTCH MD)
confirm negative margins
C. Resection of the cecum, terminal ileum, and
adjacent mesentery (en bloc resection)
D. Right hemicolectomy
E. Observation only
636 Standard therapy for a 4-cm right lobe hepatic hepatic adenomas carry a significnt risk of VON ANDRE MEDINA, DIAGNOSTIC
adenoma is? spontaneous rupture with intraperitonal MD (TOP 4 - FEB 2012 EXAM - AUG
A. Observation only bleeding. It has also a risk of malignant MED BOARDS; 2012
B. Arterial embolism to prevent further growth transformation to a well differentiated HCC. TOPNOTCH MD)
C. Laparascopic ablation Therefore, the usual recommendation is surgical
D. Surgical resection resection.
E. Liver transplantation

637 Which of the following is an indication for porcelain gallbladder, a rare pre malignant VON ANDRE MEDINA, DIAGNOSTIC
cholecystectomy in an asymptomatic patient with condition in which the wall of the gallbladder MD (TOP 4 - FEB 2012 EXAM - AUG
an incidental finding of gallstones? becomes calcified, is an absolute indication for MED BOARDS; 2012
A. Any history of abdominal pain cholecystectomy. TOPNOTCH MD)
B. Family history of complications of
cholelthiasis
C. Porcelain gallbladder
D. Frequent travel out of the country
E. Any of the above choices
638 A patient presents with biliary colic. On UTZ there Lap chole and IOC VON ANDRE MEDINA, DIAGNOSTIC
are multiple small gallstones in the gallbladder MD (TOP 4 - FEB 2012 EXAM - AUG
and the common bile duct measures 9 mm in MED BOARDS; 2012
diameter. No stone is visualized in the common TOPNOTCH MD)
bile duct. Which of the following is the most
reasonable next step?
A. Repeat UTZ in 24-48 hours
B. MRCP with contrast
C. Percutaneous cholangiography
D. Laparoscopic cholecystectomy and
intraoperative cholangiography
E. Observation only
639 The most common type of pancreatic cancer is? VON ANDRE MEDINA, DIAGNOSTIC
A. Ductal adenocarcinoma MD (TOP 4 - FEB 2012 EXAM - AUG
B. Adenosquamous carcinoma MED BOARDS; 2012
C. Acinar cell carcinoma TOPNOTCH MD)
D. Squamous cell carcinoma
E. None of the above

TOPNOTCH MEDICAL BOARD PREP SURGERY SUPEREXAM Page 84 of 94


For inquiries visit www.topnotchboardprep.com.ph or email us at topnotchmedicalboardprep@gmail.com
TOPNOTCH MEDICAL BOARD PREP SURGERY SUPEREXAM
For inquiries visit www.topnotchboardprep.com.ph or email us at topnotchmedicalboardprep@gmail.com
Item QUESTION EXPLANATION AUTHOR TOPNOTCH
# EXAM
640 The most common location of the superior about 85 % of individual have 4 parathyroid VON ANDRE MEDINA, DIAGNOSTIC
parathyroid gland is: glands that can be found within 1 cm of the MD (TOP 4 - FEB 2012 EXAM - AUG
A. Dorsal to the recurrent laryngeal nerve (RLN), junction of the inferior thyroid artery and the MED BOARDS; 2012
within 1 cm of the junction of the RLN and inferior RLN. TOPNOTCH MD)
thyroid artery
B. Ventral to the recurrent laryngeal nerve,within
1 cm of the junction of the RLN and inferior
thyroid artery
C. Dorsal to the RLN, within 3 cm of the junction
of the RLN and inferior thyroid artery
D. Ventral to the RLN, within 3 cm of the junction
of the RLN and inferior thyroid artery
E. None of the above
641 The inferior parathyroid glands are derivatives of? the Branchial pouch derivatives: 1. Ear, 2. LITO JAY MACARAIG, DIAGNOSTIC
A. 1st Branchial pouch Thymus, 3. Inf. PTG and Thyroid Gland, 4. Sup. MD (TOP 8 - FEB 2013 EXAM - AUG
B. 2nd Branchial pouch PTG MED BOARDS; 2013
C. 2nd Pharyngeal pouch TOPNOTCH MD)
D. 3rd Branchial pouch
E. 4th Branchial pouch

642 A 34 year-old man was brought to ER due to A central fever will never improve significantly LITO JAY MACARAIG, DIAGNOSTIC
motorcycle accident. Patient suffered from with the use of ordinary anti-pyretics. A MD (TOP 8 - FEB 2013 EXAM - AUG
Cerebral hematoma and undergone surgery. On centrally-acting drug like Clonidine should be MED BOARDS; 2013
his 18th hour at the ICU, patient re-bleed and had used. TOPNOTCH MD)
central fever. You are considering medical
management before re-opening his cranium. What
should you give for the fever?
A. Ibuprofen
B. Paracetamol
C. Acetaminophen
D. Clonidine
E. Naproxen
643 You are about to remove the transverse colon due LITO JAY MACARAIG, DIAGNOSTIC
to an obstructing tumor. In order to prevent MD (TOP 8 - FEB 2013 EXAM - AUG
massive bloodloss, you should ligate the MED BOARDS; 2013
A. Ileo-colic artery TOPNOTCH MD)
B. Middle colic artery
C. Ascending left colic artery
D. B and C only
E. All of the above

644 High pressure in the hypopharynx may result to this is Zenker's Diverticula, the MC esophageal LITO JAY MACARAIG, DIAGNOSTIC
development of esophageal diverticula. Most diverticula and is commonly found in the MD (TOP 8 - FEB 2013 EXAM - AUG
commonly, diverticulas are found Killian's triangle at the level of cricopharyngeus MED BOARDS; 2013
A. In the Killian's Triangle TOPNOTCH MD)
B. In the Laimer's Triangle
C. At the level of Cricopharyngeus
D. A and C
E. B and C

645 Failure of migration of neural crest cells may All choices given can be due to failure of LITO JAY MACARAIG, DIAGNOSTIC
result to migration of neural crest cells MD (TOP 8 - FEB 2013 EXAM - AUG
A. Hirschprung's disease MED BOARDS; 2013
B. Albinism TOPNOTCH MD)
C. Achalasia
D. A and C only
E. All of the above

646 A 24 year-old male patient came in due to right this is a typical case of a Carcinoid tumor. And if LITO JAY MACARAIG, DIAGNOSTIC
lower abdominal pain associated with lowgrade its size is >2cms, you should do a MD (TOP 8 - FEB 2013 EXAM - AUG
fever. (+) Rovsing's sign and leukocytosis was seen hemicolectomy, regardless of the location MED BOARDS; 2013
in CBC. Upon opening the patient's abdomen, (whether at the tip or base of the appendix) TOPNOTCH MD)
appendix is not inflamed. Rather, you saw a small,
firm, well-circumscribed yellow-brown tumor at
at the tip of the appendix, measuring only 2.1cms.
What is the best next step?
A. remove the tumor and send for histopath
exam
B. remove the whole appendix
C. do a right hemicolectomy
D. leave the tumor and close the incision
E. none of the above
647 Which of the following stetements is/are true LITO JAY MACARAIG, DIAGNOSTIC
regarding Gastrointestinal Stromal Tumor (GIST)? MD (TOP 8 - FEB 2013 EXAM - AUG
A. Arise from Interstitial cells of Cajal MED BOARDS; 2013
B. Mesenchymal in origin TOPNOTCH MD)
C. Prognosis depends on size and mitotic count
D. A and C only
E. All of the above

TOPNOTCH MEDICAL BOARD PREP SURGERY SUPEREXAM Page 85 of 94


For inquiries visit www.topnotchboardprep.com.ph or email us at topnotchmedicalboardprep@gmail.com
TOPNOTCH MEDICAL BOARD PREP SURGERY SUPEREXAM
For inquiries visit www.topnotchboardprep.com.ph or email us at topnotchmedicalboardprep@gmail.com
Item QUESTION EXPLANATION AUTHOR TOPNOTCH
# EXAM
648 All of the following clotting factors are synthesized factor VIII is synthesized by active endothelial LITO JAY MACARAIG, DIAGNOSTIC
in the liver, EXCEPT? cells MD (TOP 8 - FEB 2013 EXAM - AUG
A. Factor II MED BOARDS; 2013
B. Factor VII TOPNOTCH MD)
C. Factor VIII
D. Factor IX
E. Factor X

649 Entamoeba histolytica is the most common cause LITO JAY MACARAIG, DIAGNOSTIC
of amoebic liver abscess. And it is usually located MD (TOP 8 - FEB 2013 EXAM - AUG
at the MED BOARDS; 2013
A. Superior-anterior aspect of the Right lobe of TOPNOTCH MD)
the liver
B. Superior-anterior aspect of the Left lobe of the
liver
C. Inferior-anterior aspect of the Right lobe of the
liver
D. Inferior-anterior aspect of the Left lobe of the
liver
E. Posterior lobe of the liver
650 A 32 year old patient was brought to ER due to LITO JAY MACARAIG, DIAGNOSTIC
severe epigastric pain. Patient came from a party MD (TOP 8 - FEB 2013 EXAM - AUG
and ate chicharon bulakalak, pork barbeque, and MED BOARDS; 2013
consumed 9 bottles of beer. Upon PE, you saw TOPNOTCH MD)
ecchymosis along the patient's flank. The lesion is
called
A. Santorini's sign
B. Grey Turner's sign
C. Cullen's sign
D. Rovsing's sign
E. Ranson's sign
651 Mondor's disease is a variant of thrombophlebitis it is the thoracoepgastric vein and NOT the left LITO JAY MACARAIG, DIAGNOSTIC
in the chest involving the gastric vein that is included MD (TOP 8 - FEB 2013 EXAM - AUG
A. Lateral thoracic vein MED BOARDS; 2013
B. Left gastric vein TOPNOTCH MD)
C. Superficial epigastric vein
D. A and C only
E. All of the above

652 Batson's plexus is a venous drainage system that is LITO JAY MACARAIG, DIAGNOSTIC
a route for metastasis of MD (TOP 8 - FEB 2013 EXAM - AUG
A. Thyroid CA MED BOARDS; 2013
B. Breast CA TOPNOTCH MD)
C. Prostate CA
D. B and C only
E. All of the above

653 Mucus retention cyst is just accumulation of LITO JAY MACARAIG, DIAGNOSTIC
submucosal mucus. It is best treated with MD (TOP 8 - FEB 2013 EXAM - AUG
A. Incision and drainage MED BOARDS; 2013
B. marsupialization TOPNOTCH MD)
C. Wedge excision
D. curettage
E. Steroids

654 Which of the following statements is/are true LITO JAY MACARAIG, DIAGNOSTIC
regarding Juvenile Nasopharyngeal Angiofibroma? MD (TOP 8 - FEB 2013 EXAM - AUG
A. Highly expansile and destructive MED BOARDS; 2013
B. Typically occurs in adolescent males TOPNOTCH MD)
C. Angioembolization is the treatment of choice
D. B and C only
E. All of the above

655 The cervical lymph nodes are divided to 6 LITO JAY MACARAIG, DIAGNOSTIC
different levels. Level II is further subdivided into MD (TOP 8 - FEB 2013 EXAM - AUG
IIa and IIb by the MED BOARDS; 2013
A. Spinal accessory nerve TOPNOTCH MD)
B. Anterior belly of omohyoid
C. Posterior belly of omohyoid
D. A and B
E. A and C

656 Most salivary gland tumors are benign. However, 25% Parotid, 50% submandibular, 75% LITO JAY MACARAIG, DIAGNOSTIC
in terms of incidence of malignancy , it is more Sublingual MD (TOP 8 - FEB 2013 EXAM - AUG
common in the MED BOARDS; 2013
A. Parotid gland TOPNOTCH MD)
B. Submandibular gland
C. Sublingual gland
D. A and B
E. A and C

TOPNOTCH MEDICAL BOARD PREP SURGERY SUPEREXAM Page 86 of 94


For inquiries visit www.topnotchboardprep.com.ph or email us at topnotchmedicalboardprep@gmail.com
TOPNOTCH MEDICAL BOARD PREP SURGERY SUPEREXAM
For inquiries visit www.topnotchboardprep.com.ph or email us at topnotchmedicalboardprep@gmail.com
Item QUESTION EXPLANATION AUTHOR TOPNOTCH
# EXAM
657 During thyroidectomy, you should always look for the thyroidea ima usually rises from innominate LITO JAY MACARAIG, DIAGNOSTIC
the 5th blood supply of the thyroid which usually artery or the aorta. The innominate artery is MD (TOP 8 - FEB 2013 EXAM - AUG
arises from? AKA brachiocephalic artery MED BOARDS; 2013
A. Brachiocephalic artery TOPNOTCH MD)
B. Innominate artery
C. Aorta
D. A or B
E. All of the above

658 On a corrective surgery for PDA, the most the Left RLN is hookson the arch of aorta as it LITO JAY MACARAIG, DIAGNOSTIC
commonly injured structure is the goes up, hence commonly injured during PDA MD (TOP 8 - FEB 2013 EXAM - AUG
A. Thyroidea Ima correction. MED BOARDS; 2013
B. Superior Thyroid artery TOPNOTCH MD)
C. Left recurrent laryngeal nerve
D. Inferior thyroid artery
E. None of the above

659 Parathyroid adenoma is best treated with it is best to remove only the adenomatous gland LITO JAY MACARAIG, DIAGNOSTIC
A. Removal of the adenomatous gland because upto present, there is no better MD (TOP 8 - FEB 2013 EXAM - AUG
B. 3.5 parathyroidectomy substitute for endogenous PTH. MED BOARDS; 2013
C. Total parathyroidectomy + TOPNOTCH MD)
autotransplantation
D. B or C
E. Radiation therapy

660 You had a 64 year old smoker patient whom you Musoepidermoid CA is the most common LITO JAY MACARAIG, DIAGNOSTIC
operated on due to salivary gland tumor. The malignat tumor of salivary glands in adults. MD (TOP 8 - FEB 2013 EXAM - AUG
histopath results showed highly aggressive MED BOARDS; 2013
malignant cells. What is your first consideration as TOPNOTCH MD)
your diagnosis?
A. Adenoid cystic CA
B. Acinic cell CA
C. Pleomorphic Adenoma
D. Mucoepidermoid CA
E. Adenocystic CA
661 A 45-year old male, chronic alcoholic presents to In any patient first assess ABC's: Airway, HAZEL KAREN RAZ, MIDTERM 2 -
the ER with coffee-ground vomiting amounting to breathing, Circulation MD (TOP 6 - FEB 2013 AUG 2013
2 cups and dark tarry stools. On physical MED BOARDS;
examination, patient is afebule, BP 90/60 HR 110, TOPNOTCH MD)
RR 24, Positive ascites. What is the first step in
management of this patient?
A. Access airway, breathing and circulation
B. Establish access with IV catheters
C. Control bleeding
D. do CBC, PT, PTT
E. None of the above
662 A 25-Year old female presented to the ER with This is done to rule out ectopic pregnancy or HAZEL KAREN RAZ, MIDTERM 2 -
RLQ pain, and fever. Patient also states that she PID. MD (TOP 6 - FEB 2013 AUG 2013
had 2 episodes of vomiting, of nonbilous material MED BOARDS;
6 hours prior. On physical examination, patient is TOPNOTCH MD)
in moderate distress but with stable vital signs.
Positive rebound tenderness at the RLQ, rectal
exam was normal. CBC was done revealing
leukocytosis with predominance of neutrophils,
urinalysis was normal. What additional
examination must be ordered?
A. Ultrasound
B. Blood culture
C. Abdominal X-ray upright and supine
D. Pregnancy Test
E. None of the above
663 Bleeding from esophageal varies originate from The left gastric vein (or coronary vein) carries HAZEL KAREN RAZ, MIDTERM 2 -
which blood vessel? blood low in oxygen, tributaries derived from MD (TOP 6 - FEB 2013 AUG 2013
A. Left gastric artery both surfaces of the stomach; it runs from right MED BOARDS;
B. Left Gastric Vein to left along the lesser curvature of the stomach, TOPNOTCH MD)
C. Right Gastric Artery between the two layers of the lesser omentum,
D. Right Gastric Vein to the esophageal opening of the stomach, where
E. None of the above it receives some esophageal veins.

664 What is the first line diagnostic test for Trial of PPI is the only diagnostic test for reflux HAZEL KAREN RAZ, MIDTERM 2 -
uncomplicated reflux diseases? without dysphagia. MD (TOP 6 - FEB 2013 AUG 2013
A. Endoscopy MED BOARDS;
B. 24-h pH test TOPNOTCH MD)
C. Manometry
D. Trial of PPI
E. None of the above

665 Where does the criminal nerve of Grassi originate? Left Anterior : Right Posterior HAZEL KAREN RAZ, MIDTERM 2 -
A. Posterior (Left) Vagus branch MD (TOP 6 - FEB 2013 AUG 2013
B. Posterior (Right) Vagus Branch MED BOARDS;
C. Anterior (Left) Vagus Branch TOPNOTCH MD)
D. Anterior (Right) Vagus Branch
E. None of the above

TOPNOTCH MEDICAL BOARD PREP SURGERY SUPEREXAM Page 87 of 94


For inquiries visit www.topnotchboardprep.com.ph or email us at topnotchmedicalboardprep@gmail.com
TOPNOTCH MEDICAL BOARD PREP SURGERY SUPEREXAM
For inquiries visit www.topnotchboardprep.com.ph or email us at topnotchmedicalboardprep@gmail.com
Item QUESTION EXPLANATION AUTHOR TOPNOTCH
# EXAM
666 What type of vagotomy is described in the ff: Truncal or total abdominal vagotomy. The HAZEL KAREN RAZ, MIDTERM 2 -
Denervation of stomach (including pylorus) above main vagal trunks are divided, and surgery is MD (TOP 6 - FEB 2013 AUG 2013
crus of diaphragm, prescruing aliac and hepatic accompanied by a drainage procedure, such as MED BOARDS;
branches? pyloroplasty . TOPNOTCH MD)
A. Total Selective (total gastric) vagotomy. The main
B. Selective vagal trunks are dissected to the point where
C. Highly Selective the branch leading to the biliary tree divides,
D. Proximal Gastric and there is a cut at the section of vagus close to
E. None of the above the hepatic branch. This procedure is rarely
indicated or performed.
Highly selective vagotomy (HSV). HSV
selectively deprives the parietal cells of vagal
nerves, and reduces their sensitivity to
stimulation and the release of acid. It does not
require a drainage procedure. The branches of
Latarjet's nerve are divided from the
esophagogastric junction to the crow's foot
along the lesser curvature of the stomach.
Thoracoscopic vagotomy. Performed through
the third, sixth, and seventh left intercostal
spaces, the posterior vagus trunk is isolated,
clipped, and a segment excised.
667 What is the strongest tissue layer of the small HAZEL KAREN RAZ, MIDTERM 2 -
intestine? MD (TOP 6 - FEB 2013 AUG 2013
A. Epithelium MED BOARDS;
B. Mucosa TOPNOTCH MD)
C. Submucosa
D. Muscularis Layer
E. None of the above

668 Most common cause of small bowel obstruction in HAZEL KAREN RAZ, MIDTERM 2 -
adults? MD (TOP 6 - FEB 2013 AUG 2013
A. Hernia MED BOARDS;
B. Tumors TOPNOTCH MD)
C. Constipation
D. Adhesion
E. None of the above

669 This type of hernia is commonly foound in females Femoral hernias occur just below the inguinal HAZEL KAREN RAZ, MIDTERM 2 -
and passes anterior to Cooper's ligament and ligament, when abdominal contents pass MD (TOP 6 - FEB 2013 AUG 2013
posterior to ingunal ligament? through a naturally occurring weakness called MED BOARDS;
A. Femoral Hernia the femoral canal. TOPNOTCH MD)
B. Direct Inquinal Hernia
C. Pantaloon Hernia
D. Litre's Hernia
E. None of the above

670 Time of blood supply to the liver? Portal vein - 2/3 blood, 50% O2
HAZEL KAREN RAZ, MIDTERM 2 -
A. Portal vein provides 2/3 of blood to liver Hepatic artery - 1/3 blood, 50% O2 MD (TOP 6 - FEB 2013 AUG 2013
B. Hepatic Artery provides 2/3 of blood to liver MED BOARDS;
C. Portal vein provides 70% of O2 supply to liver TOPNOTCH MD)
D. Hepatic Artery provides 70% of O2 supply to
liver
E. None of the above

671 Most common type of gall bladder stone? Combination of both pigment and cholesterol HAZEL KAREN RAZ, MIDTERM 2 -
A. Cholesterol stone MD (TOP 6 - FEB 2013 AUG 2013
B. Pigment MED BOARDS;
C. Black Stone TOPNOTCH MD)
D. Mixed Stones
E. None of the above

672 A 40 -year old female presents to the ER with Anytime if with complications. If , 72 hours from HAZEL KAREN RAZ, MIDTERM 2 -
fever, nausea, vomiting, constant RUQ pain with a onset, proceed with surgery. If more that 72 MD (TOP 6 - FEB 2013 AUG 2013
history of bilary colic. Physical Examination shows hours, wait for 4 - 6 weeks for inflammation to MED BOARDS;
inspiratory arrest on palpation of RUQ. Diagnosis subside then operate TOPNOTCH MD)
is acute cholecystitis. When is cholecystectomy
indicated?
A. Cholecystectomy if 1 week from onset of
symptoms
B. Within 72 Hours if without complications
C. Wait 4-6 weeks if with complication
D. if less than 72 hours, wait 5 days from
resolution of inflammation
E. None of the above
673 Possible comlications if acute pancreatitis? Locoregional complications include pancreatic HAZEL KAREN RAZ, MIDTERM 2 -
A. Necrosis pseudocyst (Most common, occurring in up to MD (TOP 6 - FEB 2013 AUG 2013
B. Abscess 25% of all cases) and phlegmon / abscess MED BOARDS;
C. Hemorrhage formation, splenic artery pseudoaneurysms, TOPNOTCH MD)
D. AOTA hemorrhage from erosions into splenic artery
E. None of the above and vein, thrombosis of the splenic vein,
superior mesenteric vein and portal veins (in
descending order of frequency), duodenal
obstruction, common bile duct obstruction,
progression to chronic pancreatitis

TOPNOTCH MEDICAL BOARD PREP SURGERY SUPEREXAM Page 88 of 94


For inquiries visit www.topnotchboardprep.com.ph or email us at topnotchmedicalboardprep@gmail.com
TOPNOTCH MEDICAL BOARD PREP SURGERY SUPEREXAM
For inquiries visit www.topnotchboardprep.com.ph or email us at topnotchmedicalboardprep@gmail.com
Item QUESTION EXPLANATION AUTHOR TOPNOTCH
# EXAM
674 What is the preferred crystalloid in large volume Lactated Ringer’s is a sterile solution that is used HAZEL KAREN RAZ, MIDTERM 2 -
rescusutation? to replace fluid lost by the body. It is commonly MD (TOP 6 - FEB 2013 AUG 2013
A. PNSS used for fluid resuscitation, meaning that the MED BOARDS;
B. D5W patient needs aggressive fluid replacement for TOPNOTCH MD)
C. Plain LR their injury or illness. Lactated Ringer’s looks
D. D50/50 W like water, but it contains additives including
E. None of the above calcium, potassium, lactate, sodium and
chloride.
675 What percentage of TBW is found in the ECF? 60-40-20 rule HAZEL KAREN RAZ, MIDTERM 2 -
A. 10% MD (TOP 6 - FEB 2013 AUG 2013
B. 20% MED BOARDS;
C. 30% TOPNOTCH MD)
D. 40%
E. 15%

676 First sign / manifestation of hypovolemic shock? Direct loss of effective circulating blood volume HAZEL KAREN RAZ, MIDTERM 2 -
A. Hypotension leading to: MD (TOP 6 - FEB 2013 AUG 2013
B. Tachypnea A rapid, weak, thready pulse due to decreased MED BOARDS;
C. Tachycardia blood flow combined with tachycardia TOPNOTCH MD)
D. Oliguria
E. Orthostatic Hypotension

677 What are the surgical indications for hemothorax? A hemothorax is managed by removing the HAZEL KAREN RAZ, MIDTERM 2 -
A. Massive hemothorax >= 1000mL source of bleeding and by draining MD (TOP 6 - FEB 2013 AUG 2013
B. >150mL/hr from chest tube for 2hrs. the blood already in the thoracic cavity. Blood in MED BOARDS;
C. BT needed for hemodynamic stability the cavity can be removed by inserting a drain TOPNOTCH MD)
D. A & B (chest tube) in a procedure called a tube
E. All of the above thoracostomy. Usually the lung will expand and
the bleeding will stop after a chest tube is
inserted.
678 Patient presented with respiratory distress, This is a case of cardiac tamponade. HAZEL KAREN RAZ, MIDTERM 2 -
hypotension and distended neck veins and muffled MD (TOP 6 - FEB 2013 AUG 2013
heart sounds. ECG was done revealing low voltage MED BOARDS;
electrical alternans, What would be the 1st step in TOPNOTCH MD)
management of this patient?
A. Pericardiocentesis
B. Surgery to ID source of bleeding
C. Emergency thoracostomy
D. Fluids to raise CVP > Intrapericardial
pressures
E. None of the above
679 The following are short term complications of Cataract formation is part of the long term HAZEL KAREN RAZ, MIDTERM 2 -
electrical burns except:? complications of burns. MD (TOP 6 - FEB 2013 AUG 2013
A. Amythmia MED BOARDS;
B. Acute tubular necrosis TOPNOTCH MD)
C. Compartment syndrome
D. Cataracts
E. None of the above

680 The three levels of lymphatic drainage of the Pectoralis minor divides the levels of LN that HAZEL KAREN RAZ, MIDTERM 2 -
breast are numbered in relation to what structure? drains the breast. MD (TOP 6 - FEB 2013 AUG 2013
A. Pectoralis Major MED BOARDS;
B. Pectoralis Minor TOPNOTCH MD)
C. Serratus Anterior
D. Deltoid Muscle
E. Biceps Brachii

681 A 24 yo man presents with a 1-2 day history of Answer: D. Give topical antibiotics with OPD MICHELLE JAY MIDTERM 1 -
right eye redness, purulent discharge, mild follow-up FRANCISCO, MD (TOP 9 AUG 2013
blurring of vision and mild pain. PE reveals a Notes: Choice D is inappropriate. Condition is - FEB 2013 MED
visual acuity of 20/25 right and 20/20 left, right most likely Gonococcal. BOARDS; TOPNOTCH
conjunctival edema, injection, a clear cornea, and MD)
copious creamy purulent discharge. Which of the
following would be inappropriate in the
management?
A. Obtain ophthalmologic consultation
B. Obtain thorough sexual history
C. Initiate systemic antibiotic treatment
D. Give topical antibiotics with OPD follow-up
682 Mr. Bugoy Tigas, 20 yo male, is stabbed in the left Answer: C. Order an echocardiogram MICHELLE JAY MIDTERM 1 -
side of his chest, medial to the nipple. Upon Notes: Cardiac tamponade is characterized by FRANCISCO, MD (TOP 9 AUG 2013
examination, his blood pressure is 90/60 mmHg decreased cardiac output and increased central - FEB 2013 MED
and his pulse is 130/min. His jugular venous pulse venous pressure owing to restriction of blood BOARDS; TOPNOTCH
increases on inspiration, whereas his peripheral flow into and out of the heart as fluid in the MD)
pulse and blood pressure decreased on pericardial sac restricts filling of all the cardiac
inspiration. Breath sounds are normal bilaterally. chambers. An echocardiogram is the most
The patient’s chest x-ray film is unremarkable. sensitive and specific noninvasive test to
After receiving 2 L of isotonic saline, his blood establish the presence of fluid in the pericardial
pressure remains low, whereas his central venous sac.
pressure rises to 32 cmH2O. Which of the
following is the most appropriate next step in the
most appropriate next step in the management of
this patient?
A. Insert a chest tube into the left pleural cavity
TOPNOTCH MEDICAL BOARD PREP SURGERY SUPEREXAM Page 89 of 94
For inquiries visit www.topnotchboardprep.com.ph or email us at topnotchmedicalboardprep@gmail.com
TOPNOTCH MEDICAL BOARD PREP SURGERY SUPEREXAM
For inquiries visit www.topnotchboardprep.com.ph or email us at topnotchmedicalboardprep@gmail.com
Item QUESTION EXPLANATION AUTHOR TOPNOTCH
# EXAM
B. Increase parenteral fluids until the blood
pressure increases.
C. Order an echocardiogram.
D. Decrease venous pressure by administering a
venodilator.

683 A grossly underweight 52 yo woman with chronic Answer: D. Pneumothorax MICHELLE JAY MIDTERM 1 -
malnutrition is due to undergo major surgery. It is Notes: Pneumothorax can inadvertently occur FRANCISCO, MD (TOP 9 AUG 2013
decided to start total parenteral nutrition (TPN) as while introducing a subclavian venous catheter. - FEB 2013 MED
part of the initial therapy. While introducing a The patient will complain of sudden dyspnea. A BOARDS; TOPNOTCH
central venous catheter into the right subclavian chest x-ray film should always be taken after MD)
vein, the patient develops sudden dyspnea. Which subclavian catheter placement, to determine the
of the following is the most likely diagnosis? position of the tip of the catheter and to exclude
A. Pulmonary embolism the possibility of pneumothorax.
B. Fat embolism
C. Acute anxiety
D. Pneumothorax
684 A 60 yo alcoholic male presents with severe chest Answer: D. Administer Gastrografin swallow MICHELLE JAY MIDTERM 1 -
pain after repeated vomiting. A chest X-ray shows test FRANCISCO, MD (TOP 9 AUG 2013
a small left pleural effusion. The next step in - FEB 2013 MED
management is: BOARDS; TOPNOTCH
A. Obtain cardiac enzymes and admit to coronary MD)
care unit
B. Insert nasogastric tube, administer intravenous
fluids, and observe
C. Perform upper endoscopy
D. Administer Gastrografin swallow test
685 Three months after accidental ingestion of Answer: D. Esophagectomy with gastric pull- MICHELLE JAY MIDTERM 1 -
muriatic acid, a 30 yo female consults for up FRANCISCO, MD (TOP 9 AUG 2013
dysphagia. Barium swallow shows a 10cm - FEB 2013 MED
strictured segment of distal thoracic esophagus BOARDS; TOPNOTCH
and normal-looking stomach. What is the MD)
preferred treatment for this patient?
A. Esophageal dilatation
B. Gastric pull-up
C. Esophagectomy with colonic interposition
D. Esophagectomy with gastric pull-up
686 The criteria for diagnosis of Primary Sclerosing Answer: A. Presence of choledocholithiasis MICHELLE JAY MIDTERM 1 -
Cholangitis are the following; except, FRANCISCO, MD (TOP 9 AUG 2013
A. Presence of choledocholithiasis - FEB 2013 MED
B. No evidence of primary liver disease BOARDS; TOPNOTCH
C. Absence of history of congenital anomalies MD)
D. Stenosis of a major portion of biliary duct
system
687 Statement/s true about metabolic changes in Answer: B. There is a rapid depletion of labile MICHELLE JAY MIDTERM 1 -
injured patients; and functional stores. FRANCISCO, MD (TOP 9 AUG 2013
A. There is an obligatory decrease in energy - FEB 2013 MED
expenditure and nitrogen excretion BOARDS; TOPNOTCH
B. There is a rapid depletion of labile and MD)
functional energy stores
C. There is a positive nitrogen balance
D. All of these
688 The most common organism associated with OPSI Answer: A. Streptococcus pneumoniae MICHELLE JAY MIDTERM 1 -
is: Notes: OPSI – Overwhelming Post-Splenectomy FRANCISCO, MD (TOP 9 AUG 2013
A. Streptococcus pneumoniae Infection - FEB 2013 MED
B. Pseudomonas aeruginosa BOARDS; TOPNOTCH
C. Hemophilus influenzae MD)
D. Neisseria meningitides

TOPNOTCH MEDICAL BOARD PREP SURGERY SUPEREXAM Page 90 of 94


For inquiries visit www.topnotchboardprep.com.ph or email us at topnotchmedicalboardprep@gmail.com
TOPNOTCH MEDICAL BOARD PREP SURGERY SUPEREXAM
For inquiries visit www.topnotchboardprep.com.ph or email us at topnotchmedicalboardprep@gmail.com
Item QUESTION EXPLANATION AUTHOR TOPNOTCH
# EXAM
689 A 68 yo man presents at the ED because of such Answer: D. Acute small bowel infarction MICHELLE JAY MIDTERM 1 -
severe abdominal pain. He tells the triage nurse Notes: Acute small bowel infarction is indicated FRANCISCO, MD (TOP 9 AUG 2013
that he hadn’t been feeling well for the past couple by the sudden onset of severe abdominal pain - FEB 2013 MED
of months, primarily because he had been having with vomiting and abdominal distention out of BOARDS; TOPNOTCH
abdominal pain about 30 minutes after eating and proportion with the physical findings, absent MD)
as a consequence lost almost 10 pounds, but last bowel sounds, a striking neutrophilic
night he suddenly developed severe stomach ache. leukocytosis with left shift, lactic acidosis,
He also has been vomiting and has had several hypotension, and increased serum amylase
episodes of bloody diarrhea. Upon PE, the concentration of bowel origin.
physician notes hypotension and confirms the
abdominal pain and notices abdominal distention.
However, bowel sounds are absent, and there is no
rebound tenderness present or other relevant
findings upon abdominal examination. Laboratory
data reveal an absolute neutrophilic leukocytosis
and left shift plus lactic acidosis, and elevation of
the serum amylase level. Which of the following is
the most likely diagnosis?
A. Acute ulcerative colitis
B. Hemorrhagic pancreatitis
C. Aortoenteric fistula
D. Acute small bowel infarction
690 Ten years ago, a 79 yo female with a 25-year Answer: D. Formation of an arteriovenous MICHELLE JAY MIDTERM 1 -
history of type 2 diabetes was diagnosed with (AV) fistula FRANCISCO, MD (TOP 9 AUG 2013
diabetic nephropathy. At this time, she is Notes: To perform hemodialysis, it is necessary - FEB 2013 MED
overweight but not obese and she smokes about to surgically form an arteriovenous (AV) fistula. BOARDS; TOPNOTCH
three packs of cigarettes per week. Despite being This is accomplished by creating an opening in MD)
diagnosed with diabetic nephropathy, she had not which the artery is sewn on to a vein; this is
stopped smoking and in general had not tightly usually done in the arm. In hemodialysis, blood
controlled her blood glucose levels; consequently, is withdrawn from an artery and returned via a
her renal function has steadily deteriorated and vein.
her physician now believes she has ESRD. He
arranges for her to undergo hemodialysis. Before
undergoing hemodialysis, it was also
recommended that she have an operation. Which
of the following choices describes the surgical
procedure most likely recommended?
A. A renal biopsy
B. Insertion of a special, soft catheter through a
small slit made adjacent to the navel under local
anesthesia
C. Performing open surgery and inserting a
catheter under general anesthetic
D. Formation of an arteriovenous (AV) fistula
691 A 65 yr-old man w/ a history of chronic alcohol Answer: A MICHELLE JAY MIDTERM 1 -
abuse has been experiencing epigastric & A px w/ pancreatitis can have severe abdominal FRANCISCO, MD (TOP 9 AUG 2013
periumbilical pain asso. w/ nonbilious vomiting pain & rigidity. Surgery should be avoided - FEB 2013 MED
for 1 day. He denies any melena or hematemesis. except for complications (necrotizing panc.or BOARDS; TOPNOTCH
In the past he has had several episodes of similar symptomatic endocyst). Initial management: MD)
pain that sometimes radiated to the back and was bowel rest, IV resuscitation, parenteral nutrition
hospitalized for several days 2 months prior. No & monitoring in ICU when appropriate.
previous surgery or medical problems.P/E: BP
120/80, HR 110 bpm, dry mucous membranes.
Abdomen not distended, no surgical scars.
Diminished bowel sounds. Soft abdomen, exhibits
voluntary guarding of epigastrium. Serum amylase
level is 550 units/100 ml. What is the most
reasonable initial step in management of this
patient?
A. Establish intravenous access
B. Conduct sonographic studies to demonstrate
cholelithiasis
C. Perform CT scanning to diagnose a pancreatic
pseudocyst
D. Perform esophagogastroduodenoscopy (EGD)
to evaluate for varices and complications of
cirrhosis
E. Initiate a low-fat diet and antilipid treatment.
692 The syndrome of multi-organ system failure Answer: A MICHELLE JAY MIDTERM 1 -
(MOF): The “two-event” model of multi-organ system FRANCISCO, MD (TOP 9 AUG 2013
A. Involves sequential insults that lead to systemic failure involves an initial insult that results in a - FEB 2013 MED
hyperinflammation primed inflammatory response; can develop BOARDS; TOPNOTCH
B. Requires the documentation of active infection w/out overt infection;bimodal-within 3 days of MD)
C. Has decreased in incidence over the past decade initial insult or 6-8 days after insult.
D. Requires diagnosis within 3 days of the
systemic insult
E. Demonstrates consistent improvement after
blood transfusion
693 One of the following is a true diverticulum: Answer: B MICHELLE JAY MIDTERM 1 -
A. Zenker Parabronchial (midesophageal diverticulum) is FRANCISCO, MD (TOP 9 AUG 2013
B. Parabronchial a true diverticulum caused by traction of - FEB 2013 MED
C. Meckel inflamed mediastinal nodes. BOARDS; TOPNOTCH
D. Pharyngoesophageal MD)

TOPNOTCH MEDICAL BOARD PREP SURGERY SUPEREXAM Page 91 of 94


For inquiries visit www.topnotchboardprep.com.ph or email us at topnotchmedicalboardprep@gmail.com
TOPNOTCH MEDICAL BOARD PREP SURGERY SUPEREXAM
For inquiries visit www.topnotchboardprep.com.ph or email us at topnotchmedicalboardprep@gmail.com
Item QUESTION EXPLANATION AUTHOR TOPNOTCH
# EXAM
694 A 35 yr-old woman complains of dysphagia, Answer: A MICHELLE JAY MIDTERM 1 -
regurgitation and weight loss. Esophagography The cause of achalasia is idiopathic; however, FRANCISCO, MD (TOP 9 AUG 2013
shows narrowing of the distal end of esophagus severe emotional stress, T.cruzi infection - FEB 2013 MED
and manometry studies show significant tertiary causing destruction of Auerbach plexus & BOARDS; TOPNOTCH
waveforms. The LES has high residual pressure on drastic wt.loss have been implicated. MD)
swallowing. Which of the following has not been
implicated as a possible cause of her disease?
A. H.pylori infection
B. Severe emotional stress
C. A parasitic infection
D. Drastic weight reduction
E. Degeneration of Auerbach plexus

695 All but one meet the criteria for referral to burn Answer: C MICHELLE JAY MIDTERM 1 -
center: *Review criteria for referral to burn center FRANCISCO, MD (TOP 9 AUG 2013
A. A 50 yr.old w/ a 1% TBSA partial thickness PT burns > 10% TBSA; chemical burns..etc. - FEB 2013 MED
burn on her left hand from a cooking accident BOARDS; TOPNOTCH
B. A 30 yr-old construction worker with pain & MD)
blistering bilaterally on the knees after kneeling in
wet cement whole afternoon
C. A 25 yr-old man w/ 7 % TBSA partial thickness
burns on the chest.
D. 18 yr-old man in motor vehicle collision w/
30% TBSA burns on chest and circumferential
burns on arms
696 Refeeding syndrome is characterized by which of Answer: C MICHELLE JAY MIDTERM 1 -
the following electrolyte abnormalities? Refeeding syndrome results when glucose is FRANCISCO, MD (TOP 9 AUG 2013
A. Hyponatremia, hypokalemia,hypercalcemia administered quickly to an individual w/ poor - FEB 2013 MED
B. Hyperphosphatemi, hypokalemia, hypocalcemia nutrient intake before TPN. Subsequent rapid BOARDS; TOPNOTCH
C. Hypokalemia, hypomagnesemia, serum depletion, magnesium, phosphorus or MD)
hypophosphatemia potassium develops.
D. Hypocalcemia, hypernatremia ,
hypomagnesemia
697 Definitive treatment of a patient with sclerosing Answer: D MICHELLE JAY MIDTERM 1 -
cholangitis and biliary cirrhosis involves which of Once sclerosing cholangitis has progressed to FRANCISCO, MD (TOP 9 AUG 2013
the following? cirrhosis, the only definitive treatment is hepatic - FEB 2013 MED
A. Ursodeoxychlic acid transplant. BOARDS; TOPNOTCH
B. Corticosteroids MD)
C. Endoscopic balloon dilation & stenting
D. Hepatic transplantation

698 A 60 yr-old alcoholic man has a 24-hr history of Answer: D MICHELLE JAY MIDTERM 1 -
nausea and vomiting, abdominal pain, distention Although abdominal distention maybe because FRANCISCO, MD (TOP 9 AUG 2013
and decreased passage of stool and flatus. He of ascites, its onset is much more gradual than - FEB 2013 MED
underwent abdominoperineal resection of the 24 hrs. And it is usually painless. BOARDS; TOPNOTCH
rectum for cancer 18 months earlier, along w/ MD)
post-op irradiation and chemo. P/E reveals
distended, diffusely tender, tympanitic abdomen.
Which of the following is the least likely diagnosis?
A. pancreatitis with ileus
B. adhesive bowel obstruction
C. bowel obstruction caused by extrinsic
compression
D. alcoholic hepatitis with ascites
699 Radiation delivered to the breast after right Answer: D MICHELLE JAY MIDTERM 1 -
lumpectomy and sentinel lymph node biopsy for a Radiation for local control; chemo for systemic. FRANCISCO, MD (TOP 9 AUG 2013
1.2 cm node-neg infiltration ductal carcinomais - FEB 2013 MED
likely to be asso. with which of the following? BOARDS; TOPNOTCH
A. Decreased risk for systemic recurrence MD)
B. Can be used in lieu of chemotherapy in early
stage breast cancers.
C. Increased risk for lymphoma
D. Decreased risk for local recurrence
700 Based on the classification of the American College Answer: C MICHELLE JAY MIDTERM 1 -
of Surgeons, the classical features of hypovolemic Class I –orthostatic hypotension; Class II – FRANCISCO, MD (TOP 9 AUG 2013
shock is first noted in: tachycardia; Class III – hypotension; Class IV – - FEB 2013 MED
A. Class I profound shock BOARDS; TOPNOTCH
B. Class II MD)
C. Class III
D. Class IV

TOPNOTCH MEDICAL BOARD PREP SURGERY SUPEREXAM Page 92 of 94


For inquiries visit www.topnotchboardprep.com.ph or email us at topnotchmedicalboardprep@gmail.com
TOPNOTCH MEDICAL BOARD PREP SURGERY SUPEREXAM
For inquiries visit www.topnotchboardprep.com.ph or email us at topnotchmedicalboardprep@gmail.com

Item # ANSWER 88 C 174 E 262 B 350 A


1 D 89 B 175 C 263 A 351 C
2 C 90 D 176 A 264 A 352 A
3 B 91 B 177 D 265 C 353 B
4 B 92 D 178 E 266 E 354 B
5 B 93 B 179 A 267 B 355 A
6 C 94 D 180 E 268 D 356 C
7 A 95 A 181 B 269 A 357 A
8 A 96 C 182 A 270 E 358 C
9 A 97 D 183 D 271 D 359 D
10 A 98 B 184 C 272 B 360 A
11 E 99 A 185 A 273 B 361 E
12 D 100 C 186 B 274 C 362 A
13 A 101 B 187 A 275 D 363 B
14 D 102 B 188 C 276 A 364 D
15 D 103 A 189 D 277 C 365 C
16 B 104 C 190 E 278 C 366 E
17 A 105 E 191 B 279 D 367 B
18 E 106 Bonus, 192 E 280 E 368 D
19 A correct answer should 193 C 281 B 369 E
20 A be 11,700ml 194 B 282 D 370 B
21 B 107 E 195 D 283 C 371 C
22 C 108 B 196 D 284 C 372 D
23 D 109 C 197 B 285 E 373 A
24 A 110 D 198 E 286 D 374 B
25 B 111 D 199 A 287 E 375 C
26 C 112 B 200 C 288 C 376 E
27 A 113 C 201 D 289 C 377 B
28 C 114 E 202 D 290 A 378 D
29 D 115 B 203 B 291 E 379 A
30 A 116 D 204 B 292 D 380 A
31 A 117 C 205 C 293 D 381 E
32 B 118 A 206 B 294 E 382 D
33 B 119 E 207 E 295 B 383 D
34 C 120 B 208 E 296 E 384 C
35 E 121 D 209 B 297 A 385 A
36 B 122 C 210 D 298 C 386 A
37 C 123 D 211 A 299 E 387 E
38 C 124 D 212 E 300 D 388 B
39 A 125 C 213 D 301 D 389 B
40 B 126 A 214 E 302 D 390 A
41 E 127 B 215 C 303 D 391 B
42 B 128 D 216 B 304 A 392 D
43 B 129 D 217 D 305 B 393 C
44 B 130 B 218 A 306 A 394 B
45 A 131 B 219 C 307 D 395 C
46 D 132 A 220 B 308 B 396 A
47 C 133 D 221 B 309 C 397 D
48 A 134 A 222 C 310 B 398 B
49 A 135 B 223 C 311 D 399 D
50 D 136 A 224 B 312 C 400 B
51 C 137 D 225 A 313 B 401 D
52 C 138 B 226 C 314 B 402 B
53 C 139 D 227 C 315 A 403 D
54 B 140 B 228 C 316 B 404 B
55 C 141 D 229 C 317 B 405 B
56 A 142 C 230 B 318 C 406 C
57 A 143 C or D 231 A 319 C 407 E
58 A 144 C 232 B 320 B 408 C
59 A 145 A 233 B 321 B 409 B
60 E 146 B 234 B 322 C 410 A
61 C 147 A 235 B 323 A 411 E
62 B 148 A 236 A 324 B 412 E
63 A 149 B 237 A 325 A 413 D
64 E 150 A 238 B 326 D 414 D
65 C 151 B 239 A 327 C 415 D
66 B 152 A 240 D 328 D 416 D
67 D 153 D 241 C 329 D 417 E
68 E 154 C 242 C 330 B 418 B
69 C 155 B 243 D 331 B 419 D
70 C 156 C 244 A 332 D 420 B
71 A 157 C 245 B 333 C 421 D
72 E 158 A 246 D 334 C 422 B
73 E 159 B 247 C 335 D 423 A
74 E 160 D 248 B 336 E 424 C
75 D 161 E 249 D 337 A 425 C
76 D 162 B 250 B 338 C 426 D
77 B 163 E 251 C 339 D 427 B
78 A 164 D 252 A 340 A 428 A
79 C 165 B 253 B 341 D 429 E
80 D 166 A 254 C 342 A 430 D
81 A 167 C 255 B 343 B 431 B
82 B 168 E 256 C 344 A 432 C
83 A 169 E 257 C 345 A 433 D
84 C 170 B 258 B 346 B 434 B
85 B 171 D 259 E 347 D 435 B
86 C 172 D 260 D 348 A 436 D
87 B 173 A 261 B 349 B 437 B
TOPNOTCH MEDICAL BOARD PREP SURGERY SUPEREXAM Page 93 of 94
For inquiries visit www.topnotchboardprep.com.ph or email us at topnotchmedicalboardprep@gmail.com
TOPNOTCH MEDICAL BOARD PREP SURGERY SUPEREXAM
For inquiries visit www.topnotchboardprep.com.ph or email us at topnotchmedicalboardprep@gmail.com
438 B 527 D 616 C
439 B 528 A 617 B
440 C 529 B 618 B
441 C 530 B 619 E
442 A 531 D 620 C
443 B 532 D 621 A
444 C 533 A 622 E
445 D 534 B 623 C
446 A 535 B 624 A
447 B 536 D 625 C
448 D 537 B 626 B
449 B 538 B 627 C
450 E 539 D 628 B
451 C 540 D 629 C
452 B 541 C 630 C
453 E 542 C 631 B
454 C 543 A 632 C
455 C 544 B 633 C
456 D 545 E 634 A
457 B 546 E 635 A
458 C 547 B 636 D
459 E 548 C 637 C
460 C 549 C 638 D
461 D 550 D 639 A
462 B 551 C 640 A
463 B 552 A 641 D
464 C 553 B 642 D
465 B 554 C 643 D
466 A 555 B 644 D
467 E 556 A 645 E
468 C 557 D 646 C
469 A 558 A 647 E
470 C 559 C 648 C
471 B 560 A 649 A
472 A 561 B 650 B
473 B 562 B 651 D
474 A 563 C 652 E
475 A 564 B 653 B
476 D 565 A 654 E
477 B 566 B 655 A
478 D 567 C 656 C
479 C 568 D 657 E
480 A 569 A 658 C
481 C 570 D 659 A
482 B 571 D 660 D
483 A 572 C 661 A
484 D 573 A 662 D
485 D 574 B 663 B
486 B 575 C 664 D
487 A 576 C 665 B
488 C 577 B 666 B
489 D 578 A 667 C
490 B 579 A 668 D
491 C 580 B 669 A
492 C 581 B 670 A
493 C 582 B 671 D
494 C 583 A 672 B
495 D 584 E 673 D
496 B 585 C 674 C
497 C 586 C 675 B
498 B 587 D 676 C
499 A 588 E 677 E
500 D 589 A 678 D
501 A 590 B 679 D
502 C 591 E 680 B
503 B 592 A 681 D
504 C 593 C 682 C
505 E 594 D 683 D
506 B 595 E 684 D
507 D 596 C 685 D
508 D 597 A 686 A
509 A 598 D 687 B
510 C 599 C 688 A
511 B 600 A 689 D
512 D 601 B 690 D
513 A 602 E 691 A
514 A 603 C 692 A
515 A 604 A 693 B
516 D 605 C 694 A
517 B 606 B 695 C
518 C 607 D 696 C
519 A 608 D 697 D
520 B 609 C 698 D
521 D 610 B 699 D
522 A 611 A 700 C
523 D 612 C
524 B 613 D
525 B 614 C
526 C 615 D
TOPNOTCH MEDICAL BOARD PREP SURGERY SUPEREXAM Page 94 of 94
For inquiries visit www.topnotchboardprep.com.ph or email us at topnotchmedicalboardprep@gmail.com

Das könnte Ihnen auch gefallen